Sternstunden der Mathematik

STERNSTUNDEN DER MATHEMATIK
J.-H. ESCHENBURG
Wolfgang Meyer zum 80. Geburtstag gewidmet
Einführung
Der Titel “Sternstunden der Mathematik” ist ausgeborgt von Stefan Zweigs1 “Sternstunden der Menschheit”, den “zwölf historischen
Miniaturen”, wie der Untertitel sie beschreibt. Dort geht es um Weltgeschichte, aber es sind nicht so sehr die bekanntesten historischen
Ereignisse, an die in diesem Buch erinnert wird, sondern etwas verborgenere, in denen sich gleichwohl das Weltgeschehen fokussierte, wie
die Entdeckung des Pazifik 1513 oder das Schicksal der Familie Suter,
auf deren Besitz der kalifornische Goldrausch von 1849 begann, oder
die erste Telegrafenleitung über den Atlantik 1858, die gleich wieder
verstummte. Stefan Zweig schreibt im Vorwort zu seinem Buch: “Was
ansonsten gemächlich nacheinander und nebeneinander abläuft, komprimiert sich in einem einzigen Augenblick, der alles bestimmt und alles
entscheidet.”
Von solchen Ereignissen hat auch die Wissenschaft und besonders
die Mathematik viele zu bieten. Die Entdeckung der komplexen Zahlen durch Rafael Bombelli um 1572 ist durchaus mit der Entdeckung
des Pazifischen Ozeans zu vergleichen, und das Schicksal von Évariste
Galois war nicht weniger dramatisch und traurig als das von Johann
August Suter. Das Buch, das aus einer Vorlesungsreihe im Winter
2014/15 entstand, möchte versuchen, mathematische Ideengeschichte
nachzuzeichnen anhand einer Auswahl von Ereignissen, die ganz von
den Interessen und dem begrenzten Wissen des Autors bestimmt ist.
Die einzelnen Ereignisse stehen jeweils für eine ganze Entwicklung, die
vorher begonnen hat und nachher weiter entfaltet wird.
Allen Hörerinnen und Hörern der Vorlesung möchte ich für ihre Geduld und Ausdauer und einigen für ihre hilfreichen Kommentare danken. Mein ganz besonderer Dank gilt Erich Dorner, der das Manuskript
immer wieder gelesen und mich auf viele Fehler aufmerksam gemacht
hat.
Date: 5. Juli 2016.
1
Stefan Zweig, 1881 (Wien) - 1942 (Petrópolis, Brasilien)
1
2
J.-H. ESCHENBURG
1. Pythagoras: Verhältnis und Unendlichkeit (−500)
Die Proportionenlehre bildet einen Ausgangspunkt der Mathematikgeschichte, denn sie gibt eine Antwort auf die Frage: Was sind eigentlich
die Zahlen?
In der Antike kannte man zunächst nur die natürlichen Zahlen. Sie
dienten zum Zählen von endlichen Mengen, d.h. von Zusammenfassungen von Individuen nach bestimmten Gesichtspunkten. Insofern ist
die Mengenlehre die Grundlage der Mathematik; die Zahlen kommen
später: Bevor man zählen kann, muss man wissen, was man zählen will,
und die Vereinigung von Mengen ist ursprünglicher als die Addition von
Zahlen.
Doch schon früh, sicher schon in ägyptischer oder babylonischer Zeit
erkannte man, dass die Zahlen noch zu etwas anderem dienen konnten: zum Vergleichen von Größen. Das waren zum Beispiel Längen,
Flächen- und Rauminhalte, Massen und Gewichte, Zeitspannen. Da
es keine allgemein anerkannten Maßeinheiten gab, konnte man solche
Größen nicht einfach durch Zahlen ausdrücken. Aber man konnte zwei
von ihnen miteinander vergleichen, zum Beispiel eine größere Strecke a
mit einer kleineren b.
a
b
Zunächst kann man nur feststellen: dass a größer ist als b. Aber um
wieviel größer? Im besten Fall gewinnen wir a durch mehrfaches Aneinanderlegen von b:2
a
b
b
2
b
b
Die vier mit b bezeichneten Strecken sind allerdings durchaus unterschiedlich,
da sie sich ja an verschiedenen Orten befinden. Aber sie gehen aus der ursprünglich
b genannten Strecke durch Verschieben hervor, wobei Länge und Richtung nicht
geändert wird; in dem Sinne sind sie gleich. “Das Gleiche” ist bekanntlich nicht “dasselbe”. Gleichheit bezieht sich nur auf ein Wesensmerkmal der beiden Gegenstände,
hier die Länge und vielleicht noch die Richtung. Den Gleichheitsbegriff treffen wir
bereits beim Zählen an, also schon beim ursprünglichen Gebrauch der Zahlen: Zwei
durchaus unterschiedliche Mengen werden als gleichartig angesehen, wenn sie gleich
viel Elemente haben, also nur in dem Wesensmerkmal “Anzahl” übereinstimmen.
Wenn bestimmte Waren nach Stückzahl verkauft werden, hängt der Preis nicht
davon ab, welche Waren gekauft wurden, sondern nur von der Anzahl.
STERNSTUNDEN DER MATHEMATIK
3
In unserer Figur erhalten wir a durch 3-faches Aneinanderlegen von
(Kopien von) b, also a = 3b. Weder a noch b sind Zahlen, aber gemeinsam definieren sie eine Zahl, nämlich ihr Verhältnis a/b = 3.
Ein anderes Beispiel ist der Vergleich großer Mengen. Ich stelle mir zwei
ägyptische Bauern vor – nennen wir sie A und B – die zum staatlichen
Getreideeinkäufer nach Memphis kommen, zur Zeit von “Joseph dem
Ernährer” während der “sieben fetten Jahre”. Beide haben einen Sack
Getreide mitgebracht, A einen großen und B einen kleinen. Sie wollen
gerecht entlohnt werden. A muss mehr Lohn bekommen, aber wieviel
mehr? Den Gewichten, die der Einkäufer bereithält, trauen sie nicht.
Aber mit einer Balkenwaage kann man nicht allzu sehr betrügen: Wenn
sie vor und nach dem Beladen beider Schalen im Gleichgewicht ist,
sind die Lasten auf beiden Seiten gleich. Also lädt der Einkäufer das
Getreide von B auf die eine Waagschale und soviel von dem Getreide
von A auf die andere, bis sie im Gleichgewicht ist. Dann lässt er den zu
A gehörigen Haufen auf die Seite schaffen und wiederholt das Gleiche
mit dem restlichen Getreide von A, so oft, bis es aufgebraucht ist.
=
A
B
Auch hier erscheint eine Zahl: A hat 3-mal soviel Getreide wie B. Wir
haben nicht die Körner gezählt, es sind ja “unzählige”, sondern die
Gleichheit der vier (durchaus unterschiedlichen) kleinen Haufen als
Gleichheit der Gewichte definiert. Die “Gleichheit” unterschiedlicher
Haufen ist bereits im ursprünglichen Begriff der Zählens vorhanden:
Hätten wir die Körner zählen können und gleiche Anzahlen festgestellt, hätten wir auch die Haufen als “gleich” angesehen. Das Neue
hier liegt also nicht im Begriff der Gleichheit, sondern in der Methode
des “Zählens des Unzählbaren” durch Gewichts- oder Längenmessung;
wir sind von Zählen zum Messen übergegangen. Wann dieser wichtige
Schritt zum ersten Mal vollzogen wurde, verliert sich im Dunkel der
frühen Geschichte.
Aber was machen wir, wenn es “nicht auskommt”? In unserer zweiten
Figur passt b zweimal in a hinein, und es bleibt noch ein Rest c. Dieser
ist kleiner als b, sonst würde ja noch ein drittes Exemplar von b in a
hineinpassen.
a
b
b
c
4
J.-H. ESCHENBURG
Zunächst können wir nur sagen, dass das Verhältnis a/b zwischen 2 und
3 liegen muss. Um Genaueres zu sagen müssen wir c näher bestimmen,
indem wir c mit b vergleichen. Das geht auf gleiche Weise wie vorher:
Wieder haben wir eine große und eine kleine Strecke, statt a und b
diesmal b und c, und wieder prüfen wir, wie oft c in b hineinpasst; in
unserer Figur geht es einmal, und es bleibt ein Rest d, der kleiner als c
ist. Nun vergleichen wir d mit c; wieder passt d einmal in c hinein mit
einem Rest e, der in unserem Beispiel schließlich genau zweimal in d
aufgeht.
a
b
c
d
c
b
d
e
e e
a
b
Dasselbe in einer anderen figürlichen Darstellung:
e
e
d
b
d
c
b
b
a
c
e
STERNSTUNDEN DER MATHEMATIK
5
In unserem Beispiel ist34


a = 2b + c
d =
2e





b = c+d
c = d+e =
2e + e
= 3e
⇒
c = d+e
b = c+d =
3e + 2e
= 5e



a = 2b + c = 2 · 5 e + 3e = 13 e .
d =
2e 
Wir stellen also fest, dass b in a zwar nicht mehr ganzzahlig aufgeht,
dass es aber eine kleinere Strecke e gibt, die sowohl in a als auch in
b ganzzahlig aufgeht, 13-mal in a und 5-mal in b. Wir haben keine
Maßeinheit gebraucht, um dies festzustellen; die beiden Strecken haben
sich nämlich die für den Vergleich am besten geeignete Maßeinheit, ihr
“gemeinsames Maß” e, selbst gesucht. Damit können wir wieder das
genaue Verhältnis a/b feststellen, nämlich 13/5.
Halten wir noch einmal fest: Größen sind keine Zahlen, aber zwei
Größen a und b können miteinander verglichen werden: Sie können
gleich sein oder eine von beiden, sagen wir a, ist größer als die andere,
a > b. Die kleinere Größe b können wir mehrfach (sagen wir: p-mal) vervielfältigen und die Kopien zu einer neuen Größe pb zusammensetzen.
Wir können das so oft machen, dass pb gerade noch in a hineinpasst;
noch eine weitere Kopie von b passt nicht mehr. Wenn es nicht “aufgeht”, wenn also pb < a, dann vergleichen wir b mit dem Rest c = a−pb,
der sicher kleiner als b ist (sonst würde ja noch eine weitere Kopie von
b in a hineinpassen). Was wir vorher mit a und b gemacht haben, das
machen wir nun mit b und c: Wir kopieren die kleinere Strecke c so oft
(sagen wir: q-mal), dass qc ≤ b, aber (q+1)c > b. Damit haben wir einen
Algorithmus geschaffen, ein Rechenverfahren, das den immer gleichen
Rechenschritt auf immer neue Eingaben anwendet, die im Laufe des
Verfahrens erst produziert werden. Dieses Verfahren heißt Wechselwegnahme oder euklidischer Algorithmus, weil es von Euklid5 beschrieben
3Wir
a
b
können stattdessen auch gleich das Verhältnis a/b ausdrücken:
= 2 + cb , cb = 1 + dc , dc = 1 + de , de = 2 und damit ab = 2 + 1+ 1 1 . Einen solchen
1+ 1
2
Ausdruck nennen wir einen regelmäßigen Kettenbruch und schreiben dafür kurz
a
b = [2; 1, 1, 2]. Das hier beschriebene Verfahren der Wechselwegnahme ist dasselbe
wie die regelmäßige Kettenbruchentwicklung des Verhältnisses a/b.
4
Wenn wir die Gleichung anders herum auflösen, können wir auch das gemeinsame Maß e durch die gegebenen Größen a und b ausdrücken:
e = c−d, d = b−c, c = a−2b, ⇒ e = c−(b−c) = 2c−b = 2(a−2b)−b = 2a−5b.
5Euklid
von Alexandria, ca. 325 - 265 v.Chr. (Alexandria, Ägypten), fasste um
300 v.Chr. das gesamte mathematische Wissen seiner Zeit in seinem Lehrbuch “Elemente” zusammen. Die Proportionenlehre findet sich im 5. Buch der “Elemente”.
6
J.-H. ESCHENBURG
worden ist. Es ist aber sehr viel älter und war zum Beispiel Pythagoras6 vor 500 v.Chr. bestens bekannt. Vielleicht hat er es auf seinen
ausgedehnten Reisen nach Ägypten und Mesopotamien kennen gelernt.
Jedenfalls hat er die Bedeutung dieses Verfahrens voll erkannt und soll
in den Jubelruf “Alles ist Zahl” ausgebrochen sein, weil sich auf diese
Weise die Verhältnisse beliebiger Größen, aus welchem Bereich auch
immer sie stammen mochten (Geometrie, Astronomie, Mechanik, Musik,7 Wirtschaft), durch ein Verhältnis von Zahlen ausdrücken ließen.
Es war die Geburtsstunde der angewandten Mathematik.
Doch nur wenige Jahre später schüttete ein Schüler des Pythagoras,
vermutlich Hippasos,8 reichlich Wasser in diesen schönen Wein, durch
eine der folgenreichsten mathematischen Erkenntnisse der Antike: Es
gibt Strecken a und b, deren Verhältnis mit der Wechselwegnahme niemals genau ermittelt werden kann; immer bleibt noch ein Rest und
das Verfahren endet nie. Vermutlich geschah diese Entdeckung am
Verhältnis des Goldenen Schnitts. Dabei wird eine Strecke a so in zwei
ungleiche Teile b und c unterteilt, dass sie sich zum größeren Teil b so
verhält wie b zum Rest c, also a/b = b/c oder b+c
= cb .9
b
a
c
b
Die Teilstrecke b passt also einmal in a hinein, und der Rest c steht
zu b wieder im gleichen Verhältnis wie vorher b zu a, also b/c = a/b.
Beim Vergleich von c mit b stehen wir daher wieder vor der gleichen
6Pythagoras
von Samos, ca. 570 - 510 v.Chr.
erkannte, dass Tonverhältnisse mit bestimmten Streckenverhältnissen korrespondieren: Unterteilt man eine Saite in der Mitte, ertönt die
Oktave, drittelt man sie, hört man die Quinte über der Oktave usw. Die bekannten
Tonintervalle sind auf diese Weise einfachen Zahlenverhältnissen zugeordnet:
7Pythagoras
2/1
3/2
4/3
5/4
6/5
8
Oktave
Quinte
Quarte
Große Terz
Kleine Terz
9/8
10/9
16/15
25/24
81/80
Großer Ganzton
Kleiner Ganzton
Großer Halbton
Kleiner Halbton
Syntonisches Komma
Hippasos von Metapont, ca. 550 - 470 v.Chr., Metapont (Süditalien)
b
der Gleichung b+c
b = c lässt sich der Wert dieses Verhältnisses x = a/b =
= 1 + cb = 1 + x1 . Multiplikation mit x auf beiden
b/c ermitteln: x = cb = b+c
b
2
Seiten ergibt
√ die quadratische Gleichung 1x =√x + 1 mit der positiven Lösung
1
x = 2 (1 + 5) ≈ 1,618 (die zweite Lösung 2 (1 − 5) ist negativ). Diese Zahl x und
√
ihr Kehrwert x1 = x − 1 = 12 ( 5 − 1) ≈ 0,618 werden Goldener Schnitt genannt.
9Aus
STERNSTUNDEN DER MATHEMATIK
7
Situation; b und c bilden ein verkleinertes Abbild von a und b, weil ja
die Verhältnisse (Proportionen) gleich sind. Wieder passt c einmal in b
hinein, und für den Rest d gilt wiederum b/c = c/d.
a
c
b
d
c
e
d
e
f
Diese Situation wiederholt sich auf jeder Stufe; das Verfahren bricht
niemals ab und wir finden deshalb kein gemeinsames Maß.
Der Goldene Schnitt war in der Zeit um 500 v.Chr. nicht nur den Mathematikern, sondern auch den Künstlern bestens bekannt und fand
vielfache Verwendung. Die Schule des Pythagoras hatte sogar ein besonders enges Verhältnis dazu, denn ihr Symbol war das Pentagramm,
der Diagonalenstern des regelmäßigen Fünfecks, und je zwei Diagonalen
unterteilen sich gegenseitig im Verhältnis des Goldenen Schnittes. Dies
folgt aus der Ähnlichkeit (gleiche Form bei unterschiedlicher Größe)
der beiden in der nachfolgenden Figur schraffierten gleichschenkligen
Dreiecke:
b
b
c
c
b
Das Verhältnis von großer und kleiner Seite ist in den beiden ähnlichen
Dreiecken dasselbe, also folgt das Goldene Schnittverhältnis b+c
= cb .
b
Mit dieser Konstruktion können wir sehr anschaulich erkennen, dass
es wirklich kein gemeinsames Maß zwischen a und b oder zwischen b
und c gibt (nicht nur, dass wir keins finden konnten). Möglicherweise
ist auch Hippasos so zu seiner Erkenntnis gelangt. Dazu betrachten wir
8
J.-H. ESCHENBURG
eine Kette von immer kleineren Fünfecken, wobei die Seite sk des k-ten
Fünfecks die Diagonale dk+1 des (k + 1)-ten Fünfecks ist:
s1
s1
s2
d1
s1
d2
s2
s3
d3
d4
s4
Aus der Figur sehen wir d2 = s1 und s2 = d1 − s1 , allgemein
(∗)
dk+1 = sk ,
sk+1 = dk − sk .
Wenn d1 und s1 ein gemeinsames Maß hätten, also ganze Vielfache
einer Strecke e wären (wie klein diese auch immer sein mag), dann
wären auch d2 = s1 und s2 = d1 − s1 ganze Vielfache von e, und durch
Wiederholung des Schlusses würde dasselbe für alle dk und sk gelten:
Alle sind ganzzahlige Vielfache von e, und doch werden sie beliebig
klein und schließlich kleiner als e, ein Widerspruch!10
Diagonale und Seitenlänge des regelmäßigen Fünfecks besitzen somit
kein gemeinsames Maß, sie sind inkommensurabel. Ihr Verhältnis (das
Goldene Schnittverhältnis) lässt sich nicht mehr als Verhältnis ganzer
Zahlen schreiben; es ist irrational.11 12
Pythagoras’ Erkenntnis “Alles ist Zahl” war daher falsch, solange
man unter “Zahl” nur die natürlichen Zahlen und ihre Verhältnisse
verstand. Das Zahlverständnis änderte sich aber im Verlauf der folgenden zwei Jahrhunderte; man fing an, auch irrationale Verhältnisse
zwischen Größen als Zahlen anzuerkennen. Der Ersatz für die Darstellung von Größenverhältnissen ab als Verhältnis natürlicher Zahlen nk war
10Es
ist sehr bemerkenswert, dass gerade das ganzzahlige Gleichungssystem (∗)
zu einer Nicht-Ganzzahligkeits-Aussage führt: Diagonale und Seitenlänge sind nicht
ganzzahlige Vielfache eines gemeinsamen Maßes.
11
Die rationalen Zahlen Q (von lat./engl. “ratio” = Verhältnis) sind die Quotienten ganzer Zahlen k/n (Brüche). “Irrational” bedeutet: Kein Verhältnis ganzer
Zahlen.
12
Der Goldene Schnitt ist nicht nur irgend eine irrationale Zahl, sondern die
“irrationalste” Zahl überhaupt: In jedem Schritt passt der Rest nur einmal in die
Teilstrecke hinein, er ist also fast so groß wie diese, und deshalb sind wir maximal
weit von einem rationalen Verhältnis entfernt.
STERNSTUNDEN DER MATHEMATIK
9
das Archimedische Axiom,13 das Archimedes selbst aber dem Eudoxos14
zuschreibt und das bereits der Wechselwegnahme zugrunde liegt:
Zu je zwei Größen a, b mit a > b gibt es eine natürliche
Zahl k mit kb ≤ a < (k + 1)b.
Wenn wir diesen Grundsatz statt auf a und b auf na und b für eine
beliebig große Zahl n ∈ N anwenden, finden wir ein k ∈ N mit
k
n
a
b
k+1
n
kb ≤ na < (k + 1)b
und damit ≤ <
= nk + n1 . Das Verhältnis ab ist also fast gleich
dem ganzzahligen Verhältnis nk ; es weicht davon um höchstens n1 ab.
Dies war die erste bewusst durchgeführte Zahlbereichserweiterung
der Mathematikgeschichte. Die reellen Zahlen R waren geboren als
Größenverhältnisse, die durch Verhältnisse ganzer Zahlen zwar nicht
immer ausgedrückt, aber doch angenähert werden konnten.
Übungen
1.1. Kommensurabel = Rationales Verhältnis. Zeigen Sie: Zwei
Größen a > b haben genau dann ( ⇐⇒ ) ein rationales Verhältnis,
wenn die Wechselwegnahme abbricht, d.h. wenn sie nach endlich vielen
Schritten mit dem Rest 0 endet und damit ein “gemeinsames Maß”
produziert. Zu zeigen ist also:
a) Wenn a/b rational, dann bricht die Wechselwegnahme ab,
b) Wenn die Wechselwegnahme abbricht, dann ist a/b rational.
1.2. Anwendung auf das “Zählbare”. Bestimmung des ggT (größten gemeinsamen Teilers) von zwei natürlichen Zahlen a und b durch
Wechselwegnahme. Beispiel:
a) a = 112 und b = 91.
b) a = 544 und b = 323.
Drücken Sie umgekehrt den ggT durch die gegebenen Zahlen aus.
1.3. Gemeinsame Verfeinerung. Gegeben sind zwei gleichmäßige
Unterteilungen derselben Strecke in 5 bzw 7 gleiche Teile (a bzw. b).
Konstruieren Sie graphisch die gemeinsame Verfeinerung und überzeugen Sie sich, dass dadurch die Teilstrecke a in 7 und die Teilstrecke b in
5 gleiche Teile geteilt wird. Führen Sie auch die Wechselwegnahme für a
und b durch, wobei 5a = 7b zu beachten ist: a = b+c, 7b = 5a = 5b+5c,
2b = 5c, b = 2c + d, 5c = 2b = 4c + 2d, c = 2d, b = 2c + d = 4d + d = 5d,
a = b + c = 5d + 2d = 7d. Probieren Sie ein anderes Beispiel, etwa
13Archimedes
14Eudoxos
von Syrakus, ca. 287 - 212 v.Chr.
von Knidos, ca. 408 - 355 v.Chr.
10
J.-H. ESCHENBURG
7a = 11b. Was hat das mit der eindeutigen Primfaktorzerlegung zu
tun?
a
b
1.4. Zahlen ohne eindeutige Primfaktorzerlegung. Die Eindeutigkeit der Primfaktorzerlegung in den ganzen Zahlen Z ist nicht selbstverständlich. In manchen
√ Zahlbereichen√ist sie nicht gegeben. Das
√ einfachste
Beispiel
ist
Z[
−5]
=
{m
+
n
−5;
m,
n
∈
Z}
(mit
−5 =
√
i 5). In dieser Menge von (komplexen) Zahlen kann man unbeschränkt
addieren, subtrahieren und multiplizieren, genau wie in den ganzen
Zahlen. Solche Zahlen heißen Primzahlen, wenn sie keinen echten Teiler mehr
√zwei Zerlegungen
√ haben (außer ±1). Zeigen Sie: Die√Zahl 6 hat
in Z[ −5], nämlich 6 = 2·3 sowie 6√= (1+ −5)(1− −5). Überzeugen
Sie sich, dass 2 und 3 auch in Z[ −5] immer noch Primzahlen sind,
und zwar folgendermaßen: Aus 3 = a · b folgt auch 3 = ā ·√b̄, wobei
ā, b̄ die komplex
Konjugierten zu√a, b sind: Wenn a =√m + n −5 und
√
b = p + q −5, so ist ā = m − n −5 und b̄ = p − q −5. Zeigen Sie:
Durch Multiplizieren der Gleichungen 3 = a · b und 3 = ā · b̄ folgt die
ganzzahlige Gleichung 9 = a · ā · b · b̄ = (m2 + 5n2 )(p2 + 5q 2 ). Wie folgt
daraus, dass a oder b gleich ±1 sein muss?
1.5. Fibonacci-Zahlen: Wenn man den letzten Rest beim Goldenen
Schnitt vernachlässigt, entstehen die Fibonaccizahlen 1, 2, 3, 5, 8,
13, 21, 34, 55...: die jeweils nächste Zahl ist die Summe ihrer beiden
Vorgänger:
f1 = 1,
f2 = 2,
fk+1 = fk + fk−1
für k = 2, 3, . . . . Machen Sie sich dieses an der Figur klar.
21
13
5
3
2
1 1
1
3
2
1
8
5
8
13
STERNSTUNDEN DER MATHEMATIK
11
2. Theodoros: Quadratwurzeln und Selbstähnlichkeit
(−430)
Aus der Figur auf Seite 8 sehen wir, dass die quadratische Gleichung des Goldenen Schnittes, x2 = x + 1 mit Selbstähnlichkeit zu
tun hat: Eine Figur heißt selbstähnlich, wenn sie eine Teilfigur enthält,
die zur ganzen Figur ähnlich ist (kongruent nach Verkleinerung). Dem
antiken griechischen Mathematiker Theodoros von Kyrene (ca. 460 390 v.Chr.) verdanken wir die Erkenntnis, dass dies für viele quadratische Gleichungen der Fall ist; man kann daraus die Irrationalität der
Lösungen ablesen und diese beliebig genau berechnen.
Theodoros war der Lehrer eines sehr viel bekannteren Mathematikers, Theaitetos (415 - 369 v.Chr.), dem wir die Entdeckung des Dodekaeders und Ikosaeders sowie den Beweis der Irrationaliät aller Quadratwurzeln von Primzahlen verdanken, so wie er später in den “Elementen” des Euklid überliefert wurde und heute noch geführt wird.15
In dem gleichnamigen Dialog von Platon (ca. 428 - 348 v.Chr.) erinnert
Theaitetos an diese Leistung und vergleicht sie mit der seines Lehrers:
“Über Quadratwurzeln (‘dynamis’) zeichnete uns Theodoros hier etwas, womit er von den Quadraten von drei
und fünf Quadratfuß Flächeninhalt bewies, dass ihre
Seitenlänge nicht messbar wäre durch die einfüßige. Und
so ging er jede Quadratwurzel einzeln durch bis zum
Quadrat mit siebzehn Quadratfuß; bei dieser hielt er inne. Uns nun fiel so etwas ein, da der Quadratwurzeln
unendlich viele zu sein schienen, wollten wir versuchen,
sie zusammenzufassen in eins, wodurch wir diese Quadratwurzeln alle behandeln könnten.”
Mein kürzlich verstorbener Kollege Benno Artmann hat in einem
Artikel gezeigt, was Theodoros vermutlich gezeichnet hat und √
warum
16
er nicht weiter als bis 17 gekommen ist. Hier ist die Figur für 3, die
Theodoros vermutlich gefunden hat:
15√p
2
= nk ⇒ p = nk 2 ⇒ (∗) n2 p = k 2 . In einer Quadratzahl wie k 2 und n2
kommt jeder Primfaktor in gerader Potenz vor. In der Gleichung (∗) kommt also der
Primfaktor p links in ungerader Potenz, rechts in gerader Potenz vor, Widerspruch!
16
B. Artmann: A proof for Theodoros’ theorem by drawing diagrams, Journal of
Geometry 49 (1994). Siehe auch Janina Deininger: Ein Beweis des Theorems von
Theodoros durch graphische Darstellung, Zulassungsarbeit, Augsburg 2012.
12
J.-H. ESCHENBURG
d
b
e
d
b=1
c
c
b
a= 3
2
√
Zur geometrischen Konstruktion von 3 benötigt man ein√Rechteck
mit Höhe b = 1 und Diagonale 2, dann ist die Breite a = 3 (denn
√ 2
12 + 3 = 22 ). Dieses ist √
leicht konstruierbar. Nun führen wir die
Wechselwegnahme von a = 3 und b = 1 durch wie in der Rechteckfigur auf Seite 4: Wir versuchen, das Rechteck durch jeweils möglichst
große Quadrate auszufüllen. Zuerst spalten wir ein Quadrat der Seitenlänge b ab, danach eins der Seitenlänge c = a − b, danach eins mit
Seitenlänge d = b − c. Von dieser Sorte würde noch ein zweites Quadrat in den freien Raum passen. Aber schon beim ersten Quadrat mit
Seitenlänge d macht Theodoros eine entscheidende Beobachtung: Der
rechte untere Eckpunkt (in der Figur eingekreist) liegt auf der Diagonalen des ursprünglichen Rechtecks! Ob Theodoros das wirklich bewiesen
hat oder nur an der Zeichnung abgelesen, wissen wir nicht. Ein algebraischer Beweis dafür ist schnell gegeben: Zu zeigen ist e/d = a/b
oder, was dasselbe ist, be = ad. Dabei ist
e = c − d, d = b − c, c = a − b,
also
be = b(c − d) = b(c − b + c) = 2bc − b2 = 2b(a−b) − b2 = 2ab − 3b2 ,
ad = a(b − c) = a(b − a + b) = 2ab − a2 ,
also
be = ad ⇐⇒ 2ab − 3b2 = 2ab − a2 ⇐⇒ 3b2 = a2 ⇐⇒ a/b =
√
3.
Theodoros hat mit diesen Zeichnungen weit mehr geleistet, als ihm
sein Schüler in Platons Dialog zugesteht: Er hat nicht nur die Irrationalität der Quadratwurzeln der Primzahlen von 2 bis 17 bewiesen (“dass
ihre Seitenlänge nicht messbar wäre durch die einfüßige”), sondern seine
Zeichnungen enthalten die volle Kettenbruchentwicklung der Quadratwurzeln, aus der sie mit beliebiger Genauigkeit berechnet, d.h. durch
√
Brüche approximiert werden können. Im vorliegenden Fall a/b = 3
STERNSTUNDEN DER MATHEMATIK
13
haben wir
c b
d c
e
a
a
=1+ ,
=1+ ,
=1+ =1+
b
b c
c d
d
b
und daraus
√
a
1
1
= [1; 1, 2] .
3= =1+
=1+
1
b
1 + 1+ a
1 + 1+1+1 1
b
1+...
√
Sehen wir uns die entsprechende Figur für 2 an:
e
d
d
c
c’
b=1
c
c
c
b
a= 2
√
Auch hier wird zunächst 2 konstruiert, indem die Diagonale des Einheitsquadrates in die Horizontale gedreht√wird. Auf den ersten Blick
sieht das Bild sehr ähnlich aus wie das von 3, nur dass c = a−b kleiner
ist und daher zwei Quadrate mit Kantenlänge c übereinander passen.
Aber es gibt einen wichtigen Unterschied: Nicht nur das Rechteck oben
rechts mit Kantenlängen e und d ist ähnlich zum Ausgangsrechteck,
sondern bereits das (um 90 Grad gedrehte) Rechteck mit Kantenlängen
c′ = c + d = b − c und c, denn seine Diagonale steht senkrecht auf
der Diagonalen des Ausgangsquadrats; das Seitenverhältnis muss also (nach Vertauschen von Breite und Höhe) das gleiche sein wie beim
Ausgangsquadrat. In der Tat ist
a/b = c′ /c
⇐⇒
⇐⇒
⇐⇒
⇐⇒
⇐⇒
=
bc′
=
b(b − c)
a(a − b) = b(b − (a − b))
=
b(2b − a)
2
a − ab =
2b2 − ba
2
a
=
2b2
√
a/b
=
2.
ac
′
Aus den Relationen ab = 1 + cb und cb = 1 + cc = 1 + ab folgt die
Kettenbruchentwicklung
√
a
c
1
1
2= =1+ =1+
= [1; 2] .
a = 1+
b
b
1+ b
1 + 1 + 1+1 a
b
14
J.-H. ESCHENBURG
Hier noch die Figuren für
√
5,
√
7,
√
11 und
√
17:
e
d
dd e
d
d
c
c
b=1
b
b=1
b
a= 7 = [2;1,1,1,4]
3
a= 5 = [2;4]
d
e
b
c
b
b
c
c c
f
e
c
4
d
c
c
b=1
b
b
c
a= 11 = [3;3,6]
c
17 = [4;8]
6
5
√
Der Fall 13 liegt etwas anders; seine Kettenbruchentwicklung17 ist
[3; 1, 1, 1, 1, 6]. Mit der langen Periode und der großen √
Zahl 6 am Periodenende wäre er noch viel schwieriger zu zeichnen als 7. Aber es gibt
eine einfachere Möglichkeit, wenn man etwas allgemeinere quadratische
Gleichungen zulässt. Die einfachsten periodischen Kettenbrüche haben
Periode 1 und sind daher von der Gestalt x = k + k+1 1 = k + x1 ; ihre
k...
Gleichung x = k + x1 führt auf die quadratische Gleichung x2 = kx + 1,
√
also x = 12 (k + 4 + k 2 ). Für k = 1 erhalten wir den Goldenen Schnitt
√
√
x = 21 (1 + 5), für k = 3 ist x = 21 (3 + 13).
1
2(
13 +3)= [3;3]
b=1
b
b
d
c
c
c c
b
13
√
√
Warum kam Theodoros nicht weiter als bis 17? Weil 19 die Kettenbruchentwicklung [4; 2, 1, 3, 1, 2, 8] hat, die mit ihren 8 winzigen Quadraten am Ende einer langen Periode sehr schwer sauber zu zeichnen
ist.
17Vgl.
http://mathworld.wolfram.com/PeriodicContinuedFraction.html
STERNSTUNDEN DER MATHEMATIK
15
Aber gibt es zu jeder Quadratwurzel eine periodische Kettenbruchentwicklung? Das hat erst Lagrange18 über 2000 Jahre später gezeigt.
Der Beweis ist für uns, die wir in Variablen und Variablentransformationen denken, einfach.
√
Sehen wir uns noch einmal den Fall 3 von der algebraischen Seite
an. Das Verhältnis x1 = a/b ist die positive Lösung der Gleichung
x21 = 3. Wir betrachten nacheinander die folgenden Verhältnisse:
x1
x2
x3
x4
x5
x6
=
=
=
=
=
=
a/b
c/b
b/c
d/c
c/d
e/d
= (a − b)/b = x1 − 1 ,
=
1/x2 ,
= (b − c)/c = x3 − 1 ,
=
1/x4 ,
= (c − d)/d = x5 − 1 .
Jedes dieser Verhältnisse ist die positive Lösung einer quadratischen
Gleichung:19
x21
x22 + 2x2
1 + 2x3
1 + 2(x4 + 1)
1 − 2x4
(e)
x25 − 2x5
2
(f ) x6 + 2x6 + 1 − 2(x6 + 1)
x26
(a)
(b)
(c)
(d)
=
3,
=
2,
=
2x23 ,
= 2(x24 + 2x4 + 1)
=
2x24 ,
=
2,
=
2
=
3.
x1 = x2 + 1
x2 = 1/x3
x3 = x4 + 1
⇒
x4 = 1/x5
x5 = x6 + 1
⇒
Das letzte Verhältnis x6 erfüllt
√ also wieder dieselbe Gleichung wie das
erste x1 , d.h. x6 = x1 = 3, was wir ja schon auf Seite 12 gesehen
haben.
Um dieses Verfahren besser zu verstehen, betrachten wir nicht nur
Quadratwurzeln, Lösungen einer Gleichung x2 = c, sondern allgemeinere quadratische Gleichungen
(1)
ax2 − bx = c
mit ganzzahligen Koeffizienten a, b, c. Die beiden Lösungen
√
1
(2)
x± = (b ± b2 + 4ac)
2a
haben unterschiedliche Vorzeichen genau dann, wenn ac > 0, was wir
voraussetzen wollen. Wir dürfen annehmen, dass a und c beide positiv
18Joseph-Louis
Lagrange, 1736 (Turin) - 1813 (Paris)
beachte im Folgenden: Mit x = 1/x̃ gilt ax2 −bx = c ⇐⇒ a/x̃2 −b/x̃ = c
⇐⇒ a − bx̃ = cx̃2 .
19Man
16
J.-H. ESCHENBURG
sind. Auf die Variable x wenden wir in jedem Schritt des Verfahrens
eine von zwei mögliche Transformationen an:20
(A) Die Verschiebung nach links, x̃ = x − k mit k ∈ N, aber nicht
zu weit nach links: die positive Lösung x+ soll auch nach der
Verschiebung positiv bleiben: x+ − k > 0, d.h. k < x+ ,
(B) Die Inversion x̃ = 1/x.
Bei diesen Transformationen ändern sich die Koeffizienten; die transformierte Variable x̃ erfüllt eine neue Gleichung
ãx̃2 − b̃x̃ = c̃,
aber auch die neuen Koeffizienten ã, b̃, c̃ sind ganze Zahlen und ã, c̃ sind
positiv. Zudem hat die Diskriminante
(3)
d = b2 + 4ac
sich nicht geändert, d˜ = d. Das ist klar für die Verschiebung (A), weil
die Differenz x+ − x− = d/a (gemäß der “Mitternachtsformel” (2)) bei
Verschiebungen gleich bleibt und auch ã = a gilt. Für die Inversion (B)
gilt ã = c, c̃ = a und b̃ = −b, also wiederum d˜ = d nach (3).
Nun gibt es aber für gegebenes d nur eine begrenzte Anzahl von
Tripeln ganzer Zahlen (a, b, c) mit a, c > 0 und d = b2 + 4ac. Die
Transformationskette A-B-A-B-A-B . . . des obigen Verfahrens muss
deshalb nach endlich vielen Schritten auf eine Gleichung führen, die
im Verfahren schon einmal vorher aufgetreten ist; von da an wiederholt sich alles. Die Lösungen x der Gleichungen (1) mit ganzzahligen
Koeffizienten a, b, c und ac > 0 haben also eine periodische Wechselwegnahme oder Kettenbruchentwicklung. Stellt man diese graphisch
dar durch Ausfüllen eines Rechtecks durch Quadrate, so erhält man
stets eine selbstähnliche Figur.
Die Methode von Theodoros beruht also in der Tat auf einem allgemeinen Verfahren, das für alle Quadratwurzeln und weit darüber hinaus
Gültigkeit hat.
Übungen
2.1. Konstruierbarkeit von Quadratwurzeln. Man zeige, dass jede ungerade Zahl Differenz von zwei (sogar benachbarten) Quadratzahlen ist.
20Es
handelt sich um gebrochen-lineare Transformationen (MöbiusTransformationen), benannt nach August Ferdinand Möbius, 1790 - 1868
(Leipzig). Sie sind Erzeugende der von allen gebrochen-linearen Transformationen
mit positiver Ableitung erzeugten Gruppe.
STERNSTUNDEN DER MATHEMATIK
17
2.2. Beweis der Selbstähnlichkeit.
√ Man mache sich den Beweis der
Selbstähnlichkeit
für 3 klar und übertrage ihn auf die
√
√ des Rechtecks
Rechtecke für 5 und 11.
2.3. Einfache Perioden. Was ist die nächste “halbe” Wurzel, die
man mit einem Kettenbruch der Periode Eins darstellen kann?
2.4. “Goldenes Rechteck”. Man konstruiere
die selbstähnliche Fi√
1
gur für den Goldenen Schnitt τ = 2 (1+ 5). Der Goldene Schnitt kann
zum Beispiel wie folgt konstruiert werden:
1/2
τ
τ
1−τ
2.5. Variablenverschiebung. Zeigen Sie durch eine direkte Rechnung, dass bei der Variablensubstitution x = x̃ + k für eine beliebige ganze Zahl k die Diskriminante d erhalten bleibt: Sind ã, b̃, c̃ die
Koeffizienten der Gleichung für x̃, so gilt b2 + 4ac = b̃2 + 4ãc̃.
2.6. Anzahl der Tripel (a, b2 , c). In der Gleichung x2 = 3 ist ao = 1,
bo = 0 und co = 3, also d = b2o +4ao co = 12. Finden Sie alle ganzzahligen
Tripel (a, b2 , c) mit a, c > 0 und b2 + 4ac = 12. Welche von ihnen
kommen in dem beschriebenen Verfahren vor?
3. Archimedes: Die Rechnung mit dem Unendlichen (−220)
Archimedes ist der berühmteste Mathematiker der Antike. Er lebte
von ca. 287 bis 212 v.Chr. in Syrakus auf Sizilien. Anders als Pythagoras, der Probleme mit seinen Landesherren hatte, war Archimedes
ein guter Freund, vielleicht sogar ein Verwandter des Königs Hieron
von Syrakus. Und doch holte auch ihn die Politik auf tragische Weise
ein, als nämlich die Römer im Zweiten Punischen Krieg im Jahr 212
die Stadt Syrakus nach zweijähriger Belagerung endlich erobert hatten.
Bei der Besetzung der Stadt wurde Archimedes von einem römischen
Soldaten getötet. Archimedes hatte sich an der Verteidigung der Stadt
durch den Bau verschiedener sehr effektiver Kriegsmaschinen beteiligt; nach ihrem Fall saß er angeblich vor seinem Haus, bereits wieder
in mathematische Überlegungen versunken, als plötzlich ein römischer
Soldat vorbeikam. “Noli turbare circulos meos”, zerstöre nicht meine
Kreise, soll er noch gerufen haben, aber der Soldat hatte keinen Sinn
für Wissenschaft. Nach einem anderen Bericht hatte der Soldat Befehl,
Archimedes zu dem römischen Oberbefehlshaber Marcellus zu bringen,
18
J.-H. ESCHENBURG
wozu er auch bereit war, aber er wollte vorher noch seine mathematische Überlegung zu Ende führen. Da verlor der Soldat die Geduld
mit ihm und brachte den alten Mann kurzerhand um: eine Parabel von
Geist und Macht.21
Sehr bekannt ist auch die Geschichte von der Krone des Hieron, die
gut die Denkweise von Archimedes zeigt. Ein Goldschmied, der für den
König eine Krone gefertigt hatte, die angeblich aus reinem Gold bestand, geriet in Verdacht, ein Betrüger zu sein. Archimedes hatte eine
Idee, ihn zu überführen: Er verglich die Krone mit einem gleich schweren Stück reinen Goldes, indem er beide ins Wasser tauchte. Die Krone
21www.math.nyu.edu/
crorres/Archimedes/Death/DeathMosaicBlack.jpg
STERNSTUNDEN DER MATHEMATIK
19
verdrängte mehr Wasser als das reine Gold, hatte also ein größeres Volumen. Der Goldschmied hatte dem Gold einen Anteil des leichteren
(und billigeren) Silbers beigemischt.
Archimedes verstand also etwas von Raum- und Flächeninhalten,
und seine Denkweise war eine anschaulich physikalische.22 Mit der Idee
der Wasserverdrängung hatte er verstanden, dass sich dasselbe Volumen in jede beliebige Form bringen lässt: Größe und Form sind
vollständig unabhängig voneinander. Im Bereich der Physik hatte er damit das spezifische Gewicht (Gewicht pro Volumen) sowie das Phänomen
des Schwimmens verstanden: Ein Körper, der leichter ist als Wasser,
verdrängt genau die Wassermenge, die seinem eigenen Gewicht entspricht. Auf mathematischem Gebiet war ihm klar geworden, dass sich
jedes Volumen und jede Fläche aus kleinsten Teilen zusammensetzen
lässt, die sich auch auf andere Weise anordnen lassen und dadurch eine
Berechnung möglich machen.
Ein Beispiel dafür ist die Berechnung der Kreisfläche durch Archimedes, F = 21 ur, wobei r den Radius und u den Umfang des Kreises
bezeichnet. Dazu zerlegte er den Kreis in lauter schmale Dreiecke wie
in der linken Figur gezeigt.
9
5 4
7 6
3
8
9
r
2
1
s
8
7
6
4
5
3
2
r
1
s
Jedes Teildreieck hat Höhe r und Grundseite s,23 also Flächeninhalt
1
rs, und durch Aufsummieren aller Teildreiecke ergibt sich die Kreis2
fläche zu F = 12 ru. Wenn man die Dreiecke übereinander aufschichtet,
kann man die Kreisfläche in eine Rechteckfläche mit Breite r und Höhe
1
u verwandeln (rechte Figur).
2
Definiert man die Zahl π als das Verhältnis von Umfang u und Durchmesser 2r des Kreises,24 dann ergibt sich für die Kreisfläche F = πr2 ,
oder, wie Archimedes es formuliert hätte, π ist nicht nur das Verhältnis
von Umfang zu Durchmesser des Kreises, sondern auch viermal das
22Vgl. auch http://www.iazd.uni-hannover.de/∼erne/griechen/Archimedes.html
23Das
stimmt nicht ganz genau, aber der Gesamtfehler wird beliebig klein, wenn
die Dreiecke nur genügend schmal werden.
24Dieses Verhältnis ist für alle Kreise gleich, denn je zwei Kreise sind ähnlich,
kongruent bis auf Verkleinerung oder Vergrößerung, das Verhältnis von zwei Längen
bleibt also gleich.
20
J.-H. ESCHENBURG
Verhältnis der Kreisfläche zur Fläche des umbeschriebenen Quadrats.
Um dieses Ergebnis zu erzielen, musste Archimedes die Kreisfläche in
sehr viele (“unendlich viele”) schmale Dreiecke zerlegen; es gibt stets
noch eine kleine Abweichung von der gesuchten Formel, die aber beliebig klein wird, wenn die Dreiecke immer schmaler und immer mehr
werden.
Eine weitere Anwendung der Zerlegung in virtuell unendlich viele
Teile ist das Prinzip von Cavalieri,25 das in dieser Form eben bereits
von Archimedes stammt:
Zwei Körper haben gleiches Volumen, wenn ihre Schnitte mit jeder horizontalen Ebene den gleichen Flächeninhalt haben.
In der Tat können wir uns die Körper aus dünnen Scheiben über solchen
horizontalen Schnitten zusammengesetzt denken; da deren Volumina
übereinstimmen, haben die Körper insgesamt gleiches Volumen.
Eine Anwendung dieses Prinzips, auf die Archimedes besonders stolz
war, war seine Berechnung des Kugelvolumens. Er zeigte damit, dass
die Halbkugel vom Radius r das gleiche Volumen besitzt wie ein Kreiszylinder mit Radius r und Höhe r, aus dem ein auf der Spitze stehender
Kreiskegel mit Radius und Höhe r ausgeschnitten ist.
ρ
t
t
r
t
25Bonaventura
r
t
r
r
ρ
Francesco Cavalieri, 1598 (Mailand) - 1647 (Bologna).
STERNSTUNDEN DER MATHEMATIK
21
Schneidet man nämlich beide Körper mit einer horizontalen Ebene der
Höhe t, so ist der Schnitt mit dem ausgebohrten Kreiszylinder ein Kreisring mit innerem Radius t und äußerem r, und der Schnitt mit der
26
Halbkugel
√ ist ein Kreis, der nach dem Satz des Pythagoras 2 den Radius ρ = r2 − t2 hat. Der Kreisring hat den Flächeninhalt πr −πt2 und
der Kreis πρ2 = π(r2 − t2 ); der Flächeninhalt ist also derselbe. Nach
dem Cavalierischen Prinzip hat die Halbkugel daher dasselbe Volumen
wie der Zylinder mit ausgebohrtem Kegel.
Mit der Formel für das Kegelvolumen 31 · Grundfläche · Höhe hat die
Halbkugel also das Volumen πr2 · r − 31 πr2 · r = 32 πr3 . Das Volumen der
ganzen Kugel ist demnach 43 πr3 .
Dieselbe Überlegung, mit der Archimedes den Zusammenhang zwischen Umfang und Flächeninhalt des Kreises erkannte, führte ihn danach auch auf die Beziehung zwischen Oberfläche und Volumen der
Kugel:
Wenn man sich die Kugelfläche aus kleinen, fast ebenen Flächenstücken
(z.B. Dreiecken) zusammengesetzt denkt, dann setzt sich die Kugel aus
den Kegeln mit Höhe r über diesen Flächenstücken zusammen, und
nach der Kegelformel ist damit das Kugelvolumen V = 13 · F · r, wobei
F die Gesamtoberfläche der Kugel bezeichnet. Also ist 43 πr3 = 31 F r
und daher F = 4πr2 . Die Kugelfläche ist also genau viermal so groß
wie die Kreisfläche mit demselben Radius!
26
Der vielleicht schönste Beweis des Satzes von Pythagoras ist vermutlich indischen Ursprungs. Dabei wird ein rechtwinkliges Dreieck mit den Seitenlängen a, b, c
zu einem Quadrat mit Kantenlänge a + b ergänzt:
b
a
c
b
b²
c²
a c
c² = a² + b²
a²
Wenn man viermal die Dreiecksfläche von der Quadratfläche wegnimmt, ergibt sich
in der linken Figur c2 , in der rechten a2 + b2 , also ist c2 = a2 + b2 .
22
J.-H. ESCHENBURG
Aber wie hat Archimedes das Volumen eines Kegels oder einer Pyramide von der Höhe h über einem ebenen Flächenstück G berechnet?
h
G
t
G
Das ist viel schwieriger als die Berechnung des Flächeninhalts eines
Dreiecks, das man ja einfach zu einem Parallelogramm oder Rechteck
verdoppeln kann. Und doch führt eine ähnliche Überlegung auch hier
zum Ziel. Als erstes sehen wir, dass das Volumen nur von der Höhe h
und der Grundfläche G abhängt: Wenn zwei Kegel mit gleicher Grundfläche und gleicher Höhe gegeben sind, dann haben die Schnitte mit
einer zur Grundfläche parallelen Ebene bei beiden Kegeln den gleichen
Flächeninhalt.27 Damit sind auch die Volumina gleich, weil wir uns die
beiden Kegel aus volumengleichen flachen Scheiben über diesen Querschnitten aufgebaut denken können.
Nun können wir das Kegelvolumen berechnen. Dazu betrachten wir
eine Scheibe oder Säule mit Höhe h und dreieckiger Grundfläche G,
ein Prisma. Die unteren Eckpunkte mögen a, b, c genannt werden, die
oberen A, B, C. Dieses Prisma zerlegen wir in die drei Tetraeder (Kegel über einem Dreieck), die jeweils durch die vier Eckpunkte (abcC),
(abBC) und (aABC) gegeben sind.
A
C
G’
h
B
a
G
c
b
27Der
Schnitt mit einer Ebene in Höhe t ist eine Verkleinerung der Grundfläche
G um den Faktor (h − t)2 /h2 , denn alle Längen werden um den Faktor (h − t)/h
verkleinert.
STERNSTUNDEN DER MATHEMATIK
23
Alle drei Tetraeder haben das gleiche Volumen; für (abcC) und (aABC)
ist das unmittelbar klar, denn es sind Kegel der Höhe h über der Grundfläche G = (abc) und der Deckfläche G′ = (ABC), die gleich groß sind.
Um die noch fehlende Gleichheit zu sehen, können wir z.B. die Tetraeder (abBC) und (aABC) ansehen als Kegel mit Spitze C über den
Dreiecken (abB) und (aAB). Diese teilen das Rechteck (abBA) diagonal in zwei Hälften, haben also gleichen Flächeninhalt, und da sie in
derselben Ebene liegen, ist auch ihre Höhe die gleiche, also sind auch
die Volumina von (abBC) und (aABC) gleich.
Damit ist das Volumen von jedem der drei Tetraeder ein Drittel des
Prismavolumens Gh. Der Tetraeder (abcC) mit Grundfläche G und
Höhe h hat also das Volumen 13 Gh. Da sich jede Fläche in Dreiecke
zerlegen lässt, gilt die gleiche Formel für Kegel über beliebigen Grundflächen.
Archimedes hat nicht nur die Kreisfläche berechnet, sondern auch
die Fläche einer Parabel unterhalb einer beliebigen durchlaufenden
Geraden (“Sekante”). Dabei entwickelte er die Exhaustionsmethode
(Ausschöpfungsmethode), bei der die Fläche durch immer neue Dreiecke ausgefüllt wird. Weil alle Parabeln ähnlich sind, können wir uns
auf die “Normalparabel” beschränken, deren Koordinaten durch die
Gleichung y = x2 verbunden sind. Die Sekante trifft die Parabel in den
Punkten A und B mit den x-Werten a und b (und den y-Werten a2
und b2 ). Dann sei M der Punkt auf der Parabel, dessen x-Koordinate
m genau in der Mitte zwischen a und b liegt, also bei m = 12 (a + b).
Welchen Flächeninhalt hat das Dreieck AM B?
y
B
D
d
d
A
a
M
m
b x
a
m
b
Dieser Flächeninhalt ergibt sich wieder aus dem “Cavalieri-Prinzip”,
diesmal in vertikaler Richtung. Das rechte, flächengleiche Dreieck hat
offensichtlich den Flächeninhalt Fo = 12 (b − a)d, wobei d die Länge der
Strecke DM ist. Dabei ist D der Punkt auf der Sekante in der Mitte
zwischen A und B, vertikal über M mit der y-Koordinate 21 (a2 + b2 ).
Also ist d = 12 (a2 + b2 ) − m2 = 12 (a2 + b2 ) − 41 (a2 + b2 + 2ab) = 41 (a − b)2
und damit Fo = 81 (b − a)3 . Insbesondere hängt F nur vom Abstand
b − a ab.
24
J.-H. ESCHENBURG
Nun schöpfen wir den Flächeninhalt der Parabel aus, indem wir
durch weitere Halbierungen des Intervalls [a, b] immer weitere Dreiecke
hinzunehmen.
Die nächsten beiden Dreiecke haben jeweils den Flächeninhalt ( 12 )3 Fo ,
weil wir die “Basislänge” b − a durch 21 (b − a) zu ersetzen haben.
Der gesamte Flächeninhalt der beiden angefügten Dreiecke beträgt also F1 = 2 · ( 12 )3 Fo = ( 21 )2 Fo = 14 Fo . Im nächsten Schritt erhalten
wir 4 neue Dreiecke, jedes mit Flächeninhalt ( 14 )3 Fo , also Gesamtfläche
( 41 )2 Fo . In jedem weiteren Schritt halbieren wir die Basislänge und verdoppeln die Anzahl; im Schritt 3 also ist die hinzukommende Gesamtfläche 8 · ( 18 )3 Fo = ( 81 )2 Fo = ( 14 )3 Fo , usw. Die Gesamtfläche ist also
F = Fo (1 + 41 + ( 14 )2 + ( 14 )3 + . . . ). Aber was ist der Wert dieser unendlichen Summe? Kein Problem für Archimedes:
Der gefärbte Anteil ist 14 + ( 41 )2 + ( 14 )3 + . . . des gesamten Quadrats.
In jeder der L-förmigen Teilflächen ist der gefärbte Teil offensichtlich
jeweils 13 , also ist er insgesamt auch 13 der Gesamtfläche. Wir erhalten
daher 41 + ( 14 )2 + ( 41 )3 + · · · = 13 und 1 + 41 + ( 14 )2 + ( 41 )3 + · · · = 43 .
Die Fläche zwischen Parabel und Sekante ist demnach F = 34 Fo =
4 1
· (b − a)3 = 16 (b − a)3 . Beispiel: a = −1, b = 1, dann ist F = 34 .
3 8
y
1
−1
1
x
STERNSTUNDEN DER MATHEMATIK
25
Eine andere Fläche, die Archimedes berechnet hat, ist die der Archimedischen Spirale. Dabei dreht sich ein Strahl mit gleichmäßiger Geschwindigkeit um ein Zentrum, und gleichzeitig bewegt sich ein Punkt
auf diesem Strahl mit gleichmäßiger Geschwindigkeit nach außen.
4
3
2
5
...
1
n
n−1
Wie groß ist die Fläche bei einem Umlauf (mittlere Figur)? Dazu unterteilt man den vollen 360-Grad-Umlauf in n gleichmäßige Teile und
approximiert die Fläche der Spirale durch n gleichschenklige Dreiecke
mit Winkel 360/n Grad wie in der rechten Figur. Diese vergleicht man
mit den entsprechenden Dreiecken, die den Kreis ausfüllen, der die
Spirale umschließt. Die Seitenlängen der Spiralen-Dreiecke wachsen
proportional zu ihrer Nummer k, bis sie am Ende fast genauso groß
wie die (immer gleich großen) Kreisdreiecke sind; das Verhältnis ihrer
Flächeninhalte ist also ( nk )2 und das Verhältnis der Flächen
von Spi2 2
n−1 2
1 2
rale und Kreis also annährend ( n ) + ( n ) + · · · + ( n ) /n. Dies ist
annähernd (für große n) der Flächeninhalt unter der Parabel, denn die
gefärbten senkrechten Stäbe in der nachfolgenden Figur haben Breite
1/n und Höhe ( nk )2 für k = 1, . . . , n − 1.
y
y=x 2
1
2
3
x
1 2 3 4 5 6 ... n−1 n
n n
nn n nnn
Auf Seite 24 sahen wir, dass der Flächeninhalt über der Parabel gleich
2
ist (die Hälfte von 34 ), der Flächeninhalt unter der Parabel ist also
3
1 − 32 = 13 . Der Flächeninhalt der Archimedischen Spirale ist somit ein
Drittel der Fläche des Kreises, der sie umschließt.
Übungen
3.1. Kugelkappen. Mit der Methode von Archimedes kann nicht nur
das Volumen der Halb- oder Vollkugel berechnet werden, sondern auch
26
J.-H. ESCHENBURG
der Rauminhalt einer Kugelkappe, des Teils einer Vollkugel, der durch
einen ebenen Schnitt abgetrennt wird. Zeigen Sie für eine Kugelkappe
der Höhe h von einer Kugel mit Radius r die Volumenformel V =
π
(3r − h)h2 (siehe nachstehende Figur).
3
h
r
r−h
r
r
3.2. Integralrechnung. Weisen Sie die Flächen- und Volumenformeln
von Archimedes mit Hilfe der Integralrechnung nach!
3.3. Kreisberechnung 1. Archimedes hat den Umfang des Einheitskreises 2π nach oben und unten abgeschätzt mit Hilfe von umbeschriebenen und einbeschriebenen regulären Vielecken. Insbesondere hat er
π < 22/7 gezeigt.
t
Er beginnt mit dem regulären Dreieck und verdoppelt immer wieder die
Anzahl der Seiten. Im Schritt 1 erhält er ein umschriebenes Sechseck,
im Schritt n ein umschriebenen (3·2n )-Eck. Wie sieht die Algebra dazu
aus?
3
30
u
s
t
o
1
1
1
60
o
60
o
h
t
1
α/2
α/2
1
α
Die halbe Seitenlänge im Schritt n sei tn . Am Anfang ist
√
(4)
to = 3
STERNSTUNDEN DER MATHEMATIK
27
(linke Figur), denn das Dreieck mit den Winkeln 30, 60, 90 Grad ist die
Hälfte des gleichseitigen Dreiecks mit Seitenlänge 2. Die rechte Figur
zeigt die Rekursionsformel28
p
t2n + 1 − 1
(5)
tn+1 =
,
tn
wobei in der Figur tn = s und tn+1 = t gesetzt ist. Zeigen Sie diese
Gleichung folgendermaßen. Für h haben wir die beiden Beziehungen
h2 = u2 − t2 = (u + t)(u − t) = s(u − t)
(6)
sowie
(h + 1)2 = s2 + 1 ,
√
und aus (7) ergibt sich einerseits h = s2 + 1 − 1 und andererseits
h2 = s2 − 2h. Mit der letzteren Beziehung folgt aus (6):
(7)
u − t = h2 /s = s − 2h/s .
Andererseits ist u + t = s und damit
√
2t = (u + t) − (u − t) = 2h/s.
s2 + 1 − 1 ein, so ergibt sich die Behauptung
p
√
t = ( s2 + 1 − 1)/s = 1 − (1/s)2 − 1/s .
Setzen wir h =
(8)
3.4. Kreisberechnung 2. Die Rekursionsformel (5) wird einfacher für
die Kehrwerte von s und t. Setzen wir r = 1/s in (8) ein, so ist t =
√
1 + r2 − r, und weil
√
√
( 1 + r2 − r)( 1 + r2 + r) = 1 + r2 − r2 = 1,
√
√
ist 1 + r2 + r der Kehrwert von t = 1 + r2 − r, also
√
1/t = 1 + r2 + r .
Setzen wir also allgemein rn = 1/tn für alle n, so ergibt sich aus (5) die
einfachere Rekursionsformel
p
(9)
rn+1 = 1 + rn2 + rn .
Archimedes29 benutzt genau diese Rekursionsformel (9). Aber wie
kann er die iterierten Wurzeln effektiv berechnen? Sein Ausgangspunkt
28Da
p
1 + t2n < 1 + 21 t2n , gilt tn+1 < 21 tn . Da die Zahl der Seiten sich in jedem
Schritt verdoppelt, wird die obere Abschätzung für 2π in jedem Schritt kleiner, also
besser. Zusammen mit der unteren Schranke durch einbeschriebene Vielecke erhält
Archimedes eine beliebig genaue Approximation von 2π.
29Archimedes: Measurement of a circle, Seite 93 - 96
https://archive.org/details/worksofarchimede029517mbp
28
J.-H. ESCHENBURG
√
ist die Approximation r1 = 3 > 265/153. In der Tat ist 2652 = 70225
und 3 · 1532 = 70227. Die Rechnung läuft folgendermaßen:
p
n 153 rn ≥
153 1 + rn2 ≥ . . . > . . .
153rn+1 >
1
2
265
571
3
1162 18
4
2334 14
√ 153 · 2
2
2
> 591 81
q 153 + 571
1532 + (1162 18 )2 > 1172 81
q
1532 + (2334 14 )2 > 2339 41
571
1162 18
2334 14
4673 12
Die Länge π4 des halben 96-Ecks ist 96 · t5 , also
1/π4 =
4673 21
r5
>
96
14688
und damit
667 21
667 12
14688
1
.
π < π4 <
1 = 3+
1 < 3+
1 = 3+
7
4673 2
4673 2
4672 2
3.5. Kreisberechnung 3. Wir können die Rekursionsformeln auch zu
einer Produktformel für π umformen. Dabei brauchen wir auch einen
2
Ausdruck für 1 + rn+1
, der sich aus (9) ergibt:30
p
2
1 + rn+1
= 1
+ 1 + rn2 + rn2 + 2rn 1+ rn2
p
= 2 (1 + rn2 ) + rn 1 + rn2
p
p
p
2
= 2
(10) 1 + rn+1
1 + rn2 + rn
1 + rn2 = 2rn+1 1 + rn2 .
√
√
Hilfssatz 3.1. Wir setzen w1 = 2 + 3 und rekursiv wn+1 = 2 + wn .
Dann gilt
√
√
(11)
rn+1 = w1 · · · wn · wn .
√
Beweis. Aus (4) folgt r0 = 1/to = 1/ 3. Nach (9) ist
p
√
√
√
4/3
+
1/
3
=
3/
3
=
3,
r1 =
√
√
√
r2 =
4+ 3
= 2 + 3 = w1 .
Wir beweisen (11) gemeinsam mit der folgende Gleichung:
q
√
2
1 + rn+1
= 2 w1 · · · wn .
(12)
30Anderes
Argument:
Die quadratische Gleichung x2 − ax + b = 0 p
mit den
p
2
Lösungen x± = 1 + rn ± rn hat die Koeffizienten a = x+ + x− = 2 1 + rn2
und b = x+ x− = 1, daher gilt (10) für x = rn+1 .
STERNSTUNDEN DER MATHEMATIK
29
Für n = 1 ist die Gleichung richtig, da
√
√
1 + r22 = 1 + w12 = 1 + (2 + 3)2 = 8 + 2 3 = 4w1 .
Schritt von n − 1 auf n: Nach Induktionsvoraussetzung (IV ) gelte bereits
p
√
1 + rn2 =
2 w1 · · · wn−1
(IV )
√
√
w1 · · · wn−1 · wn−1
rn
=
für ein festes n ≥ 2. Nach (10) ist
p
p
2
1 + rn2 + rn
1 + rn+1
= 2 1 + rn2
√
√
√
√
(IV )
= 4 w1 · · · wn−1 (2 w1 · · · wn−1 + w1 · · · wn−1 · wn−1 )
√
= 4w1 · · · wn−1 (2 + wn−1 )
= 4w1 · · · wn
und damit folgt die Behauptung (12). Ferner gilt nach (9):
p
rn+1 =
1 + rn2 + rn
√
√
√
(IV )
= 2 w1 · · · wn−1 + w1 · · · wn−1 · wn−1
√
√
= √w1 · · · wn−1 (2 + wn−1 )
= √w1 · · · wn−1√· wn
w1 · · · wn · wn .
=
Da tn+1 = 1/rn+1 die halbe Seitenlänge eines den Kreis umschreibenden regulären Vielecks mit 3 · 2n+1 Ecken ist, erhalten wir für die
Länge π des Halbkreises:
(13)
2n+1
2
2
2
π ≤3· √
= 3 · √ ··· √ · √ ·
√
w1 · · · wn · wn
w1
wn
wn
Weil die Differenz gegen Null strebt und wn → 4 für n → ∞, hat
Archimedes für π/3 die folgende Produktdarstellung bewiesen:
(14)
∞
Y
2
π
2
2
= √ · ... · √ · ··· =
√ .
3
w1
wn
wn
n=1
Zur expliziten Berechnung der Nährungswerte kann man die rechte
Seite von (13) wesentlich vereinfachen, weil die Kehrwerte der wk leicht
zu berechnen sind. Wir setzen
√
w̄k := 2 − wk−1 = 4 − wk .
Dann gilt für k ≥ 1
wk w̄k = (2 +
√
wk−1 )(2 −
√
wk−1 ) = 4 − wk−1 = w̄k−1 ,
30
J.-H. ESCHENBURG
(wobei wir w0 = 3 und w̄0 = 1 setzen), also
(15)
1/wk = w̄k /w̄k−1 .
Somit ist
w̄1 w̄2
w̄n
1
=
·
· ... ·
= w̄n
w1 · · · wn
w̄0 w̄1
w̄n−1
und folglich mit (11)
r
r
√
1
1
w̄n
4 − wn
4
(16)
tn+1 = √
=√
=
=
− 1.
√
w1 · · · wn wn
wn
wn
wn
Für die Nährungswerte
πn = 3 · 2n+1 tn+1 = 3 · 2n+1 ·
ergibt sich (mit w1 = 2 +
√
3 und wn = 2 +
r
√
4
−1
wn
wn−1 ):
n
wn
πn
1
2
3
4
5
6
7
8
3.73205 . . .
3.93185 . . .
3.98288 . . .
3.99571 . . .
3.99892 . . .
3.99973 . . .
3.99993 . . .
3.99998 . . .
3.215 . . .
3.159 . . .
3.146 . . .
3.1427 . . .
3.1418 . . .
3.14166 . . .
3.14161 . . .
3.141595 . . .
Es ist π = 3.141592653 . . . und 22/7 = 3.142857 . . . ; Archimedes musste also bis zu n = 4 rechnen, also bis zum Vieleck mit 2 · 3 · 25 = 192
Seiten, um π < 22/7 zu beweisen.
4. Brunelleschi: Wo schneiden sich Parallelen? (1420)
Wir machen nun einen Sprung von 1600 Jahren. Dazwischen liegt
das europäische Mittelalter, das in mathematischer Hinsicht eine Zeit
der Dürre war. Mathematik und Naturwissenschaft fanden vorwiegend
außerhalb Europas statt, worüber noch zu reden sein wird. Aber mit
dem Ausgang des Mittelalters, mit der in Italien beginnenden Frührenaissance, änderte sich die Situation sehr schnell. Eine der ersten
Entdeckungen dieser Zeit war die Zentralperspektive, deren Bedeutung
für die Geschichte der Mathematik und Naturwissenschaft nicht hoch
genug geschätzt werden kann. Ein ganz neuer Zweig der Geometrie
nahm hier ihren Ausgangspunkt und sollte erst im 19. Jahrhundert
STERNSTUNDEN DER MATHEMATIK
31
seine höchste Blüte erreichen: die Projektive Geometrie. Ohne die Reflektion des Standpunkts des Betrachters, die bei einer perspektivischen Darstellung nötig wird, hätte man auch nicht ein neues Weltbild entwerfen können, dessen Mittelpunkt nicht mehr der menschliche Betrachter des Kosmos ist; Kopernikus31 und Kepler32 hätten die
Planetenbahnen nicht bestimmen können. Der Mann, der diese Entwicklungen angestoßen hat, ist nur wenigen Mathematikern bekannt,
obwohl fast jeder sein bedeutendstes Werk kennt: Es ist der Florenzer
Architekt Filippo Brunelleschi (1377 - 1440), der Erbauer der Kuppel
des Doms von Florenz und Entdecker der Zentralperspektive.
Parallelen schneiden sich bekanntlich nicht, aber in perspektivischen
Darstellungen scheinen sie doch einen Schnittpunkt zu haben, nämlich
auf dem Horizont. Zur korrekten Konstruktion der Zentralperspektive
braucht man nur drei einfache Regeln:
(1) Geraden werden in Geraden abgebildet,
(2) Bilder von Parallelen sind wieder parallel oder haben einen gemeinsamen Schnittpunkt,
(3) Die Schnittpunkte der Bilder aller Parallelen zu Geraden einer festen Ebene liegen auf einer gemeinsamen Geraden, dem
Horizont der Ebene.
Die einfachste Übung im perspektivischen Zeichnen ist eine Eisenbahnstrecke, die gerade auf den Horizont zuläuft und deren Schwellen
gleichmäßige Abstände haben. Dann muss man im Bild nur den Horizont, die beiden Gleise und die ersten zwei Schwellen vorgeben; die
Bilder der anderen Schwellen lassen sich konstruieren, denn alle von den
Gleisen und zwei benachbarten Schwellen gebildeten Rechtecke haben
parallele Diagonalen, deren Bilder sich (bei geradliniger Verlängerung)
auf dem Horizont schneiden.
31Nikolaus
Kopernikus, Niklas Koppernigk, 1473 (Thorn, Torun) - 1543 (Frauenburg, Frombork)
32Johannes Kepler, 1571 (Weil der Stadt) - 1630 (Regensburg)
32
J.-H. ESCHENBURG
Das Bild eines einzelnen Rechtecks, das ein beliebiges konvexes Viereck sein kann, legt jeden anderen Bildpunkt derselben Ebene eindeutig fest. Dieselben Prinzipien gelten auch für Zeichnungen räumlicher
Objekte, z.B. eines Quaders. Die vertikalen Kanten werden meistens
ebenfalls vertikal gezeichnet (diese Schar paralleler Geraden wird also
auf Parallelen abgebildet). Wenn wir das Bild des vorderen und eines
der Seitenrechtecke vorgeben, ist alles andere bestimmt.
Durch Aufsetzen eines Daches entsteht das Bild eines Giebelhauses; wir
müssen nur noch die Höhe des vorderen Giebels vorgeben, vgl. Übung
4.1.
Horizont der
Dachebene
Horizont
der Fußboden−
ebene
Horizont
der Frontebene
Heute ist uns die Perspektive von der Photographie her vertraut,
aber die Menschen früherer Jahrhunderte hatten keine solche Möglichkeit. Unser Sehen ist nicht wirklich perspektivisch, denn durch die
Beidäugigkeit und die Anpassung der Augenlinse an die Entfernung
erhält unser Gehirn eine zusätzliche Tiefeninformation. Perspektivisch
gesehen müsste ja ein Gegenstand scheinbar größer werden, wenn wir
uns ihm nähern, aber im Nahbereich ist das keineswegs der Fall; der
STERNSTUNDEN DER MATHEMATIK
33
Gegenstand scheint durchaus seine Größe beizubehalten.33 Perspektivische Darstellung setzt also eine gewisse Abstraktion des natürlichen Sehens voraus. Es hat zwar schon von der Antike an Versuche gegeben, die
räumliche Tiefe durch schräge und konvergente Linien wiederzugeben,
aber die genaue Konstruktion blieb verborgen. Sie gelang erst zwischen
1410 und 1420 dem späteren Baumeister des Doms von Florenz, Filippo
Brunelleschi, dessen Zeichnungen wir aber nur aus Berichten kennen.
Die ersten uns überlieferten perspektivischen Darstellungen stammen
von einem Freund Brunelleschis, dem Maler Masaccio (eigentlich Tommaso di ser Giovanni di Mone Cassai, 1401 - 1428). Berühmt ist sein
Fresco “Dreifaltigkeit” (1426) in der Florenzer Kirche Santa Maria Novella, in dem die Perspektive eine wichtige Funktion für die Aussage
des Bildes bekommt, weil der Standort des Betrachters miteinbezogen
wird.34 Das erste Lehrbuch der Perspektive (“De Pictura”) schrieb 1435
der Genueser Gelehrte Leon Battista Alberti (1404 - 1472).
Perspektivische Bilder sind Zentralprojektionen. Der Bildpunkt ist der
Schnitt der Bildebene mit einer durch den Urbildpunkt und einen festen
Punkt (das Projektionszentrum) verlaufenden Geraden, der Projektionsgeraden. Beim perspektivischen Sehen ist das Auge selbst das Projektionszentrum; die Projektionsgeraden sind die Lichtstrahlen, die vom
Gegenstand ausgehend das Auge erreichen, und der Bildpunkt ist der
Schnitt dieses Strahls mit der Bildebene, die man sich zwischen Auge
und Gegenstand denkt.
A
Auge
A’
B’
Bildebene
B
Gegenstände
Albrecht Dürer zeigt in seinem Lehrbuch “Underweysung der Messung”
von 1525,35 wie man den Bildpunkt auf einer zwischen Auge und Gegenstand befindlichen Glasscheibe bestimmt, indem man den Gegenstand
durch ein Loch in einem fest montiertes Gestell anpeilt. Er beschreibt
33J.-H.
Eschenburg: Is binocular visual space constantly curved? J. Math. Biology 9 (1980), 3 - 22
34http://de.wikipedia.org/wiki/Dreifaltigkeit (Masaccio)
35http://de.wikisource.org/wiki/Underweysung der Messung ...
34
J.-H. ESCHENBURG
auch ein rein mechanisches Verfahren zur Erzeugung eines perspektivischen Bildes (wir haben nur die Schreibweise modernisiert):36
“Bist du in einem Saal, so schlag eine große Nadel mit
einem weiten Öhr, die dazu gemacht ist, in eine Wand
und setz das für ein Auge. Dadurch zeuch einen starken
Faden und häng unten ein Bleigewicht daran. Danach
setz einen Tisch oder Tafel so weit von dem Nadelöhr,
darin der Faden ist, als du willst. Darauf stell ... einen
aufrechten Rahmen ..., der ein Türlein habe, das man
auf und zu mag tun. Dies Türlein sei deine Tafel, darauf
du malen willst. Danach nagel zwei Fäden, die als lang
sind als der aufrechte Rahmen lang und breit ist, oben
und mitten in den Rahmen und den anderen auf einer
Seite auch mitten in den Rahmen und lass sie hängen.
Danach mach einen langen Stift, der vorne an der Spitze
ein Nadelöhr habe. Darein fädel den langen Faden, der
durch das Nadelöhr an der Wand gezogen ist, und fahr
mit der Nadel und dem langen Faden durch den Rahmen hinaus und gib sie einem anderen in die Hand, und
warte du der anderen zwei Fäden, die an dem Rahmen
hängen. Nun gebrauche dies also: Leg eine Laute, oder
was dir sonst gefällt, so fern von dem Rahmen als du
willst, und dass sie unverrückt bleibt, solange du ihrer
bedarfst, und lass deinen Gesellen die Nadel mit dem Faden hinausstrecken auf die nötigsten Punkte der Laute,
und so oft er auf einer Stelle hält und den langen Faden
anstreckt, so schlag allweg die zwei Fäden an dem Rahmen kreuzweis gestreckt an den langen Faden und kleb
sie an beiden Orten mit einem Wachs an den Rahmen,
und heiß deinen Gesellen seinen langen Faden nachlassen. Danach schlag das Türlein zu und zeichne dieselben
Punkte, da die Fäden kreuzweise übereinander gehen,
auf die Tafel. Danach tu das Türlein wieder auf und tu
mit einem anderen Punkt aber also bis dass du die ganze Laute gar an die Tafel punktierst. Dann zeuch alle
Punkte, die auf der Tafel von der Laute worden sind,
mit Linien zusammen, so siehst du, was daraus wird.
Also magst du andere Dinge auch abzeichnen.”
36http://www.martin-missfeldt.de/perspektive-zeichnen-tutorial/perspektive-
albrecht-duerer.php
STERNSTUNDEN DER MATHEMATIK
35
Beim Fotoapparat oder seinem Vorgänger, der Lochkamera (Camera
Obscura) ist es etwas anders: Das Projektionszentrum ist der Linsenmittelpunkt oder das Loch, und die Bildebene befindet sich dahinter
auf der Rückwand der Kamera. Die Bildebene ist also nicht mehr zwischen Gegenstand und Projektionszentrum, sondern erst hinter dem
Projektionszentrum. Der Unterschied ist jedoch gering; eine Parallelverschiebung der Bildebene bewirkt lediglich eine zentrische Streckung
des Bildes. Wird die Bildebene wie im vorliegenden Fall auf die andere
Seite des Projektionszentrums verschoben, so ist der Streckungsfaktor
negativ. Das Bild wird daher im Fotoapparat um 180o gedreht, steht
also auf dem Kopf.
A
B’
A’
Linse
Bildebene
B
Gegenstände
Der französischen Festungsbaumeister Gerard Desargues (1591 - 1661)
entwickelte 1639 eine Idee, die sich als sehr weit tragend erweisen sollte. In einem perspektivischen Bild einer Ebene gibt es eine Gerade,
auf der sich die Bilder paralleler Geraden treffen, den Horizont. Ihm
entspricht aber keine Gerade der abgebildeten Ebene. Sollte man nicht
die Urbildebene um neue, “im Unendlichen liegende” Punkte erweitern, sogenannte Fernpunkte oder ideale, d.h. nur der Idee nach vorhandene Punkte, die man als Urbilder der Horizontpunkte ansehen
könnte? Die Fernpunkte müssten zusammen eine neue Gerade bilden,
die Ferngerade, das Urbild des Horizonts. Dann wäre man endlich den
lästigen Sonderfall der Geometrie los, dass zwei Geraden einer Ebene leider nicht immer einen Schnittpunkt haben, sondern manchmal
parallel sind: Die parallelen Geraden würden sich eben in den neu hinzugewonnenen Punkten, den Fernpunkten treffen, und zu jeder Klasse
paralleler Geraden würde genau ein solcher Fernpunkt gehören. Ebenso
könnte man im Raum von einer (hinzugedachten) Fernebene sprechen,
die die Schnittpunkte paralleler Geradenscharen im Raum enthalten
und die Ferngeraden aller Ebenen des Raumes enthalten müsste. Dass
es solche Punkte nicht wirklich gibt, störte die Mathematiker wenig; es
war eben eine Erweiterung der üblichen Geometrie, ähnlich wie man
die Zahlen durch Hinzunahme gedachter neuer Zahlen (z.B. ∞) erweitern konnte; man nannte sie projektive Geometrie. Als der französische
36
J.-H. ESCHENBURG
Mathematiker Jean Victor Poncelet (1788 - 1867) als Soldat unter Napoleon 1812 in russische Kriegsgefangenschaft geriet und viel Zeit, aber
keine Bücher zur Verfügung hatte, entwickelte er systematisch die Gesetze dieser Geometrie.
Wir wollen uns einen einfachen Satz der projektiven Geometrie ansehen, der von Desargues stammt:
S
A
s
C
B
A’
C’
c
a
B’
b
Satz. (Desargues): In der projektiven Ebene seien drei Geraden a, b, c
gegeben, die einen Punkt S gemeinsam haben. Gegeben seien weiterhin zwei Dreiecke ABC und A′ B ′ C ′ mit Eckpunkten auf den jeweiligen Geraden: A, A′ ∈ a und B, B ′ ∈ b und C, C ′ ∈ c. Dann liegen
die Schnittpunkte entsprechender Seiten, die Punkte AB ∧ A′ B ′ und
AC ∧ A′ C ′ und BC ∧ B ′ C ′ , auf einer gemeinsamen Geraden s.
Beweis 1 (mit Abbildungsgeometrie): Mit Hilfe einer Zentralprojektion können wir die Gerade s durch die Punkte AB ∧ A′ B ′ und
AC ∧ A′ C ′ zur Ferngeraden machen. Dann sind die Geradenpaare AB
und A′ B ′ sowie AC und A′ C ′ parallel, da sie sich auf der Ferngeraden treffen. Somit entsteht das Dreieck A′ B ′ C ′ aus dem Dreieck ABC
durch zentrische Streckung (mit Zentrum S) und daher ist auch das
dritte Geradenpaar BC und B ′ C ′ parallel (“Strahlensatz”), d.h. die
Geraden BC und B ′ C ′ schneiden sich auf der Ferngeraden. Durch
Rücktransformation, die die Ferngerade wieder auf s abbildet, folgt
die Behauptung.
Beweis 2 (mit räumlicher Geometrie): Wir können die DesarguesFigur als Projektion einer räumlichen Figur ansehen, wobei wir uns z.B.
vorstellen, dass die mittlere Gerade b weiter vorn liegt als a und c. Die
STERNSTUNDEN DER MATHEMATIK
37
beiden Dreiecke ABC und A′ B ′ C ′ definieren nun zwei unterschiedliche
Ebenen E und E ′ im Raum, die sich in einer Geraden s schneiden.
Die Seiten der beiden Dreiecke liegen in den jeweiligen Ebenen, ihre
Schnittpunkte (wenn sie existieren) also auf der Geraden s = E ∩ E ′ .
Zwei Geraden im Raum schneiden sich allerdings nur, wenn sie in einer
gemeinsamen Ebene liegen; das ist jedoch für einander entsprechende
Seiten der beiden Dreiecke der Fall; z.B. liegen AB und A′ B ′ in der von
den Strahlen a und b aufgespannten Ebene. Damit ist die Behauptung
gezeigt.
Was aber sind diese Fernpunkte wirklich? Auch dazu geben die perspektivischen Abbildungen, d.h. die Zentralprojektionen den Schlüssel.
Wir beschreiben sie nochmal mit den Begriffen der räumlichen Geometrie. Jeder Punkt x der Urbildebene U bestimmt ja genau eine Gerade
ox durch das Projektionszentrum o, und sein Bildpunkt ist der Schnitt
dieser Geraden mit der Bildebene B. Eigentlich können wir die Punkte
x der Urbildebene ganz vergessen und durch ihre Projektionsgeraden
ox ersetzen. Eine Menge von Projektionsgeraden, deren Bilder (Schnitte mit B) auf einer gemeinsamen Geraden in B liegen, sind in einer
gemeinsamen Ebene enthalten, nämlich der von der Bildgeraden und
dem Projektionszentrum aufgespannten Ebene. Wir haben daher eine
Art Lexikon gefunden: Punkte entsprechen Projektionsgeraden, Geraden entsprechen Ebenen durch o. Aber einige Geraden durch o treffen die Urbildebene U gar nicht, nämlich die zu U parallelen; ihnen
entspricht also kein Punkt von U . Das sind die neuen “idealen” Punkte von U , die nach unserem Lexikon wirklich auf einer gemeinsamen
“Geraden” (der Ferngeraden) liegen, denn sie sind ja alle in der zu U
parallelen Ebene durch o enthalten. Sie können dagegen sehr wohl die
Bildebene B schneiden, deshalb sehen wir dort den Horizont als Bild
der Ferngeraden.
B
Horiz.
U
Die ebene Projektive Geometrie ist daher nichts anderes als die Geometrie des “Büschels” der Geraden durch einen festen Punkt o im Raum,
38
J.-H. ESCHENBURG
wobei wir nur neue Worte benutzen: Eine Gerade durch o nennen wir
“Punkt” und eine Ebene durch o “Gerade”.
Wie aber steht es mit der räumlichen projektiven Geometrie? Auch
hier können wir von Fernpunkten sprechen, die alle auf der Fernebene
versammelt sind, aber um den Projektiven Raum (nämlich den um die
Fernebene erweiterten Raum) wieder durch Geradenbüschel beschreiben zu können, müssten wir eine Dimension mehr zur Verfügung haben:
Der Projektive Raum wäre dann das Büschel aller Geraden durch einen
festen Punkt im 4-dimensionalen “Raum”. Dieser Gedanke musste erst
einmal gefasst werden und das Konzept eines “Raumes” mit mehr als
drei Dimensionen entwickelt werden; es geht auf Grassmann37 zurück,
der 1844 das Buch “lineale Ausdehnungslehre” schrieb.38
Dort führt er “Systeme m-ter Ordnung” ein, die “durch je m Änderungsweisen erzeugbar” seien (ebd., Seite 30). Heute fällt uns dieser Gedanke leichter: Da die Punkte der Ebene durch Zahlenpaare (x, y), die
ebenen Koordinaten, und die des Raums durch Zahlentripel (x, y, z),
die räumlichen Koordinaten darstellbar sind, mag zwar die geometrische Vorstellung eines vierdimensionalen Raumes problematisch sein,
aber die Menge aller Zahlenquartetts (x, y, z, w) zu betrachten macht
keinerlei Schwierigkeiten; sie wird für den “4-dimensionalen Raum”
untergeschoben (4 “Änderungsweisen” in der Sprache Grassmanns),
und ebenso können wir auch “Räume” mit noch mehr (m) Dimensionen definieren (“m Änderungsweisen”). Statt der Geradenbüschel kann
man in einem m-dimensionalen Raum übrigens auch Ebenenbüschel
durch einen festen Punkt und sogar höherdimensionale Analoga (kdimensionale “Ebenen” für alle k < m) betrachten; dies sind grundlegende Strukturen der höherdimensionalen Geometrie, die zu Grassmanns Ehren “Grassmann-Mannigfaltigkeiten” genannt werden; sie verallgemeinern die Projektiven Räume beliebiger Dimension noch ein weiteres Mal.
Übungen
4.1. Perspektive. Zeichnen Sie ein Haus mit Giebeldach perspektivisch schräg von oben (Vogelperspektive). Vertikale Linien sollen auch
in der Zeichnung vertikal sein, und die rechte Kante der Giebelfront soll
vorne liegen. Die Giebelfront soll in der Realität bis zum Dachansatz
37Hermann
Günter Grassmann, 1809 - 1877 (Stettin)
38https://archive.org/details/dielinealeausde00grasgoog
STERNSTUNDEN DER MATHEMATIK
39
quadratisch sein, und die Dachschräge soll 45o betragen. Ihre Zeichnung soll auch die Horizonte aller beteiligten Ebenen enthalten (Fußboden, Wände, Dachschrägen). Geben Sie kurze Erklärungen und Begründungen für alle Konstruktionsschritte.
4.2. Photographische Abbildung. Bestimmen Sie im dreidimensionalen Raum (Koordinaten x, y, z) die Zentralprojektion mit Projektionszentrum (0, 1, 1), die die xy-Ebene (Urbildebene) auf die xz-Ebene
(Bildebene) abbildet (Zeichnung!), d.h. berechnen Sie die Zuordnungsvorschrift (x, y) 7→ (x̃, z̃). Bestimmen Sie den Horizont in der xz-Ebene
und zeigen Sie, dass die Bilder paralleler Geraden der xy-Ebene auf dem
Horizont einen Schnittpunkt besitzen (mit Ausnahme der zum Horizont
parallelen Geraden).
4.3. Dürers Heiliger Hieronymus. Bitte vertiefen Sie sich einmal in
den Holzschnitt “Der Heilige Hieronymus 39 im Gehäus” von Albrecht
Dürer (1514):
http://www.glyphs.com/art/durer/jerome.jpg
http://de.wikipedia.org/wiki/Der heilige Hieronymus im Gehäus
Wie vielfach bei Dürer ist darin eine Fülle von geometrischen Ideen
enthalten. Mit dem Zusatzwissen, dass einige Winkel in der Realität
rechte Winkel sind (Tisch, Schemel, ...) kann man aus der perspektivischen Darstellung einen exakten Grundriss des Zimmers konstruieren!
Versuchen Sie, die vorstehende Konstruktionszeichnung zu verstehen
und zu erklären, wie man den wahren Winkel zwischen Kante und Diagonale des Stuhls und damit seine genaue Form ermittelt. “Erklären”
heißt den Grund finden: Warum ist β der wahre Winkel?
39Hieronymus,
347 - 414, übersetzte das Neue Testament vom Griechischen ins
Lateinische (Vulgata): “Gloria in excelsis Deo, et in terra pax hominibus bonae
voluntatis”. Er galt als sehr tierlieb; nach einer Legende soll er einem Löwen einen
Dorn aus der Pfote gezogen haben, daher wird er immer mit dem Löwen abgebildet.
40
J.-H. ESCHENBURG
Hl. Hieronymus
E
H
G
F
g
f
Tisch
Stuhl
_
g
_
f
Löwe
ß
0o
4.4. Satz von Pascal. Der Satz von Pascal40 besagt:
Gegenüberliegende Kanten eines in einen Kegelschnitt
(z.B. Kreis oder Ellipse) einbeschriebenen Sechsecks schneiden sich auf einer gemeinsamen Geraden (linke Figur).
Die Reihenfolge der 6 Punkte ist dabei beliebig.
?
Der Beweis ist ein Beispiel für die Anwendung perspektivischer Abbildungen (projektive Geometrie). Wir können die linke Figur als perpektivische Darstellung deuten; der Horizont ist die schräge StrichpunktGerade, die durch zwei der drei Kantenschnittpunkte definiert ist. In
der “ursprünglichen” Situation (mittlere Zeichnung) sind also zwei Kantenpaare parallel, die durchgezogenen. (Der Kreis wird bei der Transformation vielleicht zu einer Ellipse verzerrt, aber durch eine Stauchung in
Richtung der längeren Ellipsenachse wird daraus wieder ein Kreis und
die zwei Kantenpaare bleiben parallel.) Damit sind die beiden markierten Winkel in der mittleren Zeichnung gleich, und nach dem Satz über
den Faßkreiswinkel (“Peripheriewinkel über einer Sehne gleich halber
40Blaise
Pascal, 1623 (Clermont-Ferrand) - 1662 (Paris)
STERNSTUNDEN DER MATHEMATIK
41
Mittelpunktswinkel”, rechte Figur) haben die beiden gepunkteten Sehnen den gleichen Mittelpunktswinkel, sie sind also kongruent und damit
spiegelsymmetrisch mit einer Spiegelachse durch den Kreismittelpunkt
(Strich-Punkt-Gerade in der mittleren Figur). Die beiden fehlenden
(gestrichelten) Sechseckseiten sind somit invariant unter dieser Spiegelung, denn ihre Endpunkte werden vertauscht. Daher sind sie senkrecht
zur Spiegelachse, also parallel.
5. Cardano: Kubische und quartische Gleichung (1545)
Die nächste “Sternstunde”, von der ich berichten möchte, ist die
Veröffentlichung des Buches “Ars magna de Regulis Algebraicis” von
Gerolamo Cardano im Jahr 1545.41 Jedes Ereignis, über das ich hier
berichte, hat seine Vorgeschichte und seine Nachwirkungen. In diesem
Fall ist die Vorgeschichte besonders interessant.
Das europäische Mittelalter war in mathematischer Hinsicht eine Zeit
des Stillstandes. Mathematik fand anderswo statt, vor allem in der
islamischen Welt, die das antike Erbe übernahm und fortführte und
mit der indischen Mathematik verband. Das Wort “Geometrie” ist
griechischen Ursprungs (Geo-metrie = Erd-Messung), das Wort “Algebra” dagegen kommt aus dem Arabischen, der gemeinsamen Sprache des islamischen Kulturraums. Es stammt von dem arabischen Verb
“jabr” = “vervollständigen”.42 In mathematischer Bedeutung tritt es
zum ersten Mal um 825 n.Chr. in dem Buchtitel “Hisab al-jabr wa-lmuqabala” (Rechenverfahren durch Vervollständigen und Ausgleichen)
des persisch-arabischen Mathematikers Al-Chwarizmi43 auf, von dessen Namen übrigens auch das Wort “Algorithmus” (Rechenverfahren)
abgeleitet wurde.44 Die beiden Worte “al-jabr” (das Vervollständigen)
und “al-muqabala” (das Ausgleichen)45 bezeichnen zwei Sorten von
Umformungen von Gleichungen zum Zwecke der leichteren Lösung:
Beseitigung negativer Terme durch Vervollständigung (al-jabr) sowie
41http://www.filosofia.unimi.it/cardano/testi/operaomnia/vol
4 s 4.pdf
Wort wurde auch in medizinischer Bedeutung verwandt, im Sinne von
Einrenken von Gliedern. In dieser Bedeutung wanderte es in das mittelalterliche Spanisch ein: Ein “Algebrista” war eine Person, die Glieder einrenkte, vgl.
http://buscon.rae.es/drae/srv/search?val=algebrista
43Abu Dscha’far Muhammad ibn Musa al-Chwarizmi, ca. 780 - 850 n.Chr., vermutlich persischer Herkunft, hat vorwiegend in Bagdad gelebt und am dortigen
“Haus der Weisheit” gelehrt und gearbeitet.
44http://de.wikipedia.org/wiki/Algorithmus
45Wir benutzen die angelsächsische Umschrift, weil sie international (und im
Internet) die Üblichere ist. Im Deutschen wird die Transskiption “al-dschabr” und
“al-murhabala” verwendet.
42Das
42
J.-H. ESCHENBURG
Verkleinerung positiver Terme auf beiden Seiten durch Ausgleich (almuqabala). Als Beispiele nennt Al-Chwarizmi:46
x2
al-jabr
sowie
⇒
= 40x − 4x2
5x2 =
40x,
50 + 3x + x2 = 29 + 10x
al-muqabala
⇒
21 + x2
=
7x.
Sehr bald schon stand das Wort “Algebra” für das gesamte Gebiet,
das der persische Gelehrte Omar Khayyam (1048 - 1131, Naishapur,
Persien) in seinem Lehrbuch “Algebra und Murhabala” folgendermaßen beschrieb: “Was man in dieser Kunst sucht, ist der Zusammenhang zwischen dem Gegebenen des Problems und der Unbekannten ...
Die Algebra besteht darin, die mathematischen Methoden zu erkennen,
vermöge derer man die so beschriebene Bestimmung der Unbekannten
entweder arithmetisch oder geometrisch vornehmen kann.”47 Das sind
neue Töne im Vergleich mit der klassischen griechischen Mathematik,
in der der Begriff “Unbekannte” noch nicht vorkam. Doch einen antiken Vorläufer gab es, Diophantos, der vermutlich im 3. nachchristlichen Jahrhundert in Alexandria lebte. In seinem Buch “Arithmetica”
behandelt er Gleichungen und verwendet bereits Unbekannte und ihre Potenzen, beeinflusst vielleicht von babylonischer Mathematik. Die
noch heute in der Mathematik übliche Bezeichnung “Diophantische
Gleichungen” für Gleichungen mit ganzzahligen Lösungen erinnert an
sein Werk. Die “Arithmetica” wurde ins Arabische übersetzt und war
bekannt.
Lassen Sie mich noch etwas bei Omar Khayyam verweilen. Er war ein
Universalgelehrter, ein “Hakim”. Neben seinem mathematischen Werk
berechnete er lange vor der gregorianischen Kalenderreform (1582) die
genaue (damalige) Länge eines Jahres, nämlich 365,24219858156 Tage, und er schuf den bis heute gültigen persischen Kalender, in dem
das Jahr mit dem Frühlingsanfang am 21. März (Norūz) beginnt. Mit
einiger Wahrscheinlichkeit war er auch am Bau der großartigen Freitagsmoschee von Isfahan beteiligt (siehe Übung 5.2). Außerdem schrieb
er eine große Zahl von Gedichten, zum Großteil Vierzeiler wie diesen:
Von allen Menschen, die ich je gekannt,
Ich nur zwei Menschen glücklich fand.
Den, der der Welt Geheimnis tief erforscht,
Und den, der nicht ein Wort davon verstand.
46http://www-history.mcs.st-and.ac.uk/Biographies/Al-Khwarizmi.html
47Sebastian
Linden: Die Algebra des Omar Chayyam, Edition Avicenna, 2012
STERNSTUNDEN DER MATHEMATIK
43
oder diesen:
O Weh um jenes Herz, in dem kein Feuer brennt.
Das nicht die hehre Glut der Liebessonne kennt;
Wer einen ganzen Tag ohn’ Liebe hingebracht,
Tut recht, wenn jenen Tag er ’nen verlornen nennt.
oder gar diesen:
Als Du das Leben schufst, schufst Du das Sterben:
Uns, Deine Werke, weiht’st Du dem Verderben.
Wenn schlecht Dein Werk war, sprich wen trifft die Schuld?
Und war es gut, warum schlägst Du’s in Scherben?
Man versteht vielleicht, wieso er in seinem späteren Leben Ärger mit
der religiösen Orthodoxie bekam, dem er sich durch eine Pilgerreise
nach Mekka entzog. Aber zwei Jahrzehnte lang (1074 - 1092) konnte er eine ruhige Forschungszeit in Isfahan genießen, dessen Herrscher
Malik-Shah seine Arbeit schätzte und ihn beschützte. In dieser Zeit entstanden auch seine mathematischen Werke, die sich mit der Kubischen
Gleichung beschäftigten, der Gleichung vom Typ
ax3 + bx2 + cx + d = 0.
Warum interessierte man sich für diese Gleichung? Eine Motivation
kam von Archimedes, der folgende Aufgabe gestellt hatte: Man finde die
Höhe h einer Kugelkappe von gegebenem Radius r, deren Volumen Vh
ein gegebener Bruchteil α des Kugelvolumens ist. Da Vh = π3 (3r − h)h2
3
(vgl. Übungsaufgabe 3.1, Seite 25), ist α = Vh / 4πr
= 4r13 (3rh − h3 ),
3
also löst h eine kubische Gleichung. Archimedes hatte eine Lösung angekündigt, aber nie gegeben. Auch andere geometrische Probleme von
Archimedes führten auf kubische Gleichungen, zum Beispiel die Konstruktion des regelmäßigen Siebenecks. Die islamischen Mathematiker
versuchten, mit den neuen Methoden der Algebra diese alten Probleme zu lösen. Omar Khayyam selbst gibt in seiner kleinen Schrift “Über
die Teilung des Viertelkreises” ein anderes Problem als Motivation, das
aus der Kreisgeometrie stammt (siehe Übung 5.1), aber er verweist ausdrücklich auch auf das Kugelabschnitts-Problem des Archimedes. Seine
Lösungsmethode ist, die Lösung einer kubischen Gleichung durch den
44
J.-H. ESCHENBURG
Schnitt zweier Kegelschnitte,48 zum Beispiel Kreis und Parabel, zu gewinnen.49
y
ay = x 2
p
yp
x2+ y 2= bx
b
2
x
xp
Im Fall, der in der Figur dargestellt ist, erfüllen die Koordinaten des
Schnittpunkts p die beiden Gleichungen yp = x2p /a und x2p + yp2 = bxp ,
also x2p + x4p /a2 = bxp , und nach Division durch xp und Multiplikation
mit a2 erhalten wir für x = xp die kubische Gleichung
x 3 + a2 x = a2 b .
Auch dies ist keine algebraische Lösung des Problems, aber Kegelschnitte können mechanisch erzeugt werden (siehe Fußnote 48) und
somit war ihr Schnitt zeichnerisch konstruierbar. Mitten in der Konstruktion hält Omar Khayyam einen Moment inne und schreibt die
folgenden bemerkenswerten Sätze:50
“Die Selbstzufriedenheit aber ist das Vorrecht der niederen Leute, denn ihre Seele kann von den Wissenschaften
nur einen winzigen Teil begreifen. Und wenn sie diesen
einmal begreifen, glauben sie, dass dieses Wenige alle
Wissenschaften umfasst und in sich vereint. Dass Gott
uns bewahre vor solcherlei Anschauungen, die uns in die
48Kegelschnitte sind die einfachsten ebenen Figuren, die nicht geradlinig berandet
sind. Es sind Schnitte eines Kreiskegels mit einer Ebene: Kreis, Ellipse, Parabel und
Hyperbel (sowie Geradenpaare). Sie lassen sich mechanisch erzeugen:
P
P
F’
F
F
F
F’
Hyperbel
Parabel
Wußing: 6000 Jahre Mathematik, Springer 2008, Seite 249
50Sebastian Linden: Die Algebra des Omar Chayyam, Edition Avicenna, 2012,
Seite VK 100
Ellipse
49H.
STERNSTUNDEN DER MATHEMATIK
45
Irre führen und daran hindern, die Wahrheit zu erkennen und unser Heil zu finden.”
Europa nahm die wissenschaftlichen Fortschritte im islamischen Kulturkreis erst nach 1200 zur Kenntnis. Dann allerdings begann eine rege Übersetzungstätigkeit, besonders in Spanien und Süditalien, wo die
beiden Kulturen in Kontakt standen. Auch viele der antiken Schriften
wurden erst durch Rückübersetzung aus dem Arabischen in Europa
wieder zugänglich, was schließlich zur “Wiedergeburt” (Renaissance)
der antiken Wissenschaft in Europa führte. Die wirtschaftlich fortgeschrittenste Region Europas vor der Entdeckung Amerikas und des
Seewegs nach Indien war Norditalien. Dort entstand das Bankwesen,
die negativen Zahlen wurden entdeckt,51 und neben den alteingesessenen Universitäten verdingten sich Rechenmeister, um den Kaufleuten
das kaufmännische Rechnen beizubringen.
Der erste substantielle Beitrag der europäischen Mathematik zur Algebra war um 1520 die Entdeckung einer echten Lösungsformel für die
kubischen Gleichung 52
x3 + ax = b.
(17)
Wie hat man wohl die Lösungsformel gefunden?53 Die Gleichung (17)
scheint schwer lösbar. Aber es gibt andere kubische Gleichungen, deren
Lösung auf der Hand liegt: Die Gleichung
(x + u)3 = v 3
(18)
hat offensichtlich die Lösung
(19)
x = v − u.
Die Idee ist nun, die einfache Gleichung (18) auf die Form der schwierigen (17) zu bringen: Es gilt
(x + u)3 = x3 + 3x2 u + 3xu2 + u3 = x3 + 3xu(x + u) + u3 ,
51Die
negativen Zahlen scheinen in Europa durch Leonardo von Pisa, genannt
Fibonacci (ca. 1170 - 1250) in seinem Liber Abaci eingeführt worden zu sein (Pisa
1202 und 1228), in dem auch die arabischen Ziffern zum ersten Mal verwendet werden. Vgl. Wolfgang Schwarz: Aus der Geschichte der Zahlentheorie, Frankfurt/M.,
2001, http://www.math.uni-frankfurt.de/∼schwarz/schwarz/g ztws98.pdf
52
Die allgemeinste kubische Gleichung ist x3 + ax2 + bx = c. Sie lässt sich aber
leicht auf die Form (17) bringen, wenn man anstelle von x den Ausdruck x̃ − α/3
einsetzt (“substituiert ”); dann ergibt sich eine kubische Gleichung in x̃ ohne x̃2 Term.
53Die folgende Darstellung verdanke ich Urs Kirchgraber (Zürich).
46
J.-H. ESCHENBURG
und weil wir x + u gemäß (19) durch v ersetzen können (hier liegt der
Trick!), verwandelt sich (18) in die Gleichung
(20)
x3 + 3uvx = v 3 − u3 .
Diese Gleichung ist tatsächlich von der Form (17) mit
(21)
a = 3uv, b = v 3 − u3 .
Wenn also die Koeffizienten a, b der “schwierigen” Gleichung (17) die
Form (21) haben, dann ist x = v − u eine Lösung. Um für gegebene
Koeffizienten a, b solche u und v zu finden, müssen wir die Gleichungen
(21) nach u und v auflösen:
a 3
a
3
3
u= , v =b+u =b+
.
3v
3v
Multiplizieren wir die letzte Gleichung noch einmal mit v 3 , so erhalten wir eine quadratische Gleichung für v 3 , nämlich v 6 = bv 3 + ( a3 )3 .
√
Quadratische Gleichungen können wir lösen: v 3 = 2b ± D mit D :=
√
( a3 )3 + ( 2b )2 , und folglich −u3 = b − v 3 = 2b ∓ D. Als Lösung x = v − u
erhalten wir daher
r
r
a 3 b 2
√
√
b
3 b
3
.
+ D+
− D, D =
(22)
x=
+
2
2
3
2
Diese Formel wurde um 1515 von dem Bologneser Mathematiker Scipione del Ferro54 entdeckt, der sie aber erst auf dem Totenbett an seiner
Schüler Antonio Maria del Fiore weitergab. Dieser, im Alleinbesitz der
Formel, forderte 1535 den venezianischen Rechenmeister Nicolo Tartaglia55 zu einem Rechenduell heraus, für das er ihm eine Reihe kubischer
Gleichungen vorlegte. In der Nacht vor dem “Duell” fand Tartaglia
selbst die Formel und trug damit den Sieg davon. Er gab sie 1539
an seinen Freund, den Arzt und Mathematiker Cardano,56 auf dessen
inständige Bitten weiter, und zwar in Gedichtform:
Quando chel cubo con le cose appresso (x3 + ax)
Se agguaglia à qualche numero discreto (= b)
Trovan dui altri differenti in esso. (u − v = b)
Dapoi terrai questo per consueto
Che’llor produtto sempre sia eguale (uv =)
Al terzo cubo delle cose neto. (( a3 )3 )
El residuo poi suo generale
54Scipione
del Ferro, 1465 - 1526, Bologna
Tartaglia, 1499 - 1557, Breschia, Venedig
56Girolamo Cardano, 1501 - 1576, Mailand, Pavia
55Nicolo
STERNSTUNDEN DER MATHEMATIK
47
√
√
Delli lor lati cubi, ben sottratti ( 3 u − 3 v)
Varra la tua cosa principale (= x).57
Aber Cardano musste schwören, die Formel für sich zu behalten.
Cardano hatte einen Freund und Assistenten. Ludovico Ferrari,58
den er bereits als Jugendlichen in sein Haus aufgenommen hatte und
der sich als sehr klug erwies. Er wurde Cardanos Sekretär und engster
Mitarbeiter und fand 1540 auch die Lösung der quartischen Gleichung59
(23)
x4 + ax2 + bx + c = 0,
indem er sie wiederum mit einer einfach zu lösenden Gleichung verglich:
(24)
(x2 + ux + v)(x2 − ux + w) = 0
Die linke Seite von (24) ist Null, wenn einer der beiden Faktoren Null
ist, also braucht man zu ihrer Lösung nur zwei quadratische Gleichungen zu lösen. Wieder ist die Idee, die einfache Gleichung (24) durch
Ausmultiplizieren auf die Form der schwierigen (23) zu bringen:
(x2 + ux + v)(x2 − ux + w) = x4 + (w + v − u2 )x2 + u(w − v)x + vw ;
für (24) = (23) muss also w + v = a + u2 und w − v = b/u und
vw = c gelten. Durch Addition und Subtraktion der ersten beiden
Gleichungen folgt 2w = a + u2 + b/u und 2v = a + u2 − b/u und
daraus 4vw = (a + u2 )2 − (b/u)2 . Andererseits ist 4vw = 4c (dritte
Gleichung), also erfüllt t = u2 die Gleichung (a + t)2 − b2 /t = 4c, und
nach Multiplikation mit t entsteht die kubische Gleichung
t3 + 2at2 + (a2 − 4c)t = b2 .
√
Eine Lösung t dieser Gleichung ergibt u = t und damit auch 2w =
a + u2 + b/u und 2v = a + u2 − b/u. Damit haben wir die Lösung
der quartischen Gleichung auf die Lösung einer kubischen und einer
quadratischen Gleichung zurückgeführt. Das war Ferraris Methode.
57http://utenti.quipo.it/base5/poetico/tartagliac.htm
Die Formeln sind die “moderne” Interpretation. Übersetzung:
Wenn der Kubus mit den Sachen daneben (x3 + ax)
gleich einer bestimmten Zahl ist, (= b)
finden sich zwei andere verschiedene in dieser. (u − v = b)
Dann wirst du dieses festhalten nach Gewohnheit,
dass ihr Produkt sei immer gleich (uv =)
dem dritten Teil des reinen Kubus der Sachen. (( a3 )3 )
Der allgemeine Rest dann
√
√
von ihren Kubikwurzeln, wohl abgezogen, ( 3 u − 3 v)
wird den Wert deiner Hauptsache ergeben. (= x)
58
Ludovico Ferrari, 1522 - 1565, Bologna
59
Wieder kann man die allgemeine quartische Gleichung x4 +a1 x3 +a2 x2 +a3 x+
a4 = 0 durch die Substitution x = x̃ − a41 auf die Form (23) ohne x3 -Term bringen.
48
J.-H. ESCHENBURG
Cardano hätte dieses Ergebnis gerne veröffentlicht, aber ohne die
Lösung der kubischen Gleichung war es unbrauchbar. Tartaglia aber
weigerte sich, seine Formel zu veröffentlichen. Doch im Jahr 1543 besuchten Cardano und Ferrari ihren Bologneser Kollegen Annibale della
Nave, den Schwiegersohn und Nachfolger von Scipione del Ferro, und
erfuhren, dass letzterer die Lösungsformel der kubischen Gleichung lange vor Tartaglia gefunden hatte. Daraufhin fühlte sich Cardano nicht
mehr an seinen Eid gebunden und veröffentlichte 1545 die Formel und
die Ergebnisse von Ferrari mit Beweisen in seinem Buch “Ars Magna”. Seitdem heißt sie Cardanosche Formel, obwohl Cardano bereits
im Vorwort seines Buches die Entstehungsgeschichte bis zurück zu den
Mathematikern des islamischen Kulturkreises getreu schildert. Danach
waren Cardano und Tartaglia nicht mehr so gut befreundet. Es gab
einen gewaltigen Streit, der 1548 zu einer Diskussion vor großem Publikum in Mailand führte. Cardano kam nicht selbst, sondern schickte
den wortgewandten Ferrari, dem Tartaglia nicht gewachsen war, denn
er stotterte.60
Übungen
5.1. Omar Khayyams Teilung des Viertelkreises. In dem Buch
“Die Teilung des Viertelkreises” von Omar Khayyam geht es um das
folgende geometrische Problem: Gesucht ist eine horizontale Sehne des
Viertelkreises, deren Länge sich zum Radius verhält wie die Differenz
von Radius und Höhe zur Höhe dieser Sehne. Setzen wir den Radius
gleich Eins, so ergibt sich:
C
t
1−y
1−y
D
y
y
x
x
B
1
A
60Tartaglia war als Jugendlicher schwer verwundet worden, als französische Trup-
pen 1512 in seine Heimatstadt Brescia eindrangen und dort ein furchtbares Blutbad
anrichteten.
STERNSTUNDEN DER MATHEMATIK
(25)
49
1−y
x
=
1
y
(linke Figur). Leiten Sie daraus für y die kubische Gleichung
(26)
y3 + y2 + y = 1
ab. Beachten Sie x2 + y 2 = 1, also x2 = 1 − y 2 = (1 + y)(1 − y). Wie
lautet die Gleichung für x? Lösen Sie dazu (25) nach y auf und setzen
Sie das Ergebnis in (26) ein.
5.2. Omar Khayyams Dreieck. Wenn man die linke Figur des Viertelkreises um die Tangente im Punkt (x|y) ergänzt, so erhält man ein
rechtwinkliges Dreieck ABC (rechte Figur). Dieses ist durch eine besondere Eigenschaft gekennzeichnet, wie Omar Khayyam gezeigt hat:
Die Länge der Hypothenuse ist die der kürzeren Kathete plus die Höhe
(über der Hypothenuse). In unserer Figur hat die kürzere Kathete die
Länge 1 (Radius des Viertelkreises) und die Höhe ist x. Zu zeigen ist
also
t = x.
Zeigen Sie dies mit der Gleichung (25), x = 1−y
= y1 − 1. Beachten
y
Sie, dass die Dreiecke ABD und BCD ähnlich sind (beide sind ähnlich
zum großen Dreieck ABC) und folgern Sie x2 = (t + 1 − y)y, also
1 = x2 + y 2 = (t + 1)y, und daraus t = y1 − 1 = x.
Bemerkung: Das Khayyam-Dreieck ist in der 1088-89 gebauten Nordkuppel der Freitags-Moschee in Isfahan vielfach realisiert, ein Indiz
dafür, dass Omar Khayyam bei ihrer Konstruktion beteiligt war.61
5.3. Cardanos Formel. (a) Finden Sie mit der Cardanoschen Formel
(22) eine Lösung x für die kubische Gleichung
(27)
x3 − 18x = 35.
(b) Wie findet man alle kubischen Gleichungen, für die die Cardanoschen Formeln “aufgehen”? Starten Sie dazu mit zwei beliebigen
Ku√
62
bikzahlen k+ und k− mit
√ k+ > k− . Berechnen
√ Sie b und D aus den
Gleichungen k+ = b/2 + D und k− = b/2 − D. Setzen Sie
(28)
61Alpay
(a/3)3 = D − (b/2)2
Özdural: A Mathematical Sonata for Architecture. Omar Khayyam and
the Friday Mosque of Isfahan. Technology and Culture 39 (1998), 699 - 715
62Eine ganze Zahl k heißt Kubikzahl, wenn es eine ganze Zahl m mit m3 = k
gibt.
50
J.-H. ESCHENBURG
√
mit D := ( D)2 . Was ist dann die Cardanosche Lösung x der Gleichung
(29)
x3 + ax = b
für dieses a und b? Aus (28) kann man nicht erkennen, dass a rational
ist, aber aus (29), wieso? Probieren Sie alles aus für die Kubikzahlen
k+ = 33 = 27 und k− = 23 = 8 (Hinweise: 192 = 361, 352 = 1225,
1225−361
= 216 = 63 ; überzeugen Sie sich selbst!) und vergleichen Sie
4
mit Teilaufgabe (a).
5.4. Lösung der quintischen Gleichung? Versuchen Sie, die quintische Gleichung
x5 + ax + b = 0 ,
(A)
nach Ferraris Methode zu lösen, indem Sie sie mit einem Produkt eines
quadratischen und eines kubischen Polynoms vergleichen,
(x3 + ux2 + vx + w)(x2 − ux + t) = 0
(B)
wobei die Zahlen u, v, w, t aus a, b zu berechnen sind. Die Lösungen von
(B) sind einfach die Lösungen einer quadratischen und einer kubischen
Gleichung. Leider führt das Verfahren auf eine Gleichung vom Grad 10
für u; zeigen Sie dies!
Anleitung: Bringen Sie die “einfache” Gleichung (B) durch Ausmultiplizieren auf die Form der “schwierigen” (A) und vergleichen Sie die
Koeffizenten in (A) und (B). Sie erhalten vier Gleichungen; die ersten
beiden, v + t = u2 und v − t = w/u, folgen aus dem Verschwinden der
Koeffizienten von x3 und x2 in (A); aus ihnen lassen sich v und t ermitteln. Die beiden anderen sind quadratische Gleichungen in w, deren
Koeffizienten noch von u abhängen. Nach Hochmultiplizieren der Nenner bilden Sie die Differenz dieser beiden Gleichungen, um den w2 -Term
wegzuheben. Damit erhalten Sie einen Ausdruck für w in Abhängigkeit
von u, den Sie in eine der beiden quadratischen Gleichungen für w einsetzen können; dies ergibt die gesuchte Gleichung für u. Für u 6= 0
erhalten Sie die Gleichung63
u10 − 2au6 − 10bu5 − 8a2 u2 − 8abu − 4b2 = 0,
(C)
und aus u lassen sich w, v, t leicht ermitteln. Aber die Gleichung (C)
für u ist wesentlich komplizierter als die Ausgangsgleichung (A). Das
Verfahren ist also gescheitert.
63Keine
Garantie für die Korrektheit der Rechnung! Wenn Sie einen Rechenfehler
finden, schreiben Sie ihn mir bitte.
STERNSTUNDEN DER MATHEMATIK
51
6. Bombelli: Die Zahl, die es nicht gibt (1572)
Die kubische Gleichung
x3 + ax = b
hat nach Cardano die Lösung
r
r
√
√
3 b
3 b
x=
+ D+
− D,
2
2
Sehen wir uns zwei Beispiele an:
D=
a 3
3
2
b
.
+
2
Beispiel 1: x3 − 6x = 9. Dann ist
a
b
9
81
81 − 32
49
= −2,
= , D = −8 +
=
= .
3
2
2
4
4
4
√
√
√
= 8 und 2b − D = 9−7
= 1,
Damit ist D = 27 und 2b + D = 9+7
2
2
√
√
3
3
3
also x = 8 + 1 = 2 + 1 = 3. Probe: 3 − 6 · 3 = 27 − 18 = 9.
Beispiel 2: x3 − 6x = 4. Dann ist
b
a
= −2,
= 2, D = −8 + 4 = −4.
3
2
Jetzt haben√wir ein Problem: Weil D negativ ist, können wir die Quadratwurzel D nicht ziehen! Die Lösungsmethode scheint zu versagen.
Cardano wusste keinen Rat und gab diesem Fall den Namen Casus
irreducibilis (unlösbarer Fall). Das war eigentlich ein Skandal, denn
die Gleichung x3 − 6x = 4 hat offensichtlich Lösungen, zum Beispiel
x = −2, denn x3 − 6x = −8 + 12 = 4. Aber Cardano wusste nicht, wie
man diese Lösung mit seiner Formel finden sollte.
Gut 20 Jahre nach Cardanos “Ars Magna” unternahm ein Ingenieur namens Rafael Bombelli64 einen neuen Anlauf und war erfolgreich. Bombelli hat nie eine Universität besucht, aber er scheint einen
sehr guten Privatlehrer gehabt zu haben, den Ingenieur und Architekten Pier Francesco Clementi. Als dieser um 1548 vom Kirchenstaat
für ein Wasserbauprojekt angestellt wurde, folgte Bombelli seinen Spuren und arbeitete zwischen 1551 und 1560 an der Trockenlegung des
Chiana-Tals, einer zauberhaften Landschaft zwischen Umbrien und der
Toskana, die damals wie Bombellis Heimatstadt Bologna zum Kirchenstaat gehörte. In einem heutigen Touristenhinweis lesen wir über das
Chiana-Tal: “Bei den Römern Kornkammer, dann versumpft, durch
Wasserbauingenieure mit einer Flussumkehr trocken gelegt, wiederum
eine der fruchtbarsten Landschaften und vor allem Heimat der größten
und ältesten Rinderrasse Europas: der Razza Chianina.” Dies also war
64Rafael
Bombelli, 1526 - 1572, Bologna
52
J.-H. ESCHENBURG
ein Verdienst von Bombelli. Nebenher interessierte er sich schon früh
für Algebra; wir wissen nicht, ob Clementi ihn auch auf dieses Wissensgebiet aufmerksam gemacht hat, das ja gerade im Zentrum der
Aufmerksamkeit stand, und Bologna hatte dabei eine gewichtige Rolle
gespielt. Diese Geschichte kannte Bombelli gut. Insbesondere hatte er
das Buch von Cardano gelesen und schätzte es sehr hoch, hatte aber
das Gefühl, dass vieles einfacher gesagt werden konnte. So schrieb er
ein eigenes Algebra-Buch auf Italienisch, “l’Algebra”, in dem er u.a. die
Ergebnisse von del Ferro, Tartaglia, Cardano und Ferrari vorstellte. Er
stieß dabei auch auf den “Casus Irreducibilis”, gab sich aber nicht mit
Cardanos Resignation zufrieden, sondern löste den “unlösbaren Fall”.
Wir erklären seine Methode am Beispiel 2 oben. Bombelli wusste,
dass negative Zahlen wie −4 keine Quadratwurzel haben, denn das
Quadrat negativer wie positiver Zahlen ist positiv, Minus mal Minus
ergibt Plus.65 Aber wir könnten einmal so tun, als gäbe es solche Zahlen
doch (sie wurden später “imaginäre” Zahlen genannt, Zahlen, die nur in
der Vorstellung existieren) und damit wie gewohnt rechnen.√Eigentlich
genügt sogar eine einzige “imaginäre” Zahl, nämlich
i := −1; denn
√
2
2
2
dann wäre i = −1 und (2i) = 4i = −4, also −4 = 2i. Die Lösung
gemäß Cardanos Formel (22) ist demnach
√
√
(30)
x = 3 2 + 2i + 3 2 − 2i.
Doch was sollen wir mit einem solchen Ergebnis anfangen? Wie sollen
wir die 3. Wurzel aus 2 + 2i ziehen? Das wusste auch Bombelli nicht.66
Aber die Umkehrung, die 3. Potenz solcher Zahlen konnte er immerhin
berechnen, zum Beispiel:
(−1 + i)3 = −1 + 3i − 3i2 + i3
= −1 + 3i + 3 − i
= −1 + 3 + (3 − 1)i
= 2 + 2i
und ebenso (−1 − i)3 = 2 − 2i. Diese Rechnung ist ein Glücksfall für
unser Beispiel: Die dritte Potenz von −1 + i und −1 − i ergibt genau
die Zahlen, deren dritte Wurzel wir suchen; diese sind also Kubikzahlen, dritte Potenzen bekannter√Zahlen, wie auch 1 und 8 in Beispiel 1
Kubikzahlen waren. Also ist 3 2 ± 2i = −1 ± i und aus (30) erhalten
65Auch
diese Erkenntnis war noch nicht so alt. Erst seit Kurzem hatte man gelernt, mit negativen Zahlen zu rechnen. Ursache hierfür war das besonders in Oberitalien aufkommende Banken- und Kreditwesen. Damit erst hatte man die reellen
Zahlen vervollständigt und war vom Zahlenstrahl zur Zahlengeraden übergegangen.
66Später konnte man das durchaus, siehe Abschnitt 8, Seite 72.
STERNSTUNDEN DER MATHEMATIK
53
wir
x = (−1 + i) + (−1 − i) = −2.
Wie durch Zauberei sind die “imaginären” Wurzeln negativer Zahlen
verschwunden und wir erhalten die uns schon bekannte Lösung x = −2.
“Ein ausschweifender Gedanke nach Meinung vieler”, bekennt Bombelli.67 “Ich selbst war lange Zeit der gleichen Ansicht. Die Sache schien
mir auf Sophismen mehr als auf Wahrheit zu beruhen, aber ich suchte
so lange, bis ich den Beweis fand.”
Bombellis Algebra-Lehrbuch erschien in seinem Todesjahr 1572. Eine wahre Sternstunde der Mathematik: Bombelli hatte es gewagt, die
Grenzen der bisherigen Vorstellung (“Quadratwurzeln negativer Zahlen gibt es nicht”) zu verlassen, und war damit zu richtigen Ergebnissen
gelangt! Er hatte zum zweiten Mal in der Geschichte der Mathematik
(nach der Entdeckung der irrationalen Zahlen in der Antike) eine dramatische Erweiterung des Zahlbegriffs gewagt. Es sollte mehr als zwei
Jahrhunderte dauern, bis die “imaginären Zahlen” ihrer Mystik ganz
entkleidet und voll akzeptiert waren. Summen von reellen und imaginären Zahlen, wie sie bei den Rechnungen aufgetreten sind, nennt
man komplexe (“zusammengesetzte”) Zahlen.
Man muss sich bei den komplexen Zahlen von einigen gewohnten
Vorstellungen trennen. Zum Beispiel stimmt es nicht mehr, dass eine
Größe sich durch Hinzufügen (Addition) einer anderen vermehrt, aber
das war ja schon bei den negativen Zahlen nicht mehr wahr. Dieses
Phänomen wurde unter dem Namen Interferenz zu einer Grundtatsache
der Physik des 20. Jahrhunderts, aus der die komplexen Zahlen deshalb
nicht mehr wegzudenken sind.68
Außerdem muss man sich eine neue geometrische Vorstellung von
den Zahlen machen: Zahlenstrahl und Zahlengerade werden durch die
67Zitiert
nach Moritz Cantor, Vorlesungen über Geschichte der Mathematik,
Berlin 1900-1908, Band 2, S. 625. Siehe auch J. Bewersdorff, Algebra für Einsteiger,
DOI 10.1007/978-3-658-02262-4 2, 9783658022617-c1.pdf, Seite 12
68 Wenn man Licht durch einen dünnen Spalt fallen lässt, so gibt es auf einem Schirm dahinter eine gewisse Helligkeitsverteilung mit einem breiten Maximum in der Mitte. Öffnet man einen zweiten Spalt sehr dicht daneben, dann
wird die Helligkeit keineswegs überall zunehmen, weil in bestimmten Richtungen
der Wellenberg der einen Welle auf das Wellental der anderen Welle fällt und
Auslöschung eintritt. Nach der Quantenmechanik tritt dieses Phänomen nicht nur
bei Licht, sondern auch bei Materie auf, weil sie ebenfalls Wellencharakter hat. Siehe
nachfolgende Figur sowie www.quantenphysik-schule.de, www.didaktik.physik.unierlangen.de/quantumlab
54
J.-H. ESCHENBURG
Zahlenebene abgelöst. Diesen Schritt hat erst 1797 Caspar Wessel69
vollzogen, der auch kein Mathematiker war, sondern Landvermesser.
Seine dänisch geschriebene Arbeit blieb allerdings lange unbemerkt.
Unabhängig von ihm hat 1806 der Buchprüfer und Amateurmathematiker Argand70 eine ähnliche Interpretation vorgelegt, die 1813 durch
Vermittlung des Mathematikers Jacques Français71 bekannt wurde.
i
−4
−3
−2
−1
0
1
2
3
4
√
Wo könnten wir die neue Zahl i = −1 auf unserer Zahlengeraden
unterbringen? Nicht rechts von der Null, denn dort sind die positiven
Zahlen, deren Quadrat ja wieder positiv ist, also nicht −1. Auch nicht
links davon, denn das Quadrat negativer Zahlen ist ebenfalls positiv
(Minus mal Minus = Plus). Die Null selbst ist es auch nicht, denn ihr
Quadrat ist Null, nicht −1. Auf der Zahlengeraden ist einfach kein Platz
für diese Zahl; sie wird also daneben untergebracht werden müssen,
und damit kommen wir in die zweite Dimension, in die Ebene. Mit der
Zahl i haben wir uns freilich noch viele weitere Zahlen “eingehandelt”:
die Vielfachen von i und deren Summen mit den gewöhnlichen reellen
Zahlen. In der Ebene finden sie alle ihren Platz: Der Zahl x + yi (für
reelle Zahlen x, y) werden wir den Punkt (x, y) mit den kartesischen
Koordinaten x und y zuweisen.
hell
dunkel
dunkel
hell
hell
Wellenberg
Wellental
69Caspar
Wessel, 1745 (Vestby, Norwegen) - 1818 (Kopenhagen)
Robert Argand, 1768 (Genf) - 1822 (Paris); auf ihn geht auch ein elementarer Beweis des Fundamentalsatzes der Algebra zurück, siehe Abschnitt 8.
71Jacques Frédéric Français, 1775 (Saverne) - 1833 (Metz)
70Jean
STERNSTUNDEN DER MATHEMATIK
−4
−3
55
−1+2i
2i
1+2i
2+2i
3+2i
−2+i
−1+i
i
1+i
2+i
3+i
−2
−1
0
1
2
3
−1−i
−i
4
1−i
Alle diese Zahlen zusammen bilden die Menge der komplexen Zahlen,
die mit dem Symbol C bezeichnet wird:
C = {x + yi : x, y ∈ R}
Die Rechenoperationen haben wir schon gesehen; sie ergeben sich aus
den (weiterhin gültigen) Rechenregeln zusammen mit der Gleichung,
die i definiert, i · i = −1:
(x + yi) ± (u + vi)
(x + yi)(u + vi)
1
x + yi
Die letzte Gleichung folgt
= (x ± u) + (y ± v)i ,
= (xu − yv) + (xv + yu)i ,
x − yi
x
y
= 2
=
−
i.
x + y2
x2 + y 2 x2 + y 2
durch Erweitern mit x − yi, denn
(x + yi)(x − yi) = x2 + y 2 .
Diese Gleichungen zeigen, dass auch für die komplexen Zahlen die vier
Grundrechenarten erklärt und die üblichen Rechenregeln erfüllt sind.
Einen solchen Zahlbereich nennen wir heute einen Körper; auch die
rationalen und die reellen Zahlen bilden einen Körper. Dieser Begriff
wurde allerdings erst 1871 durch Richard Dedekind72 eingeführt.
Wir wollen auch für komplexe Zahlen eigene Buchstaben benutzen,
z.B. z und w (später werden wir x auch für komplexe Variable verwenden). Für z = x + yi nennen wir x den Realteil und y den Imaginärteil
und schreiben
x = Re z, y = Im z.
Wir können nun die Addition bei komplexen Zahlen geometrisch ebenso
wie bei reellen Zahlen deuten, nämlich als Aneinanderlegen von zwei
Stäben, die jetzt allerdings unterschiedliche Richtungen haben können.
72Julius
Wilhelm Richard Dedekind, 1831 - 1916 (Braunschweig)
56
J.-H. ESCHENBURG
3i
z+w
z
2i
i
−2
−1
0
w
1
2
3
4
−i
−2i
Dies entspricht der Addition von zwei Größen, die nicht nur eine Länge,
sondern auch eine Richtung (“Phase” genannt) haben; solche Größen
gibt es in der Tat in der Physik, vgl. Fußnote 68.73 Die geometrische Bedeutung der Multiplikation werden wir noch kennenlernen (Abschnitt
8).
Aber es gibt noch eine weitere Rechenoperation in den komplexen
Zahlen, die wir von den reellen Zahlen her nicht kennen: Die komplexe
Konjugation , die jeder komplexen Zahl x + yi die komplexe Zahl x − yi
zuordnet, diese also quasi an der x-Achse spiegelt. Wir bezeichnen sie
mit einem Querstrich:
z = x + yi 7→ z = x − yi .
Sie bietet keinerlei rechnerische Schwierigkeit; im Gegenteil wurde sie
eingeführt wegen ihrer spektakulären Recheneigenschaften, die viele
Rechnungen vereinfachen:
(31)
z ± w = z ± w,
z · w = z · w,
z/w = z/w.
Wie bei reellen gibt es auch bei komplexen Zahlen z = x + yi einen
Absolutbetrag (“Betrag”), der die Größe dieser Zahl unabhängig von
der Richtung misst:
p
√
(32)
|z| = x2 + y 2 = zz.
Nach Pythagoras (vgl. Fußnote 26) ist dies gerade die Länge der Strecke
von 0 nach z in der xy-Ebene. Damit wird eine enge Beziehung zwischen
den komplexen Zahlen und der Geometrie der Ebene begründet; jeden
73Man
kann sich fragen, ob es nicht auch Größen gibt, die eine räumliche Richtung haben; das müsste sich beim Addieren von drei von ihnen bemerkbar machen.
Solche Größen wurden in der Tat im 19. Jahrhundert unter dem Namen Vektor
eingeführt; man kann sie aber nur addieren und subtrahieren, nicht multiplizieren
und dividieren. Um diesen Mangel zu beheben, musste man eine weitere Dimension
hinzunehmen; siehe Abschnitt 10.
STERNSTUNDEN DER MATHEMATIK
57
Satz der ebenen Geometrie könnte man als Satz über komplexe Zahlen
ausdrücken und umgekehrt.74 Rein rechnerisch erfüllt der Betrag einer
komplexen Zahl dieselben Rechenregeln wie der einer reellen Zahl:
Satz.
(33)
(34)
|z + w| ≤ |z| + |w|
|z| · |w| = |zw|.
Beweis. Obwohl (33) viel anschaulicher ist, ist die zweite Gleichung
(34) einfacher zu beweisen: |z|2 |w|2 = zzww = z w z w = zwzw =
|zw|2 mit (31). Die erste Gleichung (33) folgt, weil Re z ≤ |z| für jede
komplexe Zahl z, da
p
√
|z| = x2 + y 2 ≥ x2 = |x| ≥ x = Re z ,
und insbesondere
Re (zw) ≤ |zw| = |z||w| = |z||w|.
(∗)
Andererseits ist75
(∗∗)
2 Re (zw) = zw + zw = zw + zw ,
denn für w = u + vi ist w = u − vi = u + vi = w. Daher gilt:
|z + w|2
= (z + w)(z + w)
= zz + zw + wz + ww
(∗∗)
= |z|2 + 2 Re (zw) + |w|2
(∗)
|z|2 + 2|z||w| + |w|2
(|z| + |w|)2 .
≤
=
Gleichung (33) heißt Dreiecksungleichung, denn sie sagt, dass im
Dreieck ∆(0, z, z + w) eine Seite kürzer ist als die beiden anderen zusammen; der direkte Weg von 0 nach z + w ist kürzer als der Umweg
über z.
|w|
z+w
z
|z|
|z+w|
0
74Ein
75
Beispiel hierfür bietet die Übungsaufgabe 6.3.
1
(z − z̄)
Für jedes z ∈ C gilt Re z = 21 (z + z̄) und Im z = 2i
58
J.-H. ESCHENBURG
Übungen
6.1. Kubische Gleichung, Casus Irreducibilis. Lösen Sie die kubische Gleichung
x3 − 9x = 10
mit Hilfe der Cardanoschen
(22). Hinweis: Berechnen Sie zuvor
√ Formel
3
die komplexe Zahl (1 ± 2 i) .
6.2. Beispiel von Bombelli: Lösen Sie die kubische Gleichung
x3 − 15x = 4.
Berechnen Sie zuvor (2 ± i)3 .
6.3. Multiplikation mit i und Jim Knopfs Vergesslichkeit. Jim
Knopf hat seine Weihnachtsgeschenke vergraben, um sie vor der Wilden 13 in Sicherheit zu bringen und ins Neue Jahr hinüberzuretten.
Damit er sie wiederfindet, hat er sich den Ort aufgeschrieben: “Gehe vom Schneemann S zum Weihnachtsbaum W, von da im rechten
Winkel gleichviele Schritte nach links und zünde dort eine Kerze K1
an. Gehe danach vom Schneemann zum Glühweinstand G, von da im
rechten Winkel gleichviele Schritte nach rechts und zünde dort eine
zweite Kerze K2 an. Die Geschenke sind bei X genau in der Mitte
zwischen den beiden Kerzen vergraben.” Die Verfolgung der Wilden
13 zieht sich jedoch etwas hin, und erst nach Ostern kehrt Jim wieder nach Lummerland zurück. Er findet den Weihnachtsbaum und die
Reste des Glühweinstandes, aber den Ort des (längst geschmolzenen)
Schneemanns hat er falsch in Erinnerung. Trotzdem findet er die Geschenke nach der Anleitung sofort wieder - können Sie das erklären?
Fassen Sie die Punkte der Ebene als komplexe Zahlen auf ! Die Drehung um 90 Grad nach links ist die Multiplikation mit i. Also z.B.
K1 = W + i(W − S).76
K2
X
K1
G
W
S
76Diese
Aufgabe verdanke ich meinem früheren Doktoranden Robert Bock.
STERNSTUNDEN DER MATHEMATIK
59
7. Pascal: Gott würfelt nicht, aber der Mensch (1654)
“Gott würfelt nicht”, das ist ein Zitat, das Albert Einstein77 zugeschrieben wird.78 Einstein konnte sich nicht mit der Deutung der Quantenmechanik abfinden, gemäß der die Physik prinzipiell Ereignisse nicht
sicher vorhersagen kann, sondern nur eine Wahrscheinlichkeitsverteilung für ihr Eintreten bestimmt; die Wahrscheinlichkeit wurde damit
zu einem Grundbegriff der Physik. Als Begründer einer mathematischen Theorie der Wahrscheinlichkeit gilt Blaise Pascal; er entwickelte
diese anhand von Problemen, die mit Würfelspielen zusammenhingen.
Auch er hatte einen Bezug zu Gott, einen ganz anderen als Einstein,
und in der berühmten Pascalschen Wette brachte er ihn sogar mit der
Theorie der Wahrscheinlichkeiten zusammen; auch davon soll am Ende
kurz die Rede sein.
Blaise Pascals Vater Étienne Pascal war Richter am obersten Steuergerichtshof in Clermont-Ferrand; später verkaufte (!) er dieses Amt an
seinen Bruder und zog nach Paris. Der etwas kränkliche Blaise wurde
von seinem Vater und von Hauslehrern unterrichtet, die Beschäftigung
mit Mathematik war ihm jedoch verboten - eine wunderbare Methode, um Interesse zu wecken. Als er mit 12 Jahren ohne jede Anleitung
die Winkelsumme im Dreieck fand, wurde dieses Verbot aufgehoben; er
durfte den Euklid studieren. Mit 14 Jahren wurde er von seinem Vater
in den Kreis des Theologen und Mathematikers Mersenne79 eingeführt,
ein Zentrum für einige der besten Köpfe im damaligen Paris. Dort
lernte er u.a. Desargues kennen und bewundern, und mit 16 schrieb
er seine erste Arbeit über projektive Geometrie (Satz von Pascal) (siehe Aufgabe 4.4, Seite 40) und präsentierte sie diesem illustren Kreis.
1639 zog die Familie nach Rouen, wo Étienne Pascal eine Stelle als
Steuereinnehmer in der Normandie antrat. Um seinem Vater die Arbeit
zu erleichtern, arbeitete Blaise Pascal zwischen 1642 und 1645 an der
Konstruktion einer Rechenmaschine, der Pascaline,80 von der bis 1652
77Albert
Einstein, 1879 (Ulm) - 1955 (Princeton)
Quantenmechanik ist sehr achtunggebietend. Aber eine innere Stimme
sagt mir, daß das noch nicht der wahre Jakob ist. Die Theorie liefert viel, aber dem
Geheimnis des Alten bringt sie uns kaum näher. Jedenfalls bin ich überzeugt, daß
der nicht würfelt.” (aus einem Brief an Max Born vom 4.12.1926)
79Marin Mersenne, 1588 - 1648 (Paris), besonders bekannt durch die Mersenneschen Primzahlen: Das sind Primzahlen der Gestalt 2p − 1; wenn eine solche Zahl
prim ist, muss auch p eine Primzahl sein, denn (2m )n − 1 wird von 2m − 1 geteilt. Der Umkehrschluss ist jedoch falsch: 211 − 1 = 2047 = 23 · 89. Die größte
bekannte Primzahl (Stand von 2013) ist vom Mersenneschen Typ: 257.885.161 − 1.
http://de.wikipedia.org/wiki/Mersenne-Primzahl
80
http://www.youtube.com/watch?v=3h71HAJWnVU
78“Die
60
J.-H. ESCHENBURG
etwa 50 Exemplare gebaut wurden; 9 von ihnen sind noch erhalten.81
Geschäftlich gesehen war das Unternehmen aber ein Fehlschlag, nur
wenige Exemplare der Pascaline konnten verkauft werden. 1647 kehrte
die Familie nach Paris zurück. Blaise Pascal beschäftigte sich mit Experimenten zum Luftdruck und wies insbesondere die Höhenabhängigkeit
des Luftdrucks experimentell nach.82 Dahinter stand auch die philosophische Frage nach der Existenz des Vakuums, an das z.B. Descartes83
nicht glauben konnte, aber Pascals Messungen wiesen auf das Gegenteil
hin, denn der Luftdruck nahm mit der Höhe ab.
1651 starb der Vater Étienne Pascal. Blaise Pascal war nun vermögend
und wandte sich eine Weile lang der eleganten Welt von Paris zu. Dort
lernte er auch den “Chevalier de Méré” kennen,84 einen Schriftsteller
und leidenschaftlichen Spieler. Der wunderte sich über ein Phänomen,
das er bei seinen Spielen beobachtet hatte: Bei einem Würfelspiel mit
einem Würfel liegt die Chance, mindestens einmal eine Sechs zu würfeln,
über 50 Prozent, wenn man mindestens viermal würfeln darf. Wenn
man dagegen mit zwei Würfeln mit gleicher Chance mindestens eine
Doppelsechs werfen möchte, dann benötigt man mehr als die sechsfache Zahl von Würfen, obwohl die Chance für jeden einzelnen Wurf im
zweiten Fall genau ein Sechstel von der im ersten Fall ist.85 Das hielt er
laut Pascal für einen großen Skandal und folgerte, dass die Arithmetik
falsch sein müsse. Pascal kommentiert: “Car il a très-bon esprit, mais il
n’est pas géomètre, c’est, comme vous savez, un grand défaut”.86 Vermutlich ist diese Kennzeichnung etwas ungerecht, denn de Méré kannte
81
Damit ist Pascal wohl der erste, der eine funktionierende Rechenmaschine wirklich gebaut hat. Vor Pascal hatte 1623 Wilhelm Schickard, 1592 (Herrenberg bei
Tübingen) - 1635 (Tübingen), eine “Rechenuhr” konstruiert, an der Kepler großes
Interesse zeigte; leider wurde sie vor ihrer Fertigstellung durch Feuer zerstört. Sie
wurde 1984 nach Schickards Aufzeichnungen nachgebaut und erwies sich als funktionsfähig. Pascal hatte vielleicht davon gehört, aber niemand wusste damals, ob
Schickards Konstruktion tatsächlich funktionierte.
82
Die Druckeinheit Pascal (Pa) erinnert an diese Leistung. Der Luftdruck wird
heute in Hektopascal (1 hPa = 100 Pa) gemessen; ein Hektopascal entspricht in der
älteren Bezeichnung genau einem Millibar. Der Luftdruck auf Meereshöhe beträgt
im Mittel etwas mehr als 1000 hPa = 1 bar.
83
René Descartes (Cartesius), 1596 (Touraine) - 1650 (Stockholm)
84
Richtiger Name: Antoine Gombaud, 1607 - 1684
85
Siehe Übungsaufgaben 7.1 und 7.2.
86
“Denn er ist von sehr gutem Verstand, aber er ist kein Geometer, was, wie Sie
wissen, ein großer Fehler ist.” Oeuvres de Blaise Pascal, Tome Quatrième, Paris
1819, Seite 367, Brief von Pascal an Fermat vom 29. Juli 1654,
http://books.google.de/books?id=N7EGAAAAQAAJ
Englische Übersetzung: http://www.york.ac.uk/depts/maths/histstat/pascal.pdf
STERNSTUNDEN DER MATHEMATIK
61
sehr wohl das obige Ergebnis, er konnte nur nicht verstehen, warum die
Proportionalität nicht galt. Darauf hat Pascal eigentlich keine Antwort
gegeben; erst de Moivre87 hat diese Frage in seinem 1718 erschienenen
Buch “The Doctrine of Chances” behandelt.
De Méré stellte noch eine zweite Frage, die Pascal wesentlich intensiver beschäftigte und zu einem langen Briefwechsel mit Fermat88 führte.
Es kam häufiger vor, dass Glücksspiele abgebrochen werden mussten,
bevor noch ein Sieger feststand. Sollte dann jeder Mitspieler einfach seinen Einsatz zurückbekommen, oder gab es eine Möglichkeit, die bisher
schon erzielten Ergebnisse jedes Spielers zu berücksichtigen? Im einfachsten Fall werfen zwei Spieler immer wieder eine Münze und wetten
auf unterschiedliche Seiten der Münze (Kopf oder Zahl). Gewonnen hat,
wer als erster eine vorher festgesetzte Anzahl von Runden gewonnen
hat. Wenn bei Abbruch des Spieles dem einen Spieler noch 2 gewonnene Runden, dem anderen aber 4 fehlen, welchen Anteil am Einsatz
sollen beide Spieler bekommen? Pascal schreibt dazu:89
“Das Geld, welches die Spieler aufs Spiel gesetzt haben,
gehört ihnen nicht mehr, denn sie haben diesen Besitz
aufgegeben; doch im Gegenzug haben sie das Recht erworben, zu erwarten, was ihnen der Zufall entsprechend
den vorher vereinbarten Bedingungen bereithält. Da es
jedoch ein freiwilliges Gesetz ist, können sie es nach Belieben auflösen und das Spiel verlassen, egal an welcher
Stelle es sich befindet; und so, wie sie beim Eintritt
in das Spiel das Recht auf Zufall erwarben, können sie
nun die Erwartung des Zufalls eintauschen, so dass jeder
Mitspieler wieder in den Besitz einer gewissen Summe
tritt. In diesem Fall muss die Regelung dessen, was jedem einzelnen Mitspieler zukommen soll, so festgelegt
werden, dass die Summe im Verhältnis zu dem Anrecht
steht, das sie auf die Gewinnaussicht haben, so dass es
für jeden Mitspieler vollkommen gleich ist, entweder das
zu nehmen, was man ihm zuteilt, oder das Glücksspiel
87
Abraham de Moivre, 1667 (Vitry-le-François) - 1754 (London)
Pierre de Fermat, 1607 - 1665 (Castres bei Toulouse)
89
“Usage du Triangle Arithmetique pour determiner les partys qu’on doit faire
entre deux Joueurs qui jouent en plusieurs parties”,
http://gallica.bnf.fr/ark:/12148/btv1b86262012/f27.image
Siehe auch Jaques Attali: Blaise Pascal. Biographie eines Genies, Klett-Cotta, Stuttgart 2006, Seite 173.
88
62
J.-H. ESCHENBURG
fortzusetzen: Diese gerechte Wiederaufteilung nennt sich
Teilung (parti).”
Doch wie berechnet man die Gewinnaussichten jedes einzelnen Spielers?
Pascal verwendet stillschweigend eine Definition der Gewinnchance, die
wohl erst 1812 durch Laplace formuliert worden ist:90
Die Gewinnchance ist (bei gleicher Chance für jeden
Ausgang) das Verhältnis der Anzahl der günstigen Ausgänge zur Anzahl aller möglichen Ausgänge des Spiels.
Wie hoch ist zum Beispiel die Chance, bei n Münzwürfen genau kmal zu gewinnen? Das Spiel besteht aus n Münzwürfen; die möglichen
Ausgänge sind daher alle Kopf-Zahl-Sequenzen (oder 0-1-Sequenzen)
der Länge n, bei n = 3 also die acht Sequenzen 111, 110, 101, 100,
011, 010, 001, 000. Da auf jedem Platz der Sequenz die Entscheidung
zwischen 0 und 1 (Kopf oder Zahl) fällt, ist die Anzahl der möglichen
Ausgänge bei einer Runde gleich 2 (Sequenzen 1 und 0), bei zwei Runden gleich 4 (Sequenzen 11, 10, 01, 00), bei drei Runden 8 usw., bei n
Runden also 2n . Unter denen sind die günstigen Ausgänge diejenigen
Sequenzen, die an genau k Stellen eine 1 stehen haben, für n = 3 und
k = 2 zum Beispiel die Sequenzen 110, 101, 011. Für die Anzahl dieser
Sequenzen wurde später das Symbol ( nk ) (“n über k”) eingeführt:
( nk ) = Anzahl der 0-1-Sequenzen der Länge n mit genau k Einsen.
Offensichtlich gibt es für k = 0 und k = n je genau eine solche Sequenz,
nämlich 00...0 und 11...1, also ist ( n0 ) = ( nn ) = 1. Außerdem gilt die
Rekursionsformel
(35)
n
n
( n+1
k ) = ( k−1 ) + ( k ) ,
denn die “günstigen” Sequenzen (genau k Einsen) der Länge n + 1
teilen sich auf in solche, die mit 1 enden und solche die mit 0 enden.
Die auf 1 endenden haben noch genau k − 1 weitere Einsen auf Plätzen
n
) Stück. Die auf 0 endenden
zwischen 1 und n, davon gibt es also ( k−1
Sequenzen dagegen entsprechen den günstigen Sequenzen der Länge n,
von denen es ( nk ) Stück gibt.
90
Pierre Simon de Laplace, 1749 (Beaumont-en-Auge, Normandie) - 1827 (Paris),
Théorie Analytique des Probabilités (1812)
STERNSTUNDEN DER MATHEMATIK
63
Daraus lassen sich die Zahlen ( nk ) rekursiv berechnen. Wenn wir
nämlich eine Tabelle nach folgendem Muster anlegen,
(20 )
(00 )
(10 )
(21 )
(11 )
(22 )
...
(n0 )
n
(k−1
)
...
...
(n+1
k )
(nk )
(nn )
...
...
dann steht an beiden Rändern eine Eins, da ( n0 ) = ( nn ) = 1, und jede
Zahl im Inneren ist nach (35) die Summe der beiden schräg über ihr
stehenden Zahlen:
1
1
1
1
(36)
1
1
2
3
4
5
1
1
3
6
10
1
4
10
1
5
1
...
Pascal führt für das Dreieck (36) den Namen “arithmetisches Dreieck”
ein.91 Heute wird es Pascalsches Dreieck genannt, obwohl es bereits
Omar Khayyam, Tartaglia und Cardano bekannt war. Aber erst Pascal
hat seine Eigenschaften systematisch beschrieben.92
Damit kann Pascal nun das eingangs gestellte Problem lösen, welche
Anteile vom Einsatz den zwei Spielern A und B zustehen, denen noch 2
bzw. 4 gewonnene Runden fehlen.93 Nach spätestens 5 weiteren Runden
ist das Spiel beendet; die möglichen Spielausgänge sind also die 0-1Sequenzen der Länge 5. Die für A günstigen sind die mit 2, 3, 4 oder 5
Einsen; die für B günstigen sind die verbleibenden Sequenzen mit 0 oder
1 Einsen, also 5 oder 4 Nullen. Die Aufteilung des Einsatzes geschieht
also im Verhältnis ( 52 ) + ( 53 ) + ( 54 ) + ( 55 ) = 10 + 10 + 5 + 1 = 26 zu
( 50 )+( 51 ) = 1+5 = 6, also im Verhältnis 13:3 zugunsten des Spielers A.
Übrigens kam Fermat mit anderen Methoden zum gleichen Ergebnis.
91http://gallica.bnf.fr/ark:/12148/btv1b86262012/f6.image
92
“Traité du triangle arithmetique” von 1654, erschienen posthum 1665.
http://gallica.bnf.fr/ark:/12148/btv1b86262012/
Englische Übersetzung:
http://www.cs.xu.edu/math/Sources/Pascal/Sources/arith triangle.pdf
93http://gallica.bnf.fr/ark:/12148/btv1b86262012/f33.image
64
J.-H. ESCHENBURG
Neben der Rekursionsformel gab Pascal auch eine direkte Formel zur
Berechnung der Zahlen ( nk ) an:94
(37)
( nk ) =
n · (n − 1) · . . . · (n − k + 1)
n!
=
.
1 · 2 · ... · k
k! · (n − k)!
Diese Formel sieht man einfacher mit Hilfe einer etwas anderen Vorstellung von ( nk ). Wir stellen uns vor, dass die Nummern der Plätze in
einer 0-1-Sequenz, auf denen eine Eins steht (die “guten” Plätze), aus
einer Trommel gezogen werden, die jede Zahl von 1 bis n genau einmal
enthält. So geschieht es zum Beispiel beim Lotto 6 aus 49. Zu jeder Ziehung gehört eine 0-1-Sequenz und umgekehrt. Zum Beispiel entspricht
die Ziehung der Zahlen 3, 15, 22, 28, 34, 47 der 0-1-Sequenz mit Länge
49, die nur auf den Plätzen mit den Nummern 3, 15, 22, 28, 34, 47
eine Eins stehen hat und sonst lauter Nullen. Wenn ich einen Sechser
gewinnen möchte, muss ich genau diese Zahlen auf meinem Lottoschein
angekreuzt haben. Unter den ( 49
6 ) möglichen Sequenzen mit genau 6
Einsen gibt es also nur eine einzige günstige; meine Gewinnchance ist
demnach 1/ ( 49
6 ).
Aber ich kann auch anders argumentieren: Eine Kugel nach der anderen wird gezogen. Für die Ziehung der ersten gibt es 49 Möglichkeiten,
für die zweite noch 48, weil eine Kugel ja schon fehlt, für die dritte 47, für die vierte 46, für die fünfte 45 und für die sechste noch 44
Möglichkeiten. Insgesamt gibt es also 49·48·47·46·45·44 mögliche Ziehungen. Welche von denen sind für mich günstig, wenn ich die obigen
Zahlen getippt habe? Nur die, wo genau diese Zahlen gezogen werden, allerdings in beliebiger Reihenfolge. Für die erste Kugel gibt es
also nur noch 6 Möglichkeiten (3, 15, 22, 28, 34, 47), für die zweite dann noch 5, weil eine Zahl ja schon vergeben ist, für die dritte 4, die vierte 3, die fünfte 2 und die sechste ist dann festgelegt.
Die Zahl der günstigen Ziehungen ist also 6 · 5 · 4 · 3 · 2 · 1 = 6!
(“Sechs-Fakultät”)95 und die Gewinnchance somit der Kehrwert von
49 · 48 · 47 · 46 · 45 · 44/6! = 49 · 47 · 46 · 3 · 44 = 13 983 816. Die gleiche
Überlegung für n und k statt 49 und 6 ergibt die Formel (37).96
Pascal gibt noch eine weitere Anwendung seines “arithmetischen Dreiecks”, die im nächsten Abschnitt unserer Vorlesung eine Rolle spielen
94http://gallica.bnf.fr/ark:/12148/btv1b86262012/f17.image
95n!
ist das Produkt der ganzen Zahlen von 1 bis n.
wir den mittleren Ausdruck von (37) mit (n − k)! erweitern, ergibt sich
der rechte Ausdruck.
96Wenn
STERNSTUNDEN DER MATHEMATIK
65
wird, nämlich die allgemeine binomische Formel:97
n
(38) (a + b) =
n
X
( nk ) an−k bk
k=0
= an + nan−1 b + ( n2 ) an−2 b2 + ( n3 ) an−3 b3 + · · · + bn .
Zur Begründung dieser Formel multipliziert man aus, lässt aber in jedem Summanden die Reihenfolge der Faktoren zunächst unverändert:
(a + b)2 =
=
=
3
(a + b) =
=
=
=
(a + b)(a + b)
(a + b)a + (a + b)b
aa + ba + ab + bb ,
(a + b)2 (a + b)
(aa + ba + ab + bb)(a + b)
(aa + ba + ab + bb)a + (aa + ba + ab + bb)b
aaa + baa + aba + bba + aab + bab + abb + bbb .
Man sieht das Bildungsgesetz: (a+b)2 ist die Summe aller a-b-Sequenzen
der Länge 2. Bei (a + b)3 = (a + b)2 (a + b) wird an alle diese Zweiersequenzen noch jeweils ein a oder ein b angehängt, und so entstehen
alle a-b-Sequenzen der Länge 3. Ebenso98 ist (a + b)n die Summe aller
a-b-Sequenzen der Länge n. Unter denen gibt es ( nk ) Sequenzen,
in denen genau k-mal b und (n − k)-mal a vorkommt, und jede von
ihnen hat unabhängig von der Reihenfolge den Wert an−k bk . Das ergibt
(38).
Wegen ihrer Rolle als Koeffizienten in der binomischen Formel99 heißen die Zahlen ( nk ) Binomialkoeffizienten.
Pascal hatte also im Sommer des Jahres 1654 herausgefunden, was ein
abgebrochenes Spiel wert war und hatte seine dabei gewonnenen Erkenntnisse auf verschiedene andere Gebiete angewandt. Nachdem er in
der Nacht von 23. November 1654 eine Art religiöse Vision erlebte,100
widmete er sich mit gleicher Intensität der Frage, was ein (durch den
97
“Usage du triangle arithmetique pour trouver les puissances des Binomes et
Apotomes”, http://gallica.bnf.fr/ark:/12148/btv1b86262012/f40.image
98Genau genommen ist das eine vollständige Induktion über n. Wir haben den
Induktionsschluss n → n+1 für den Fall n = 2 ausgeführt und damit das allgemeine
Schema gezeigt: Wenn man an alle a-b-Sequenzen der Länge n jeweils ein a oder
ein b anhängt, erhält man alle a-b-Sequenzen der Länge n + 1.
99Ein Binom ist eine Potenz einer Summe mit zwei Summanden.
100In seinem “Mémorial”, das er von da an stets bei sich trug, schrieb er dazu:
“Feuer. ‘Gott Abrahams, Gott Isaaks, Gott Jakobs’, nicht der Philosophen und
Gelehrten. Gewissheit, Gewissheit, Empfinden: Freude, Friede. Gott Jesu Christi.”
66
J.-H. ESCHENBURG
Tod) abgebrochenes Leben wert war. Berühmt geworden ist das Fragment Nr. 233 aus seinen um 1660 verfassten “Pensées”,101 in dem es
um die “Pascalsche Wette” geht. Diese betrifft keine geringere Frage
als die Existenz Gottes. Die Entscheidung ist für Pascal klar: “Wettet
also, ohne zu zögern, dass Gott ist ... Wenn ihr gewinnt, so gewinnt ihr
alles, wenn ihr verliert, so verliert ihr nichts”.
Man kann Pascals Überlegung vielleicht so zusammenfassen: Es gibt
zwei Möglichkeiten:
A: Gott existiert.
B: Er existiert nicht.
Wir wissen nicht, ob A oder B stimmt. Nach Pascal müssen wir uns
dennoch für A entscheiden: Wenn A stimmt, ist der ewige Lohn für ein
gottesfürchtiges Leben unermesslich hoch, wenn aber B stimmt und wir
auf A gesetzt haben, entgehen uns lediglich die unvergleichlich viel kleineren möglichen Gewinne eines zeitlich begrenzten gottlosen Lebens.
Man könnte die Pascalsche Wette als “best-case”-Denken bezeichnen:
Gott existiert; was kann uns also noch passieren? Wir dürfen handeln,
als wüssten wir, dass die Welt im Kern gut ist. Die Bedeutung dieses
Gedankens sieht man am besten durch seine Umkehrung, die zu dem
uns viel vertrauteren “worst-case”-Denken führt.
Stellen wir uns eine sehr große drohende Gefahr vor. Wir glauben, dass
ihre Folgen im Fall des Eintreffens vernichtend sein werden. Wieder
gibt es zwei Hypothesen:
A: Die Gefahr wird eintreten.
B: Sie wird nicht eintreten.
Wir wissen nicht, ob A oder B stimmt. Aber wieder fällt die Entscheidung für A aus: Wenn A stimmt, können wir durch unser Handeln einen
unermesslichen Verlust abwenden, wenn aber B stimmt und wir auf A
gesetzt haben, müssen wir nur begrenzte Verluste (“Nebenwirkungen”)
in Kauf nehmen. Wir gelangen also zur Entscheidung, dass wir uns mit
allen uns zur Verfügung stehenden Mitteln der Gefahr entgegenstemmen müssen, als ob ihr Eintreffen sicher wäre. Je schlimmer die Gefahr
ist, je größer wir sie uns vorstellen, desto weniger Skrupel dürfen wir
bei der Wahl der Mittel haben, mit denen wir sie bekämpfen.
Dieses Argument ist im technischen Bereich unerlässlich; wir müssen
eine Brücke vor ihrem Bau durchrechnen, um zu vermeiden, dass ein
Sturm sie zerbricht. Aber wenn die drohende Gefahr von Menschen
101http://abu.cnam.fr/cgi-bin/go?penseesXX1,318,337
engl. http://www.gutenberg.org/files/18269/18269-h/18269-h.htm
STERNSTUNDEN DER MATHEMATIK
67
ausgeht, wenn sie also mit Konflikten zwischen einzelnen Menschen,
Gruppen, Nationen oder Gesellschaften zu tun hat, kann dieses Denken
zu einer tödlichen Falle werden. Denn der Mensch hat die Fähigkeit, die
Gefahr, die von anderen Menschen ausgeht, mit unbedingter Sicherheit
abzuwenden, indem er den Tod dieser Menschen herbeiführt. Dem obersten Ziel der Abwendung der Gefahr ist alles andere unterzuordnen. Das
Böse ist dann nicht nur erlaubt, man ist sogar verpflichtet, es zu tun,
wenn es zur Erreichung des Ziels notwendig ist. Dieses Argument spielt
im politischen Diskurs eine große Rolle und hat (als “Nebenwirkung”)
die schlimmsten Katastrophen der Weltgeschichte herbeigeführt. Der
Erste Weltkrieg aus deutscher Sicht ist ein Beispiel.102 Aktueller ist
das Beispiel des Irakkriegs von 2003, von dessen Folgen sich der Nahe Osten nicht so bald erholen wird.103 Pascals Wette könnte man als
einen Versuch der Immunisierung gegen diesen Irrweg verstehen.
Übungen
7.1. de Mérés Beobachtung (1). Zeigen Sie, dass es bei 4 Würfen
mit einem Würfel weniger Fälle gibt, bei denen keine 6 fällt, als solche,
bei denen mindestens einmal eine 6 geworfen wird. Die Zahlen sind 625
sowie 1296 − 625 = 671.
102Der
deutsche Generalstabschef Helmuth von Moltke war überzeugt, dass gegen Russland ein Präventivkrieg geführt werden müsse. Zu Außenminister Jagow
sagte er im Mai 1914, “in 2 bis 3 Jahren werde Russland seine Rüstungen beendet haben. Die militärische Übermacht unserer Feinde wäre dann so groß, dass
er nicht wüsste, wie wir ihrer Herr werden könnten. Es blieb seiner Ansicht nach
nichts übrig, als einen Präventivkrieg zu führen, um den Gegner zu schlagen, solange wir den Kampf noch einigermaßen bestehen könnten. Der Generalstabschef
stellte mir (Jagow) demgemäß anheim, unsere Politik auf die baldige Herbeiführung
eines Krieges einzustellen.“ (aus dem Nachlass Jagow des Auswärtigen Amts, zitiert nach: Fritz Fischer, Griff nach der Weltmacht, Düsseldorf 2013, S. 50). Eine
Angriffsabsicht der Entente gegen Deutschland gab es nicht, es lag also “Fall B”
vor. Die Wette auf Fall A hat das ganze Elend des 20. Jahrhunderts herbeigeführt.
103Am 28. Januar 2003 sprach George W. Bush “zur Lage der Nation”
(www.presidentialrhetoric.com/speeches/bushpresidency.html):
“Stellen Sie sich diese 19 Flugzeugentführer [des 11. Septembers] mit anderen Waffen und anderen Plänen vor, dieses Mal von Saddam Hussein ausgerüstet. Genügen
würden eine Ampulle, ein Kanister, eine Kiste, ausgeschüttet in diesem Land, um
uns einen Tag des Schreckens zu bringen, wie wir noch keinen je erlebt haben. Wir
werden alles in unserer Macht Stehende tun, um sicher zu stellen, dass jener Tag
niemals kommen wird.” Wir wissen, dass “Fall B” vorlag; die Wette auf A aber hat
millionenfaches Leid über die Welt und besonders den Nahen Osten gebracht, und
ein Ende ist nicht absehbar.
68
J.-H. ESCHENBURG
7.2. de Mérés Beobachtung (2). Zeigen Sie, dass die Wahrscheinlichkeit, bei 24 Würfen mit zwei Würfeln keine Doppelsechs zu werfen,
bei (35/36)24 = 50,86 Prozent liegt.
7.3. Gerechte Aufteilung nach Pascal. “Beispielsweise nimmt man
an, dass zwei Spieler A und B einen Einsatz von jeweils 32 Fr. leisten
und vereinbaren, eine symmetrische Münze zu werfen, wobei im Falle des Resultats ‘Kopf’ Spieler A einen Punkt erhält, im Falle von
‘Zahl’ dagegen Spieler B. Außerdem macht man aus, dass derjenige
Spieler, der zuerst 7 Punkte erreicht, den gesamten Einsatz von 64 Fr.
bekommt. Aus einem bestimmten Grund muss das Spiel nun jedoch
unterbrochen werden in der Situation, dass Spieler A über 5 Punkte
und Spieler B über 4 Punkte verfügt, und es stellt sich die Frage, wie
der Einsatz gerecht aufgeteilt werden soll.”104 Zeigen Sie, dass der Einsatz im Verhältnis von 11:5 aufzuteilen ist, d.h. Spieler A erhält 44 Fr.
und Spieler B bekommt 20 Fr.
7.4. Binomialkoeffizienten. Wir können die Binomialkoeffizienten
( nk ) auch durch die Formel (38) definieren. Zeigen Sie (35) mit dieser Definition.
7.5. Binomische Formel (1). Zeigen Sie mit der binomischen Formel
für (1 ± 1)n , dass gilt:
n
n
P
P
(a)
( nk ) = 2n , (b)
(−1)k ( nk ) = 0.
k=0
k=0
Die Summe der Binomialkoeffizienten ist 2n , ihre Wechselsumme ist
Null. Das erste Ergebnis überrascht uns nicht; wieviele 0-1-Sequenzen
der Länge n gibt es nämlich insgesamt?
7.6. Binomische Formel (2). Berechnen Sie 118 = (10 + 1)8 mit
Hilfe der binomischen Formel.
8. Gauß: Der Fundamentalsatz der Algebra (1799)
Der Fundamentalsatz der Algebra besagt, dass nicht nur quadratische, kubische und quartische Gleichungen eine Lösung haben, sofern
man komplexe Zahlen zulässt, sondern überhaupt alle Gleichungen von
ganz beliebigem Grad n:
(39)
104Alexander
xn + a1 xn−1 + a2 xn−2 + · · · + an = 0 ,
Odefey: Blaise Pascal, Seite 9
http://www.hs.uni-hamburg.de/DE/GNT/exk/pdf/pascal.pdf
STERNSTUNDEN DER MATHEMATIK
69
wobei auch die Koeffizienten a1 , . . . , an komplexe Zahlen sein dürfen.
Der erste Beweis dieses Satzes wurde 1746 von d’Alembert105 veröffentlicht, der Beweis enthielt jedoch Lücken, die erst 1806 durch Argand
geschlossen wurden. Vier Jahre später gab Euler einen Beweis, der aber
ebenfalls lückenhaft war. Der erste vollständige Beweis wird in der Dissertation von Gauß106 von 1799 gesehen. Aber von Bombelli bis Gauß
war ein weiter Weg, auf dem die binomische Formel von Pascal eine
nicht unwichtige Rolle spielten sollte.
Bevor man sich an eine so gewaltige Aufgabe wie die Lösung der allgemeinen Gleichung (39) wagen konnte, war die viel einfachere Gleichung
xn = a
(40)
zu lösen, also die n-te Wurzel zu ziehen. Für positive reelle Zahlen a war
das schon seit der Antike bekannt, aber nun sollte a eine beliebige komplexe Zahl sein. Diese Aufgabe wurde mit Hilfe der Exponentialfunktion
gelöst, die auf Jakob Bernoulli107 und Leonhard Euler108 zurückgeht.
Jakob Bernoulli, nach Pascal einer der Begründer der Wahrscheinlichkeitstheorie (“Ars Coniectandi”, erschienen 1713), beschäftigte sich unter anderem mit der Zinseszins-Rechnung. Bei einem jährlichen Zinssatz x würde ich am Ende eines Jahres das (1 + x)-fache meines zu
Jahresbeginn eingezahlten Kapitals ausgezahlt bekommen. Nun aber
will ich darauf bestehen, dass das Kapital nicht nur einmal, sondern nmal pro Jahr verzinst wird, natürlich mit dem entsprechend geringeren
Zinssatz nx für die kürzeren Perioden, und die Zinsen sollen jedesmal
zum Kapital geschlagen und in der nächsten Periode ebenfalls verzinst
werden. Dann erhalte ich am Ende des Jahres das (1 + nx )n -fache meines ursprünglichen Kapitals. Was passiert, wenn die Zahl n sehr groß
wird, gegen unendlich strebt? Das macht sicher keine Bank mit, aber
bei Wachstumsprozessen in der Natur wird jeder Zuwachs fast sofort
wieder zum weiteren Wachstum beitragen. Euler fand einen anderen
Weg, diesen Ausdruck zu berechnen, indem er den Binomischen Lehrsatz und die Berechnung von ( nk ) von Pascal verwendete:
n
1+
n
x n X n xk X n(n − 1) . . . (n − k + 1) xk
.
=
(k) k =
k
n
n
n
k!
k=0
k=0
105Jean-Baptiste
le Rond, genannt d’Alembert, 1717 - 1783 (Paris)
Carl Friedrich Gauß, 1777 (Braunschweig) - 1855 (Göttingen)
107Jakob Bernoulli, 1655 - 1705 (Basel)
108Leonhard Euler, 1707 (Basel) - 1755 (St. Petersburg)
106Johann
70
J.-H. ESCHENBURG
k
Der Vorfaktor von xk! in der letzten Summe hat gleich viele Faktoren
im Zähler und im Nenner:
n(n − 1) . . . (n − k + 1)
n n−1
n−k+1
= ·
· ... ·
.
k
n
n
n
n
Die rechte Seite ist offensichtlich kleiner als 1 und zudem sehr nahe bei
k
1, wenn k viel kleiner als n ist. Aber wenn k groß ist, dann wird xk!
sehr klein sein, deshalb haben diese Summanden auch gemeinsam nur
ein sehr kleines Gewicht. Somit erhalten wir die von Euler gefundene
Formel
∞
x n X xk
(41)
lim 1 +
=: exp(x) .
=
n→∞
n
k!
k=0
Den Ausdruck auf der rechten Seite nannte er die Exponentialfunktion von x, weil die Rechenregeln wie bei Exponenten sind, die ja bei
Multiplikation addiert werden: 23 · 22 = 23+2 . Ebenso konnte Euler für
beliebige x, y zeigen:
(42)
exp(x + y) = exp(x) exp(y).
Um dies zu zeigen, wird wieder
j m−j Formel
Pm (38) 1von Pascal
Pm die1 binomische
m
1
j m−j
x
y
=
(x + y)m =
verwendet: m!
j=0 j!(m−j)! x y
j=0 m! j
und damit
∞ X
m
X
xj y m−j
(43)
exp(x + y) =
.
j!
(m
−
j)!
m=0 j=0
Andererseits gilt aber auch
∞ m
∞ X
∞
X
xj y k (∗) X X xj y m−j
;
=
exp(x) exp(y) =
j!
k!
j!
(m
−
j)!
m=0 j=0
j=0 k=0
bei (∗) wurden die Summanden der Doppelsumme umgeordnet: Statt
über j und k wurde über j und m := j + k summiert und k durch m − j
ersetzt, also über die Diagonalen statt die horizontalen Reihen in der
folgenden Figur summiert (die schraffierten Bereiche sollen andeuten,
dass diese Summanden - auch gemeinsam - sehr klein sind und keine
wesentliche Rolle spielen).
k
N
2
1
0
0
1
2
N
j
STERNSTUNDEN DER MATHEMATIK
71
Euler hat der Zahl
∞
X
1 1
1
1
1
=2+ + +
+
+ · · · = 2,718281828 . . .
exp(1) =
k!
2
6
24
120
k=0
den Namen e gegeben. Dann ist exp(2) = exp(1 + 1) = exp(1)2 = e2
und entsprechend exp(n) = en . Deshalb schreibt man für alle Zahlen x
analog ex für exp x. Diese Funktion wird sehr schnell groß, da sie sich
ja aus positiven Potenzen zusammensetzt.
Was aber passiert, wenn wir für x imaginäre Zahlen x = it, t ∈ R
einsetzen? Das Ergebnis ist überraschend: Die Werte der Funktion t 7→
eit wachsen in keiner Richtung, sondern behalten für alle t den Betrag
Eins. Dazu müssen wir |eit |2 = eit · eit berechnen. Allgemein gilt für die
komplexe Konjugation:
X
X
xk /k! =
ex =
xk /k! = ex
k
k
für alle x ∈ C. Speziell für x = it mit t ∈ R ist x = −it, also eit = eit =
e−it und damit
|eit |2 = eit · eit = eit e−it = eit−it = e0 = 1.
(44)
Die komplexe Zahl eit liegt also für alle t ∈ R auf der Einheitskreislinie!
Die geometrische Bedeutung der Zahl t im Exponenten von eit ist
dabei der Winkel α zwischen 1 und eit , im Bogenmaß gemessen. Um
dies zu sehen, unterteilen wir den Bogen durch eine große Anzahl n
von Zwischenpunkten xk = eitk/n mit k = 0, . . . , n.
x3
x2
x1
x = e it
−1
1
0
Dann gilt
α ≈
n
X
k=1
|xk−1 −xk | =
n
X
|e
i(k−1)t/n
k=1
it/n
−e
ikt/n
e
− 1 n→∞
it/n
= n|1 − e | = |it| −→ |t| ,
it/n |=
n
X
k=1
|ei(k−1)t/n | |1−eit/n |
| {z }
=1
72
J.-H. ESCHENBURG
denn der für beliebige komplexe Zahlen x definierte Ausdruck
ex − 1
1
1 2 1 3
1
1
=
x + x + x + . . . = 1 + x + x2 + . . .
x
x
2!
3!
2!
3!
ist stetig und hat bei x = limn→∞ itn = 0 den Wert 1. Speziell bei t = π
haben wir gerade den Halbkreis durchmessen und erreichen die Zahl
−1; wir gelangen also zu der berühmten Eulerschen Beziehung
eiπ = −1 oder eiπ + 1 = 0 ,
die die wichtigsten Konstanten der Mathematik 0, 1, i, e, π in einer Gleichung zusammenfasst.
Nun können wir auch die komplexe Multiplikation geometrisch interpretieren: Wenn wir zwei komplexe Zahlen x = eit und y = eis auf
der Einheitskreislinie miteinander multiplizieren, so addieren sich die
beiden Winkel t und s:
x · y = eit · eis = ei(s+t) .
Wenn die Zahlen x und y dagegen beliebigen positiven Abstand vom
Nullpunkt haben, so liegen jedenfalls x/|x| und y/|y| auf der Einheitskreislinie und lassen sich demnach in der Form eis und eit darstellen,
also gilt
x · y = |x|eis · |y|eit = |x||y|ei(s+t) .
Die Beträge werden multipliziert, die Winkel addiert.
Insbesondere können wir jetzt aus einer beliebigen
komplexen Zahl a
p
n
it
eine n-te Wurzel ziehen: Für a = |a|e ist x := |a| · eit/n eine n-te
Wurzel von a, denn xn = |a| · eit = a.
e it
a= a e
it
_
√a e it/3
3
0
1
Nun können wir uns der Dissertation von Carl Friedrich Gauß und
dem Fundamentalsatz der Algebra zuwenden.109 Gauß war 22 Jahre
109Hubert Mania: Gauß. Eine Biographie. Rowohlt, Reinbeck bei Hamburg, 2008.
STERNSTUNDEN DER MATHEMATIK
73
alt, als er 1799 seine Dissertation110 an der Universität Helmstedt, der
Landesuniversität des Herzogtums Braunschweig einreichte. Das Resultat stammte eigentlich schon aus dem Jahre 1797, als Gauß mit einem
Stipendium seines Herzogs Carl Wilhelm Ferdinand111 in Göttingen
studierte (1795 - 1798). Das war eine besondere Gunst des Herzogs,
denn Göttingen lag im Kurfürstentum Hannover, also im “Ausland”.
Gauß hatte vor allem die Göttinger Universitätsbibliothek mit ihren
damals bereits 160 000 Bänden als Argument angeführt; vor allem
der Zugang zu den Veröffentlichungen der europäischen Akademien
ermöglichte ihm das Studium der Mathematik auf der Höhe seiner Zeit.
Dies trug sehr bald schon Früchte. Er studiert u.a. die algebraischen
Arbeiten von Lagrange, und bereits ein halbes Jahr nach seiner Immatrikulation in Göttingen findet er mit den Mitteln der Algebra, dass das
regelmäßige Siebzehneck mit Zirkel und Lineal konstruierbar ist; seit
2000 Jahren wurde kein solches Beispiel mehr gefunden. Diese Studien
führen Gauß bereits ab 1796 zu einer sehr viel umfangreicheren Arbeit,
den “Disquisitiones Arithmeticae”, deren Herzstück das quadratische
Reziprozitätsgesetz ist.112 Nebenher beschäftigt er sich mit vielen anderen Dingen, darunter auch mit dem Fundamentalsatz der Algebra.
Er liest die Arbeiten von d’Alembert und Euler und stellt sehr grundlegende Mängel darin fest, die er 1797 mit einem eigenen ganz neuen
Beweis beheben kann.
Nachdem 1798 das herzogliche Stipendium abgelaufen ist, kehrt Gauß
nach Braunschweig zurück und nimmt Kontakt mit dem Helmstedter
110“Demonstratio
nova theorematis omnem functionem algebraicam rationalem
integram unius variabilis in factores reales primi vel secundi gradus resolvi posse”
(deutsch: Neuer Beweis des Satzes, dass jede ganze rationale algebraische Funktion
in einer Variablen in reelle Faktoren ersten oder zweiten Grades zerlegt werden
kann). Gauß vermied die Erwähnung der komplexen Zahlen und formulierte sein
Ergebnis “reell”.
111Herzog Ferdinand war schon früh auf das junge Mathematikgenie aufmerksam gemacht worden und hatte ihm 1792 ein Stipendium für die beste Schule des
Landes, das Collegium Carolinum in Braunschweig ausgesetzt. Einer der von Gauß
verehrten Lehrer dort war ein entfernter Verwandter von mir, Johann Joachim
Eschenburg, 1743 (Hamburg) - 1820 (Braunschweig), Shakespeare-Übersetzer und
Freund Lessings; mit seinem Sohn Wilhelm Eschenburg (1778 - 1861) verband ihn
eine lebenslange Freundschaft.
112Dieses Gesetz handelt von Quadratzahlen modulo p. Zum Beispiel ist 13 eine
Quadratzahl (“quadratischer Rest”) modulo 17, denn 8 · 8 = 64 = 13 + 3 · 17 ≡
13 mod 17. Für zwei Primzahlen p und q sagt das Gesetz in den meisten Fällen:
p Quadratzahl modulo q ⇐⇒ q Quadratzahl modulo p.
Einzige Ausnahme: Wenn p+1 und q+1 beide durch 4 teilbar sind, ist q Quadratzahl
modulo p oder p Quadratzahl modulo q, aber beides zugleich kann nicht gelten.
74
J.-H. ESCHENBURG
Mathematiker Pfaff113 auf, dem er seine Dissertation vorlegen möchte.
Eigentlich möchte er die “Disquisitiones Arithmeticae” einreichen, aber
die Drucklegung verzögert sich immer wieder. So entschließt er sich auf
Anraten Pfaffs, eine überarbeitete Fassung seiner Studien zum Fundamentalsatz der Algebra einzureichen.114 Pfaff nimmt mehrfach Einfluss
auf die Arbeit und versucht (mit begrenztem Erfolg), sie “benutzerfreundlicher” zu gestalten. Er ist nicht nur der Doktorvater von Gauß,
sondern wird auch sein enger Freund; den Jahrhundertwechsel und die
Zeit bis Ostern danach verbringen sie zusammen in Helmstedt.
Die erste Hälfte der Dissertation setzt sich mit den Versuchen der
Vorgänger auseinander, d’Alembert, Euler und Lagrange. Gauß stellt
fest, dass sie implizit die Existenz einer Lösung x immer schon voraussetzen und nur noch zeigen, dass x dann von der Form u + vi für reelle
Zahlen u, v ist. Er selbst geht im zweiten Teil völlig neue Wege, um die
Existenz zu zeigen; sie führen aus der Algebra heraus in ein Gebiet, das
im 19. Jahrhundert “Analysis Situs” (Analyse der Lage) genannt wurde, heute heißt es Topologie. Gauß betrachtet zunächst den Ausdruck
auf der linken Seite der Gleichung (39),
f (x) = xn + a1 xn−1 + a2 xn−2 + · · · + an
(45)
für alle komplexen Zahlen x, gleich, welchen Wert f (x) hat. Statt einer
Lösung der Gleichung (39) sucht er damit eine Nullstelle der Funktion
f (x). Er zeigt zunächst, dass eine Kreislinie KR = {x : |x| = R}
mit großem Radius R von den Nullstellenmengen des Real- und des
Imaginärteils von f (x) immer abwechselnd geschnitten wird.
Im f = 0
Im f > 0
113
Im f = 0
Re f = 0
Im f < 0
Re f < 0
Im f < 0
Re f < 0
Re f = 0
Im f > 0
?
Im f > 0
Re f > 0
Re f > 0
Im f = 0
Re f < 0
Im f > 0
xo
L’
L
Re f = 0
Re f > 0
Im f = 0
Re f = 0
Im f < 0
Re f < 0
Im f < 0
Im f > 0
Johann Friedrich Pfaff, 1765 (Stuttgart) - 1825 (Halle an der Saale)
http://edoc.hu-berlin.de/dissertationen/historisch/gausscarolo/HTML/index.html
Englisch: http://archive.larouchepac.com/node/12482
Version zum Doktorjubiläum 1849:
http://reader.digitale-sammlungen.de/de/fs1/object/display/bsb10053475 00039.html
114
STERNSTUNDEN DER MATHEMATIK
75
Die Linien, auf denen Im f (x) = 0 gilt, müssen deshalb z.B. in das
Gebiet {x : Re f (x) > 0} eindringen und es auch wieder verlassen,
weil sie an anderen Kreispunkten enden. Deshalb müssen sie die Begrenzungslinie {x : Re f (x) = 0} dieses Gebietes kreuzen. An einem
solchen Kreuzungspunkt xo sind Re f (xo ) und Im f (xo ) gleichzeitig
Null, also ist xo eine Nullstelle von f . In unserer Beispielfigur links ist
f (x) = x2 + 1 mit Re f = u2 − v 2 + 1 und Im f = 2uv für x = u + vi.
Das Argument ist allerdings nicht ganz so einfach. Wir sehen ja an der
mit L bezeichneten gestrichelten Linie in der linken Figur, dass eine
Verbindungslinie vom Typ {Im f (x) = 0} durchaus ganz im Bereich
{Re f (x) < 0} bleiben kann. Die andere gestrichelte Linie dagegen
verlässt den Bereich {Re f (x) > 0}. Im Allgemeinen kennen wir die
Linien {Im f (x) = 0} nur auf der Kreislinie KR und im Außenbereich;
wie sie innerhalb des Kreises miteinander verbunden sind, wissen wir
nicht (rechte Figur). Dennoch können wir schließen, dass mindestens
ein Paar von benachbarten Kreispunkten vom Typ Im f = 0 (in der
Figur durch ungefüllte Kreise markiert) mit gestrichelten Linien verbunden sind. Eine gestrichelte Verbindungslinie L (mit Im f = 0 auf
L) zwischen zwei “ungefüllten” Punkten des Kreises teilt nämlich die
Kreisscheibe in zwei Teile (“Seiten”). Wenn auf jeder Seite noch weitere “ungefüllte” Punkte liegen, müssen diese untereinander verbunden
sein. Wir betrachten die Seite, auf der weniger (oder gleich viele) solcher Punkte liegen. Diese sind entweder untereinander verbunden oder
mit Punkten der anderen Seite; im letzteren Fall wird L von ihrer Verbindungslinie geschnitten. In beiden Fällen sind auch näher beieinander
liegende “ungefüllte” Punkte miteinander verbunden. Durch Wiederholung des Arguments folgt am Ende, dass ein Paar “ungefüllter” Nachbarpunkte verbunden ist; die Verbindung heiße L′ (siehe linke Figur).
Auf einer Seite von L′ und insbesondere auf dem Kreisbogen zwischen
den beiden Punkten hat Im f also keine Nullstellen mehr. Auf diesem Kreisbogen sitzt aber genau ein “gefüllter” Punkt (einer vom Typ
Re f = 0); dieser ist mit einem anderen “gefüllten” Punkt außerhalb
des durch L′ begrenzten Gebietes verbunden, und die Verbindungslinie
(wo Re f = 0) muss L′ (wo Im f = 0) schneiden.
Wie kommt Gauß aber zu der Aussage, dass eine Kreislinie KR mit
großem Radius abwechselnd von {x : Re f (x) = 0} und {x : Im f (x) =
0} geschnitten wird? Das sieht man durch Ausklammern der höchsten
Potenz xn in (45):
(46)
f (x) = xn 1 + a1 /x + a2 /x2 + · · · + an /xn .
76
J.-H. ESCHENBURG
Wenn |x| = R genügend groß ist, dann ist |ak /xk | = |ak |/Rk sehr klein
im Vergleich zu 1, und es gilt f (x) ≈ xn . Wenn x einmal den Kreis KR
durchläuft, dann durchläuft xn den Kreis vom Radius Rn genau n-mal,
da der Winkel immer mit n multipliziert wird. Die Ausdrücke Re f (x)
und Im f (x) sind die Projektionen dieser Bewegung auf die horizontale bzw. die vertikale Achse; sie wechseln das Vorzeichen, sobald xn
die jeweils andere Achse passiert. Dies geschieht abwechselnd für die
horizontale und die vertikale Achse, was zu zeigen war.
Heute wird dieses etwas komplizierte Argument durch den Begriff der
Umlaufszahl wesentlich vereinfacht, die aber (laut Grünbaum und Shephard)115 erst 1865 durch Möbius eingeführt wurde. Die Umlaufszahl
einer geschlossenen Linie in der komplexen Ebene, die nicht den Nullpunkt trifft, ist die Anzahl der Umrundungen des Nullpunktes bei einem ganzen Durchlauf der Linie. Wenn wir die geschlossene Linie stetig deformieren, dann bleibt die Umlaufszahl die gleiche, solange wir
nicht den Nullpunkt treffen. Dies wenden wir an auf die f -Bilder zweier
Kreislinien, KR = {x : |x| = R} für einen sehr großen Radius R und
Kr = {x : |x| = r} für einen sehr kleinen Radius r.
an
0
f(Kr)
f(Kρ)
f(KR)
Auf KR gilt f (x) ≈ xn , wie wir gesehen haben; damit ist die Umlaufszahl von f (KR ) gleich n, denn bei der n-ten Potenz wird der Kreis
KR n-mal durchlaufen. Auf Kr aber gilt f (x) ≈ an nach (45), denn
|ak xk | = |ak |rk ist sehr klein. Wir dürfen annehmen, dass an ungleich
Null ist, denn sonst haben wir schon eine Nullstelle gefunden: f (0) = 0.
Da sich f (Kr ) nahe bei an aufhält, liegt es in einem kleinen Winkelintervall und kann den Nullpunkt nicht umlaufen; die Umlaufszahl von
f (Kr ) ist also gleich 0. Wenn wir den Radius des Kreises langsam von r
nach R wachsen lassen, muss die Umlaufszahl bei irgend einem Radius
115http://www.ams.org/journals/tran/1990-322-01/S0002-9947-1990-1024774-
2/S0002-9947-1990-1024774-2.pdf
STERNSTUNDEN DER MATHEMATIK
77
ρ zwischen r und R springen; dort muss die Null also überschritten
werden, d.h. 0 liegt auf f (Kρ ), also f (xo ) = 0 für ein xo mit |xo | = ρ.
Bemerkung: Heutzutage ist der kürzeste Beweis des Fundamentalsatzes wohl folgender: Die Funktion f kann zu einer stetigen Funktion auf
Ĉ := C ∪ {∞} ausgeweitet werden, indem wir f (∞) = ∞ setzen; diese Hinzunahme des Punktes ∞ zur komplexen Ebenen führt von der
Zahlenebene zu einer Kugelfläche, der Riemannschen Zahlenkugel,116
siehe Seite 112. Bilder kompakter Mengen unter stetigen Abbildungen
sind kompakt, also ist f (Ĉ) kompakt. Andererseits ist f auch offen,
d.h. Bilder offener Mengen unter f sind offen.117 Somit ist f (Ĉ) auch
offen. Eine gleichzeitig offene und kompakte nichtleere Teilmenge von
Ĉ ist aber die ganze Menge Ĉ, also ist f (Ĉ) = Ĉ und insbesondere folgt
0 ∈ f (Ĉ).
Übungen
8.1. Negative Potenzen. Zeigen Sie exp(−x) = 1/ exp(x). Beachten
Sie exp(0) = 1.
8.2. Rationale Potenzen von e. Zeigen Sie für alle n ∈ N:
√
(a) exp(n) = en , (b) exp( n1 ) = n e
√
n m
und folgern Sie exp( m
e .
)
=
n
√
8.3. Bombellis Kubikwurzel. Bestimmen Sie 3 2 ± 2i (vgl. Seite
51) mit Hilfe der komplexen e-Funktion, siehe Seite 72.
8.4. Kubische Gleichung. Skizzieren Sie die entsprechende Figur zu
der auf Seite 74 für die Polynome f1 (x) = x3 − 6x − 9 und f2 (x) =
x3 − 6x − 4, vgl. Seite 51.
116Georg Friedrich Bernhard Riemann, 1826 (Breselenz bei Dannenberg, Elbe)
- 1866 (Selasca bei Verbania, Lago Maggiore)
117Zu zeigen ist: Das f -Bild jeder offenen Kreisscheibe B (x ) um jedes x enthält
ǫ o
o
eine offene Kreisscheibe Bδ (f (xo )) um f (xo ). Um dies zu sehen, schreiben wir f (x)
mit der Substitution x = (x − xo ) + xo als Polynom in x − xo und erhalten f (x) =
(x − xo )n + b1 (x − xo ) + · · · + bk (x − xo )k + bn für ein k ∈ {1, . . . , n − 1}, wobei bk 6= 0
und bn = f (xo ). Durch Ausklammern von (x − xo )k folgt f (x) = g(x)(x − xo )k + bn
mit g(xo ) = bk . Da bk 6= 0, können wir aus g(x) für
p x nahe xo die k-te Wurzel
ziehen, und es folgt f (x) = h(x)k + bn mit h(x) = k g(x)(x − xo ).
√ Die Funktion
h(x) ist umkehrbar nahe xo , denn ihre Ableitung in xo ist h′ (xo ) = k bk 6= 0. Somit
enthält h(Bǫ (xo )) eine offene Kreisscheibe Bρ (0) (da h(xo ) = 0), und f (Bǫ (xo )) =
h(Bǫ (xo ))k + bn ⊃ Bδ (bn ) mit δ = ρk .
78
J.-H. ESCHENBURG
8.5. Beweis von Argand. Wir nehmen an, dass f (x) gemäß (45)
keine Nullstelle besitzt. Dann hat |f (x)| irgendwo
ein (positives) Minimum, denn für große |x| ist auch |f (x)| groß. Nach
Verschieben der Variablen x können wir annehmen, dass das Minimum
bei 0 angenommen wird: minx |f (x)| = |f (0)| = |an |. Statt f (x) betrachten wir zunächst nur seine letzten beiden nichtverschwindenden
Terme, d.h. die Funktion g(x) = an−k xk + an für ein k < n (wenn
nämlich die Koeffizienten von x,
x2 , . . . , xk−1 alle gleich Null sind). Die
p
Nullstelle von g(x) ist xo = k −an /an−k . Berechnen Sie g(txo ) und
zeigen Sie, dass |f (txo )| ≤ |g(txo )| + |f (txo ) − g(txo )| < |an | für kleine
t > 0, ein Widerspruch, weil |an | ja das Minimum von |f (x)| war! Beachten Sie, dass xk+1 die niedrigste x-Potenz von f (x) − g(x) ist und
daher für kleine t > 0 gilt: |f (txo ) − g(txo )| ≤ Ctk+1 für eine Konstante
C.
9. Galois: (Un-)Lösbarkeit von Gleichungen (29.5.1832)
Im Süden von Paris gibt es eine Straße namens Rue de la Glacière
(Straße der Eisgrotte), die um 1860 während des zweiten Kaiserreichs
bebaut wurde, wie überhaupt ein großer Teil des alten Paris aus dieser
Zeit stammt. Dreißig Jahre zuvor war diese Gegend noch ganz ländlich
geprägt; es gab dort einen See, aus dem im Winter das Eis entnommen
wurde, um im Sommer damit Lebensmittel zu kühlen. In der Nähe
dieses Ortes fand am 30. Mai 1832 ein Duell statt. Einer der beiden
Kontrahenten, ein 20-jähriger junger Mann, wurde in den Bauch geschossen und liegen gelassen; Stunden später fand ihn ein Bauer und
brachte ihn in ein nahe gelegenes Krankenhaus, wo er am nächsten Tag
an Bauchfellentzündung starb.
STERNSTUNDEN DER MATHEMATIK
79
Der arme Junge, der da so elend starb, hieß Évariste Galois,118 und in
seinem kurzen Leben war er zu einem der bedeutendsten Mathematiker
seiner Zeit geworden. Das Bild119 zeigt ihn mit 15; es wurde von seiner
drei Jahre älteren Schwester gezeichnet. Am Abend vor dem Duell,
über dessen Ausgang er keinen Zweifel hegte, schrieb er einen langen
Brief an seinen Freund Auguste Chevalier, in dem er noch einmal seine
wichtigsten mathematischen Ergebnisse zusammenfasste.120
118Évariste
Galois, 1811 (Bourg-la-Reine) - 1832 (Paris)
Galois#mediaviewer/File:Evariste galois.jpg
120http://langevin.univ-tln.fr/notes/Galois/
http://fr.wikipedia.org/wiki/Lettre testamentaire d’Évariste Galois
http://www.ias.ac.in/resonance/Volumes/04/10/0093-0100.pdf (englisch)
119http://fr.wikipedia.org/wiki/Évariste
80
J.-H. ESCHENBURG
Galois war in vieler Hinsicht das genaue Gegenteil seines Zeitgenossen
Gauß, zu dem er gerne Kontakt aufgenommen hätte. Gauß hatte ein
langes und insgesamt recht glückliches Leben, während das von Galois von außen betrachtet eine Kette von Misserfolgen war und mit 20
endete. Gauß kam aus der Provinz, Galois lebte seit seiner Schulzeit
in Paris. Gauß war unpolitisch, Galois mischte sich in die politischen
Auseinandersetzungen seiner Zeit ein. Gauß stammte aus ärmlichen
Verhältnissen, die Familie von Galois dagegen gehörte der gebildeten
und liberalen Mittelschicht an: Sein Vater war Bürgermeister der kleinen Stadt Bourg-la Reine südlich von Paris, seine Mutter stammte aus
einer alten Juristenfamilie. Das außerordentliche mathematische Talent
trat bei Gauß und bei Galois gleichermaßen früh in Erscheinung, aber
während Gauß kompetente und mächtige Förderer fand, waren nur
wenige Lehrer121 von Galois bereit, ihn angemessen zu fördern, wohl
weil er sich mehr für die Werke großer Mathematiker wie Legendre122
und Lagrange interessierte als für den aktuellen Schulstoff. Während
Gauß sich schnell einen gesicherten Platz in der wissenschaftlichen Welt
eroberte, blieb Galois der Erfolg weitgehend versagt. Er bewarb sich
zweimal, 1828 und 1829, an der École Polytechnique, der Hohen Schule
Frankreichs für die Mathematik, und zweimal wurde er abgewiesen, das
zweite Mal unter besonders tragischen Umständen: Am 2. Juli 1829,
zwei Wochen vor der entscheidenden Prüfung, hatte sich sein geliebter Vater das Leben genommen, nachdem er in einer politischen Affäre
mit Schmutz beworfen worden war. Immerhin bekam Galois Ende 1829
einen Studienplatz an der École Préparatoire, der späteren École Normale Superieure.
Ebenso wie Gauß trieb Galois während der Schul- und Universitätszeit
eigene mathematische Forschung. Eins seiner Themen war die Auflösbarkeit algebraischer Gleichungen. Zwar war die Existenz von Lösungen
durch den Fundamentalsatz der Algebra gesichert, aber wie konnte man
sie berechnen? Gab es Formeln für die Lösungen wie bei den Gleichungen vom Grad 2,3,4? Abel123 hatte vor kurzem gezeigt, dass es eine solche Formel für die allgemeine Gleichung vom Grad 5 nicht geben konnte. Was genau war das Gesetz für die Existenz einer Lösungsformel?
Die Antwort auf diese Frage bildete den Inhalt von zwei Arbeiten von
1829, mit denen sich Galois um den Preis der Pariser Akademie bewarb.
121Eine
Ausnahme war sein Mathematiklehrer Louis Richard.
Legendre, 1752 - 1833 (Paris)
123Niels Henrik Abel 1802 - 1829 (Norwegen)
122Adrien-Marie
STERNSTUNDEN DER MATHEMATIK
81
Trotz der Unterstützung durch Cauchy124 erhielt er den Preis nicht (er
wurde an Jacobi125 und posthum an Abel vergeben). Schlimmer war,
dass die Arbeiten verloren gingen; sie waren an Fourier126 zur Begutachtung gegeben worden, der am 30. Mai 1830 starb, und in seinem
Nachlass wurden sie nicht aufgefunden. Der nächste Schlag für Galois
war die Julirevolution von 1830, die er zunächst sehr begrüßte. Die
Bourbonen, die nach dem Sieg über Napoleon wiedereingesetzt worden
waren, mussten nun endgültig abtreten, und der “Bürgerkönig” Louis
Philippe von Orleans kam an die Regierung. Aber Galois’ Fürsprecher
Cauchy ging außer Landes und verweigerte auch später den Treueeid
auf die neue Regierung. Galois selbst wurde wegen eines Artikels gegen
den stockkonservativen Rektor im Dezember 1830 von der Universität
verwiesen; er hatte inzwischen allerdings ein erstes Examen, die License, abgelegt. Im Januar 1831 legte er der Akademie eine erweiterte
Fassung der verloren gegangenen Arbeiten vor, “Mémoire sur les conditions de résolubilité des équations par radicaux” (Abhandlung über die
Bedingungen der Auflösbarkeit der Gleichungen durch Radikale”)127
Sie wurde an Poisson128 zur Begutachtung geschickt; der lehnte sie im
Juli 1831 als unverständlich ab, ermutigte Galois allerdings, noch eine
verbesserte Version vorzulegen.
Doch inzwischen hatte die Politik den Dingen eine andere Wendung
gegeben. Der “Bürgerkönig” war für den Republikaner Galois und viele
andere eine große Enttäuschung. Nach einer verbotenen Demonstration
am 14. Juli, dem Jahrestag der Revolution von 1789, wurde Galois verhaftet (bereits zum zweiten Mal) und zu einem halben Jahr Gefängnis
verurteilt, wo er einen Selbstmordversuch unternahm. Im März 1832
wurde er wegen einer Cholera-Epidemie in eine Klinik verlegt und Ende April entlassen. Kurz danach kam es zu dem verhängnisvollen Duell,
angeblich wegen einer Frau, die er bei seinem Klinikaufenthalt kennen
gelernt hatte, die aber nach anderen Berichten gar nichts von ihm wissen wollte. Auch politische Gründe wurden schon vermutet, aber Galois’ Kontrahent war politisch mit ihm einer Meinung. Die Umstände
bleiben ungeklärt.
Galois hatte seinen Freund Chevalier in seinem Brief gebeten, seine
Arbeiten in Umlauf zu bringen und insbesondere an Gauß und Jacobi
124http://www.persee.fr/web/revues/home/prescript/article/rhs
01514105 1971 num 24 2 3196
125Carl Gustav Jacob Jacobi, 1804 (Potsdam) - 1851 (Berlin)
126Jean Baptiste Joseph Fourier, 1768 - 1830 (Paris)
127Radikale sind (iterierte) Wurzelausdrücke in den Koeffizienten der Gleichung,
wie in der Cardanoschen Gleichung (22), Seite 46.
128Siméon Denis Poisson, 1781 - 1840 (Paris)
82
J.-H. ESCHENBURG
zu senden. Das tat er getreulich; er schrieb sie ab und verschickte sie.
Von Gauß und Jacobi scheint aber nie eine Reaktion gekommen zu sein.
Die Bedeutung der Schriften erkannte erst 1843 Joseph Liouville,129 der
den Zusammenhang mit Cauchys Theorie der Permutationen sah und
sie in seinem Journal veröffentlichte.
Die Ideen von Galois können wir gut an einer Lösungsmethode für
die quartische Gleichung
f (x) := x4 − ax3 + bx2 − cx + d = 0
(47)
aufzeigen, die ganz anders als die von Ferrari ist. Warum ist diese Gleichung “durch Radikale” lösbar, die quintische Gleichung aber nicht?
Die Zahlen a, b, c, d (“Koeffizienten”) sind gegeben. Nach dem Fundamentalsatz der Algebra gibt es eine Lösung x, sogar insgesamt vier
Lösungen130 x1 , x2 , x3 , x4 ; diese sollen berechnet werden. Wir betrachten zunächst das viel einfachere Umkehrproblem: Gegeben die Lösungen,
was sind die Koeffizienten? Das eindeutig bestimmte normierte131 quartische Polynom mit den Nullstellen x1 , . . . , x4 ist offensichtlich
f (x) = (x − x1 )(x − x2 )(x − x3 )(x − x4 ),
denn die rechte Seite wird genau dann Null, wenn einer der Faktoren
Null wird. Durch Ausmultiplizieren und Vergleich mit (47) können wir
a, b, c, d in Abhängigkeit von ~x := (x1 , x2 , x3 , x4 ) berechnen (“Vietascher Wurzelsatz”):132
(48)
a
b
c
d
=
x1 + x2 + x3 + x4
= x1 x2 + x1 x3 + x1 x4 + x2 x3 + x2 x4 + x3 x4
=
x1 x2 x3 + x1 x2 x4 + x1 x3 x4 + x2 x3 x4
=
x1 x2 x3 x4
=
=
=
=
ǫ1 (~x),
ǫ2 (~x),
ǫ3 (~x),
ǫ4 (~x).
Die Ausdrücke auf der rechten Seite sind ganz besondere Funktionen
in den Nullstellen x1 , . . . , x4 (die wir nun als Variable betrachen): Der
Wert dieser Funktionen ist unabhängig von der Reihenfolge der vier
129Joseph
Liouville, 1809 - 1882 (Paris)
x1 eine Lösung ist, ist das Polynom f (x) durch x − x1 teilbar und
g(x) = f (x)/(x − x1 ) ist ein Polynom vom Grad 3, das wiederum eine Nullstelle x2
besitzt, usw.
131Ein Polynom f (x) heißt normiert, wenn der höchste Koeffizient (von xn )
gleich Eins ist: f (x) = xn + a1 xn−1 + · · · + an .
132François Viète, 1540 - 1603 (Paris)
130Wenn
STERNSTUNDEN DER MATHEMATIK
83
Variablen.133 Derartige Funktionen heißen symmetrisch. Die hier vorkommenden symmetrischen Funktionen ǫ1 , ǫ2 , ǫ3 , ǫ4 , sind besonders einfach. In der Tat lassen sich alle anderen symmetrischen Funktionen
(als Polynomausdrücke) aus ihnen zusammensetzen, deshalb heißen sie
elementarsymmetrisch. Dies geschieht mit einem Algorithmus, der Sir
Isaac Newton134 zugeschrieben wird, siehe Übung 9.3. Jeden symmetrischen Ausdruck in ~x = (x1 , . . . , x4 ) können wir also aus den Koeffizienten a, b, c, d berechnen, ohne die Lösungen x1 , . . . , x4 zu kennen. Das
genügt allerdings noch nicht, um auch x1 , . . . , x4 zu berechnen.
Betrachten wir zunächst den sehr viel einfacheren Fall der quadratischen Gleichung x2 − ax + b = 0 mit den beiden Lösungen x1 , x2 . Nach
Vieta gilt a = x1 + x2 und b = x1 x2 . Wir könnten x1 , x2 berechnen,
wenn wir neben der Summe x1 + x2 = a auch ihre Differenz x1 − x2
kennen würden. Diese ist nicht symmetrisch in x1 , x2 , aber ihr Quadrat
ist symmetrisch und kann daher aus a und b berechnet werden:
(x1 − x2 )2 = (x1 + x2 )2 − 4x1 x2 = a2 − 4b.
√
Also ist x1 − x2 = ± a2 − 4b, woraus sich die bekannte Formel für
x1 , x2 ergibt.
Im Fall der quartischen Gleichung suchen wir Entsprechungen zu
x1 − x2 , nämlich Ausdrücke y1 , y2 , y3 , die aus a, b, c, d berechenbar sind
und aus denen wiederum die Lösungen x1 .x2 , x3 , x4 berechnet werden
können (“Resolventen”). Die y1 , y2 , y3 müssen Funktionen von ~x sein
wie die Koeffizienten a, b, c, d, aber wir wählen sie so, dass ihre Werte
nicht mehr unter allen 24 Umordnungen (Permutationen) unverändert
bleiben, sondern nur unter einigen wenigen, nämlich den gleichzeitigen Vertauschungen von zwei Variablenpaaren, (12)(34), (13)(24) und
(14)(23);135 wir wollen sie “halbsymmetrisch” nennen. Die einfachsten
Ausdrücke dieser Art sind:
(49)
y1 = (x1 + x4 )(x2 + x3 )
y2 = (x2 + x4 )(x3 + x1 )
y3 = (x3 + x4 )(x1 + x2 )
Auf Grund seiner Bildung ist y1 unverändert (“invariant”) unter der
Permutation (14)(23), aber erstaunlicherweise auch unter (12)(34) und
133Diese
Eigenschaft ist sehr speziell; für die Funktion φ(x1 , x2 ) = x21 x2 zum
Beispiel gilt sie nicht, denn etwa für x1 = 1 und x2 = 2 ist φ(1, 2) = 1 · 2 = 2, aber
φ(2, 1) = 4 · 1 = 4.
134Sir Isaac Newton, 1643 (Lincolnshire)- 1727 (Kensington bei London)
135(14)(23) bezeichnet die gleichzeitige Vertauschung in den Paaren (1,4) und
(2,3), usw.
84
J.-H. ESCHENBURG
(13)(24), weil
(x1 + x4 )(x2 + x3 ) = (x2 + x3 )(x1 + x4 ) = (x3 + x2 )(x4 + x1 ),
und dasselbe gilt für y2 und y3 . Diese neuen Ausdrücke y1 , y2 , y3 sind
die Lösungen einer kubischen Gleichung, nämlich
0 = (y − y1 )(y − y2 )(y − y3 ) = y 3 − uy 2 + vy − w ,
(50)
wobei die Koeffizienten u, v, w wieder die elementarsymmetrischen Funktionen in ~y = (y1 , y2 , y3 ) sind:
u =
y1 + y2 + y3
= ǫ1 (~y )
v = y1 y2 + y1 y3 + y2 y3 = ǫ2 (~y )
w =
y1 y2 y3
= ǫ3 (~y )
(51)
Setzt man (49) ein, so werden diese Ausdrücke zu Funktionen in ~x, und
zwar zu symmetrischen Funktionen:
Wenn alle yj (~x) halbsymmetrisch sind und f (~y ) symmetrisch in ~y , dann ist f (~y (~x)) symmetrisch in ~x.
Diese symmetrischen Funktionen in ~x können wir wieder durch die elementarsymmetrischen, also die Koeffizienten ausdrücken, zum Beispiel
u =
=
(52)
y1 + y2 + y3
x1 x2 + x1 x3 + x4 x2 + x4 x3
+ x2 x3 + x2 x1 + x4 x3 + x4 x1
+ x3 x1 + x3 x2 + x4 x1 + x4 x2
=
2b .
Ähnlich, aber mit deutlich
die anderen Koeffizienten.
u
v
(53)
w
mehr Rechenaufwand136 erhalten wir auch
=
2b
= b2 + ac − 4d
= abc − a2 d − c2
Damit kennen wir die Koeffizienten von (50). Diese kubische Gleichung
können wir lösen und damit die Zahlen y1 , y2 , y3 ermitteln. Aus denen
sind die x1 , x2 , x3 , x4 leicht zu berechnen, wenn wir x1 +x2 +x3 +x4 = a
berücksichtigen. Setzen wir zi = xi + x4 , so gilt z.B. y1 = (x1 + x4 )(x2 +
x3 ) = (x1 + x4 )(a − (x1 + x4 )) = az1 − z12 , und ebenso für i = 1, 2, 3
(54)
yi = azi − zi2 .
Das ist eine quadratische Gleichung für zi , die wir lösen können. Da
z1 + z2 + z3 = a + 2x4 , haben wir damit x4 ermittelt und dann auch
xi = z i − x4 .
136Siehe
z.B. “Algebra”, Seite 40-42 auf www.math.uni-augsburg.de/∼eschenbu
STERNSTUNDEN DER MATHEMATIK
85
Galois erkannte, dass Permutationen der Nullstellen den Schlüssel
zum Verständnis der Gleichung darstellen. Permutationen sind die möglichen Anordnungen einer Reihe von n Gegenständen, zum Beispiel der
Zahlen 1, . . . , n. Für n = 3 zum Beispiel gibt es die sechs möglichen
Anordnungen 123, 132, 213, 231, 312, 321; für n Gegenstände gibt es
n! = n · (n − 1) · . . . · 2 · 1 Anordnungen. Aber Galois arbeitet nicht
so sehr mit den Anordnungen, sondern den zugehörigen Umordnungen.
Eine Umordnung ist der Prozess, die Handlung, die Zahlen 1, . . . , n in
eine bestimmte Reihenfolge zu bringen, zum Beispiel 123 in die Reihenfolge 312; heute nennen wir eine solche Handlung eine bijektive
Abbildung der Menge {1, . . . , n} auf sich selbst. Das Entscheidende
ist, dass wir eine Reihe von Handlungen (oder Abbildungen) hintereinander ausführen (“verketten”) können. Wir können sozusagen mit
ihnen “rechnen”, wie wir mit Zahlen rechnen: Aus zwei Handlungen
wird durch Verkettung eine neue Handlung. Auch kann jede dieser
Handlungen durch eine andere Handlung (“Umkehrung”) rückgängig
gemacht werden. Eine solche “Rechenwelt” mit Verkettung und Umkehrung nennen wir heute eine Gruppe.
Diese Bezeichnung wird schon von Galois verwendet; er spricht in seinem Brief an Chevalier (vgl. Seite 79) von der “Gruppe der Gleichung”
(le groupe de l’équation); wir nennen sie heute die Galoisgruppe. Sie
enthält nicht alle Permutationen der Nullstellen x1 , . . . , xn , sondern nur
solche, die die zwischen den Nullstellen bestehenden Gleichungen erhalten: Wenn eine Gleichung φ(x1 , . . . , xn ) = 0 gilt, dann muss für eine
Permutation s, die zur Galoisgruppe gehört, auch φ(xs1 , . . . , xsn ) = 0
gelten, wobei s die Zahlen 1, 2, . . . , n umordnet zu s1, s2, . . . , sn. Diese
speziellen Permutationen bilden selbst wieder eine Gruppe, eine Untergruppe der Permutationsgruppe, d.h. eine Teilmenge, aus der die Rechenoperationen “Verkettung” und “Umkehrung” nicht hinausführen.
Für die Gleichung xn = a zum Beispiel gilt folgendes: Wenn x eine Lösung ist, dann ist auch ωx für ω = ei·2π/n eine Lösung, weil ja
ω n = ei·2π = 1. Bei richtiger Nummerierung137 gilt also die Beziehung
xi+1 = ωxi zwischen den Lösungen der Gleichung xn = a. Die Elemente
s der Galoisgruppe müssen diese Beziehung erhalten: xs(i+1) = ωxsi . Da
andererseits ωxsi = xsi+1 , muss s(i + 1) = s(i) + 1 gelten. Die einzigen
Permutationen, die diese Eigenschaft haben, sind die Verkettungen der
zyklischen Permutation (12 . . . n) : 1 → 2 → · · · → n → 1 mit sich
selbst. Solche Gruppen, die nur aus Verkettungen eines einzelnen Elements bestehen, heißen zyklisch.
137Dabei
ist der Index i eine Zahl zwischen 1 und n, und der nachfolgende Index
i + 1 wird “modulo n” gerechnet: Nach n kommt wieder 1, n + 1 “=” 1.
86
J.-H. ESCHENBURG
Was hat es aber mit den Permutationen (14)(23), (24)(31), (34)(12)
auf sich, den drei simultanen Paarvertauschungen, die bei der Lösung
der quartischen Gleichung auftraten? Sie bilden (gemeinsam mit der
Permutation id, die gar nichts verändert, der identischen Abbildung)
eine Untergruppe. Aber noch mehr gilt: Wenn ich eine dieser simultanen Paarvertauschungen “konjugiere”, d.h. danach eine Permutation
s ausführe und ihre Umkehrung s−1 davor, dann bekomme ich wieder
eine simultane Paarvertauschung: Wenn zum Beispiel (14)(23) mit der
Permutation s konjugiert wird, die 1234 zu 2314 umordnet, dann wird
das Paar (14) durch s zu (24), und (23) wird zu (31) und ich erhalte die
simultane Paarvertauschung (24)(31). Solche Untergruppen, die unter
Konjugation erhalten bleiben, nennen wir heute Normalteiler. Galois
beschreibt sie folgendermaßen in seinem Brief:138
“Mit anderen Worten, wenn eine Gruppe G eine andere
H enthält, dann kann die Gruppe G zerlegt werden in
Teilmengen,139 deren jede man erhält, indem man ein
und dieselbe Substitution auf die Permutationen von H
anwendet, in der Form, dass G = H + Hs + Hs′ + . . . .140
Und man kann sie auch zerlegen in Teilmengen mit den
gleichen Substitutionen G = H + tH + t′ H + . . . . Diese
zwei Sorten von Zerlegungen stimmen gewöhnlich nicht
überein. Wenn sie übereinstimmen, wird die Zerlegung
“rein” genannt.
Das ist die Definition des Normalteilers, denn sH = Hs ist äquivalent
zu sHs−1 = H. Die Teilmenge sH von G nennen wir eine Nebenklasse
von H. Eine entscheidende Beobachtung von Galois ist, dass man nicht
nur die Permutationen s, sondern auch die ganzen Nebenklassen verketten und umkehren kann, wenn H ein Normalteiler ist:141 sH ·tH = stH
und (sH)−1 = s−1 H. Die Nebenklassen eines Normalteilers H bilden
also wieder eine Gruppe, die wir G/H nennen und die viel weniger
138“En
d’autres termes, quand un groupe G en contient un autre H, le groupe G
peut se partager en groupes, que l’on obtient chacun en opérant sur les permutations
de H une meme substitution; en sorte que G = H + H S + H S’ + ... Et aussi, il
peut se décomposer en groupes qui ont toutes les memes substitutions G = H + T
H + T’ H + ... Ces deux genres de décompositions ne coincident pas ordinairement.
Quand elles coincident, la décomposition est dite propre.”
139Galois schreibt “groupes” auch für “Teilmengen”.
140+ bedeutet hier disjunkte Vereinigung von Teilmengen.
141Das Problem mit der Definition sH · tH = stH ist, dass sH = shH für jedes
h ∈ H, aber shtH nicht unbedingt gleich stH ist. Aber wenn H Normalteiler ist,
dann ist ht = th′ für ein h′ ∈ H und shtH = sth′ H = stH.
STERNSTUNDEN DER MATHEMATIK
87
Elemente als G hat, weil wir ja viele Elemente in einer Nebenklasse
zusammenfassen.
Galois verwendet die Normalteiler (die “reinen Zerlegungen”) der
Galoisgruppe, um das Problem der Gleichungsauflösung in zwei Teilprobleme zu zerlegen:142
“Es ist leicht zu sehen, wenn die Gruppe einer Gleichung
keine reine Zerlegung besitzt, dass man diese Gleichung
dann wohl transformieren kann, aber die Gruppen der
transformierten Gleichungen werden immer die gleiche
Anzahl von Permutationen haben. Im Gegensatz dazu,
wenn die Gruppe einer Gleichung eine reine Zerlegung
besitzt von der Art, dass sie sich aufteilt in M Teilmengen von je N Permutationen, dann wird man die gegebene Gleichung mit Hilfe von zwei Gleichungen lösen
können: Die eine wird eine Gruppe von M Permutationen sein, die andere eine von N Permutationen.”
In unserem Beispiel der quartischen Gleichung haben wir das gesehen:
Die Gruppe G ist die volle Permutationsgruppe der vier Wurzeln (genannt S4 ), weil es bei der allgemeinen Gleichung keine speziellen Gleichungen zwischen den Wurzeln gibt (außer den Gleichungen (48), die
nach Vieta immer gelten). Die simultanen Paarvertauschungen bilden
zusammen mit id einen Normalteiler H mit 4 Elementen. Die Gruppe
G/H hat damit nur noch 24/4 = 6 Elemente und kann als die volle Permutationsgruppe von drei Gegenständen angesehen werden; das sind
die drei Resolventen y1 , y2 , y3 , die die kubische Gleichung (50) erfüllen.
Diese entstehen aus y1 durch Anwenden aller möglichen Permutationen
der Variablen x1 , . . . , x4 , aber da jedes yj unter H invariant ist,143 ist es
statt G nur noch die Gruppe G/H, die sie permutiert. Die Galoisgruppe unserer Gleichung (47) ist nun viel kleiner geworden, weil wir die
zusätzlichen Beziehungen (54) haben, (xi + x4 )2 − a(xi + x4 ) + yi = 0,
die von den Permutationen der neuen Galoisgruppe erhalten werden
müssen; da bleiben nur noch die simultanen Paarvertauschungen übrig,
142“Il
est aisé de voir que quand le groupe d’une équation n’est susceptible
d’aucune décomposition propre, on aura beau transformer cette équation, les groupes des équations transformées auront toujours le meme nombre de permutations.
Au contraire, quand le groupe d’une équation est susceptible d’une décomposition
propre, en sorte qu’il se partage en M groupes de N permutations, on pourra
résoudre l’équation donnée au moyen de deux équations : l’une aura un groupe
de M permutations, l’autre un de N permutations.”
143Die Untergruppen, die y invariant lassen, sind konjugiert zu der, die y inj
1
variant lässt; die Gruppe H ist der Schnitt dieser drei Gruppen. Das zeigt noch
einmal, warum H ein Normalteiler ist, nämlich invariant unter Konjugationen.
88
J.-H. ESCHENBURG
d.h. die Gruppe H. Nicht die ursprüngliche Gleichung (47), sondern diese zusätzlichen Beziehungen (54) gestatten es uns nun, die Nullstellen
zu finden; wir brauchen dazu jetzt nur noch quadratische Gleichungen
zu lösen.
Das Problem der Auflösbarkeit durch Radikale wird von Galois nun
in aller Allgemeinheit gelöst:144
“Wenn man also alle möglichen reinen Zerlegungen der
Gruppe ausgeschöpft hätte, wird man bei Gruppen ankommen, die man transformieren könnte, aber deren
Permutationen immer dieselbe Ordnung hätten. Wenn
diese Gruppen jede eine Primzahl von Permutationen
enthält, ist die Gleichung durch Radikale lösbar, andernfalls nicht.”
Wir würden diesen Gedanken heute so ausdrücken: Wir betrachten alle
möglichen Folgen von Untergruppen
G = G0 ⊃ G1 ⊃ G2 ⊃ · · · ⊃ Gr = {id}
mit der Eigenschaft, dass Gk+1 stets ein Normalteiler in Gk ist, und die
nicht mehr verfeinert werden kann, ohne diese Eigenschaft zu zerstören.
Wenn dann alle Gruppen Gk /Gk+1 von Primzahlordnung145 sind, dann
ist die Gleichung auflösbar; wenn es keine solche Folge gibt, ist sie es
nicht. Die Galoisgruppe der allgemeinen quintischen Gleichung zum
Beispiel ist die Gruppe S5 der Permutationen von fünf Gegenständen
mit Ordnung 120, und die einzige solche Folge ist S5 ⊃ A5 ⊃ {id},
wobei A5 die Untergruppe von Ordnung 60 ist, die alle geraden Permutationen enthält (solche, die durch eine gerade Anzahl von Paarvertauschungen entstehen). Diese Gruppe hat keine Normalteiler mehr und ist
nicht von Primzahlordnung.146 Deshalb kann die quintische Gleichung
nicht durch Radikale aufgelöst werden.
144“Lors
donc qu’on aura épuisé sur le groupe d’une équation tout ce qu’il y a
de décompositions propres possibles sur ce groupe, on arrivera des groupes qu’on
pourra transformer, mais dont les permutations seront toujours en meme ordre. Si
ces groupes ont chacun un nombre premier de permutations, l’équation sera soluble
par radicaux; sinon non.”
145Die Ordnung einer Gruppe ist die Anzahl ihrer Elemente.
146 Man kann die Gruppen S und A geometrisch realisieren als Drehgruppen
4
5
von Würfel und Ikosaeder. Der Normalteiler H ⊂ S4 besteht aus den 180-GradDrehungen um die drei Raumachsen; diese werden bei jeder Drehung nur permutiert. Beim Ikosaeder dagegen gibt es kein Raumachsensystem, das unter allen Drehungen invariant bleibt. So ist es nicht erstaunlich, dass man keinen Normalteiler
findet.
STERNSTUNDEN DER MATHEMATIK
89
Was ist das Bedeutende an dieser Entdeckung? Galois hat nicht mehr
nach Lösungswegen für eine Gleichung gesucht, sondern die Frage der
Existenz von Lösungswegen gestellt. Um diese Frage zu beantworten,
ordnete er jeder Gleichung ein sehr viel einfacheres mathematisches
Objekt zu, eine endliche Gruppe, und er zeigte, dass sich die Frage der
Auflösbarkeit an der Struktur dieser Gruppe entscheiden lässt. Gleichzeitig hat er damit ein ganz neues mathematisches Gebiet begründet,
die Gruppentheorie. Gruppen treten überall in der Mathematik auf,
wo es Symmetrien gibt; die Gruppentheorie ist die Abstraktion des
Symmetriebegriffs. Da Symmetrien (und Symmetriebrechungen) eine
äußerst wichtige Rolle in Mathematik und Physik spielen, ist dieser
Moment der Mathematikgeschichte von größter Bedeutung.
Übungen
9.1. Permutationen und Würfeldrehungen. Zeigen Sie, dass es
gleichviele Permutationen von vier Gegenständen wie Würfeldrehungen
gibt, nämlich 24. Folgern Sie, dass die Würfeldrehungen genau den
Permutationen der vier Raumdiagonalen des Würfels entsprechen. Die
Permutationen (12)(34), (13)(24), (14)(23) entsprechen dabei den 180Grad-Drehungen um die drei Raumachsen.
Hinweis: Statt der Würfeldrehungen zählt man besser die Lagen des
Würfels: Jede der 6 Seiten kann oben liegen, jede der vier Kanten kann
vorn liegen.
9.2. Lösung der quartischen Gleichung: Finden Sie die Lösungen
x1 , . . . , x4 der Gleichung
(55)
x4 − 2x3 − 13x2 + 14x + 24 = 0
mit a = 2, b = −13, c = −14, d = 24. Zeigen Sie mit (53) für die
Koeffizienten von (50):
u = −26
v = 45
w = 72.
Die zugehörige kubische Gleichung lautet also
(56)
y 3 + 26y 2 + 45y − 72 = 0.
Zeigen Sie, dass y1 = 1 eine Lösung (56) ist. Wir können die linke Seite
von (56) also durch y −1 teilen und erhalten (y 3 +26y 2 +45y −72) : (y −
1) = y 2 +27y+72. Lösen Sie die quadratische Gleichung y 2 +27y+72 = 0
und finden Sie damit alle drei Lösungen y1 , y2 , y3 von (56). Berechnen
Sie zi = xi + x4 (i = 1, 2, 3) mit (54), zi2 − 2zi = −yi . Für y1 = 1 gibt es
nur eine Lösung, für y2 , y3 jeweils zwei. Welche jeweils die richtige ist,
90
J.-H. ESCHENBURG
muss ausprobiert werden.147 Welches die richtigen Lösungen sind, kann
man auch durch Einsetzen in die elementarsymmetrischen Funktionen
testen; die so entstehenden Zahlen müssen die Koeffizienten sein.
9.3. Newtons Algorithmus. Jedes Polynom φ(~x) ist Linearkombination (Summe von Vielfachen) von Monomen xk11 xk22 . . . xknn . Wenn φ
symmetrisch ist, dann kommen mit jedem Monom auch alle “permutierten” Monome (d.h. mit permutierten Variablen) bei gleichem Vorfaktor vor; zum Beispiel tritt mit x1 x32 x23 auch x3 x31 x22 = x31 x22 x3 auf.
Die Summe aller permutierten Monome, das “symmetrisierte Monom”,
bezeichnen wir mit dem Symbol [k1 , . . . , kn ], wobei die ki absteigend
geordnet sind: k1 ≥ k2 ≥ · · · ≥ kn . Diese Ausdrücke ordnen wir nun “lexikographisch”:148 Wir sagen “[k1 , . . . , kn ] > [l1 , . . . , lm ]”, wenn kj > lj
an der ersten Stelle j, wo kj 6= lj (also wenn ki = li für i < j und
kj > lj ). Das niedrigste Monom ist offensichtlich das, wo alle ki = 0
sind, d.h. die Konstante Eins; zwischen [k1 , . . . , kn ] und Eins liegen
endlich viele andere Monome.
Wenn wir nun ein beliebiges symmetrisches Polynom φ gegeben haben, suchen wir darin das höchste symmetrisierte Monom bezüglich
dieser Ordnung, a[k1 , . . . , kn ] für einen Koeffizienten a 6= 0, und gehen
über zu φ1 = φ − φ̃ mit
φ̃ = aǫk11 −k2 ǫk22 −k3 . . . ǫknn .
(57)
Der höchste Term von φ̃ ist149
[(x1 )k1 −k2 (x1 x2 )k2 −k3 . . . (x1 . . . xn )kn ] = [xk11 xk22 . . . xknn ]
ebenso wie der höchste Term von φ, deshalb verschwindet dieser Term
in φ1 , und der höchste Term von φ1 ist damit niedriger als der von
φ. Jetzt wiederholen wir das Verfahren mit φ1 anstelle von φ und erhalten ein symmetrisches Polynom φ2 mit noch niedrigerem höchsten
symmetrisierten Monom. Da unter jedem symmetrisierten Monom nur
endlich viele andere liegen, erreichen wir nach endlich vielen Schritten
ein konstantes Polynom. Fügen wir alles zusammen, so haben wir f als
Polynom (Summe von Vielfachen von Produkten der Variablen) in den
Variablen ǫ1 , . . . , ǫn geschrieben.150
147Ergebnis:
−1, 2, −3, 4
einem Lexikon sind die Wörter ebenso geordnet: “Akte” steht vor “Aktie”,
denn die ersten drei Buchstaben stimmen überein und der vierte entscheidet die
Reihenfolge, da “e” im Alphabet vor “i” kommt.
149Die Potenz von x zum Beispiel ist k −k +k −k +· · ·+k
1
1
2
2
3
n−1 −kn +kn = k1 .
150Vgl. Lou van der Waerden, Algebra I.
148In
STERNSTUNDEN DER MATHEMATIK
91
9.4. Lagrange-Resolventen. Wir haben schon gesehen, dass die Galoisgruppe der Gleichung xn = a zyklisch von Ordnung n ist. Es gilt
auch die Umkehrung: Wenn die Galoisgruppe G nur aus den Verkettungen der zyklischen Permutation s = (12 . . . n) besteht und n eine
Primzahl ist, dann kann jede Lösung xi durch Ziehen einer n-ten Wurzel gewonnen werden. Dazu betrachten wir die Lagrangeschen Resolventen:151 Für ωk = ei·2kπ/n , k = 1, . . . , n setzen wir
n
X
n−1
n−2
yk (~x) = ωk x1 + ωk x2 + · · · + xn =
ωkn−j xj .
j=1
Wenden Sie die zyklische Permutation s = (12 . . . n) auf yk an, d.h.
berechnen Sie syk (~x) = yk (s~x) mit
s~x = (xs1 , xs2 , . . . , xsn ) = (x2 , x3 , . . . , x1 );
zeigen Sie syk = ωk yk . Folgern Sie, dass bk := yk (~x)n invariant ist unter s und damit unter der ganzen Galoisgruppe. Solche Ausdrücke, die
invariant unter der Galoisgruppe sind, sind ohne Kenntnis der xj zu
berechnen, also “bekannt”, ähnlich wie die Koeffizienten
u, v, w im Bei√
n
152
spiel der quartischen Gleichung. Also ist yk = bk eine n-te Wurzel.
AusP
den yk sind aber die xj leicht zu berechnen: Wenn man die Summe k yk bildet und die Terme
P zu gleichen xj zusammenfasst, dann
“überlebt” nur n · xn , weil
k ωk = 0. Betrachtet man stattdessen
P j
k ωk yk für irgendein j, dann verschiebt sich der “Überlebensindex”
von n zu n − j, d.h. die Summe ist nxn−j .
9.5. Lösung der kubischen Gleichung nach Lagrange. Die kubische Gleichung
x3 − ax2 + bx − c = 0
lässt sich mit den Lagrangeschen Resolventen lösen. Dazu setzen wir
√
ω = ei·2π/3 = (−1 + 3 i)/2
und betrachten die Ausdrücke
u+ = ω 2 x1 + ωx2 + x3 = ω̄x1 + ωx2 + x3 ,
u− = ω̄ 2 x1 + ω̄x2 + x3 = ωx1 + ω̄x2 + x3 .
Ihre dritten Potenzen y± = (u± )3 sind invariant unter der zyklischen
Permutation (123), also “halbinvariant”. Sie erfüllen die quadratische
Gleichung
(58)
y 2 − py + q = 0 mit p = y+ + y− , q = y+ y− = (u+ u− )3 .
151Réflexions
sur la résolution algébrique des équations (1771/72)
http://gallica.bnf.fr/ark:/12148/bpt6k229222d/f206
152Beweis z.B. in “Algebra”, Seite 92, www.math.uni-augsburg.de/∼eschenbu
92
J.-H. ESCHENBURG
Zeigen Sie zuerst
u+ u− = [x21 ] − [x1 x2 ],
wobei die eckigen Klammern bedeuten, dass alle Permutationen des
Terms hinzugefügt werden sollen: [x21 ] = x21 +x22 +x23 und [x1 x2 ] = x1 x2 +
x1 x3 + x2 x3 . Nun können Sie q berechnen, d.h. mit a, b, c ausdrücken.
Beachten Sie dabei [x1 x2 ] = b und a2 = [x21 ] + 2[x1 x2 ], also gilt [x21 ] =
a2 − 2b und damit
q = (a2 − 3b)3 .
Zeigen Sie weiterhin
p = y+ + y− = −3[x21 x2 ] + 2[x31 ] + 12x1 x2 x3 .
Auch diese symmetrische Funktion wollen wir in elementarsymmetrische zerlegen, also mit a, b, c ausdrücken. Wir erinnern dazu an die
Formeln
a3 =
=
ab =
=
(x1 + x2 + x3 )3
[x31 ] + 3[x21 x2 ] + 6x1 x2 x3 ,
(x1 + x2 + x3 )(x1 x2 + x1 x3 + x2 x3 )
[x21 x2 ] + 3x1 x2 x3 .
Da x1 x2 x3 = c, können Sie daraus [x21 x2 ] und [x31 ] berechnen:
und somit
[x21 x2 ] = ab − 3c ,
[x31 ] = a3 − 3ab + 3c ,
p = 2a3 − 9ab + 27c .
Die Lösungen der quadratischen Gleichung (58) sind
p
y± = (p ± p2 − 4q)/2.
Berechnen Sie daraus x1 , x2 , x3 im Fall a = 0. Eine der drei Lösungen
ist die von Cardano (22).
10. Graves: Die Grenze des Zahlenreichs (26.12.1843)
Was sind die Zahlen? Wir haben diese Frage mehrfach in dieser Vorlesung gestellt, immer dann, wenn es um eine Erweiterung unseres Zahlbegriffs ging: von den rationalen zu den reellen Zahlen (Pythagoras und
Hippasos) und von den reellen zu den komplexen Zahlen (Bombelli).
Können wir diesen Erweiterungsprozess noch fortsetzen? Die komplexen Zahlen sind Paare reeller Zahlen, indem wir (x, y) statt x + yi
schreiben, und die Multiplikation (x+yi)(u+vi) = xu−yv +(xv +yu)i
kann als Multiplikationsregel für Paare reeller Zahlen gelesen werden:
(x, y)(u, v) = (xu − yv, xv + yu).
STERNSTUNDEN DER MATHEMATIK
93
Wenn man diese Formel ein wenig umformt,
(59)
(x, y)(u, v) = (xu − v̄y, vx + yū),
dann kann diese Paarmultiplikation noch zweimal angewandt werden:
Die Paare komplexer Zahlen mit dieser Multiplikationsregel (59) bilden die Quaternionen H (Olinde Rodriguez und Hamilton)153 und die
Paare von Quaternionen bilden die Oktaven oder Oktonionen O (Graves und Cayley).154 Danach ist Schluss, wie Hurwitz155 1898 gezeigt
hat. Das macht die Oktaven zu etwas Besonderem: Sie markieren die
größtmögliche Ausweitung unserer Zahlen mit den vier Grundrechenarten und einem Betrag, der die üblichen Rechenregeln erfüllt.156
Olinde Rodrigues hat in einer Arbeit von 1840 wenig geräuschvoll die
Quaternionen eingeführt als Hilfsmittel zur Berechnung von Drehungen im Raum; nur wenige Kollegen nahmen davon Notiz. Sir William
Rowan Hamilton, Professor der Astronomie am Trinity College in Dublin, wusste nichts davon. Er hatte 1833 ohne Kenntnis der Arbeiten
von Wessel und Argand selbst herausgefunden, dass komplexe Zahlen
sich als Paare reeller Zahlen und damit als Punkte der Ebene schreiben
ließen. Seither versuchte er, auch für Tripel reeller Zahlen eine Multiplikation zu finden in der Hoffnung, damit die Geometrie des Raumes
besser beschreiben zu können. Diese 10 Jahre währenden Bemühungen
waren selbst im Familienkreis bekannt, und seine Kinder sollen ihn jeden Morgen gefragt haben: “Well, Papa can you multiply triplets?”
Und jedesmal musste er zugeben, dass er es nicht konnte. Doch am 16.
Oktober 1843, auf dem Weg zu einem Treffen der Royal Irish Academy,
kam ihm der entscheidende Einfall: “Und hier dämmerte mir die Idee,
dass wir in irgend einem Sinne eine vierte Raumdimension zulassen
müssen, um mit Tripeln zu rechnen. Ein elektrischer Stromkreis schien
sich zu schließen, und ein Funke blitzte auf.”157 Die vier Dimensionen werden von der Eins sowie den Einheitsvektoren i, j, k in Richtung
153Benjamin Olinde Rodrigues, 1795 (Bordeaux) - 1851 (Paris),
Sir William Rowan Hamilton 1805 - 1865 (Dublin)
154John Thomas Graves (1806 (Dublin) - 1870 (Cheltenham)
Arthur Cayley, 1821 (Richmond, Surrey) - 1895 (Cambridge)
155Adolf Hurwitz, 1859 (Hildesheim) - 1919 (Zürich)
156Ohne die zweite Voraussetzung (Betrag) wurde erst 1958 von Michel Kervaire,
1927 (Tschenstochau, Polen) - 2007 (Genf) und John Milnor (geb. 1931, lebt in
Stony Brook bei New York) mit Methoden der algebraischen Topologie bewiesen,
dass endlich dimensionale reelle “Divisionsalgebren” (Bereiche, auf denen die vier
Grundrechenarten definiert sind) nur in den Dimensionen 1,2,4,8 existieren können.
157http://www-history.mcs.st-and.ac.uk/Biographies/Hamilton.html
http://en.wikipedia.org/wiki/William Rowan Hamilton
94
J.-H. ESCHENBURG
der drei Raumachsen aufgespannt. Die entscheidende Multiplikationsformel ritzte er in die Steine der Broom Bridge bei Dublin, die er gerade
überquerte:
(60)
i2 = j 2 = k 2 = ijk = −1.
Die irischen Mathematiker haben dafür gesorgt, dass dieses Ereignis
am 16. Oktober jeden Jahres an der Broom Bridge gefeiert wird.
Hamilton stand im Briefwechsel mit seinem Freund John Thomas
Graves, einem Juristen und Mathematiker, wie Hamilton aus Irland
gebürtig, der jetzt in London lebte. Er teilte ihm mit, dass seine Bemühungen um die Multiplikation von Tripeln, an denen auch Graves beteiligt gewesen war, endlich zu einem überraschenden Erfolg geführt
hatten. Wenn es aber vierdimensionale Zahlen gab, so dachte Graves,
könnte es dann nicht auch Zahlen von noch höherer Dimension geben?
Und wirklich: Am zweiten Weihnachtstag desselben Jahres, also am
26.12.1843, fand Graves die Oktaven. Hamilton war höchst erstaunt
und versprach, in der Royal Irish Academy davon zu berichten, aber er
vergaß es, vielleicht, weil seine Quaternionen ihn zu sehr faszinierten.
So wurde Graves’ Beitrag erst zwei Jahre später bekannt, nachdem der
STERNSTUNDEN DER MATHEMATIK
95
junge Star von der altehrwürdigen englischen Universität Cambridge,
Arthur Cayley, die Oktaven wiederentdeckt hatte. Obwohl Hamilton
auf die Priorität von Graves hingewiesen hat, werden sie bis heute
noch manchmal “Cayley-Zahlen” genannt. Zu der Zeit bezog Cayley
ein Stipendium, das es ihm erlaubte, sich ganz der Mathematik zu
widmen. Als es auslief, studierte auch er Jura und arbeitete 14 Jahre
lang als Rechtsanwalt, bis er 1863 zum Professor für Reine Mathematik
in Cambridge berufen wurde.
Hamilton und Graves entdeckten nicht die Formel (59), die erst von
Cayley stammt, sondern (60) und ihr Oktaven-Analog.158 Wir werden
ihren Weg gehen und sehen, dass dabei auch der Satz von Hurwitz herauskommt. Wir denken uns also einen n-dimensionalen Raum A = Rn
(Menge aller Folgen von je n reellen Zahlen a1 , . . . , an , Bezeichnung:
a = (a1 , . . . , an )) mit der üblichen euklidischen Geometrie, die auf
dem Längenbegriff nach Pythagoras
beruht: Die Länge oder der Bep
2
trag eines Vektors a ist |a| = a1 + · · · + a2n . Die “Punkte” a in diesem Raum stellen wir uns besser als gerichtete Strecken vom Ursprung
0 = (0, . . . , 0) bis zum Punkt a vor und nennen sie Vektoren; dieser
Begriff stammt von Hamilton. Vektoren können wir addieren und subtrahieren, indem wir sie aneinander legen; algebraisch entspricht dies
der Addition und Subtraktion der Komponenten.
a+b
3
2
b
1
−2
0
−1 0
−1
a
1
2
3
4
−2
Wir können Vektoren zudem mit reellen Zahlen multiplizieren und sie
damit verkürzen oder verlängern. Nun möchten wir sie auch miteinander multiplizieren; wir suchen also eine Regel, nach der man zwei
Vektoren a, b ∈ A einen weiteren Vektor ab ∈ A zuordnet, wobei das
Distributivgesetz a(b+c) = ab+ac und (b+c)a = ba+ca erfüllt sein soll,
sowie die Vertauschbarkeit mit der Skalarmultiplikation: a(tb) = t(ab)
für reelle Zahlen t. Außerdem sollen die Beträge (Längen) der beteiligten Vektoren das Gesetz
(61)
158
|ab| = |a||b|
Bei den Oktaven braucht man außer i, j, k noch vier weitere Einheitsvektoren,
die manchmal mit l, p, q, r bezeichnet werden. Statt ijk = −1 haben wir ijklpqr =
−1.
96
J.-H. ESCHENBURG
erfüllen. Wenn a ein Einheitsvektor ist, |a| = 1, dann sagt diese Gleichung, dass die Multiplikation mit a die Längen erhält: |ab| = |b| für
alle b, und ebenso |ab| = |a| wenn |b| = 1. Damit werden aber auch die
Winkel erhalten; zum Beispiel stehen zwei Einheitsvektoren a, b genau
√
dann senkrecht aufeinander, wenn für ihren Abstand gilt: |a − b| = 2
(Pythagoras);
b
2
1
0
a
1
da die Abstände durch die Multiplikation mit a erhalten bleiben, muss
auch das Senkrecht-Stehen (“Orthogonalität”) erhalten bleiben. Einen
solchen Raum A nennen wir eine normierte Algebra, weil der Betrag
oft auch “Norm” genannt wird.159 In A liegt insbesondere die Eins 1
und ihre reellen Vielfachen; wir nennen diese Menge die reelle Gerade
R = R · 1 in A. Den Anteil des Raumes A, der senkrecht auf R steht
(die “Hyperebene” R⊥ ) wollen wir mit A′ bezeichnen, genauer:
A′ = {a ∈ A : a ⊥ 1}.
1) Wenn a ∈ A′ und |a| = 1, dann ist a(ax) = −x für alle x ∈ A.
Beweis: (1 + a)((1 − a)x) = (1 − a)x + a(x − ax) = x − a(ax) und
|1 + a||1 − a||x| = 2|x|. Wegen |a(ax)| = |x| kann |x − a(ax)| = 2|x|
nur gelten, wenn die Vektoren x und −a(ax) gleichgerichtet sind, d.h.
wenn a(ax) = −x (siehe Figur).
x−a(ax)?
−a(ax)?
x
−a(ax)
2) Für alle b ∈ A′ und x ∈ A ist b(bx) = −|b|2 x und b2 = −|b|2 .
Beweis: Wir wenden 1) an auf a = b/|b|, falls b 6= 0.
3) Wenn a, b ∈ A′ und a ⊥ b, dann ist ab ∈ A′ und ab ⊥ a, b.
Beweis: Wir dürfen |a| = 1 annehmen. Da b ⊥ a, ist ab ⊥ a2 = −1. Da
b ⊥ 1 ist ab ⊥ a. Da a ⊥ 1, ist ab ⊥ b.
159Das
Wort “Algebra” hat hier eine besondere Bedeutung und steht nicht für
ein mathematisches Gebiet, sondern für einen Raum von Vektoren, die man nicht
nur addieren, sondern auch multiplizieren kann. Eine Algebra heißt normiert, wenn
der Betrag (die Norm) das Gesetz (61) erfüllt.
STERNSTUNDEN DER MATHEMATIK
97
4) Wenn a, b ∈ A′ und a ⊥ b, dann gilt ab = −ba.
Beweis: Wir dürfen |a| = |b| = 1 annehmen (a, b “orthonormal”). Dann
ist (a + b)2 = a2 + b2 + ab + ba = −2 + (ab + ba), aber andererseits
(a + b)2 = −|a + b|2 = −2 nach 2). Also ist ab + ba = 0.
5) Zwei orthonormale a, b ∈ A′ erzeugen die Quaternionen H ⊂ A.
Beweis: Den Einheiten i, j, k von Hamilton entsprechen die Vektoren
a, b, ab, denn (ab)2 = −1 und (ab)a = −a(ab) = b nach 3) und 4).
6) Ist c ⊥ H, so ist cH = Hc ⊥ H
Beweis: Für a, b ∈ H ∩ A′ und |a| = 1 ist ca ⊥ b, da (ca)a = −c ⊥ ba.
7) Sind a, b, c ∈ A′ orthogonal und c ⊥ ab, so gilt (ab)c = −a(bc).
Ein Tripel (a, b, c) mit (ab)c = −a(bc) nennen wir anti-assoziativ.
Beweis: Nach 6) ist auch bc ⊥ a und damit a(bc) = −(bc)a = (cb)a.
!
Somit ist (ab)c = −(cb)a zu zeigen. Einerseits gilt
((a + c)b)(a + c) = (ab + cb)(a + c)
= (ab)a + (cb)a + (ab)c + (cb)c
= 2b + (cb)a + (ab)c ,
andererseits ((a + c)b)(a + c) = |a + c|2 b = 2b, also (cb)a + (ab)c = 0.
8) Orthonormale c, d ∈ A′ mit c ⊥ H und d ⊥ (H + cH) gibt es nicht.
Beweis: Es seien a, b ∈ A′ ∩ H orthonormal. Dann gilt einerseits
a(b(cd)) = −a((bc)d) = (a(bc))d ,
(A)
denn die Tripel (b, c, d) und (a, bc, d) sind nach 7) anti-assoziativ. Andererseits gilt aber:
a(b(cd)) = −(ab)(cd) = ((ab)c)d = −(a(bc))d ,
(B)
denn die Tripel (a, b, cd), (ab, c, d) und (a, b, c) sind anti-assoziativ. Aber
(A) und (B) widersprechen sich!
Wir haben also den Satz von Hurwitz gesehen, dass R, C, H, O die
einzigen normierten Divisionsalgebren sind. Dieses Resultat hat starke
Konsequenzen in der Geometrie, die lange noch nicht vollständig ausgelotet sind. Ein Beispiel sind die kompakten kontinuierlichen Gruppen.
Wir haben endliche Gruppen kennengelernt, zum Beispiel die Permutationsgruppe Sn von n Gegenständen oder ihre Untergruppe An , die
nur die geraden Permutationen enthält. Eine Gruppe ohne echte Normalteiler nennt man einfach. Die Klassifikation der einfachen endlichen
Gruppen wurde erst 1982 abgeschlossen; es gibt eine Reihe von unendlichen Serien (wie An , n ≥ 5) sowie 26 einzelne “sporadische” Gruppen;
die größte unter ihnen hat ca. 8 · 1053 Elemente.
98
J.-H. ESCHENBURG
Es gibt aber auch “kontinuierliche” Gruppen (Lie-Gruppen);160 ihre
Elemente treten in unendlichen Familien auf, die von reellen Parametern abhängen. Ein Beispiel ist die Gruppe aller Drehungen um einen
festen Punkt in der Ebene oder im Raum; der Drehwinkel ist ein solcher Parameter. Die Voraussetzung “Kompaktheit” ist ein Ersatz für
die Endlichkeit. Die Klassifikation der kompakten einfachen Liegruppen ist eine viel einfachere Aufgabe als die der einfachen endlichen
Gruppen; sie wurde bereits um 1890 von Wilhelm Killing161 und 1894
in der Dissertation von Élie Cartan durchgeführt.162 Es gibt vier unendliche Serien (mit den Buchstaben A, B, C, D bezeichnet) und fünf
Ausnahmegruppen. Die vier Serien umfassen die Drehgruppen über R
(getrennt für gerade und ungerade Dimension) sowie ihre Analoga über
C und H, während die fünf Ausnahmegruppen G2 , F4 , E6 , E7 , E8 mit O
zu tun haben sollten. Cartan fand 1908, dass die erste dieser Gruppen,
G2 , die Automorphismengruppe von O ist (vgl. Übung 10.3), und Freudenthal163 klärte die Beziehung von O zu F4 und E6 . Die Beziehung zu
E7 und E8 ist trotz der Anstrengungen von Freudenthal, Tits,164 Vinberg165 und anderen noch immer nicht vollständig geklärt.
Übungen
10.1. Konjugation in normierten Algebren. Jede normierte Algebra A lässt sich orthogonal zerlegen als A = R ⊕ A′ , wobei R = R · 1
die reelle Gerade ist (bestehend aus den reellen Vielfachen der Eins)
und A′ = R⊥ ihr orthogonales Komplement ist. Jedes a ∈ A lässt sich
also eindeutig zerlegen in a = ao + ~a mit ao ∈ R und ~a ∈ A′ . Die Konjugation in A weist jedem Element a = ao + ~a das Element ā = ao − ~a
zu. Zeigen Sie:
a) Die Konjugation ist ein Anti-Automorphismus, ab = b̄ā.166
b) aā = āa = |a|2 für alle a ∈ A.
10.2. Divisionsalgebren. Zeigen Sie, dass jede normierte Algebra A
eine Divisionsalgebra ist, das heißt, dass jede Gleichung ax = b oder
160Marius
Sophus Lie, 1842 (Nordfjordeid) - 1899 (Kristiania = Oslo)
Killing, 1847 (Burbach bei Siegen) - 1923 (Münster)
162Élie Joseph Cartan, 1869 (Dolomieu, Dauphiné) - 1951 (Paris)
163Hans Freudenthal, 1905 (Luckenwalde in Brandenburg) - 1990 (Utrecht),
Oktaven, Ausnahmegruppen und Oktavengeometrie (1951),
http://www.maths.ed.ac.uk/∼aar/papers/freudselecta.pdf
164Jaques Tits, geb. 1930 in Uccle/Ukkel (Belgien), lebt in Paris.
165Ernest Borissowitsch Vinberg, geb. 1937, lebt in Moskau.
166Hinweis: Es gilt ~
a~b = −~b~a, siehe 4).
161Wilhelm
STERNSTUNDEN DER MATHEMATIK
99
xa = b für beliebige a, b ∈ A mit a 6= 0 eine eindeutig bestimmte
Lösung hat. Berechnen Sie diese!
10.3. Automorphismen der Oktaven. Ein Automorphismus von O
ist eine umkehrbare Abbildung σ auf O, die mit Addition und Multiplikation vertauschbar ist: σ(a + b) = σ(a)σ(b) und σ(ab) = σ(a)σ(b),
und die außerdem den Betrag erhält: |σ(a)| = |a|. Wenn wir σ auf
drei orthonormale Vektoren i, j, l ∈ O′ mit l ⊥ ij anwenden (“CayleyTripel”), dann erhalten wir ein anderes Cayley-Tripel. Umgekehrt bestimmt jedes Cayley-Tripel (a, b, c) (orthonormal in O′ mit c ⊥ ab)
einen eindeutigen Automorphismus σ mit σ(i) = a, σ(j) = b, σ(l) = c,
denn O ist orthogonale direkte Summe von H und cH, wobei H die
Quaternionenalgebra ist, die von a, b, ab statt von i, j, ij aufgespannt
wird. Zeigen Sie damit, dass die Cayley-Tripel umkehrbar-eindeutig
den Automorphismen zugeordnet sind. Zeigen Sie weiterhin, dass die
Cayley-Tripel (und damit die Automorphismen) eine 14-parametrige
Schar bildet: Der erste Vektor a muss ein Einheitsvektor in O′ sein,
also in der 6-dimensionalen Einheitssphäre von O′ = R7 liegen. Der
zweite Einheitsvektor b muss senkrecht zu a sein und damit in der 6dimensionalen Hyperebene a⊥ ⊂ O′ liegen, d.h. in der 5-dimensionalen
Einheitssphäre in diesem Raum. Der letzte Einheitsvektor c schließlich
muss senkrecht zu a, b, ab sein; er liegt also in einem Unterraum der
Dimension 7 − 3 = 4, und die Einheitssphäre darin ist 3-dimensional.
Insgesamt hat die Automorphismengruppe also die Dimension 6+5+3.
10.4. Automorphismen der Quaternionen. Zeigen Sie, dass jeder
Automorphismus σ von H ein orthogonales Paar von Vektoren a, b ∈
H′ definiert, nämlich a = σ(i), b = σ(j), und umgekehrt: Zu jedem
orthogonalen Paar a, b ∈ H′ gibt es genau einen Automorphismus σ mit
a = σ(i), b = σ(j) und σ(k) = σ(ij) = ab (und natürlich σ(1) = 1).
Je zwei orthonormale Vektorpaare in R3 = H′ lassen sich durch genau
eine Drehung aufeinander abbilden; die Automorphismengruppe von
H ist also die Drehgruppe im Raum, SO3 . Drehungen in H′ lassen
sich realisieren durch die Konjugationen Ada : x 7→ axā mit a ∈ H
mit |a| = 1: Ist a = ao + ~a, so ist ~a die Drehachse von Ada , und
der Drehwinkel berechnet sich aus ao : Ist ao = cos α, dann ist der
Drehwinkel gleich 2α.
11. Riemann: Die Geometrie des Raumes (10.6.1854)
Was ist Geometrie? Einerseits ist sie seit der griechischen Antike
eins der ältesten Teilgebiete der Mathematik und damit Teil unserer
100
J.-H. ESCHENBURG
Gedankenwelt, andererseits ist sie auch die Wissenschaft des uns umgebenden Raums, der doch Teil unserer Erfahrungswelt ist. Immanuel
Kant167 versuchte eine philosophische Begründung, warum die Geometrie des Raums ganz auf die Seite der Erkenntnisse “a priori” (vor aller
Erfahrung) gehört. In der “Kritik der Reinen Vernunft”168 schrieb er:
“Der Raum ist eine notwendige Vorstellung a priori, die
allen äußeren Anschauungen zum Grunde liegt. Man
kann sich niemals eine Vorstellung davon machen, dass
kein Raum sei, ob man sich gleich ganz wohl denken
kann, dass keine Gegenstände darin angetroffen werden.
Er wird also als die Bedingung der Möglichkeit der Erscheinungen, und nicht als eine von ihnen abhängende
Bestimmung angesehen, und ist eine Vorstellung a priori,
die notwendigerweise äußeren Erscheinungen zum Grunde liegt. Auf diese Notwendigkeit a priori gründet sich
die apodiktische Gewissheit aller geometrischen Grundsätze, und die Möglichkeit ihrer Konstruktionen a priori. Wäre nämlich diese Vorstellung des Raumes ein a
posteriori erworbener Begriff, der aus der allgemeinen
äußeren Erfahrung geschöpft wäre, so würden die ersten
Grundsätze der mathematischen Bestimmung nichts als
Wahrnehmungen sein. Sie hätten also alle Zufälligkeit
der Wahrnehmung, und es wäre eben nicht notwendig,
dass zwischen zwei Punkten nur eine gerade Linie sei,
sondern die Erfahrung würde es so jederzeit lehren...
Man würde also nur sagen können, so viel zur Zeit noch
bemerkt worden, ist kein Raum gefunden worden, der
mehr als drei Abmessungen hätte.”
Als Bernhard Riemann am 10. Juni 1854 seinen Habilitationsvortrag vor der Philosophischen Fakultät der Universität Göttingen hielt,
war dies noch immer der philosophische Hintergrund seiner Zuhörer.
Allerdings war die “Gewissheit aller geometrischen Grundsätze”, wie
sie durch Euklids Postulate der ebenen Geometrie formuliert waren,169
bereits ein wenig erschüttert worden, insbesondere das fünfte Postulat:
“Zwei Geraden, die von einer Geraden geschnitten werden, wobei die innen liegenden beiden Winkel zusammen
kleiner sind als zwei rechte, treffen sich auf der Seite, auf
167Immanuel
Kant, 1724 - 1804 (Königsberg)
Ästhetik, erster Abschnitt: Von dem Raume,
http://www.gutenberg.org/cache/epub/6342/pg6342.txt
169http://www.opera-platonis.de/euklid/eb1/eb102.htm
168Transzendentale
STERNSTUNDEN DER MATHEMATIK
101
der die Winkel liegen, die zusammen kleiner sind als zwei
rechte.”
?
Dieses Postulat war schon immer, durch 2000 Jahre Mathematikgeschichte hindurch, Gegenstand von Diskussionen gewesen. Im Gegensatz zu den anderen Postulaten170 wurde es als beweisbedürftig angesehen, und Legendre hatte noch vor wenigen Jahrzehnten verschiedene
Beweisversuche unternommen. Doch kürzlich hatten unabhängig voneinander der Russe Lobachevski,171 der Ungar János Bolyai172 und auch
Gauß (der aber seine Ergebnisse aus Vorsicht nie veröffentlicht hat) eine in sich stimmige Geometrie gefunden, in der das 5. Postulat nicht
mehr gilt: die nichteuklidische Geometrie. Damit waren nicht nur alle
Beweisversuche für das 5. Postulat ad absurdum geführt, sondern man
musste jetzt das Wort “Geometrie” in die Mehrzahl setzen: Es gab
offensichtlich nicht nur eine denkbare Geometrie, sondern mindestens
zwei.
Riemann hatte in Göttingen und Berlin studiert und 1851 in Göttingen
bei Gauß seine Dissertation eingereicht: “Grundlagen für eine allgemeine Theorie der Functionen einer veränderlichen complexen Grösse”,173
in der auch der nach ihm benannte Abbildungssatz vorkommt. 1854 habilitierte er sich in Göttingen mit der Schrift “Über die Darstellbarkeit
einer Function durch eine trigonometrische Reihe”, in der insbesondere
die Grundlagen des Riemannschen Integralbegriffs entwickelt werden.
Seine vielleicht berühmteste Arbeit, “Über die Anzahl der Primzahlen
unter einer gegebenen Größe”, die in Zusammenhang mit der Riemannschen Vermutung steht, lag noch in der Zukunft (1859). Zum Abschluss
des Habilitationsverfahrens war noch eine Probevorlesung erforderlich,
gehalten vor der gesamten Philosophischen Fakultät, zu der die Mathematik gehörte. Dafür mussten drei Themenvorschläge eingereicht werden, von denen die Fakultät kurzfristig einen aussuchte. In Riemanns
1701.
Von einem beliebigen Punkt zu einem anderen ist eine gerade Strecke zu
ziehen. 2. Eine gerade Strecke ist beliebig verlängerbar. 3. Um einen beliebigen
Punkt ist mit beliebigem Radius ein Kreis beschreibbar. 4. Alle rechten Winkel
sind unter sich gleich.
171Nikolai Iwanowitsch Lobatschewski, 1792 (Nischni Nowgorod) - 1856 (Kasan)
172János Bolyai, 1802 (Clausenburg/Cluj Napoca) - 1860 (Neumarkt/Târgu Mureş, heute Rumänien)
173 http://www.maths.tcd.ie/pub/HistMath/People/Riemann/Papers.html
102
J.-H. ESCHENBURG
Fall sorgte vermutlich Gauß dafür, dass nicht das bevorzugte erste der
vorgeschlagenen Themen, sondern das dritte ausgesucht wurde, das von
seinem eigentlichen Forschungsgebiet am weitesten entfernt lag: “Über
die Hypothesen, welche der Geometrie zu Grunde liegen”.174
Mit diesem Vortrag betrat Riemann in mehrfacher Hinsicht Neuland:
• Die anschauliche Dimensionszahl 3 ist unerheblich.
• Der zugrundeliegende Raum (Topologie) und der auf ihm definierte Abstandsbegriff (Geometrie) werden getrennt.
• Der zugrundeliegende Raum ist nur im Kleinen bestimmt.
• Riemann unterscheidet zwischen dem Raum selbst und seiner
Beschreibung durch Koordinaten.
• Er entwickelt einen sehr weit gefassten Begriff von Geometrie,
der die euklidische und die nichteuklidische Geometrie als einfache Spezialfälle enthält.
• Seine Geometrie ist (laut Hermann Weyl)175 eine “Nahwirkungstheorie” [12, Seite 45], ähnlich wie mit dem Begriff des elektrischen und magnetischen Feldes durch Faraday und Maxwell eine
Nahwirkungstheorie des Elektromagnetismus geschaffen wurde.
Die Rolle des Feldes spielt die Riemannsche Metrik.
Das Wort “Hypothesen” im Titel des Vortrags ist bereits ein Einspruch
gegen Kant, denn die Eigenschaften des Raumes stehen nach Riemann
eben nicht mit apodiktischer Gewissheit vor aller Erfahrung fest, sondern die Gültigkeit oder Ungültigkeit von Hypothesen ist eine Erfahrungstatsache.
Riemann entwickelt zunächst den Begriff einer “n-fach ausgedehnten
Größe” oder “Mannigfaltigkeit”, einer Menge, deren Elemente lokal jeweils durch n Zahlen (Koordinaten oder Parameter) festgelegt sind.
Zwei Teilmengen einer solchen Größe (“Quanta”) sind nur dann vergleichbar, wenn sie am gleichen Ort liegen und ineinander enthalten
sind, und selbst dann kann man nur von “größer” und “kleiner” sprechen und muss die Frage nach dem “Wieviel” unbeantwortet lassen. Es
ist wie am Anfang unserer ersten Vorlesung, bevor die Idee des Messens
geboren wurde (vgl. Seite 2, besonders Fußnote 2).
a
b
174Siehe
vorige Fußnote 173 sowie [12].
Weyl, 1885 (Elmshorn bei Hamburg) - 1955 (Zürich)
175Hermann
STERNSTUNDEN DER MATHEMATIK
103
Dennoch lassen sich auch auf dieser Stufe bereits Aussagen treffen: die
Dimensionszahl, ob die Größe begrenzt oder unbegrenzt, in sich geschlossen ist (wie eine Kugelfläche) oder nicht, ob man geschlossene
Linien oder Flächen darin aufeinander oder auf einen Punkt deformieren kann und vieles andere. Das sind die topologischen Aussagen. Riemann kannte sich damit aus, weil die Definitionsbereiche der von ihm
betrachteten Funktionen, die “Riemannschen Flächen”, sich in dieser
Hinsicht stark voneinander unterschieden.
Um Größen an verschiedenen Orten in Beziehung setzen zu können,
benötigt man eine längentreue Übertragung von Abständen: “Zum
Messen wird also ein Mittel erfordert, die eine Größe als Maßstab für
die andere fortzutragen.”
a
b
b
b
b
Dazu ist eine zusätzliche Struktur erforderlich, die durch die Mannigfaltigkeit selbst noch nicht gegeben ist: an jedem Ort ein Maßstab,
ein Mittel zur Längenmessung. Die Unterscheidung zwischen dem zugrundeliegenden Raum und der darauf bestehenden Geometrie wird
hier wohl zum ersten Mal getroffen; in der herkömmlichen Geometrie
gehörten Begriffe wie Gerade und Abstand wie selbstverständlich zum
Punktraum dazu. Erst durch die Trennung der Konzepte gibt es die
begriffliche Möglichkeit, von verschiedenen geometrischen Strukturen
auf dem gleichen Raum zu sprechen.
Einen Maßstab denken wir uns als kurz; wenn wir größere Distanzen messen wollen, dann durch wiederholtes Aneinanderlegen dieses
Maßstabes. Die Punkte der Mannigfaltigkeit sind durch Koordinaten
gegeben; wir wollen den Abstand ds von zwei nahe benachbarten Punkten x, x′ mit den Koordinaten x = (x1 , . . . , xn ) und x′ = x+dx messen.
Dieser Abstand wird von den Koordinaten von x und dx abhängen und
stets positiv sein, solange dx 6= 0. Riemann wählt für ds2 eine positive
quadratische Form in den Koordinaten von dx,
X
(62)
ds2 =
gij (x)dxi dxj ,
wobei die Koeffizienten gij Funktionen von x sind, die so gewählt sind,
dass die rechte Seite von (62) für alle Wahlen von dx 6= 0 positiv
ist. Eine solche punktabhängige quadratische Form (62) nennen wir
heute eine Riemannsche Metrik. Wenn wir den Punkt x im Moment
einmal festhalten, dann können wir nach Änderung der Koordinaten
104
J.-H. ESCHENBURG
annehmen, dass
gij (x) = δij =
1 für i=j
0 für i6=j
,
denn jede positive quadratische Form lässt sich durch KoordinatenP 2
wechsel auf diese Normalform bringen. Dann wird ds2 =
dxi , das
heißt, im Kleinen gilt für den Abstand der Satz des Pythagoras und
damit die euklidische Geometrie.
x’
ds
x
dx2
dx1
In diesem eingeschränkten Sinne stimmt Riemann eigentlich dem Kantschen Apriorismus zu:176 Zwar glauben wir nicht mehr, mit Sicherheit
sagen zu können, was die beiden Geraden177 in der Figur auf Seite 101
hinter der nächsten Hecke treiben, aber in unserem nächsten Umkreis
setzen wir doch annähernd die Gültigkeit der euklidischen Geometrie
voraus. Damit ist aber Platz geschaffen nicht nur für zwei mögliche Geometrien, sondern für unendlich viele. In der Dimension n = 2 (Flächen)
ist dies gar nichts Neues; schon lange hat man Flächen, auch krumme Flächen, durch Zahlenpaare x1 , x2 parametrisiert, bei der Erdoberfläche zum Beispiel durch die geographische Länge und Breite. Wenn
man Abstände auf der Fläche in diesen Koordinaten ausgedrückt, entsteht genau der Ausdruck (62). Riemann konnte daher diesen Fall als
anschauliches Beispiel bei Konstruktionen in der Riemannschen Geometrie benutzen (z.B. in der untenstehenden Figur); Riemanns Doktorvater Gauß hatte in seiner Arbeit “Disquisitiones Generales circa Superficies Curvas” (Allgemeine Untersuchungen über krumme Flächen)
von 1828 die Flächengeometrie in diesem Sinne sehr genau studiert.178
Wie aber kann man zwei Riemannsche Metriken voneinander unterscheiden, wo sie doch lokal alle annähernd gleich aussehen? Vielleicht
gehen sie alle durch Koordinatentransformationen auseinander hervor?
Riemann zeigt durch Zählen der Parameter, dass dies nicht der Fall sein
Funktionen gij , aber
kann: Jede Riemannsche Metrik besteht aus n(n+1)
2
nur n Funktionen (die Koordinaten x1 , . . . , xn ) können wir willkürlich
176Riemann
diskutiert allerdings kurz auch andere mögliche Formen für ds. Hermann von Helmholtz (1821 - 1894) zeigt in seiner Schrift “Über die Tatsachen, die
der Geometrie zum Grunde liegen” von 1868, dass die freie Beweglichkeit im Kleinen notwendig zu der Riemannschen Metrik führt.
http://gdz.sub.uni-goettingen.de/dms/load/img/?IDDOC=53331
177
Geraden im Riemannschen Sinn sind kürzeste Linien bezüglich der Riemannschen Metrik; sie werden auch Geodäten genannt.
178 Engl. Übersetzung: http://www.gutenberg.org/files/36856/36856-pdf.pdf
STERNSTUNDEN DER MATHEMATIK
105
wählen. Es bleiben n(n−1)
Funktionen, die nicht durch Koordinaten2
wahl beeinflusst werden können. Riemann versucht, diese invarianten
Funktionen zu ermitteln.
Dazu betrachtet er ein sehr spezielles Koordinatensystem (“Normalkoordinaten” um x), bei dem die von einem Punkt x ausgehenden
kürzesten Linien (Geodäten) längentreu auf radiale Strahlen im Rn
abgebildet werden, wobei die Winkel zwischen den Geodäten einerseits
und den zugehörigen Strahlen andererseits gleich sind.
Für jede beliebige Funktion f (x) kann der Wert von f in einem
Nachbarpunkt x + δx durch Polynome in δx angenähert werden (Satz
von Taylor):179
X
X
f (x + δx) = f (x) +
ak δxk +
akl δxk δxl + . . .
k
k,l
(konstant + linear + quadratisch + ... in der Variablen δx). Diesen
Satz wendet Riemann auf die metrischen Koeffizienten gij in Normalkoordinaten um x an, und er stelltP
fest, dass für f = gij der lineare
Term verschwindet, also gij = δij + aijkl δxk δxl + . . . und somit
k,l
X
gij (x + δx)dxi dxj =
i,j
X
i
dx2i +
X
aijkl δxk δxl dxi dxj + . . . .
i,j,k,l
Dabei ist der quadratische Term von einer besonderen Form:180
X
1 X
(63)
aijkl δxk δxl dxi dxj = −
Rijkl ∆xij ∆xkl
12
i,j,k,l
i,j,k,l
mit ∆xij := dxi δxj − dxj δxi . Die biquadratische Form (63) in den eindimensionalen Variablen dxi und δxj ist also in Wahrheit eine quadratische Form in den zweidimensionalen Variablen (“Flächenvariablen”)
∆xij . Die Koeffizienten Rijkl sind die Komponenten des berühmten
Riemannschen Krümmungstensors. Für die Koeffizienten Kij := Rijji
179Brook
Taylor, 1685 (Edmonton, Middlesex) - 1731 (London)
Grund ist das Gauß-Lemma für Normalkoordinaten, gij,i = 0. Siehe
Übung 11.2. Vgl. auch Hermann Weyls Kommentar, [12], Seite 57. Im zweiten
Teil von Riemanns Preisschrift für die Pariser Akademie von 1861 hat er einige
Gedanken des Habilitationsvortrags ausgearbeitet:
http://www.maths.tcd.ie/pub/HistMath/People/Riemann/Paris/Paris.pdf,
Seite 11 - 16, besonders (II) auf Seite 14.
180Der
106
J.-H. ESCHENBURG
gibt Riemann eine geometrische Deutung, die diesen Namen rechtfertigt: Es ist die Gauß-Krümmung im Punkt x für die Fläche, die von
allen Geodäten durch x in der xi xj -Ebene gebildet wird. Gauß hatte in
seinen “Allgemeinen Untersuchungen über krumme Flächen” von 1828
(vgl. Fußnote 178) die Krümmung K einer Fläche mit Hilfe der Geometrie des umgebenden Raums definiert, nämlich als Produkt der größten
und der kleinsten Krümmung von Schnittlinien der Fläche mit Ebenen,
die die Flächennormale (die Gerade senkrecht zur Fläche) enthalten.
Aber dann zeigte er, dass K sich wunderbarerweise allein aus den metrischen Koeffizienten gij berechnen lässt und also unverändert bleibt,
wenn man die Fläche längentreu verbiegt; genau diesen Ausdruck in
gij und seinen ersten und zweiten Ableitungen findet Riemann wieder
in Kij . In diesen n(n−1)
Zahlen meint er die Invarianten der Metrik
2
gefunden zu haben; das ist allerdings im Allgemeinen falsch.181
Der Riemannsche Krümmungstensor R hat später noch eine andere
geometrische Bedeutung erhalten. Christoffel, Lipschitz, Levi-Civita
und Hermann Weyl182 haben noch ein weiteres Element der euklidischen Geometrie (außer Abständen und Winkeln) in die Riemannsche
Geometrie übertragen: die Parallelverschiebung von Vektoren (Punktepaaren) auf der Mannigfaltigkeit. Eine natürliche Parallelverschiebung
gibt es nicht nur in der euklidischen Ebene, sondern auch auf krummen Flächen, und man hätte nur ihre Darstellung in Koordinaten auf
den höherdimensionalen Fall übertragen müssen, aber seltsamerweise
181Der Kümmungstensor (R
ijkl ) wird durch die Koeffizienten Rijji nicht eindeutig festgelegt. Er kann, wie Riemann gesehen hat, als (symmetrische) Matrix R auf
-dimensionalen Raum Λ2 der Flächenvariablen ∆xij aufgefasst werden;
dem n(n−1)
2
die Koeffizienten Rijji sind aber nur irgendwelche Matrixkoeffizienten von R und
keineswegs die Eigenwerte. Das kann auch nicht durch eine geeignete Koordinatentransformation erreicht werden, weil die Eigenvektoren von R im Allgemeinen
keine “zerlegbaren Vektoren” sind, also vom Typ ei ∧ ej , sondern Summen solcher
Vektoren.
182Elwin Bruno Christoffel, 1829 (Monschau) - 1900 (Straßburg),
Rudolf Otto Sigismund Lipschitz, 1832 (Königsberg) - 1903 (Bonn),
Tullio Levi-Civita, 1873 (Padua) - 1941 (Rom)
STERNSTUNDEN DER MATHEMATIK
107
scheint erst Levi-Civita in einer Arbeit von 1917 diesen Gedanken gefasst zu haben,183 und nicht etwa Christoffel 50 Jahre zuvor. Mit der
Parallelverschiebung wird ein Vergleich von Vektoren auf der Mannigfaltigkeit an verschiedenen Orten möglich. Damit können wir Vektoren
(eigentlich: Vektorfelder, d.h. Vektoren in jedem Punkt der Mannigfaltigkeit) sogar differenzieren, weil wir die Differenz von zwei Vektoren
bilden können, die sich an unterschiedlichen Orten befinden. Insbesondere können wir längs der Koordinatenlinien differenzieren und erhalten die partiellen Ableitungen von Vektorfeldern; wir nennen sie ∇i
statt ∂i . Schreibt man sie in Koordinaten, so ist ∇i = ∂i + Γi , wobei
bei der Anwendung von Γi nicht mehr abgeleitet, sondern nur noch
multipliziert wird; die Γi selbst werden aus den metrischen Koeffizienten gij und ihren ersten Ableitungen gebildet. Diese neue “Levi-CivitaAbleitung” erfüllt die gleichen Rechenregeln wie die gewöhnliche partielle Ableitung, mit einer Ausnahme: Die Regel ∂i ∂j = ∂j ∂i überträgt
sich nicht so einfach, vielmehr ist [∇i , ∇j ] := ∇i ∇j − ∇j ∇i = Rij der
Riemannsche Krümmungstensor. Tatsächlich ist
(64)
Rij = [∂i + Γi , ∂j + Γj ] = ∂i Γj − ∂j Γi + [Γi , Γj ]
ein Ausdruck in den Koeffizienten gij und ihren ersten und zweiten
partiellen Ableitungen.
Wie Hermann Weyl in seinem Kommentar zu Riemanns Vorlesung bemerkt, hat Riemann für die Geometrie einen Schritt getan, der für
die Physik bereits vollzogen war (besonders durch Faraday und Maxwell): der Übergang von der Fernwirkungs- zu einer Nahwirkungstheorie. Elektrische Ladungen oder Massen an verschiedenen Orten ziehen
sich an, aber es gibt etwas dazwischen, das diese Anziehung vermittelt,
das elektrische Feld oder das Gravitationsfeld. Ebenso gibt es etwas,
das den Abstand weit entfernter Punkte vermittelt: die Riemannsche
Metrik.
Durch die Trennung von zugrundeliegender Mannigfaltigkeit und
darauf definierter Metrik wurde es möglich, Geometrie und Physik auf
eine neue Weise zu verbinden: Die Geometrie stellt nicht nur die Bühne,
auf der sich die Physik abspielt, wie Newton und Kant es noch gesehen
haben, sondern die Geometrie wird durch die Physik selbst verändert
und bestimmt. Dieser Schritt wurde allerdings erst 60 Jahre später
durch Albert Einstein in seiner Allgemeinen Relativitätstheorie vollzogen: Die Gravitationskraft bestimmt die Krümmung und damit die
183Tullio
Levi-Civita: Nozione di parallelismo in una varietà qualunque e conseguente specificazione geometrica della curvatura riemanniana, Rend. Circ. Mat.
Palermo 42 (1917), 173 - 205
108
J.-H. ESCHENBURG
Geometrie von Raum und Zeit. Wir werden im Abschnitt 7 darüber
berichten. Soweit hat Riemann nicht gedacht, aber die letzten Worte
seines Vortrags weisen in diese Richtung: “Es muss also entweder das
dem Raume zugrunde liegende Wirkliche eine diskrete Mannigfaltigkeit bilden [wo man Messen durch Zählen ersetzen kann, J.E.] oder der
Grund der Maßverhältnisse außerhalb, in darauf wirkenden bindenden
Kräften gesucht werden. . . . Es führt dies hinüber in das Gebiet einer
anderen Wissenschaft, in das Gebiet der Physik, welches wohl die Natur
der heutigen Veranlassung nicht zu betreten erlaubt.”
Übungen
11.1. Diagonalisierung quadratischer Formen. a) Diagonalisieren
Sie die quadratische Form g(x, y) = x2 + 2axy + by 2 , indem Sie die
quadratische Ergänzung a2 y 2 hinzufügen und wieder abziehen; dann
2
2 2
2
wird g(x, y) = (x + ay)
√ + (b − a )y . Falls b > a (Fall 1), substituieren
wir x + ay = x̃ und b−a2 y = ỹ und erhalten
g(x, y) = x̃2 + ỹ 2 . Wenn
√
andererseits b < a2 (Fall 2), muss man a2 −b y = ỹ substituieren und
erhält g(x, y) = x̃2 − ỹ 2 .
b) Eine P
quadratische Form in n Variablen x = (x1 , . . . , xn ) ist vom Typ
g(x) = i,j gij xi xj . Für jedes Paar i, j mit gij 6= 0 können wir das gleiche Verfahren durchführen, müssen aber durch gii oder gjj dividieren.
Allerdings gibt es den Sonderfall gii = gjj = 0, aber gij = a 6= 0. In
dem Term axi xj substituieren wir dann xi = x̃i + x̃j und xj = x̃i − x̃j
und erhalten axi xj = ax̃2i − ax̃2j . Bei positiven quadratischen Formen
kann dieser Fall also gar nicht auftreten. Insgesamt erhält man im Fall
positiver quadratischer Formen wieder eine Summe von Quadraten, im
allgemeinen Fall können auch negative Quadrate auftreten.
11.2. Riemanns Definition des Krümmungstensors. In Normalkoordinaten (Exponentialkoordinaten) mit Zentrum x gilt gij = δij
nicht nur in x, sondern längs der ganzen Geraden (Geodäten) Rei , auf
der sich nur die Koordinate xi ändert und alle anderen xk konstant Null
sind. Das ist das Gauß-Lemma.184 Also folgt gij,i = 0, wobei der Index
i hinter dem Komma die partielle Ableitung in Richtung xi bedeutet.
Durch weitere partielle Ableitung nach xk erhalten wir gij,ik = 0 für alle
i, j, k. Durch Polarisierung (die Richtung ei ist nicht ausgezeichnet; wir
können jeden Einheitsvektor dafür einsetzen) folgt, dass der Ausdruck
184Der
Tangentenvektor der Geodäten Rei hat konstante Länge 1, und die Normalkoordinaten (Exponentialkoordinaten) erhalten rechte Winkel zu den radialen
Strahlen.
STERNSTUNDEN DER MATHEMATIK
109
gij,kl in dem Indexpaar i, k antisymmetrisch ist: gij,kl = −gkj,il . Außerdem ist gij,kl in den Indexpaaren i, j und k, l natürlich symmetrisch.
PRiemann behauptet, dass die 2. Ableitung (die Hesse-Matrix) von
gij dxi dxj folgendermaßen umgeformt werden kann:
X
X
(∗)
gij,kl dxi dxj δxk δxl =
Rijkl ∆xij ∆xkl
i,j,k,l
i,j,k,l
für irgendwelche Koeffizienten Rijkl , wobei ∆xij = dxi δxj − dxj δxi .
Multiplizieren Sie die rechte Seite dieser Gleichung aus, d.h. berechnen
Sie ∆xij ∆xkl , und vergleichen Sie mit der linken Seite. Dann sehen Sie,
dass (∗) besagt, dass manche der gij,kl gleich sein müssen; es sind dies
genau die Gleichheiten, die wir oben aus dem Gauß-Lemma abgeleitet
haben.
12. F. Klein: Ikosaeder und quintische Gleichung (1884)
Lösungsformeln für die Gleichungen dritten und vierten Grades wurden schon im 16. Jahrhundert gefunden (vgl. Abschnitt 5), aber die allgemeine Gleichung 5. Grades widerstand allen Versuchen, eine Lösungsformel zu finden. Drei Jahrhunderte später gelangten Ruffini (1799,
1813)185 und Abel (1824, 1826) zu der Erkenntnis, dass es keine Lösungsformel geben kann, die außer den vier Grundrechenarten nur Wurzeln
beliebigen Grades benutzt, vgl. Seite 88 in Abschnitt 9. Und doch gibt
es einen Lösungsweg; er benutzt noch eine neue Rechenart jenseits
der Grundrechenarten und des Wurzelziehens. Er wurde um 1858 unabhängig voneinander von drei Mathematikern gefunden: Brioschi,186
Hermite187 und Kronecker188. Dieser Lösungsweg ist ebenso speziell wie
der der Gleichungen dritten und vierten Grades und nicht auf höheren
Grad verallgemeinerbar. Felix Klein189 legte 1884 [6] eine ganz eigene
Version davon vor, von der ich hier berichten möchte;190 sie legt den
geometrischen Grund für diese merkwürdige Tatsache offen.
Wie bei Galois (Abschnitt 9) spielen die Permutationen eine große
Rolle, diesmal von 5 Gegenständen, den fünf Lösungen der quintischen
185Paolo
Ruffini, 1765 (Valentano) - 1822 (Modena)
Brioschi, 1824 - 1897 (Mailand)
187Charles Hermite, 1822 (Dieuze, Lothringen) - 1901 (Paris)
188Leopold Kronecker, 1823 (Liegnitz) - 1891 (Berlin)
189Felix Klein, 1849 (Düsseldorf) - 1925 (Göttingen)
190J.-H. Eschenburg, L. Hefendehl-Hebeker: Die Gleichung 5. Grades: Ist Mathematik erzählbar? Math. Semesterberichte 47 (2000), 193 - 220, www.math.uniaugsburg.de/∼eschenbu
186Francesco
110
J.-H. ESCHENBURG
Gleichung.191 Ähnlich wie die Permutationen von 4 Gegenständen durch
die Drehungen des Würfels ausgedrückt werden können (vgl. Fußnote
146), haben die Permutationen von 5 Gegenständen192 mit den Drehungen eines anderen platonischen Körpers zu tun, des Ikosaeders, das
von zwanzig gleichseitigen Dreiecken mit 12 Eckpunkten und 30 Kanten
begrenzt wird.
Wir können das Ikosaeder so positionieren, dass 6 der 30 Kanten parallel zu den drei Raumachsen liegen; die Mittelpunkte dieser Kanten
können wir durch die Kanten eines einbeschriebenen Oktaeders193 verbinden. Nach einer Drehung des Ikosaders übernehmen 6 andere Kanten diese Rolle. Somit enthält das Ikosaeder 30/6 = 5 Oktaeder, die
durch die Drehungen des Ikosaeders permutiert werden. Auf diese Weise
definiert jede Ikosaederdrehung eine Permutation der Menge der einbeschriebenen Oktaeder, die wir mit der Zahlenmenge {1, . . . , 5} identifizieren können, und verschiedene Drehungen definieren verschiedene
191
Wenn keine Gleichungen (“Relationen”) zwischen den Lösungen einer Gleichung xn + a1 xn−1 + · · · + an = 0 außer denen des Vietaschen Wurzelsatzes (Seite 82) bekannt sind, ist die Galoisgruppe (Seite 85) die volle Permutationsgruppe Sn . Sie kann aber immer auf den Normalteiler An ⊂ Sn reduziert werden
durch Hinzunahme
Q folgender Relation: Die Diskriminante ist das symmetrische
Polynom D = i6=j (xi − xj ). Dieses lässt sich mit Newtons Algorithmus durch
die Koeffizienten a1 , . . . , an ausdrücken. In D kommt aber jeder Faktor doppelt
vor, mit unterschiedlichem Vorzeichen. Dies kann man durch Ziehen der Quadratwurzel
vermeiden √
und erhält damit eine neue Gleichung zwischen den Lösungen,
Q
±D =: ∆, die nicht unter allen Permutationen invariant ist,
i<j (xi − xj ) =
sondern nur unter den geraden Permutationen.
192
Man muss sich dabei auf die geraden Permutationen beschränken, solche, die
aus einer geraden Anzahl von Paarvertauschungen zusammengesetzt sind. Sie bilden
eine halb so große Teilmenge A5 der Menge S5 aller Permutationen, A5 hat also 60
Elemente.
193
Das Oktaeder ist die Doppelpyramide über einem Quadrat, begrenzt von 8
gleichseitigen Dreiecken.
STERNSTUNDEN DER MATHEMATIK
111
Permutationen. Die Ikosaedergruppe wird dadurch zu einer Untergruppe der Permutationsgruppe S5 . Weil das Ikosaeder 20 Flächen hat,
deren jede nach oben gedreht werden kann, und jede Fläche von drei
Kanten berandet wird, deren jede nach vorne gedreht werden kann, gibt
es 20 · 3 = 60 Positionen des Ikosaeders und ebenso viele Drehungen.
Die Ikosaedergruppe wird damit zu einer 60-elementigen Untergruppe
der Gruppe S5 , die selbst 5! = 120 Elemente besitzt, und die einzige
solche Untergruppe ist die A5 . Die Drehgruppe des Ikosaeders ist also
isomorph zur A5 . Wenn wir jedes der 20 Dreiecke durch seine Symmetrieachsen in 6 Teildreiecke zerlegen und das so entstandene Muster
auf die Kugelfläche auftragen, die die Ikosaederecken enthält, dann entstehen auf der Kugelfläche 120 sphärische Dreiecke mit Winkeln π2 , π3
und π5 . Jedes zweite Dreieck wird gefärbt, und die Drehgruppe des
Ikosaeders bildet alle gefärbten Dreiecke ebenso wie alle ungefärbten
aufeinander ab.
q
π/5
π/2
0
π/3
1
Der Schlüssel zur Lösung der quintischen Gleichung ist eine Abbildung q von der Kugelfläche auf die Kugelfläche, die die gefärbten
Dreiecke auf die obere Halbkugel und die ungefärbten auf die untere Halbkugel abbildet und dabei invariant unter der Ikosaedergruppe
ist, q(gx) = q(x) für jeden Punkt x der Kugelfläche und jede Ikosaederdrehung g. Insbesondere werden Eckpunkte, Kantenmittelpunkte und
Flächenmittelpunkte auf je einen Punkt auf dem Äquator, der Randlinie der oberen oder unteren Halbkugel abgebildet; in Hinblick auf die
Identifizierung der Kugelfläche mit C ∪ {∞}, die wir gleich erklären,
bezeichnen wir diese drei Punkte mit 0, 1, ∞. Wir werden diese Abbildung q als Ikosaederfunktion bezeichnen; ihre Umkehrung wird die
neue Rechenart sein, die wir zur Auflösung der quintischen Gleichung
benötigen.
112
J.-H. ESCHENBURG
N
S
1−t
1
0
t
w
p=(w;t)
2
x
IR
Um die Ikosaederabbildung explizit zu berechnen, muss man zunächst
die Kugelfläche mit Ĉ = C ∪ {∞} identifizieren (Riemannsche Zahlenkugel). Das geschieht mit Hilfe der Stereographischen Projektion, einer
Abbildung, die die Kugelfläche (Sphäre) S ⊂ R2 × R (mit Ausnahme ihres höchsten Punkts, des “Nordpols” N = (0; 1)) auf R2 = C
abbildet. Sie ist geometrisch definiert: Jeder Punkt p = (w; t) der
Sphäre wird vom Nordpol N aus in gerader Linie auf einen Punkt
x der Äquatorebene R2 × {0} ⊂ R2 × R projiziert. Der Nordpol selbst
wird nicht projiziert; ihm ordnen wir den unendlich fernen Punkt ∞
außerhalb von C zu. Wegen der Ähnlichkeit der Dreiecke in der Figur
w
finden wir x : 1 = w : (1 − t), also x = 1−t
, das ist die analytische (formelmäßige) Darstellung dieser Abbildung.194 Aber diese ist gar nicht
nötig, wenn wir zwei Eigenschaften der Stereographischen Projektion
kennen: Sie ist kreistreu und winkeltreu, d.h. Kreise in S werden auf
Kreise in C = R2 abgebildet, und die Winkel, unter denen sie sich
schneiden, sind auf S und C die gleichen.195 Allein mit dieser Information können wir das stereographische Bild des Ikosaeders zeichnen und
berechnen.
194Umgekehrt ist p = N + s(x − N ) = (1 − s)N + sx für ein s 6= 0 und 1 =
|p|2 = (1 − s)2 + s2 |x|2 , also 0 = −2s + s2 (|x|2 + 1) und daraus s = 2/(|x|2 + 1) und
1−s = (|x|2 − 1)/(|x|2 + 1), also p = |x|21+1 (2x; |x|2 − 1).
195Vgl. z.B. “Geometrie”, Seite 84, auf www.math.uni-augsburg.de/∼eschenbu
STERNSTUNDEN DER MATHEMATIK
113
Dabei haben wir eine der 12 Ikosaederecken auf den Ursprung 0 in der
Mitte abgebildet; dann geht die gegenüberliegende Ecke auf den Punkt
∞, deshalb sehen wir nur 11 der 12 Ecken. Die Kanten des Ikosaeders
auf der Kugelfläche sind Großkreisbögen, die sich als Symmetrieachsen der anschließenden beiden Ikosaederdreiecke und danach als gegenüberliegende Kanten fortsetzen. In unserem Bild sind es Kreise, wobei die Kanten durchgezogen und die Symmetrieachsen gestrichelt sind.
Klar ist die Fünfersymmetrie in unserem Bild; die fünf zum Ursprung
0 benachbarten Eckpunkte bilden ein regelmäßiges Fünfeck. Ebenso
bilden die fünf weiter entfernten Eckpunkte ein größeres Fünfeck, das
gegenüber dem ersten um einen halben Fünfeckwinkel (36 Grad) verdreht ist; das entnimmt man sofort der Ikosaedersymmetrie. Unklar
sind zunächst die Durchmesser der beiden Fünfecke. Hier gibt es eine
Überraschung: Wir wissen zwar, dass die Stereographische Projektion
Kreise in Kreise abbildet, aber über die Mittelpunkte der Bildkreise
kann man im Allgemeinen nichts aussagen. Doch im vorliegenden Fall
sind die Mittelpunkte der Kantenkreise selbst Eckpunkte des Ikosaeders
(linke Figur).
114
J.-H. ESCHENBURG
α+1
α
α
2
3
72
α+α
18
144
0
18
2
α+α4 1
0
3
α +α
3
α
α4
Der Grund ist die Winkeltreue: Der Kreis in der linken Figur geht durch
die zwei Eckpunkte links und schließt den richtigen Winkel 72 Grad mit
der Kante ein, die diese mit dem Eckpunkt 0 verbindet (der radialen
Strecke); da der Kreis durch diese Angaben eindeutig bestimmt ist, ist
er das Bild des Kantengroßkreises. Der Punkt auf diesem Kreis, der
vom Ursprung 0 aus dem linken unteren Eckpunkt gegenüber liegt,
muss also wieder ein Eckpunkt des Ikosaeders sein. Die rechte Figur
zeigt, dass man die Eckpunkte damit bestimmt hat: sie sind
0, ∞, αk (α + α3 ), αk (α2 + α3 ) für k = 0, 1, 2, 3, 4
mit α := ei·2π/5 . Wie können wir das sehen? Da diese Punkte schon in
der richtigen Richtung liegen und es auf den Maßstab nicht ankommt,
brauchen wir nur noch zu überprüfen, dass die Punkte α +α3 und α +1
wirklich auf einem gemeinsamen Kreis um α + α4 liegen, dass also die
Abstände der beiden Punkte zu α + α4 die gleichen sind:
|(α + α3 ) − (α + α4 )| = |α3 − α4 | = |α3 (1 − α)| = |1 − α| ,
|(α + 1) − (α + α4 )| = |1 − α4 | = |1 − ᾱ| = |1 − α| .
Da nun die Kantenkreise bekannt sind, können wir auch die Position
der Kanten- und Flächenmittelpunkte (als Schnittpunkt von Kantenkreisen) bestimmen.
Nun suchen wir eine Abbildung q, die jedes der kleinen Kreisdreiecke in der Figur auf Seite 114 abwechselnd auf die obere oder untere
Halbebene C± = {u + iv : ±v > 0} abbildet. Die drei Eckpunkte des
Kreisdreiecks sollen dabei auf 0, 1, ∞ gehen: die “6-zählige” Ecke (mit
6 Verbindungen) auf 0, die 4-zählige auf 1 und die 10-zählige auf ∞
STERNSTUNDEN DER MATHEMATIK
115
(auf der erweiterten reellen Achse).196 Diese Funktion q kann man als
rationale Funktion (Quotient von Polynomen) q = q1 /q2 aus diesen
Angaben explizit berechnen. Da q in den 6-zähligen Punkten den Wert
0 annehmen soll, muss q1 dort seine Nullstellen haben. In jedem solchen Punkt s ist der Dreieckswinkel π/3 (siehe Figuren auf Seite 111,
114); dieser Winkel soll durch q zu π gestreckt werden. Deshalb muss
sich q(x) nahe x = s verhalten wie die dritte Potenz (x − s)3 , die ja
bekanntlich die Winkel in s verdreifacht. Da dies in jedem 6-zähligen
Punkt gilt, können wir q1 = h3 ansetzen für ein Polynom h, das genau in diesen 20 Punkten einfache Nullstellen besitzt und damit vom
Grad 20 ist. Mit dem Vietaschen Wurzelsatz sehen wir, dass h proportional ist zum Produkt der Linearfaktoren (x − s), wobei s alle 20
sechszähligen Punkte durchläuft. Das Ergebnis ist (nachdem man diese
Punkte berechnet hat)
(65)
h(x) = x20 − 228x15 + 494x10 + 228x5 + 1 .
In den 10-zähligen Ecken t soll q einen Pol und somit 1/q = q2 /q1
eine Nullstelle haben, also muss q2 dort eine Nullstelle haben. Der Dreieckswinkel in t ist π/5 und soll zu π aufgebogen werden, was mit einer
fünften Potenz möglich ist: q2 (x) ∼ (x − t)5 . Somit ist q2 = f 5 für ein
Polynom f , das in jedem der 11 Eckpunkte eine einfache Nullstelle hat:
f (x) = x
5
Y
j=1
(x − tj )
10
Y
(x − tk ) = xf1 (x)f2 (x)
k=6
mit tj = aαj und tk = bαk für a = α + α4 und b = α2 + α3 .197 Die
Funktion f ist nicht
Q schwer zu berechnen. Die Koeffizienten des Teilpolynoms f1 (x) = 5j=1 (x − tj ) sind nach dem Vietaschen Wurzelsatz
aj = ±aj ǫj (~
α), wobei α
~ = (α1 , . . . , α5 ) und ǫj die j-te elementarsymmetrische Funktion ist. Da α1 , . . . , α5 die Nullstellen des Polynoms x5 − 1
sind, ist ǫ5 (~
α) = 1 und ǫj (~
α) = 0 für j 6= 5. Also ist f1 (x) = x5 − a5 ,
Q
5
5
und entsprechend ist f2 (x) = 10
k=6 (x − tk ) = x − b , und
f1 (x)f2 (x) = x5 − a5 x5 − b5
mit
a5 = (α + α−1 )5 = 2 + 5(α3 + α2 ) + 10(α + α4 ) = 2 + 5b + 10a,
196Dass
dies möglich ist, sagt gerade der Riemannsche Abbildungssatz; dass sich
die Abbildung über den Rand der Dreiecke holomorph fortsetzt, ist das Schwarzsche Spiegelungsprinzip. Beides wird aber nicht wirklich gebraucht, weil wir die
Abbildung direkt berechnen können.
197Die Zahl a = α + α4 ist der reziproke Goldene Schnitt, und b = α2 + α3 der
negative Goldene Schnitt, siehe Figur auf Seite 7.
116
J.-H. ESCHENBURG
b5 = (α2 + α−2 )5 = 2 + 5(α + α4 ) + 10(α2 + α3 ) = 2 + 5a + 10b.
Mit a + b = α + α2 + α3 + α4 = −1 folgt a5 + b5 = 4 − 5 − 10 = −11
und ab = (α + α4 )(α2 + α3 ) = α3 + α4 + α6 + α7 = −1, also
f1 (x)f2 (x) = x10 − (a5 + b5 )x5 + (ab)5
= x10 + 11x5 − 1 .
Somit erhalten wir
(66)
f (x) = x(x10 + 11x5 − 1) .
Unsere Funktion q ist proportional zu h3 /f 5 ; diese Funktion hat auch
in der noch fehlende Ikosaederecke ∞ einen Pol vom Grad 5, weil der
Zählergrad 60 um 5 höher ist als der Nennergrad 55. Den Proportionalitätsfaktor ermitteln wir aus der noch fehlenden Bedingung in
den Kantenmittelpunkten: dort gilt q = 1, es sind also Nullstellen von
q1 − q2 . Damit erhalten wir
(h/12)3
.
f5
Diese rationale Funktion q übernimmt die Rolle der k-ten Potenz,
der rationalen Funktion x 7→ xk ; ihre Umkehrfunktion198 wird die neu
benötigte Rechenart sein, analog zur k-ten Wurzel, der Umkehrung der
k-ten Potenz.
(67)
q=
Gegeben sei nun eine allgemeine Gleichung 5. Grades. Durch eine Variablentransformation (Tschirnhaus-Transformation),199 vgl. Übung 12.1,
kann man die ersten beiden Koeffizienten zum Verschwinden bringen
und die Gleichung in der Form
(68)
x5 + ax2 + bx + c = 0
schreiben. Weil der x4 -Koeffizient verschwindet, gilt die lineare Beziehung
(69)
ǫ1 (~x) = x1 + · · · + x5 = 0.
Weil auch der x3 -Koeffizient verschwindet, gilt die quadratische
P 2Bezie2
hung ǫ2 (~x) = 0, aber wegen p2 = ǫ1 − 2ǫ2 (mit p2 (~x) = i xi ) und
ǫ1 = 0 kann diese Beziehung auch in der Form
(70)
198Die
p2 (~x) = x21 + · · · + x25 = 0
Funktion q ist natürlich nicht eindeutig umkehrbar: Ist x ein Punkt im
Inneren eines der Dreiecke, sagen wir, eines gefärbten, so hat q(x) in jedem anderen
gefärbten Dreieck ebenfalls ein Urbild x′ mit q(x′ ) = q(x). Alle diese 60 Urbilder
stehen für die Umkehrfunktion zur Auswahl, ähnlich wie ja auch die k-te Wurzel
√
k y nicht eindeutig ist, sondern k verschiedene Werte annehmen kann.
199Ehrenfried Walther von Tschirnhaus, 1651 (bei Görlitz) - 1708 (Dresden)
STERNSTUNDEN DER MATHEMATIK
117
geschrieben werden. Das ist wegen (69) eigentlich eine Gleichung in
vier Variablen, da z.B. x5 = −(x1 + · · · + x4 ). Außerdem genügt es,
~x = (x1 , . . . , x5 ) nur bis auf ein Vielfaches zu bestimmen,200 also nur
~x′ = t~x mit unbekanntem t ∈ C. Denn ǫ3 (~x′ ) = t3 ǫ3 (~x) = −t3 a und
x′ )
4 (~
leicht zu berechnen. Wir können
analog ǫ4 (~x′ ) = t4 b, also ist t = − aǫ
bǫ3 (~
x′ )
also eine der fünf Koordinaten willkürlich gleich Eins setzen; dann wird
(70) zu einer Beziehung zwischen drei Koordinaten. Die Lösungsmenge
dieser quadratischen Gleichung ist eine Fläche (zwei Koordinaten lassen sich beliebig vorgeben, die dritte kann aus der Gleichung berechnet
werden), eine “Quadrik” Q. Wir kennen solche Quadriken aus der reellen Geometrie, zum Beispiel den Hyperboloiden
QH = {(x, y, z) : x2 + y 2 − z 2 = 1}.
Das Besondere: Auf dieser krummen Fläche liegen zwei Scharen von
Geraden! Diese zweifache Geradenschar sieht man noch einfacher, wenn
man (70) (nach Einsetzen von x5 = −(x1 + · · · + x4 ) und x4 = 1) durch
eine lineare Variablensubstitution201 ~x = ~x(λ, µ, ν) (vgl. Übung 12.4)
auf die Form
(71)
λµ = ν
gebracht hat, denn für jedes konstante λ oder µ beschreibt diese Gleichung eine Gerade. Über den reellen Zahlen gibt es zwar auch Quadriken ohne Geraden, zum Beispiel die Sphäre, die Lösungsmenge der
Gleichung x2 + y 2 + z 2 = 1, aber über den komplexen Zahlen können
wir jede (nicht-entartete) Quadrik durch eine lineare Substitution auf
die Form (71) bringen.202
Jede Permutation σ ∈ S5 erhält die Gleichungen (69), (70) und damit ihre Lösungsmenge, die Quadrik Q. Außerdem bildet σ Geraden
auf Geraden ab. Wenn σ eine gerade Permutation ist (σ ∈ A5 ), so
werden die Geraden der beiden Scharen λ = const und µ = const auf
200Wir
suchen damit statt ~x das Element [~x] des Projektiven Raums.
genommen ist diese Substitution erst nach Homogenisierung linear,
siehe Übung 12.4.
202Mit der Substitution z = iz̃ wird zum Beispiel aus der Kugelgleichung x2 +
2
y + z 2 = 1 die Hyperboloidgleichung x2 + y 2 − z̃ 2 = 1.
201Genau
118
J.-H. ESCHENBURG
Geraden der gleichen Schar abgebildet, die ungeraden Permutationen
dagegen vertauschen die beiden Scharen. Jedes σ ∈ A5 wirkt also auf
den Punkten (λ, µ) in der Form (λ, µ) 7→ (σ1 (λ), σ2 (µ)), und die Abbildungen σj (j = 1, 2) sind Transformationen der Kugelfläche Ĉ für
jedes σ ∈ A5 . Es ist nicht schwer zu erraten, dass diese Transformationen Ikosaederdrehungen sind; es gibt keine andere Wirkung der A5
als Gruppe von Transformationen der Sphäre. Also sind q(λ) und q(µ)
invariant unter der A5 und daher (nach Newton, vgl. Seite 90) aus den
Koeffizienten a, b, c berechenbar.203 Durch Umkehrung von q gewinnen
wir λ aus q(λ) und µ aus q(µ) und aus λ, µ und ν = λµ den Vektor ~x′
und daraus schließlich ~x.
Es bleibt die Frage, wie die Funktion q invertiert wird. Klein kann
sich dazu auf H.A. Schwarz204 berufen, der eine Potenzreihe für die
lokalen Umkehrungen gefunden hat. Schwarz betrachtete dazu lineare
Differentialgleichungen205
(72)
y ′′ + py ′ + qy = 0
für gegebene reelle rationale Funktionen p und q mit Polen in 0, 1, ∞.
In der Nähe jedes Punktes xo 6= 0, 1, ∞ gibt es zwei linear unabhängige
Lösungen y1 , y2 , aus denen sich alle anderen Lösungen zusammensetzen
lassen; das Paar (y1 , y2 ) bezeichnet man auch als Grundlösung. Insbesondere kann man jede andere Grundlösung (ỹ1 , ỹ2 ) damit ausdrücken:
(73)
ỹ1 = ay1 + by2 ,
ỹ2 = cy1 + dy2
für konstante a, b, c, d. Schwarz betrachtet stattdessen die Quotien, kurz: η̃(x) = g(η(x)) für
ten η = y1 /y2 und η̃ = ỹ1 /ỹ2 = aη+b
cη+d
ax+b
die gebrochen-lineare Funktion g(x) = cx+d . Wir können annehmen,
dass y1 , y2 auf dem reellen Intervall (0, 1) reell sind. Auf den anschließenden Intervallen (−∞, 0) und (1, ∞) sind andere Grundlösungen
reell, sagen wir (y1− , y2− ) und (y1+ , y2+ ). Alle diese Funktionen lassen
sich “analytisch”, d.h. als Lösungen von (72) in die obere Halbebene
C+ = {u + iv : v > 0} fortsetzen. Dort erfüllen sie Relationen vom Typ
203Wenn ein Polynom f (~
x) nur unter A
statt Sn invariant ist, setzen wir f ∗ (~x) =
f (x2 , x1 , x3 , . . . ). Dann sind f + f und (f − f ∗ )2 invariant unter Sn und daher
rationale Funktionen der elementarsymmetrischen Polynome ei . Damit sind auch
f und f ∗ aus den Koeffizienten berechenbar.
204Hermann Amandus Schwarz, 1843 (Hermsdorf, Schlesien) - 1921 (Berlin):
Über diejenigen Fälle, in denen die Gaussische hypergeometrische Reihe eine algebraische Funktion ihres vierten Elementes darstellt, J. Reine u. Angew. Math. 75
(1873), 292 - 335
205Eine Differentialgleichung ist eine Gleichung, in der eine Funktion y(x) und
ihre Ableitungen (Änderungen) y ′ , y ′′ , . . . vorkommen.
∗
n
STERNSTUNDEN DER MATHEMATIK
119
(73), und für die zugehörigen Quotienten η ± = y1± /y2± gilt η = g ± (η ± ).
Gebrochen-lineare Funktionen sind kreistreu (wobei auch Geraden zu
den Kreisen gerechnet werden müssen). Weil J − = η − (−∞, 0) und
J + = η + (1, ∞) Teile der erweiterten reellen Geraden R ∪ {∞} sind,
sind η(−∞, 0) = g − (J − ) und η(1, ∞) = g + (J + ) Kreisbögen, und η(C+ )
ist ein Kreisbogen-Dreieck. Durch Spiegelung an diesen Kreisbögen
(Schwarzsches Spiegelungsprinzip) findet man analytische Fortsetzungen von η in die untere Halbebene. Durch Wiederholungen dieses Prozesses entstehen viele Kreisbogendreiecke auf der Sphäre Ĉ, die jeweils
an einer Kante aneinander grenzen, aber sich sonst im Allgemeinen
in unübersichtlicher Weise überlappen. Die Funktionen g ± erzeugen
eine Gruppe von gebrochen-linearen Transformationen von Ĉ, die im
Allgemeinen sehr groß ist; sie heißt die Monodromiegruppe der Differentialgleichung (72). Schwarz untersucht nun die Fälle, in denen ein konsistentes Muster ohne Überlappungen auf der Sphäre entsteht. Dafür
verantwortlich sind die Innenwinkel des Kreisbogendreiecks η(C+ ), die
aus den Polen der gegebenen Funktionen p und q bestimmt werden
können. In einem dieser Fälle sind die Winkel π/3, π/2, π/5 in den
Punkten η(0), η(1), η(∞), und wir erhalten die Figur auf Seite 111,
wobei η eine Umkehrfunktion von q auf der oberen Halbsphäre ist. Da
wir y1 und y2 als Lösungen von (72) in Potenzreihen entwickeln können
(“hypergeometrische Reihe”), ist η = y1 /y2 berechenbar.
Was ist so bedeutsam an dieser Arbeit von Felix Klein, dass wir
sie unter die “Sternstunden” einreihen? Er löst ein Problem, über das
Mathematiker 300 Jahre lang nachgedacht haben. Die Lösung benutzt
die Gruppe A5 in vierfacher Weise:
(1) als Permutationsgruppe,
(2) als Galoisgruppe der quintischen Gleichung,
(3) als Ikosaedergruppe,
(4) als Monodromiegruppe der Differentialgleichung (72).
Erst das Auftreten der gleichen Gruppe in vier unterschiedlichen Zusammenhängen macht die Lösung der quintischen Gleichung möglich.
Die Mathematik entwickelt nicht nur allgemeine Theorien, sondern
untersucht auch sehr spezielle Phänomene. Die fünf Platonischen Körper
und unter ihnen besonders das Ikosaeder sind solche speziellen Phänome,
und sie stehen mit anderen Phänomenen in Verbindung, sogar mit den
auf Seite 98 erwähnten Ausnahmegruppen E6 , E7 , E8 (siehe Vorwort
von P. Slodowy206 zu Kleins Buch [6]).
206Peter
Slodowy, 1948 (Leverkusen) - 2002 (Bonn)
120
J.-H. ESCHENBURG
Übungen
12.1. Tschirnhaus-Transformation. Gegeben sei die quintische Gleichung
x5 + bx3 + cx2 + dx + e = 0
mit den Lösungen x1 , . . . , x5 . Zeigen Sie, dass Sie die Gleichung durch
die Transformation
x̃i = txi + x2i − p2 /5
für geeignetes t mit p2 := x21 + · · · + x25 auf die Form
˜ + ẽ = 0
x̃5 + c̃x̃2 + dx̃
˜ ẽ aus b, c, d, e berechenbar sind. Genauer:
bringen können, wobei c̃, d,
Das Polynom
f˜(x̃) =
5
Y
i=1
˜ + ẽ
(x̃ − x̃i ) = x̃5 + ãx̃4 + b̃x̃3 + c̃x̃2 + dx̃
erfüllt ã = 0 für jedes t und b̃ = 0 für geeignetes t. Benutzen Sie die
Potenzsummen pk = xk1 + · · · + xk5 ; diese sind symmetrisch und lassen sich daher durch b, c, d, e ausdrücken. Die genaue Formel ist nicht
gefragt. Beachten Sie aber die Formel (ǫ1 )2 = p2 + 2ǫ2 .
12.2. Ikosaederdrehungen. Zeigen Sie, dass es 60 verschiedene Drehungen des Ikosaeders gibt, nämlich 60 verschiedene Positionen. Beachten Sie, dass jede Seite nach oben und jede Kante nach vorn gedreht
werden kann. Überlegen Sie sich andererseits, dass es 120 Permutationen von 5 Gegenständen gibt: Der erste Gegenstand hat 5 Plätze
zur Wahl, der zweite noch 4, der dritte noch 3 usw. Die Hälfte davon, also 60, sind gerade Permutationen. Schließen Sie daraus, dass die
Drehungen des Ikosaeders genau den geraden Permutationen der fünf
einbeschriebenen Oktaeder entsprechen.
12.3. Geraden auf dem einschaligen Hyperboloid. Das einschalige Hyperboloid besteht aus allen Punkten mit Koordinatentripeln
(x, y, z), die die Gleichung
x2 + y 2 − z 2 = 1
erfüllen. Finden Sie die Geraden auf dieser Fläche. Es genügen zwei;
die übrigen entstehen dann durch Drehung um die z-Achse.
STERNSTUNDEN DER MATHEMATIK
121
12.4. Hyperboloid und Produktfläche. Eine Gleichung φ(~x) = 0
heißt homogen vom Grad k, wenn φ(t~x) = tk φ(~x) für alle Skalare t
und Vektoren ~x. Eine inhomogene Gleichung in drei Variablen wie die
Hyperboloid-Gleichung x2 + y 2 − z 2 = 1 (die linke Seite hat Grad 2, die
rechte Grad 0) kann man zu einer homogenen Gleichung in 4 Variablen
machen (“homogenisieren”), indem man die Summanden, die nicht den
höchsten Grad haben, mit der fehlenden Potenz der vierten Variablen
multipliziert, hier zum Beispiel
x2 + y 2 − z 2 = w 2 .
(1)
Setzt man die vierte Variable gleich Eins, so erhält man die ursprüngliche Gleichung zurück. Ebenso bei der Produktfläche: Die inhomogene
Gleichung (71) λµ = ν wird mit Hilfe einer vierten Variablen ρ homogenisiert zu
λµ = νρ.
(2)
Finden Sie eine lineare Transformation, die die beiden homogenisierten
Gleichungen (1) und (2) ineinander überführt. Hinweis: Nutzen Sie
y 2 − z 2 = (y + z)(y − z) und w2 − x2 = (w + x)(w − x).
13. Einstein: Philosophisches Rätsel gelöst (25.11.1915)
Was ist der Unterschied zwischen Ruhe und Bewegung? Ist Bewegung nicht nur ein relatives Phänomen? Ich kann in einem bequemen
ICE-Sessel ruhen, und doch bewegt sich der Zug mit 200 km/h durch
die Landschaft. Sind also Ruhe und Bewegung nur Schein, hervorgerufen durch das von mir willkürlich gewählte Bezugssystem (der fahrende
Zug oder die Landschaft), ohne objektive Bedeutung? Selbst die Geometrie, zum Beispiel der Unterschied zwischen “gerade” und “krumm”
scheint sich durch Wahl des Bezugssystems zu verändern: Wenn ich geradeaus durch einen Eisenbahnwaggon laufe, der in einer Kurve fährt,
befinde ich mich in einem Bezugssystem, das das Krumme (meine
Bewegung bezüglich der Landschaft) gerade erscheinen lässt (meine
Geradeaus-Bewegung durch den Waggon). Allerdings merke ich jetzt
einen Unterschied zur Ruhe oder zur geradlinigen Bewegung: Es werden nämlich seitliche Kräfte auf mich ausgeübt, denen ich Widerstand
entgegensetzen muss, um meine Geradeausbewegung fortzusetzen. Es
scheint also doch nicht “alles nur relativ” zu sein.
Die Wahl des Bezugssystems ist ein zentrales Problem der Geistesgeschichte. Seit über 5000 Jahren haben die Menschen die Himmelskörper
beobachtet, Sonne, Mond, die Planeten (Venus, Merkur, Mars, Jupiter
und Saturn), Kometen und Fixsterne. Sie nahmen die unterschiedlichen Bewegungen und Konstellationen wahr und ordneten ihnen sogar
122
J.-H. ESCHENBURG
Bedeutung für das menschliche Schicksal zu. Den Griechen gelang es,
diese Bewegungen auch theoretisch zu beschreiben und damit vorhersagbar zu machen (Ptolemäisches Weltbild).207 Das Bezugssystem für
die Beschreibung dieser Bewegungen war dabei ganz selbstverständlich
die Erde; diese erschien fest, die Himmelkörper dagegen bewegten sich
(geozentrisches System). Das gesamte Mittelalter hielt an dieser Vorstellung fest, erst die Renaissance war in der Lage, sie aufzugeben.
Im kopernikanischen Weltbild,208 später durch Kepler verfeinert, wurde die Sonne zum Zentrum und Ruhepol des Bezugssystems, die Erde
aber den Planeten gleichgeordnet (heliozentrisches System). Die von
der Erde aus zu beobachtenden komplizierten Planetenbewegungen ließen sich damit viel einfacher erklären, nämlich als Relativbewegung der
Bewegung zweier Himmelskörper, Erde und Planet. Die Aufgabe der
Vorstellung, die Erde und damit der Mensch stünden im Mittelpunkt
des Weltsystems, kam einer Revolution gleich (“kopernikanische Wende”) und führte zu der bekannten Auseinandersetzung zwischen Galilei209 und der katholischen Kirche, die sich den Menschen zu groß und
die übrige Schöpfung zu klein vorgestellt hatte.
Noch weit über das heliozentrische System hinaus ging Newton: Auch
die Sonne bewegt sich; selbst die Galaxie, zu der sie gehört, die Milchstraße, ruht nicht. Wo also war das in Ruhe befindliche Bezugssystem?
Newton machte einen radikalen Schnitt: Das ruhende Bezugssystem
war überhaupt nicht mehr an irgendwelchen Himmelskörpern festzumachen, es war vielmehr der von der Materie losgelöste “absolute”
Raum selbst. Er war die Bühne, auf der sich die Physik abspielte; die
Erde und alle Himmelskörper bewegten sich in ihm. Aber warum gab
es überhaupt ein solches “absolut ruhendes” Bezugssystem? Konnte
es nicht sein, dass Ruhe und Bewegung wirklich nur relative Begriffe
waren, immer auf ein zufällig gewähltes Koordinatensystem bezogen?
Newton gab eine physikalische Begründung für die Berechtigung, absolut von Ruhe oder Bewegung zu reden: Es gibt wahrnehmbare Käfte,
ähnlich wie in der eingangs geschilderte Situation der Bewegung im Zug
bei Kurvenfahrt. Lässt man einen Eimer mit Wasser um seine Achse
rotieren, so wird nach einer Weile (durch die Reibung) das Wasser im
Eimer genauso schnell rotieren wie der Eimer, sich also gegenüber dem
Eimer in Ruhe befinden. Aber die Wasseroberfläche wird (durch die
Zentrifugalkraft) gewölbt sein, anders als in der Situation ohne Rotation. Man kann also zwischen dem ruhenden und dem bewegten System
207Claudius
Ptolemäus, ca. 100 - 160 n.Chr. (Alexandria)
Kopernikus: De revolutionibus orbium coelestium, Nürnberg 1543
209Galileo Galilei, 1564 (Pisa) - 1642 (Arcetri bei Florenz)
208Nikolaus
STERNSTUNDEN DER MATHEMATIK
123
unterscheiden. Kant hat Newtons Ansatz auch philosophisch gerechtfertigt: Die Wahrnehmung des Raumes mit allen seinen Gesetzen (denen der euklidischen Geometrie) ist nicht Ergebnis unserer Erfahrung,
sondern “a priori”, vor aller Erfahrung gegeben; sie ist eine Bedingung
der Möglichkeit von Erfahrung, denn wir weisen allen Erscheinungen
der physikalischen Erfahrung bereits Plätze in diesem Raum zu.
Der österreichische Physiker und Philosoph Ernst Mach210 hat wohl
als erster die Schwäche von Newtons Eimer-Argument herausgearbeitet:
“Der Versuch Newtons mit dem rotirenden Wassergefäss
lehrt nur, dass die Relativdrehung des Wassers gegen die
Gefässwände keine merklichen Centrifugalkräfte weckt,
dass dieselben aber durch die Relativdrehung gegen die
Masse der Erde und die übrigen Himmelskörper geweckt
werden. Niemand kann sagen, wie der Versuch verlaufen
würde, wenn die Gefässwände immer dicker und massiger, zuletzt mehrere Meilen dick würden.”211
Einstein hat später vom “Machschen Prinzip” gesprochen; er verstand
darunter die Hypothese, dass die Trägheitskräfte durch die Gesamtheit
der im Universum vorhandenen Materie verursacht werden.
Eigentlich war Newtons Ansatz schon bei seinem Entstehen nicht
ganz konsistent, denn bereits Galilei hatte festgestellt, dass die Physik (damals im Wesentlichen die Mechanik) in zwei zueinander gleichförmig-geradlinig bewegten Bezugssystemen genau gleich beschrieben
wird. Mit anderen Worten, in einem zum “absoluten Raum” gleichförmig-geradlinig bewegten Bezugssystem herrscht dieselbe Physik, und
wir könnten dieses bewegte System ebensogut als den ruhenden “absoluten Raum” ansehen und den ruhenden als (in umgekehrter Richtung)
bewegt. Solche Systeme, in denen dieselbe Physik herrscht wie im absoluten Raum, wurden Inertialsysteme212 genannt. Statt des absoluten
Raums gab es also eigentlich bereits vor Newton eine Familie gleichberechtigter Bezugssysteme, die untereinander durch eine Gruppe von
linearen Transformationen von R4 = R3 × R, den Galileitransformationen verbunden waren. Wir bezeichnen zwei solche Systeme mit (x, t)
210Ernst
Waldfried Josef Wenzel Mach, 1838 (Chirlitz bei Brünn) - 1916 (Vaterstetten bei München)
211Ernst Mach: Die Mechanik in ihrer Entwicklung, Leipzig 1883,
http://www.archive.org/stream/diemechanikinih03machgoog#page/n230/mode/2up
212Der Begriff “Inertialsystem” (nach lat. “iners” = träge) wurde zuerst 1883
von dem Physiker und Physiologen Gustav Ludwig Lange, 1863 (Gießen) - 1936
(Weinsberg bei Heilbronn) verwendet.
124
J.-H. ESCHENBURG
und (x′ , t′ ). Wird das “gestrichene” System gegenüber dem “ungestrichenen” mit der Geschwindigkeit v in x-Richtung bewegt, so hat sich
sein Ursprung x′ = 0 zum Zeitpunkt t um die Strecke tv vom Ursprung
x = 0 des ungestrichenen entfernt; diese Strecke muss also im gestrichenen System gegenüber dem ungestrichenen abgezogen werden:
(74)
x′ = x − tv, t′ = t
Die Raumkoordinate ändert sich also, nur die Zeitkoordinate t bleibt
dieselbe; sie ist “absolut”.
Newtons Theorie des absoluten Raums hatte allerdings im 19. Jahrhunderts eine kurze Renaissance, als man die elektrodynamischen Erscheinungen zu untersuchen begann, insbesondere die Ausbreitung elektromagnetischer Wellen (Licht). Diese schienen doch einen Unterschied
in der Physik der verschiedener Inertialsysteme zu begründen, da die
Lichtgeschwindigkeit in einem auf die Lichtquelle zurasenden Bezugssystem größer sein müsste als in einem, das sich von der Quelle entfernt. Man postulierte ein Medium, in dem sich die Lichtwellen ausbreiteten, den sogenannten Äther. Diese Vorstellung stammt bereits
aus der Antike; Aristoteles213 verstand darunter das fünfte Element
(neben Feuer, Wasser, Luft und Erde), in dem sich die Himmelskörper
oberhalb des Mondes bewegten. Das Koordinatensystem, in dem der
Äther ruhte, war also gegenüber allen anderen Systemen zu bevorzugen. Aber die Versuche, Ende des 19. Jahrhunderts durch Messungen von Laufzeit-Differenzen (Michelson-Interferometer) die Bewegung
des erdgebundenen Bezugssystems gegenüber dem Äther nachzuweisen, brachten ein negatives Ergebnis:214 Die Lichtgeschwindigkeit war
in allen Richtungen dieselbe, in Richtung der Erdrotation, in Gegenrichtung und senkrecht dazu. Das war mit der Äthertheorie schwer zu
vereinbaren; schließlich bewegen wir uns durch die Erddrehung in jedem
Moment mit 1000 km/h nach Osten. Es gab aber keinen “Ätherwind”.
Das Galileische Relativitätsprinzip schien auch für die Elektrodynamik
gültig zu sein.
Aber damit geriet man in einen Widerspruch zu Grundregeln der
Physik: Die Lichtgeschwindigkeit ließ sich nicht zur Geschwindigkeit eines Bezugssystems addieren. Ich kann einem Dieb, der meine Geldbörse
geklaut hat, nachlaufen und ihn vielleicht sogar einholen. In meinem
mitgeführten Bezugssystem wird seine Geschwindigkeit dann negativ,
d.h. er scheint rückwärts auf mich zuzukommen. Aber wenn der Dieb
durch einen Lichtstrahl ersetzt wird, ist das nicht möglich: Wenn ich
213Aristoteles,
384 v.Chr. (Stagira, Chalkidike) - 322 v.Chr (Chalkis, Euböa)
214http://de.wikipedia.org/wiki/Michelson-Morley-Experiment
STERNSTUNDEN DER MATHEMATIK
125
einer Lichtwelle nachjage, kann ich ihre Geschwindigkeit in meinem Bezugssystem nicht verringern. Einstein löste in seiner Speziellen Relativitätstheorie (1905) nach Vorarbeiten von Lorentz215 und Poincaré216
diesen Widerspruch, indem er die absolute Zeit der absoluten Lichtgeschwindigkeit opferte: Die Transformationen zwischen den Inertialsystemen mussten das Ausbreitungsgesetz der (zur Zeit 0 vom Ursprung ausgehenden) Lichtwellen erhalten. Wenn sich eine Lichtwelle
vom Ursprung 0 mit Geschwindigkeit c in alle Richtungen gleichmäßig
ausbreitet, dann ist sie zur Zeit t am Ort x mit |x| = ct. Den Faktor c
können wir Eins setzen, wenn wir die Zeit in Sekunden und den Raum in
Lichtsekunden messen, die Entfernung, die das Licht in einer Sekunde
zurücklegt (ca. 300 000 km). Die Transformation in das mit Geschwindigkeit v (einem Bruchteil der Lichtgeschwindigkeit) bewegte System
muss also die “Diagonale” x = t erhalten; die Galileitransformation
(74) tut das nicht. Sie muss deshalb durch eine andere Transformation
ersetzt werden, die Lorentztransformation:
(75)
x′ = γ(x − tv), t′ = γ(t − vx)
√
mit γ = 1/ 1 − v 2 .217
t
t’
x=t
t
t’
x=t
x’
Galilei
x,x’
Lorentz
x
Hermann Minkowski218 interpretierte die Transformationen (75) als eine Art Drehungen in der xt-Ebene in R3 ×R, wobei aber die euklidische
Metrik ds2 = |dx|2 + dt2 durch die Minkowski-Metrik
(76)
ds2 = |dx|2 − dt2
ersetzt worden ist.
Einsteins radikaler Schritt hatte starke Auswirkungen auf die Mechanik. Die spektakulärste Konsequenz war die Gleichheit von Masse
und Energie, die berühmte Gleichung E = mc2 bzw. E = m, wenn
215Hendrik
Antoon Lorentz, 1853 (Arnheim) - 1928 (Haarlem)
Henri Poincaré, 1854 (Nancy) - 1912 (Paris)
217Der Faktor γ sorgt dafür, dass die Transformationsmatrix A = γ 1 −v Dev
−v 1
terminante Eins hat, also Flächeninhalte erhält. Andernfalls hätte die Umkehrtransformation A−v eine andere Determinante, det A−v = 1/ det Av , was nicht möglich
ist, weil die Richtungen von v und −v gleichberechtigt sind.
218
Hermann Minkowski, 1864 (Kaunas, Litauen) - 1909 (Göttingen)
216Jules
126
J.-H. ESCHENBURG
man c = 1 setzt. Sie sollte sich beim Abwurf der Atombombe tragisch bestätigen,219 wo ein winziges Quantum Masse in eine ungeheure
Energiemenge umgesetzt wurde.
Im Jahr 1905, als Einstein diese Arbeit schrieb, arbeitete er noch am
Schweizer Patentamt in Bern. Im gleichen Jahr erschienen weitere bedeutende Arbeiten von ihm, darunter die über den Photoelektrischen
Effekt, in dem Grundlagen der Quantentheorie gelegt wurden; für sie
und nicht für die Relativitätstheorie erhielt er 1921 den Nobelpreis
für Physik. Aber seine größte Aufgabe lag noch vor ihm. So bedeutsam die Spezielle Relativitätstheorie auch war, sie brachte keinen Fortschritt für das Problem der Inertialsysteme. Keineswegs waren alle Bezugssysteme gleichberechtigt. Wie bei Galilei und Newton gab es noch
immer eine ausgezeichnete Klasse von Bezugssystemen, die durch die
Lorentztransformationen ineinander transformiert wurden. Zwar war
der “König”, Newtons absoluter Raum, entthront worden, aber er wurde lediglich durch die “Aristokratie” der Inertialsysteme ersetzt, wie es
eigentlich schon zur Zeit Galileis gewesen war. Allerdings war der Raum
jetzt von der Zeit nicht mehr zu trennen; die beiden waren zu einer unauflösbaren Einheit, der vierdimensionalen “Raumzeit” R4 = R3 × R
verschmolzen (siehe die Lorentz-Figur auf Seite 125). Aber die Raumzeit als Ganzes schien doch wieder “a priori” zu sein, vor aller Erfahrung, die unbewegte Bühne, auf der sich die Physik abspielte. Dort gab
es bereits eine Geometrie, die übliche Geometrie des R4 , die es gestattete, zwischen “gerade” und “krumm” oder besser “gleichförmig geradlinig” und “beschleunigt” zu unterscheiden. Die Minkowski-Metrik
(76) hatte die Rolle des euklidischen Abstands übernommen mit den
Lorentztransformationen als den “Drehungen”. Die Raumzeit war also
mit einer voll ausgebildeten Geometrie a priori ausgestattet, ganz im
Sinne von Kant und Newton. Dieses Bild zu korrigieren und die physikalische Quelle der Geometrie zu finden, die Riemann schon erahnt
hatte (siehe Abschnitt 11), war die größte Leistung Einsteins: die Allgemeine Relativitätstheorie.220 Völlig überraschend löste er damit nicht
219Einstein
war sein ganzes Leben lang Pazifist gewesen. Aber im August 1939
war er Mitunterzeichner eines Briefes von Wissenschaftlern an den amerikanischen
Präsidenten, in dem davor gewarnt wurde, dass Hitler in den Besitz einer Atombombe gelangen könnte. Der Brief entfaltete erst zwei Jahre später eine Wirkung, als
die USA das Manhattan-Projekt starteten, das zum Bau der Atombomben führte,
die im August 1945 auf Hiroshima und Nagasaki abgeworfen wurden.
220 Albert Einstein: Die Grundlagen der allgemeinen Relativitätstheorie.
Annalen der Physik 354 (1916), S. 769 - 822.
http://www.physik.uni-augsburg.de/annalen/history/einsteinpapers/1916 49 769-822.pdf
STERNSTUNDEN DER MATHEMATIK
127
nur das Problem der Relativität aller Bezugssysteme, sondern eröffnete
auch ein ganz neues Verständnis für die im Großen bedeutsamste physikalische Grundkraft, die Gravitation. Er griff dabei den Gedanken
von Ernst Mach auf, der in gewissem Sinn an den historischen Ausgangspunkt zurückführte: Die “guten” Koordinatensysteme sind vielleicht doch (wie das früher selbstverständlich benutzte erdgebundene
System) an den physikalischen Massen festgemacht, aber nicht mehr
lokal (an der Erde, an der Sonne, an der Galaxie), sondern im Großen.
Einstein geht aber viel weiter als Mach und postuliert, dass die gesamte
raumzeitliche Geometrie durch die Massenverteilung erst erzeugt wird.
Der Ausgangspunkt seiner Überlegung ist, dass Inertialsysteme am genauesten in frei fallenden, also nur unter dem Einfluss der Schwerkraft
bewegten Systemen realisiert werden.
Wir wollen uns zunächst die Gravitationstheorie Newtons ins Gedächtnis
zurückrufen. Sie beruht auf zwei Gesetzen:
(1) Newtonsche Bewegungsgleichung: Die Bahn x = x(t) eines Teilchens der Masse m in einem Kraftfeld F (x) wird bestimmt durch das
Gesetz Kraft = Masse mal Beschleunigung. Wenn wir die Geschwindigkeit mit x′ = Änderung von x und die Beschleunigung mit x′′ =
Änderung von x′ bezeichnen, erhalten wir
(77)
mx′′ = F (x).
(2) Newtonsches Gravitationsgesetz: Eine große Masse M am Ursprung
O wirkt auf ein Teilchen der Masse m am Ort x. Newton hat in seinem
Hauptwerk “Philosophiae Naturalis Principia Mathematica” (1687) die
Frage gelöst, welche mathematische Gestalt die Gravitationskraft F (x)
haben muss, damit die von Kepler beschriebenen Ellipsenbahnen als
Lösungen von (77) herauskommen. Die Antwort war: Die Kraft musste
radial auf die Masse M gerichtet sein und quadratisch mit der Entfernung abnehmen. Sie musste außerdem proportional zu den beteiligten
Massen M und m sein.
mM
(78)
F (x) = − 2 ex
|x|
wobei ex = x/|x| der Einheitsvektor in Richtung x ist.
Auffällig ist, dass derselbe Faktor m in beiden Gleichungen (77) und
(78) vorkommt, obwohl seine Bedeutung in beiden Gleichungen eigentlich völlig unterschiedlich ist: Im Bewegungsgesetz (77) bezeichnet er
die träge Masse, d.h. den Widerstand, die ein Körper einer Beschleunigung entgegensetzt, im Gravitationsgesetz (78) dagegen die schwere
Masse, das Maß, in dem die Gravitationskraft ihre Wirkung auf den
Körper entfaltet. Die Gleichheit von träger und schwerer Masse ist bei
128
J.-H. ESCHENBURG
Newton lediglich eine empirische Tatsache, bei Einstein aber wird sie zu
einem Eckpfeiler seiner Theorie. Verbindet man die beiden Gesetze, so
lässt sich die Masse m kürzen, und es entsteht die Differentialgleichung
(79)
x′′ = −
M
ex =: f (x).
|x|2
Sie zeigt, dass die Bewegung unter der Schwerkraft von der Masse, vom
Gewicht des bewegten Körpers unabhängig ist.
Nun möchte man die eine Masse M durch eine beliebige Massenverteilung ρ(x) ersetzen, die wir uns aus lauter Punktmassen zusammengesetzt denken können. Dazu müssen wir das Gravitationsfeld der
M
Punktmasse, f (x) = − |x|
2 ex , noch ein wenig anders beschreiben. Das
Vektorfeld f steht senkrecht auf der Kugelfläche vom Radius r um
0, und |f (x)| ist proportional zu 1/r2 , während die Kugelfläche selbst
den Flächeninhalt 4πr2 hat (siehe Abschnitt 3), das Produkt von beidem ist also konstant, unabhängig von r. Stellt man sich f wie das
Geschwindigkeitsfeld der Partikel eines Gases vor (“Strömungsfeld”),
so fließt durch jede Kugelfläche das gleiche Gasvolumen hindurch; in
den Bereich zwischen zwei Kugelflächen fließt also genauso viel hinein wie wieder heraus. In unserer hydrodynamischen Redeweise gibt
es sozusagen keine Quellen und keine Senken für das Strömungsfeld f
außerhalb des Ursprungs 0. Ein solches Vektorfeld heißt “quellenfrei”
oder divergenzfrei, div f = 0. Es gibt auch kein anderes divergenzfreies
radiales Vektorfeld. Außerdem gibt es noch eine zweite Eigenschaft: f
beschreibt die Veränderung, den Gradienten einer reellwertigen Funktion φ, genauer: f = −∇φ (wobei ∇ für “Gradient” steht); diese Funktion φ heißt Potential zum Vektorfeld f . In unserer hydrodynamischen
Einkleidung bedeutet dies: Das Strömungsfeld des Gases (der “Wind”)
kommt von den unterschiedlichen Druckverhältnissen und ist in jeder
Richtung proportional zum Druckabfall; das Potential entspricht dem
Gasdruck an jeder Stelle. Im Fall der einzelnen Masse, (79), ist
(80)
φ=−
M
r
(denn die Ableitung von −1/r ist 1/r2 ). Die beiden Eigenschaften “divergenzfrei” und “Gradient” von f bleiben erhalten, wenn wir die einzelne Masse durch eine beliebige Ansammlung von Massen ρ(x) ersetzen; allerdings gilt die Divergenzfreiheit nur dort, wo keine Massen
sind, wo also ρ(x) = 0 ist. Die Massen sind sozusagen die “Quellen”
des Gravitationsfeldes. Die obigen Gleichungen verallgemeinern sich
STERNSTUNDEN DER MATHEMATIK
129
deshalb wie folgt:
(81)
x′′ = f (x), (a)
f
= −∇φ, (b)
div f = 4πρ. (c)
Die erste Gleichung gibt an, wie Massen durch das Gravitationsfeld
bewegt werden (Bewegungsgleichung), die anderen beiden, wie Massen
das Gravitationsfeld erzeugen (Feldgleichung): die zweite und dritte
zusammen ergeben die Poissongleichung
(82)
div ∇φ = −4πρ.
In Einsteins Gravitationstheorie wird daraus folgendes: Die Raumzeit ist nicht mehr R4 = R3 × R, sondern eine beliebige 4-dimensionale
Mannigfaltigkeit M mit Koordinaten x1 , x2 , x3 , x4 = t und mit einer
quadratischen Form
(83)
2
ds =
4
X
gij dxi dxj
i,j=1
wie bei Riemann, vgl. (62) auf Seite 103. Aber diese Metrik lässt sich
punktweise nicht länger auf die euklidische Metrik dx21 +dx22 +dx23 +dx24
transformieren, sondern auf die Minkowski-Metrik dx21 +dx22 +dx23 −dx24
(“Lorentzmetrik”). Deshalb gilt nun nicht mehr ds2 (v) > 0 für jeden
Vektor v 6= 0 auf M . Vektoren mit ds2 (v) < 0 heißen zeitartig, solche
mit ds2 (v) = 0 heißen lichtartig. Diese Metrik ist aber nicht a-priori
gegeben, sondern physikalisch bestimmt, und zwar durch das Analogon
der Feldgleichung (81)(c), wie wir gleich sehen werden.
Aber zunächst kommen wir zum Analogon der Bewegungsgleichung
(81)(a). Fast alles in der Lorentz-Geometrie ist wie in der Riemannschen Geometrie; insbesondere gibt es Geodäten, die Analoga von Geraden. Schwere Teilchen, die keinen anderen als Gravitationskräften ausgesetzt sind, bewegen sich auf zeitartigen Geodäten, d.h. die Vektoren
tangential an die Geodäte sind zeitartig, Licht dagegen läuft auf lichtartigen Geodäten. Die Bewegungsgleichung wird also zur Geodätengleichung221
X
(84)
x′′k = −
Γkij dx′i dx′j .
i,j
221Die
Geodäte hat einen eigenen Parameter, der nicht mehr die Zeitkoordinate
ist wie bei Newton in (77), sondern die Eigenzeit, d.h. die Bogenlänge bezüglich der
Lorentzmetrik.
130
J.-H. ESCHENBURG
Der Vergleich mit (81)(a) suggeriert, dass die rechte Seite, also die
Christoffelsymbole Γkij , dem Newtonschen Gravitationfeld f entsprechen muss. Genau wie dieses nach (81)(b) aus ersten Ableitungen des
Potentials φ besteht, entstehen auch die Γkij durch Differentiation einer
anderen Größe, nämlich der Komponenten gij der Metrik (83) gemäß
der Levi-Civita-Gleichung,
X1
g kl (−gij,k + gjk,i + gki,j )
(85)
Γlij =
2
k
(wobei g kl die Komponenten der Umkehrmatrix zu (gij ) sind). Die Rolle
des Gravitationspotentials φ wird also bei Einstein von der Metrik (gij )
übernommen.
Was aber ist das Analogon der Feldgleichung (81)(c)? Dazu muss
man zuvor bedenken, dass bereits in der Speziellen Relativitätstheorie
die Massendichte ρ nicht unter Lorentztransformationen invariant ist.
Sie wird deshalb ersetzt durch eine weitere quadratische Form T =
(Tij ) den Massetensor oder Energie-Impuls-Tensor, der alle vorhandenen Massen und Energien (z.B. auch die von elektromagnetischen Feldern) zusammenfasst; diese ist Lorentz-invariant. Die Energie-ImpulsErhaltung drückt sich durch die Divergenzfreiheit dieses Tensors aus.
Auf der anderen Seite der Feldgleichung muss wie in (82) ein Ausdruck stehen, der Ableitungen zweiter Ordnung (Veränderungen von
Veränderungen) der Größe enthält, die die Rolle des Potentials übernommen hat, also der Metrik (gij ). Dies trifft für den Krümmungstensor
zu:
X
m
m
l
m
(86)
Rijk
= ∂i Γm
Γljk Γm
jk − ∂j Γik +
il − Γik Γjl
l
Aber der gesuchte Ausdruck muss wie der Massetensor auch eine divergenzfreie quadratische Form sein. Es gibt bis auf Vielfache nur eine
solche quadratische Form, nämlich den Einsteintensor mit den Komponenten222
X
X
1
k
m
Gij =
Rkij
− sgij mit s =
Rmkl
g kl
2
k
k,l,m
P k
einfache Spurbildung des Krümmungstensors Ricij = k Rkij
nennt man
P
m
g kl heißt Skalarkrümmung.
Riccitensor, die zweifache Spurbildung s = k,l,m Rmkl
Der Einsteintensor ist divergenzfrei auf Grund der 2. Bianchi-Identität für die Ableitung des Krümmungstensors, die wiederum eine Konsequenz der Symmetrie der
gewöhnlichen dritten Ableitungen ist.
222Die
STERNSTUNDEN DER MATHEMATIK
131
Tatsächlich lauten die Einsteinschen Feldgleichungen223
(87)
G = 8πT.
Einstein suchte lange, bis er auf der geometrischen Seite ein Äquivalent
des Massetensors gefunden hatte. In der Endphase dieser Suche im November 1915 hält er jeden Donnerstag einen Vortrag vor der Preußischen Akademie der Wissenschaften, in der er jedesmal die Version der
Vorwoche teilweise widerruft und überarbeitet. Am 25. November 1915
hat er es schließlich geschafft (vgl. [9]): Er hat den Einsteintensor gefunden, und damit erblickt die verblüffend einfache Grundformel der neuen
Gravitationstheorie, die Einsteinsche Feldgleichung (87), das Licht der
Welt.
Damit ist das Programm der Relativitätstheorie erfüllt. Die Geometrie der Raum-Zeit ist nicht mehr a-priori, sondern physikalisch bestimmt: Die Massen- und Energieverteilung, die ihren Ausdruck in dem
Tensor T findet, bestimmt den Einsteintensor der Metrik g, ein Differentialausdruck zweiter Ordnung in den Komponenten gij . Die Metrik
ds2 = g ist Lösung der Differentialgleichung (87) und somit durch die
Massenverteilung bestimmt. Mach hat Recht: Trägheitskräfte treten
bei Abweichung von der Geodäten auf, und diese sind durch die Geometrie und damit durch die Massenverteilung, nämlich den Massetensor
T bestimmt.
Wie in der Newtonschen Theorie beschreibt (84) die Bewegung der
Massen durch das Gravitationsfeld,
“geometry tells matter how to move,”
und (84) die Erzeugung des Gravitationsfeldes durch die Massen,224
“matter tells geometry how to curve.”
Ebenso wie bei Newton ist der Unterschied zwischen felderzeugenden
und vom Feld bewegten Massen eigentlich künstlich; in Wirklichkeit
handelt es sich um ein gekoppeltes Differentialgleichungssystem für
Feld und Massen. Eine etwas genauere Analyse zeigt sogar, dass (84)
eine Konsequenz von (87) ist; die Einsteinsche Feldgleichung (87) ist
also das Gesetz der Gravitation.
Das erste Beispiel ist wieder das Gravitationspotential des sphärisch
symmetrischen Sterns, das in der Newtonschen Theorie φ = − Mr lautete; der entsprechende Ausdruck in der Einsteinschen Theorie wurde
223Seite
806, Gleichung (51) in der in Fußnote 220 zitierten Arbeit von Einstein.
Dort steht eigentlich die Gleichung Ric = κ(T − 21 τ g) mit τ = Spur T . Nimmt man
auf beiden Seiten die Spur (mit Spur g = 4), so folgt s = κ(τ − 2τ ) = −κτ , also ist
G = Ric − 12 sg = κ(T − 12 τ g + 21 τ g) = κT .
224C.W. Misner, K.S. Thorne, J.A. Wheeler: Gravitation, Freeman 1973
132
J.-H. ESCHENBURG
1916 von Karl Schwarzschild225 gefunden:
ds2 = −(1 + 2φ)dt2 + (1 + 2φ)−1 dr2 + r2 dω 2
(88)
wobei wieder φ = − Mr und dω 2 die Metrik der Kugelfläche vom Radius
Eins im euklidischen Raum bezeichnet. Bei der Sonnenfinsternis vom
29. Mai 1919226 wurde Einsteins Theorie zum ersten Mal getestet. Sie
sagte nämlich eine Gravitationswirkung auf das Licht voraus, das sich ja
auf Lichtgeodäten in der Metrik (88) ausbreitet; das Licht eines hinter
der Sonne stehenden Sterns musste demnach durch die Sonne abgelenkt
werden, ein Effekt, der bei einer totalen Sonnenfinsternis beobachtbar
wurde. Bereits in der speziellen Relativitätstheorie war allerdings ein
solcher Effekt zu erwarten gewesen, denn wegen der Gleichheit von
Masse und Energie musste auch das Photon eine Gravitationswirkung
zeigen, aber der durch die Schwarzschild-Metrik vorausgesagte Effekt
war (auf Grund des Faktors 2 vor φ) doppelt so groß. Bei zwei Messungen in Brasilien und Afrika wurde dieser Wert bestätigt, was der
Einsteinschen Gravitationstheorie zum Durchbruch verhalf.
Manchmal wird behauptet, philosophische Probleme würden niemals
gelöst. Das Beispiel Einsteins zeigt, dass diese Behauptung falsch ist:
Er hat ein uraltes philosophisches Problem gelöst: Gibt es einen absoluten Unterschied zwischen Ruhe und (beschleunigter) Bewegung?
Die Antwort lautet Ja, und verantwortlich dafür ist die Geometrie, die
durch die Massenverteilung bestimmt wird, die sich wiederum nach den
Gesetzen der Geometrie entwickelt.
Übungen
13.1. Die Massendichte ist nicht Lorentz-invariant. Zeigen Sie,
dass sich die Massendichte ρ in der Speziellen Relativitätstheorie unter
Lorentz-Transformationen ändert (nicht dagegen unter Galilei-Transformationen). In der Figur unten nehmen wir an, dass jeweils ein Massenpunkt der Masse M auf einem Gitterpunkt auf der x-Achse sitzt
und im ungestrichenen System ruht. Die “Weltlinien” der Massenpunkte, die zu jeder Zeit t ihren Ort x angeben, werden im (x, t)-System
Parallelen zur t-Achse sein (der Ort x verändert sich nicht: Ruhe). Nun
betrachten wir ein mit konstanter Geschwindigkeit v geradlinig bewegtes System (x′ , t′ ). Der x′ -Abstand der Massenpunkte ist der Lorentz′
Abstand der Schnitte der Weltlinien
p mit der x -Achse. Der Lorentz2
Abstand von (xo , to ) zu (x1 , t1 ) ist (x1 − xo ) − (t1 − to )2 . Man zeige
damit, dass der Abstand der Massenpunkte im (x′ , t′ )-System um den
225Karl
Schwarzschild, 1873 (Frankfurt, Main) - 1916 (Potsdam)
eclipse of May 29, 1919
226http://en.wikipedia.org/wiki/Solar
STERNSTUNDEN DER MATHEMATIK
133
√
Faktor 1 − v 2 kleiner geworden ist; ihre Dichte ist also gewachsen. Dabei ist noch nicht √
berücksichtigt, dass die Masse im bewegten System
um den Faktor 1/ 1 − v 2 größer geworden ist, was die Massendichte
noch einmal vergrößert.
t
t’
x’
M
M
M
M
M
x
13.2. Das Zwillingsparadoxon. Wenn einer von zwei Zwillingen sich
zu einer Reise mit Geschwindigkeit v zum nächsten Fixstern
√ α-Centauri
entschließt, wird er nach seiner Rückkehr um den Faktor 1 − v 2 jünger
sein als sein Bruder, denn um diesen Faktor ist Zeit t′ im System des
Reisenden verkürzt: Die Zeit für die Hinreise ist der Lorentzabstand
zwischen den “Ereignissen” (Ort√und Zeit) der Abreise, (0, 0) und der
Ankunft, (vt/2, t/2), also (t/2) 1 − v 2 , und die Gesamtzeit bis zur
Rückkehr ist die doppelte.
t t’
α−cen
t/2
vt/2
x
Dieses von der Relativitätstheorie vorhergesagte Phänomen wurde zeitweilig als Paradoxon empfunden, weil man doch die Rollen der Zwillinge umkehren könne: Relativ zum reisenden Zwilling entferne sich doch
auch der zu Hause Gebliebene und kehre wieder zurück, also könne
man ebenso für den umgekehrten Altersunterschied nach der Rückkehr
argumentieren. Aber die Systeme sind nicht gleichberechtigt: Nur der
zurückbleibende Zwilling bewegt sich auf einer Geodäten (einer Geraden), der Reisende nicht: Seine Weltlinie hat einen Knick. Sein System ist kein “Inertialsystem” mehr. Man sieht hieran sehr schön, wie
die Koordinaten-unabhängige “affine Struktur” der Raumzeit, d.h. die
Geraden oder Geodäten, den Unterschied zwischen den Koordinatensystemen definiert.
13.3. Die De-Sitter-Raumzeit. Die einfachste Lorentz-Mannigfaltigkeit nach dem Minkowskiraum ist der De-Sitter-Raum,227 so etwas wie die Einheitssphäre im Minkowskiraum von einer Dimension
227Willem
de Sitter, 1872 (Sneek, Westfriesland) - 1934 (Leiden)
134
J.-H. ESCHENBURG
mehr. Man betrachtet den Raum Rn+1 mit der Minkowskimetrik ds2 =
P
n
2
2
n+1
: |x|L = 1} mit
i −dxn+1 und darin die Quadrik Q = {x ∈ R
i=1 dx
P
|x|2L = ni=1 x2i − x2n+1 . Für n = 2 ist dies das einschalige Hyperboloid,
siehe die Figur auf Seite 117. Die Lorentzmetrik ist die Einschränkung
der auf Rn+1 definierten Minkowskimetrik auf Tangentialvektoren von
Q. Die auf Q liegenden Geraden (siehe Figur auf Seite 117) sind genau
die Lichtgeodäten; machen Sie sich das bitte klar.
14. Gödel: Ist die Mathematik axiomatisierbar? (1931)
Mathematik hat die Aufgabe, das Verborgene auf das Offensichtliche
zurückzuführen. Machen wir uns das an einem einfachen Beispiel aus
der ebenen Geometrie klar: Die Summe der Innenwinkel in jedem Dreieck beträgt 180 Grad. Das ist zweifellos etwas Verborgenes: Wirklich
in jedem Dreieck? Es gibt doch unendlich viele verschiedene Dreiecke,
“die meisten” hat noch kein Mensch je gesehen, und doch soll der Satz
auch for sie gelten! Ein Nachmessen der Winkel kann diesen Satz im
Rahmen der Messgenauigkeit zwar bestätigen, aber niemals beweisen.
Der Beweis geschieht anders, nämlich durch Rückführung auf bekannte
Tatsachen vermittels einer Konstruktion, hier durch Ziehen der Parallele zur Grundseite durch den höchsten Punkt des Dreiecks; dort finden
wir die drei Winkel wieder versammelt, und sie fügen sich “offensichtlich” zum gestreckten Winkel 180 Grad zusammen:
Wir haben verschiedene “offensichtliche” Tatsachen dabei benutzt, z.B.
Existenz und Eigenschaften der Parallele sowie die Tatsache, dass die
Winkel an der Grundlinie bei 180-Grad Drehung um die markierten
Mittelpunkte der Seiten ihre Größe beibehalten.
Schon recht früh in der Mathematikgeschichte entstand das Bedürfnis
nach einem Katalog der “offensichtlichen” Tatsachen, auf die (allein)
man sich bei Beweisen berufen durfte. Der Katalog sollte möglichst
klein sein, aber doch vollständig in dem Sinn, dass wirklich alle wahren
Sätze davon ableitbar sein sollten. Ein erstes System dieser Art stellte
Euklid um 300 v.Chr. in seinem Buch “Elemente” für die ebene Geometrie auf. Dieses Buch wurde über viele Jahrhunderte in Europa und im
vorderen Orient wieder und wieder kopiert und übersetzt und erlangte
STERNSTUNDEN DER MATHEMATIK
135
Vorbildcharakter auch für andere Wissenszweige: “Ordine Geometrica Demonstrata”, nach geometrischer Ordnung bewiesen, das war der
höchste Standard der Erkenntnis.228
Erst um 1900 wurden diese frühen Bemühungen auf eine moderne
Grundlage gestellt und auf alle Gebiete der Mathematik ausgeweitet.
Es kam in rascher Folge zu Axiomatisierung verschiedener Teilbereiche
der Mathematik: Arithmetik (Peano 1898), ebene Geometrie (Hilbert
1899), Analysis (Hilbert 1902), Mengenlehre (Zermelo 1907 und Fraenkel 1921), Wahrscheinlichkeitstheorie (Kolmogorov 1933).
Die Axiome wurden jetzt weniger als ein Katalog einfacher wahrer Aussagen angesehen, sondern als definierende Eigenschaften eines Begriffs
oder eines Gebietes, das sich mit diesem Begriff befasste: Eine Gruppe zum Beispiel ist eine Struktur, die die Gruppenaxiome erfüllt: eine
Menge G mit einem ausgezeichneten Element e ∈ G, genannt Neutralelement, und zwei Abbildungen m : G × G → G: (g, h) 7→ gh
(Gruppenmultiplikation) und j : G → G: g 7→ g −1 (Inversion) mit
folgenden Eigenschaften:
(1) Assoziativität: g(hk) = (gh)k für alle g, h, k ∈ G,
(2) Neutralelement: ge = eg = g für alle g ∈ G,
(3) Inverses: gg −1 = g −1 g = e für alle g ∈ G.
Das sind die definierenden Eigenschaften einer Gruppe und damit die
Axiome der Gruppentheorie; alle gültigen Sätze über Gruppen müssen
sich durch eine Kette logischer Schlüsse auf sie zurückführen lassen.
Beispiel: Das Neutralelement ist eindeutig, genauer: Falls es ein Elemente e′ ∈ G mit (2)’ ge′ = g für alle g ∈ G, dann ist e′ = e..
Beweis: (2)’ ⇒ ee′ = e und (2) ⇒ ee′ = e′ , also e = e′ .
In dieser Auffassung gleicht jedes Gebiet der Mathematik einem Schachspiel. Im Fall der Gruppentheorie entsprechen die Schachfiguren (König,
Läufer usw.) dem Neutralelement e sowie den beiden Abbildungen m
und j, die Anfangsaufstellung sind die Axiome. Die Zugregeln werden
durch die Logik gegeben (z.B.: Wenn ge′ = g für alle g ∈ G gilt, dann
dürfen wir jedes Element von G für g substituieren, insbesondere das
Element e, also folgt ee′ = e), und die Stellungen, die durch erlaubte
Zugfolgen zustande kommen, entsprechen den gültigen Sätzen. Aber
wer die Zugregeln und die Anfangsstellung kennt, kann deshalb noch
nicht Schach spielen, und die Schachbücher listen nicht alle möglichen
Stellungen auf. Gute Schachspieler und Mathematiker kennen ganz andere Wegweiser zu interessanten Stellungen und Sätzen.
228Berühmtestes
Beispiel ist die “Ethik” von Baruch Spinoza: “Ethica ordine
geometrica demonstrata”, erschienen posthum 1677.
136
J.-H. ESCHENBURG
Dennoch ist es die “Lebenslüge” der Mathematiker, dass vom rein logischen Standpunkt ihre Wissenschaft einem solchen Spiel gleicht und
alle ihre Sätze logisch aus den Axiomen gefolgert werden können. Diese
Rückführung ist die Methode nicht der Wahrheitsfindung, sondern der
Bestätigung, der Verifikation, vergleichbar mit dem Experiment in den
Naturwissenschaften.
Im ersten Drittel des 20. Jahrhunderts kam es zum Grundlagenstreit
innerhalb der Mathematik. Die Auseinandersetzung betraf die Frage,
welche Mittel zum Beweis eines Satzes zugelassen waren. Eine Gruppe
von Mathematikern um L.E.J. Brouwer229 und H. Weyl – sie nannten sich Intuitionisten – wollten nur noch konstruktive Beweise zulassen. Reine Existenzbeweise ohne Konstruktion, etwa die Existenz
des Grenzwerts einer beschränkten monotonen Folge,230 wurden nicht
mehr anerkannt; auch das Prinzip des ausgeschlossenen Dritten (Ein
Satz oder sein Gegenteil ist wahr, etwas Drittes gibt es nicht, “Tertium non datur”) wurde abgelehnt. Dagegen stand Hilbert auf und
formulierte ein Programm, um den klassischen Bestand der Mathematik zu bewahren: Alles sollte nach vorgegebenen Regeln auf Axiome
zurückgeführt werden. Diese Denkrichtung innerhalb der Mathematik
wurde Formalismus genannt. Ein wichtiger Teil des Hilbertschen Programms war der Beweis der Widerspruchsfreiheit der Axiome. Schon
in seinem berühmten Vortrag auf dem Internationalen Mathematikerkongress 1900 in Paris231 stellte Hilbert als zweites seiner 23 Probleme
229Luitzen
E. J. Brouwer, 1881 (bei Rotterdam) - 1966 (Blaricum, Niederlande)
Folge besteht aus einer Aufzählung von unendlich vielen reellen Zahlen
a1 , a2 , a3 , . . . , an , . . . Sie konvergiert gegen einen Wert a, wenn sich die Dezimalbruchentwicklung von an zu der von a stabilisiert, wenn n genügend groß ist. Sie
ist monoton wachsend, wenn a1 ≤ a2 ≤ . . . , und beschränkt, wenn es eine Zahl
b gibt, die von keinem der Folgenglieder a1 , a2 , a3 , . . . übertroffen wird. Um den
Grenzwert a zu finden, wird b schrittweise verkleinert, z.B. auf den Wert genau in
der Mitte zwischen einem der Folgenglieder und b, und jedesmal wird überprüft, ob
diese kleinere Zahl b′ noch immer von keinem Folgenglied übertroffen wird. Dieser
Nachweisversuch führt aber nicht notwendig zu einem Ergebnis: Vielleicht sind die
ersten hundert Millionen Folgenglieder kleiner als b′ , und dennoch könnte es ein
Folgenglied mit einer noch weit größeren Nummer geben, das b′ übertrifft. Wir sind
zwar davon überzeugt, dass entweder alle Folgenglieder kleiner oder gleich b′ sind
oder aber ein Folgenglied – und von da an alle weiteren – die Zahl b′ übertrifft
(das ist das Prinzip des ausgeschlossenen Dritten), aber wir sind nicht in der Lage,
mit Sicherheit zu entscheiden, welche der beiden Alternativen eintritt. Deshalb ist
dieser Beweis nicht konstruktiv.
231
“Wer von uns würde nicht gerne den Schleier lüften, unter dem die Zukunft
verborgen liegt, um einen Blick zu werfen auf die bevorstehenden Fortschritte unserer Wissenschaft und in die Geheimnisse ihrer Entwicklung während der künftigen
230Eine
STERNSTUNDEN DER MATHEMATIK
137
den Beweis der Widerspruchsfreiheit der Axiome der Zahlentheorie.232
Nach dem Ersten Weltkrieg konkretisierte er sein Programm:
(HP1) “Alles, was bisher die eigentliche Mathematik
ausmacht, wird nunmehr streng formalisiert, so dass die
eigentliche Mathematik oder die Mathematik im engeren Sinne zu einem Bestande an beweisbaren Formeln
wird.”
(HP2) “Zu dieser eigentlichen Mathematik kommt eine
gewissermaßen neue Mathematik, eine Metamathematik
hinzu, die zur Sicherung jener dient [...]. In dieser Metamathematik kommt – im Gegensatz zu den rein formalen
Schlussweisen der eigentlichen Mathematik – das inhaltliche Schließen zur Anwendung, und zwar zum Nachweis
der Widerspruchsfreiheit der Axiome.”233
Wie war es zu diesem Grundlagenstreit gekommen in einer Wissenschaft, die fester gefügt schien als jede andere? Anlass war eigentlich die
Entwicklung des grundlegendsten Zweiges der Mathematik, der Mengenlehre, durch Georg Cantor234 ab 1878. Nicht die Zahlen sind ja eigentlich die grundlegenden Objekte der Mathematik, denn bevor man
zählen kann, muss man wissen, was man zählen will; man muss also die
zu zählende Gesamtheit definiert haben, die Menge. Cantor gibt 1895
die folgende berühmt gewordene Definition einer Menge:235
Unter einer “Menge” verstehen wir jede Zusammenfassung M von bestimmten wohlunterschiedenen Objekten
m unserer Anschauung oder unseres Denkens (welche
die “Elemente” von M genannt werden) zu einem Ganzen.
Jahrhunderte! Welche besonderen Ziele werden es sein, denen die führenden mathematischen Geister der kommenden Geschlechter nachstreben? Welche neuen Methoden und neuen Tatsachen werden die neuen Jahrhunderte entdecken auf dem
weiten und reichen Felde mathematischen Denkens?”
http://www.digizeitschriften.de/dms/img/?PPN=GDZPPN002498863
232
Das erste Hilbertsche Problem war die Kontinuumshypothese, auf die wir
weiter unten eingehen werden.
233
Hilbert, David: Neubegründung der Mathematik. Erste Mitteilung. Abhandlung Mathematisches Seminar Hamburg 1, 157 - 177.
234
Georg Ferdinand Ludwig Philipp Cantor, 1845 (St. Petersburg) - 1918 (Halle
a.d. Saale)
235
Georg Cantor: Beiträge zur Begründung der transfiniten Mengenlehre, Math.
Ann. 46 (1895), 481 - 512, https://en.wikipedia.org/wiki/Georg Cantor, Weblink
der “gesammelten Abhandlungen”, S. 282
138
J.-H. ESCHENBURG
Die Problematik dieser Definition liegt in dem Wort “Zusammenfassung”. Diese muss ja entweder durch explizite Aufzählung oder aber
anhand einer gemeinsamen Eigenschaft geschehen, die sprachlich ausgedrückt werden muss. Sprachliche Formulierungen können aber problematisch sein und zu Widersprüchen führen, besonders dann, wenn
sie auf sich selbst angewandt werden können. Es war erst Bertrand
Russell,236 dessen Hinweis auf diese Problematik Beachtung fand, obwohl auch Cantor selbst sie schon erkannt hatte.237 Russells um 1901
gefundene Antinomie war folgende: Mengen können Elemente anderer Mengen sein; zum Beispiel bilden die Teilmengen einer Menge M
die Elemente eine anderen Menge, der Potenzmenge von M . Damit ist
es im Prinzip denkbar, dass eine Menge auch sich selbst als Element
enthält, auch wenn diese Vorstellung etwas abenteuerlich anmutet. Die
von Russell definierte Menge R enthielt als Elemente genau die Mengen
ohne diese pathologische Eigenschaft; R ist die “Menge aller Mengen,
die sich nicht selbst als Element enthalten”:
(R)
R = {M : M 6∈ M }
Was ist problematisch an dieser Definition? Die Frage, ob R selbst
Element von R ist oder nicht: R ∈ R oder R 6∈ R? Aber wenn R 6∈ R,
dann ist R nach Definition (R) Element von R, und wenn R ∈ R, dann
ist R 6∈ R nach (R), ein vollkommener Widerspruch!
Russell selbst soll seine Antinomie einmal so umschrieben haben: In
einem kleinen englischen Ort wird ein Barbier angestellt; er soll genau
die Einwohner der Dorfes rasieren, die sich nicht selbst rasieren. Das
scheint vernünftig, aber nach einiger Zeit erhält der Barbier von der
236Bertrand
Arthur William Russell, 1872 - 1970 (Wales)
Cantor wies z.B. 1899 nach, dass der Begriff “Menge aller Mengen” widersprüchlich ist: Diese universelle Menge U müsste insbesondere ihre eigene Potenzmenge P (U ) (die Menge aller Teilmengen von U ) als Teilmenge enthalten. Cantor
hatte aber schon den Begriff Mächtigkeit (Kardinalität) eingeführt (zwei Mengen
sind gleichmächtig, wenn es eine umkehrbare Abbildung zwischen ihnen gibt) und er
hatte gezeigt, dass die Potenzmenge P (A) einer Menge A zu keiner Teilmenge von
A gleichmächtig ist. Der Beweis hierzu ist ähnlich wie Russells Antinomie: Wenn
doch, gäbe es eine surjektive Abbildung f : A → P (A); “surjektiv” heißt, dass jede
Teilmenge B ⊂ A (jedes Element B ∈ P (A)) im Bild von f liegt, d.h. es gibt b ∈ A
mit f (b) = B. Nun bestehen die Möglichkeiten b ∈ f (b) oder b 6∈ f (b). Analog zur
Russelschen Menge bilden wir die Teilmenge R ⊂ A, wo die zweite Alternative eintritt: R = {a ∈ A : a 6∈ f (a)}. Dann ist R = f (r) für ein r ∈ A. Gilt nun r ∈ f (r)?
Wenn nein, wenn also r 6∈ R = f (r), dann erfüllt r die definierende Eigenschaft
von R, also gilt doch r ∈ R. Wenn ja, hat r die Eigenschaft nicht, die die Elemente
von R auszeichnet, also r 6∈ R. Das ist ein Widerspruch, und damit kann es keine
surjektive Abbildung f : A → P (A) geben.
237
STERNSTUNDEN DER MATHEMATIK
139
Gemeinde eine Abmahnung wegen Überschreitung seiner Kompetenzen:
Er habe sich selbst rasiert, das sei aber im Vertrag nicht vorgesehen,
denn er dürfe nur die Leute rasieren, die sich nicht selbst rasieren. Von
nun an geht unser Barbier jeden Morgen zu einem Freund und lässt
sich von ihm rasieren. Aber nach kurzer Zeit erhält er wieder ein amtliches Schreiben, diesmal wegen Vernachlässigung seiner Dienstpflichten:
Wenn er sich nicht selbst rasiere, also zu den Leuten gehöre, die sich
nicht selbst rasieren, habe er laut Vertrag die Pflicht, sich zu rasieren,
das habe er versäumt!
Derartige Paradoxien waren um 1900 modern. Das Musterbeispiel steht
allerdings schon in der Bibel: Titus 1,12, wo ein Kreter mit den Worten zitiert wird: “Alle Kreter lügen immer”. In unserem Zusammenhang ist die Version des französische Mathematikers Jules Richard238
von Bedeutung: Man stelle sich die Liste aller (sprachlich formulierten)
möglichen Eigenschaften von natürlichen Zahlen vor, z.B. “ungerade”
oder “prim” oder “Quadratzahl”. Diese Liste ist unendlich lang, aber
man kann sie durchnummerieren, zum Beispiel, indem man sie nach
der Anzahl der verwendeten Buchstaben und, bei gleicher Buchstabenanzahl, nach Alphabet wie in einem Lexikon sortiert. Zu jeder Zahl n
gehört eine Eigenschaft mit dieser Nummer. Die Eigenschaft Nummer
n kann auf die Zahl n zutreffen oder auch nicht; im letzteren Fall wird n
Richardsche Zahl genannt. Richardsche Zahl zu sein ist zweifellos eine
mögliche Eigenschaft von Zahlen, deshalb kommt sie in unserer Liste
vor und hat dort eine Nummer, nR . Frage: Ist nR selbst eine Richardsche Zahl? Wenn ja, dann trifft die Eigenschaft, Richardsche Zahl zu
sein, nicht auf sie zu, sie ist also doch keine Richardsche Zahl. Wenn
nein, erfüllt sie die Eigenschaft, ist also doch eine Richardsche Zahl. Ein
blanker Widerspruch, der daher rührt, dass der Begriff “Eigenschaft”
nicht genau gefasst ist.
Die Problematik solcher widersprüchlichen sprachlichen Ausdrücke liegt
in ihrem Selbstbezug: Die Menge R ist möglicherweise Element von sich
selbst, der Barbier muss selbst rasiert werden, der Kreter lügt selbst
bei seinem Eingeständnis zu lügen und die der Eigenschaft “Richardsche Zahl” bezieht sich auf eine Liste, die sie selbst enthält. Russell
versuchte in seinem grundlegenden Werk “Prinzipia Mathematica” (gemeinsam mit A.N. Whitehead),239 diese Problematik durch Einführung
unterschiedlicher Typen von Dingen zu vermeiden. Insbesondere wurde
zwischen Mengen und Klassen unterschieden: die Elemente von Klassen
238Jules
Antoine Richard, 1862 (Blet, Dept. Cher) - 1956 (Chteauroux)
North Whitehead , 1861 (Ramsgate, England) - 1947 (Cambridge,
Massachusetts)
239Alfred
140
J.-H. ESCHENBURG
sind Mengen. Die Mengen, die sich sich nicht als Element enthalten,
bilden selbst keine Menge mehr, sondern nur noch eine Klasse, ebenso
wie auch alle Mengen zusammen. Damit entfällt der Selbstbezug und
der Widerspruch. Hilberts Programm trennt die Sprachebenen Mathematik und Metamathematik (HP2) und vermeidet damit Selbstbezüge.
Das ist der Hintergrund, die Bühne für Kurt Gödel, den “Helden”
dieses Kapitels.240 Er stammte aus einer begüterten deutschsprachigen Familie in Brünn (heute: Brno), das damals zu Österreich-Ungarn
gehörte. 1919 wurde die Stadt Teil der neugegründeten Tschechoslowakei. Gödel, der nur schlecht tschechisch sprach, nahm 1923 die österreichische Staatsbürgerschaft an und zog 1924 nach Wien. Dort studierte er zunächst theoretische Physik, bald aber auch Philosophie und Mathematik, wo er sich besonders mit Fragen der Logik und Mengenlehre
beschäftigte. Durch Vermittlung der Wiener Mathematiker Karl Menger und Hans Hahn241 nahm er ab 1929 an den Treffen des berühmten
“Wiener Kreises” teil, einer Gruppe von Wissenschaftlern (Philosophen, Mathematiker, Natur- und Sozialwissenschaftler), die sich von
1924 - 1936 regelmäßig im Wiener Mathematischen Seminar trafen;
auch Menger und Hahn gehörten dazu. “Ein schmächtiger und ungewöhnlich stiller junger Mann”, so beschreibt Menger seinen begabtesten Studenten, “er sprach fast nie und deutete Zustimmung oder Widerspruch durch eine kaum merkbare Bewegung seines Kopfes an.”242
Menger war mit dem “Grundlagenstreit” bestens vertraut, denn er hatte zwischen 1925 und 1927 fast zwei Jahre als Assistent von Brouwer
in Amsterdam verbracht, bevor er 1927 nach Wien berufen wurde. Im
März 1928 wurde Brouwer (von Physikern!) nach Wien eingeladen und
hielt dort zwei Vorträge, die Gödel wohl sehr beeindruckt haben.
1928 fand der Internationale Mathematikerkongress in Bologna statt.
Zum ersten Mal nach dem Krieg nahmen auch die deutschen Mathematiker wieder teil. Hilbert hatte sich vehement dafür eingesetzt, gegen
einige Widerstände u.a. von Brouwer. Auch Menger und Hahn waren
unten den Teilnehmern. Dort stellte Hilbert sein Programm vor:243
240Kurt
Friedrich Gödel, 1906 (Brünn, Brno) - 1978 (Princeton, New Jersey)
Menger, 1902 (Wien) - 1985 (Chicago), Hans Hahn, 1879 - 1934 (Wien)
242Karl Siegmund: Sie nannten sich Der Wiener Kreis, Springer 2015, Seite 198
243D. Hilbert: Probleme der Grundlegung der Mathematik. Vortrag gehalten
auf dem Internationalen Mathematiker-Kongress Bologna 1928. Math. Ann. 102,
1 - 9. Zitiert nach Erhard Scholz: Die Gödelschen Unvollständigkeitssätze und
das Hilbertsche Programm einer “finiten” Beweistheorie, http://www2.math.uniwuppertal.de/∼scholz/preprints/goedel.pdf
241Karl
STERNSTUNDEN DER MATHEMATIK
141
Mit dieser Neubegründung der Mathematik, die man
füglich als eine Beweistheorie bezeichnen kann, glaube ich die Grundlagenfragen der Mathematik als solche endgültig aus der Welt zu schaffen, indem ich jede
mathematische Aussage zu einer konkret aufweisbaren
und streng beweisbaren Formel mache und dadurch den
ganzen Fragenkomplex in die Domäne der reinen Mathematik versetze.[...] Die Beweistheorie [...] verschafft uns
das Hochgefhl der Überzeugung, dass wenigstens dem
mathematische Verstande keine Schranken gezogen sind
und dass er sogar die Gesetze des eignen Denkens aufzuspüren vermag.
Gödel leistete in der Folgezeit selbst einen wichtigen Beitrag zum Hilbertschen Programm und löste zwei der offenen Probleme aus dem neu
erschienenen Lehrbuch “Grundzüge der theoretischen Logik” von Hilbert und seinem Schüler Wilhelm Ackermann.244 Insbesondere bewies
er eine Vollständigkeitseigenschaft245 der Arithmetik im Hilbertschen
Kalkül. Diese Arbeit, “Die Vollständigkeit des Logik-Kalküls”, nahm
Hans Hahn als Dissertation an, und 1930 erwarb Gödel in Wien den
Doktorgrad.
Trotz seines Erfolges war Gödel keineswegs vom Hilbertprogramm überzeugt, und noch vor Abschluss seiner Promotion hatte er die Grundzüge
seiner Unvollständigkeitssätze entwickelt, die diesem Programm widersprachen.246 Auf einer “Tagung für exakte Erkenntnislehre” im September 1930 in Königsberg konnte er seine Ergebnisse fast beiläufig in
der Abschlussdiskussion ankündigen. 1931 erschien seine bahnbrechende Arbeit “Über formal unentscheidbare Sätze der Principia mathematica und verwandter Systeme”. Darin die zwei Hauptsätze:
UV1. Wenn ein solches System widerspruchsfrei ist, so ist es unvollständig, d.h. es gibt Sätze in dem System, deren Richtigkeit
innerhalb des Systems nicht entschieden werden kann.
UV2. Das System kann seine eigene Widerspruchsfreiheit nicht beweisen.
Satz UV2 gab eine negative Antwort auf das zweite Hilbertsche Problem: Die Widerspruchsfreiheit der Arithmetik, der Theorie der natürlichen Zahlen 0, 1, 2, 3, . . . , kann innerhalb der Arithmetik nicht bewiesen werden.
244
Wilhelm Friedrich Ackermann, 1896 (bei Herscheid) - 1962 (Lüdenscheid)
Ein Satz in dem System ist ableitbar genau dann, wenn er gültig ist in jedem
Modell, in dem die Axiome gelten.
246
K. Siegmund: Sie nannten sich Der Wiener Kreis, aaO, Seite 201
245
142
J.-H. ESCHENBURG
Ein Grundgedanke von Gödel ist, dass die saubere Trennung zwischen
den Sprachebenen “Mathematik” und “Metamathematik” in Hilberts
Programm (HP2) nicht aufrecht erhalten werden kann, sobald das System groß genug ist und z.B. die natürlichen Zahlen enthält. Dazu
ordnet er allen zugelassenen Zeichen und Zeichenreihen (Formeln und
Sätzen) der mathematischen Sprache eine Zahl zu, die Gödelzahl. Zum
Beispiel247 werden den Rechensymbolen 0 = “Null”, s = “Nachfolger”
(1 = s0, 2 = ss0 usw.), =, +, ·, (, ), x (Variable), y (zweite Variable) die Zahlen 11 bis 19 zugeordnet und den logischen Symbolen ∀ =
“für alle”, ∃ = “es existiert”, ∧ = “und”, ∨ = “oder”, ¬ = “nicht”
die Zahlen von 21 bis 25. Für weitere Variable kann man das Symbol
′
mit der Gödelzahl 31 verwenden (x′ , x′′ , . . . ). Eine Formel setzt sich
aus mehreren Zeichen zusammen; ihre Gödelzahl entsteht, indem die
den einzelnen Zeichen zugeordneten Zahlen hintereinander geschrieben
werden. In Sequenzen von Formeln werden die einzelnen Formeln jeweils durch eine Null abgetrennt. Jeder mathematische Satz und auch
jeder mathematische Beweis wird somit zu einer Zahl, und die Aussagen
über mathematische Formeln, Sätze und Beweise, die metamathematischen Aussagen, übersetzen sich in Sätze über Zahlen. Diese wiederum sind Teil der mathematischen Sprache, der Arithmetik; durch die
Übersetzung von Formelsequenzen in Gödelzahlen wird die Metamathematik also wieder in die Mathematik abgebildet, und zwar injektiv,
d.h. der Prozess lässt sich rückgängig machen.
Sodann formuliert Gödel einen metamathematischen Satz, der nichts
anderes aussagt, als dass er selbst aus den Axiomen der Arithmetik
nicht ableitbar sei: “Ich bin nicht ableitbar”. Dieser Satz wird mit Hilfe
der Gödelzahlen in einen Satz (G) der Arithmetik umgewandelt. Dieser
kann aber aus den Axiomen der Arithmetik nicht ableitbar sein, da er ja
behauptet, es nicht zu sein. Aber ebenso wenig kann erwiesen werden,
dass er falsch ist, denn wäre “Ich bin nicht ableitbar” falsch, dann dann
wäre dieser Satz ja doch ableitbar, obwohl er das Gegenteil behauptet.
Aber “Ich bin nicht ableitbar” ist kein Satz unserer formalen Sprache
(was soll “ich” genau heißen?), deshalb ist der Übersetzungsprozess
nötig, und der Satz (G) muss erst konstruiert werden.248
247https://de.wikipedia.org/wiki/Beweise
der gödelschen Unvollständigkeitsätze
stützen uns auf die Darstellung von Ernest Nagel und James R. Newman: Der Gödelsche Beweis, R. Oldenburg, Wien und München 1964, besonders
die Seiten 78 - 80 und 86 - 92, sowie den schon erwähnten Wikipedia-Artikel
https://de.wikipedia.org/wiki/Beweise der gödelschen Unvollständigkeitsätze.
Eine englische Übersetzung der Originalarbeit von Gödel ist online zu erhalten:
http://hirzels.com/martin/papers/canon00-goedel.pdf
248Wir
STERNSTUNDEN DER MATHEMATIK
143
Wir müssen zunächst zwei mathematische Prozesse mit Hilfe von Gödelzahlen verfolgen, die Beweisbarkeit und die Substitution. Wir sagen,
dass zwei Zahlen x, y ∈ N zueinander in der Beziehung Beweis(x, y)
stehen, wenn x die Gödelzahl einer Formelkette ist, die die Formel mit
der Gödelzahl y beweist. Jede Formel in der zu x gehörigen Kette muss
nach den logischen Prinzipien aus den Axiomen der Arithmetik oder
aus den voranstehenden Formeln gebildet werden, und die Schlussformel (nach der letzten Null) muss y sein. Damit ist auch die Eigenschaft
beweisbar für die Gödelzahl y einer Formel definiert:
(89)
beweisbar(y) ⇐⇒ ∃x : Beweis(x, y).
Andererseits sagt die Formel
(90)
¬(beweisbar(y))
aus, dass es zu der Formel mit der Gödelzahl y keinen Beweis gibt.
Die Substitution erkären wir zunächst an einem Beispiel. Die Formel
(91)
∃x (x = sy)
besagt, dass die Zahl y einen Nachfolger y +1 hat. Die Gödelzahl dieser
Formel ist
m = 21 18 16 18 13 12 19 17.
Man ersetze nun in dieser Formel die Gödelzahl 19 für das Variablensymbol y durch diese Zahl m. So entsteht die Zahl
sub(m, 19) := 21 18 16 18 13 12 21 18 16 18 13 12 19 17 17,
die Gödelzahl der Formel ∃x (x = sm), die besagt: “die Gödelzahl der
Aussage ‘y besitzt einen Nachfolger’ besitzt selbst einen Nachfolger”.
Für eine beliebige Gödelzahl y soll z := sub(y, 19) die Zahl sein, die aus
y dadurch entsteht, dass in der Formel mit der Gödelzahl y die Teilsequenz 19 überall durch die Zahl y selbst ersetzt wird. Die neue Zahl z
ist die Gödelzahl der Formel, die aus der Formel mit der Gödelzahl y
entsteht, wenn dort das Variablensymbol y überall durch die Gödelzahl
y ersetzt wird.
Die Formel ¬(beweisbar(y)) in (90) besitzt eine Gödelzahl, die wir n
nennen wollen. Die Gödelsche Formel ist nun
(G)
¬(beweisbar(g)), g := sub(n, 19)).
In Worten lautet diese Formel: Setze in die Formel (90), “die Formel mit
Gödelzahl y ist nicht beweisbar”, an die Stelle von y die Gödelzahl eben
dieser Formel (90) ein. Die Gödelzahl dieser neuen Formel ist g. Diese
Zahl tritt also in doppelter Funkton auf: innerhalb von (G), nämlich
als Gödelzahl der von (G) behaupteten nicht beweisbaren Formel, und
144
J.-H. ESCHENBURG
außerhalb von (G), nämlich als Gödelzahl von (G) selbst. In diesem
Sinne kodiert (G) wirklich den Satz “Ich bin nicht beweisbar”.
Wäre die Formel (G) mit der Gödelzahl g beweisbar (d.h. aus den
Axiomen der Arithmetik ableitbar), dann wäre ihre Aussage ja richtig.
Diese besagt rückübersetzt gerade, dass die Formel (G) nicht beweisbar ist, ein Widerspruch! Wäre (G) dagegen widerlegbar, dann wäre
ihr Inhalt falsch, der besagt, dass (G) nicht beweisbar ist, (G) wäre
also beweisbar. Das ist ebenfalls ein Widerspruch. Beide Annahmen,
“(G) beweisbar” und “(G) widerlegbar”, führen also zu Widersprüchen,
was nicht sein kann, wenn die Axiome der Arithmetik widerspruchsfrei
sind. Die Formel (G) ist daher weder beweisbar noch widerlegbar, sie
ist unentscheidbar. Wenn man weiterhin an das Prinzip vom ausgeschlossenen Dritten (“Tertium non datur”) glaubt, muss man daher an
eine Quelle der Wahrheit glauben, die außerhalb der arithmetischen
Axiome angesiedelt ist.
Damit ist der erste Gödelsche Unvollständigkeitssatz (mit einigen
Ungenauigkeiten) bewiesen:
UV1. Wenn die Axiome der Arithmetik widerspruchsfrei sind, dann ist die Arithmetik unvollständig, d.h. es
gibt Sätze wie (G), die weder bewiesen noch widerlegt
werden können.
Durch Formalisierung dieses Satzes wird nun der Satz UV2 gezeigt,
der besagt, dass die Widerspruchsfreiheit nicht abgeleitet werden kann.
Die Voraussetzung von UV1 kann man nach dem logischen Prinzip
“Ex Falso Quodlibet” (aus Falschem kann man Beliebiges schließen) so
umschreiben: Es gibt eine Formel, die nicht ableitbar ist,
(W )
∃y : ¬(beweisbar(y)).
Die Behauptung von UV1 können wir durch “(G) ist nicht beweisbar”
ersetzen, formal:
¬(beweisbar(g))
(“Die Formel mit der Gödelzahl g ist nicht beweisbar”). Das ist aber
genau der Inhalt der obigen Formel (G), und deshalb ergibt Satz UV1
die ableitbare Formel
(U V 1)
(W ) ⇒ (G)
Wäre die Widerspruchsfreiheit der Arithmetik in unserer Sprache beweisbar, dann wäre die Voraussetzung (W ) von (UV1) ableitbar, damit
aber auch die Behauptung (G). Aber das ist falsch, wie wir gesehen
haben: (G) ist ja gerade nicht beweisbar. Deshalb kann (W ) nicht ableitbar sein, also gibt es keinen Widerspruchsbeweis.
STERNSTUNDEN DER MATHEMATIK
145
Gibt es unentscheidbare Formeln, die mathematisch interessanter
sind als (G)?249 Ein Kandidat war die Kontinuumshypothese, um die
es in Hilberts erstem Problem geht. Cantor hatte gezeigt, dass die
Mächtigkeit der Potenzmenge P (N) der Menge N = {0, 1, 2, 3, . . . }
strikt größer ist als die von N selbst, siehe Fußnote 237. Aber gibt es
noch eine Mächtigkeitsstufe dazwischen? Gibt es Teilmengen von P (N),
die strikt mächtiger sind als N, aber nicht so mächtig wie P (N)?250
Gödel zeigte 1936, dass sich die Kontinuumshypothese aus den Axiomen von Zermelo und Fraenkel nicht widerlegen lässt, und Paul Cohen251 bewies 1963, dass sie sich daraus auch nicht ableiten lässt, wofür
er 1966 die Fields-Medaille erhielt.
15. Bott: Der Periodizitätssatz (1959)
Bernhard Riemann hat den Unterschied zwischen Geometrie und
Topologie, zwischen Abstands- und Zusammenhangsverhältnissen eines Raumes erkannt: Die gleiche “Mannigfaltigkeit” kann verschiedene
“Geometrien” tragen (vgl. Abschnitt 173). Die Topologie behandelt
Eigenschaften einer Mannigfaltigkeit,252 die von der Geometrie weitgehend unabhängig sind und bei Verformungen erhalten bleiben. Trotz
früher Ansätze bei Euler, Gauß und Riemann hat es lange gedauert, bis
die Topologie als ein eigenes mathematisches Gebiet anerkannt wurde.
Als Startpunkt gilt der Artikel “Analysis Situs” (das ist der ältere Namen für “Topologie”) von 1895 von Henri Poincaré.253 Die erste internationale Konferenz, mit der sich das Gebiet etablierte, fand 1935 in der
sowjetischen Hauptstadt Moskau statt, wo sich die damalige topologische Weltelite versammelte. Viele bis heute entscheidende Anstöße und
249Siehe
dazu auch Kapitel 16
Frage gewinnt an Bedeutung, weil P (N) zur Menge R der reellen Zahlen gleichmächtig ist: Jeder Punkt im Einheitsintervall [0, 1] lässt dadurch sich genau festlegen, dass man das Intervall wieder und wieder halbiert und jedesmal
bestimmt, ob der Punkt in der oberen oder der unteren Hälfte des zuletzt betrachteten Teilintervalls liegt; das ist die Dualbruch-Entwicklung. Solche “oben-unten”oder 0-1-Folgen entsprechen den Teilmengen T ⊂ N, wobei T aus den Nummern
der Folgenglieder mit Wert 1 (oder “oben”) besteht.
251
Paul Joseph Cohen, 1934 (Long Branch, N.J., USA) - 2007 (Stanford,
Kalifornien)
252
allgemeiner: eines topologischen Raumes, ein Begriff, der 1914 von Felix Hausdorff, 1868 (Breslau) - 1942 (Bonn), in seinem Buch “Grundzüge der Mengenlehre”
geprägt wurde. Wir werden oft einfach “Raum” sagen.
253
Jules Henri Poincaré, 1854 (Nancy) - 1912 (Paris),
http://gallica.bnf.fr/ark:/12148/bpt6k4337198.image.r=langFR.f7.pagination
250Die
146
J.-H. ESCHENBURG
begriffliche Entwickungen wurden hier zum ersten Mal präsentiert.254
Eines der Themen war die kürzlich entwickelte Homotopietheorie, vorgestellt von Hurewicz,255 von der hier die Rede sein soll.
Ein Ziel der Topologie ist die Klärung der Zusammenhangsverhältnisse
eines Raumes P oder, negativ ausgedrückt, das Auffinden und Bestimmen seiner “Löcher”. Ein Loch in der Straße (ein “2-dimensionales
Loch”) kann man umlaufen, man kann aber nicht hindurchlaufen ohne hineinzufallen. Das Umlaufen kann man mathematisch durch eine
stetige Abbildung der Kreislinie S1 = {v ∈ R2 : |v| = 1} nach P beschreiben, die das Loch umschließt, und dass es unbedeckt ist heißt,
dass man diese Abbildung nicht stetig auf die Kreisscheibe D2 = {v ∈
R2 : |v| ≤ 1} fortsetzen kann. Stetige Abbildungen der S1 können solche
Löcher also “entdecken”. Es gibt aber auch andere Sorten von Löchern,
3-dimensionale, wie die im Schweizer Käse. Ein solches Loch wird von
eine stetigen Abbildung der S1 nicht entdeckt, weil diese sich problemlos auf die Kreisscheibe D2 fortsetzen lässt; man muss die Scheibe ja
nur etwas ausbeulen, um das kugelförmige Loch zu vermeiden. Aber
wir können die Kreislinie S1 durch die Kugelfläche S2 ersetzen256 und
diese so in den “Käse” P abbilden, dass ihr Bild das Loch umhüllt:
Eine solche Abbildung lässt sich nicht auf die Vollkugel D3 fortsetzen,
denn das Loch verhindert es. Es gibt auch 1-dimensionale Löcher, wenn
nämlich P in zwei disjunkte Teilmengen P± zerfällt, P = P− ∪ P+ , die
nicht durch einen Weg, eine stetige Abbidung [−1, 1] → P verbunden werden können. Dann kann keine Abbildung f : S0 = {±1} → P
mit f (±1) ∈ P± auf D1 = [−1, 1] stetig fortgesetzt werden, denn eine Fortsetzung wäre ja ein Weg zwischen P− und P+ . Die maximalen
zusammenhängenden Teilmengen einer Menge P heißen ihre Zusammenhangskomponenten.
Allgemein wird ein k+1-dimensionales Loch definiert durch eine stetige Abbildung f : Sk → P , die nicht stetig auf Dk+1 fortgesetzt werden
kann. Natürlich werden wir das gleiche Loch beschreiben, wenn wir die
Abbildung f etwas deformieren. Solche Deformationen nennen wir Homotopien: Zwei Abbildungen f, g : Sk → P heißen homotop, wenn es
eine stetige Abbildung F : Sk × [0, 1] → P, F (v, s) =: fs (v) gibt mit
f1 = f und f0 = g. Ein (k + 1)-dimensionales Loch wird also durch
eine Homotopieklasse [f ] von Abbildungen f : Sk → P definiert, die
254
Hassler Whitney: Moscow 1935: Topology moving to America,
http://www.ams.org/samplings/math-history/hmath1-whitney10.pdf
255
Witold Hurewicz 1904 (Lódź, Polen) - 1956 (Uxmal, Mexiko)
256Für jedes k ≥ 0 ist Sk = {v ∈ Rk+1 : |v| = 1} die “k-Sphäre”, und Dk+1 =
{v ∈ Rk+1 : |v| ≤ 1} die “(k + 1)-Scheibe”.
STERNSTUNDEN DER MATHEMATIK
147
nicht zur konstanten Abbildung homotop sind (sonst wäre das Loch
zugedeckt).257
Zwei k+1-dimensionale Löcher [f ], [g] in P kann man zu einem neuen
verschmelzen oder “addieren”, wie die folgende Figur andeutet.
f(N)
N
f
N
f(S)
m
g(N)
f*g
m
g
S
S
g(S)
In der Figur sind N und S Nord- und Südpol der Sphäre Sk (die Punkte
in Sk mit letzter Koordinate ±1), und m ist ein beliebiger Großkreisboden (Meridian) zwischen N und S. Für zwei stetige Abbildungen
f, g : Sk → P definieren wir f ∗ g : Sk → P , indem wir sie auf jedem Meridian m angeben: Die Einschränkung (f ∗ g)|m wird mit Hilfe
von f |m und g|m wie in der Figur angedeutet definiert: Erst wird f (m)
durchlaufen, dann der gestrichelte Verbindungsweg, dann g(m).258 Mit
dieser “Addition” bilden die k-dimensionalen Löcher eine kommutative Gruppe, die k-te Homotopiegruppe πk (P ). Dabei ist die “Null” die
triviale Homotopieklasse, die Klasse der konstanten Abbildung.
Die Mannigfaltigkeit P , deren Homotopiegruppen (Löcher) Bott259
bestimmte, war die Orthogonale Gruppe On , die aus den längentreuen
linearen Transformatinen des Rn besteht, sowie ihre Verwandten Un
und Spn , wo Rn durch Cn und Hn ersetzt wird. Eine Abbildung g :
Rn → Rn heißt linear, wenn g(v + w) = gv + gw und g(tv) = tgv für
257Da
die Scheibe Dk+1 als die Schar konzentrischer Sphären Skr mit Radius r ∈
[0, 1] aufgefasst werden kann (wobei Sk0 der Mittelpunkt 0 ist), ist eine Homotopie
F = (fs ) : Sk × [0, 1] → P mit f1 = f und f0 = p = const dasselbe wie eine
Fortsetzung von f auf Dk+1 .
258Wir müssen dazu natürlich voraussetzen, dass die Punkte f (S) und g(N )
durch einen Weg verbindbar sind. Im Allgemeinen ist die Definition der Homotopiegruppen etwas komplizierter, und π1 muss gar nicht kommutativ sein. Die hier
behandelten Räume P sind aber symmetrische Räume, vgl. Fußnote 268, und für
diese ist π1 kommutativ. Die angegebene Konstruktion ist daher richtig für k = 1.
Für k ≥ 2 müssen wir zur “universellen Überlagerung” P̃ übergehen, wobei die
Homotopiegruppen nicht verändert werden.
259Raoul Bott, 1923 (Budapest) - 2005 (Carlsbad, Kalifornien)
148
J.-H. ESCHENBURG
alle v, w ∈ Rn und t ∈ R. Sie heißt längentreu, wenn |gv| = |v| für alle
v ∈ Rn . Diese Transformationen g bilden eine Gruppe mit der Komposition (Hintereinanderausführung) als Gruppenoperation. Die Eins
der Gruppe ist die identische Abbildung I, das Inverse ist die Umkehrabbildung. Das ist die Symmetriegruppe der höherdimensionalen
euklidischen Geometrie; sie ist deshalb von grundlegender Bedeutung
in der Mathematik. Bott ist es gelungen, alle k-dimensionalen Löcher
in On zu finden, wenn n genügend groß ist (n >> k). Der wichtigste
Schritt dazu ist der Bottsche Periodizitätssatz,260 der in der einfachsten
Version besagt:
Satz. Für n >> k ist πk (On ) 8-periodisch, d.h. πk (On ) = πk−8 (On ).
Raoul Bott (geb. 1923) wuchs in Budapest und Bratislava (Slovakei)
auf. Seine Mutter, die schon 1935 starb, war Jüdin. Seine Stiefeltern sahen die politische Katastrophe in Europa heraufziehen und emigrierten
1938 mit ihm nach Kanada. Dort studierte er zunächst Elektrotechnik,
dann Mathematik an der McGill-Universität in Montreal. 1947 promovierte er am Carnegie Institute of Technology in Pittsburgh und löste
ein damals berühmtes Problem der Netzwerktechnik. Das brachte ihm
1949 durch Vermittlung von Hermann Weyl eine Einladung an das Institute of Anvanced Study in Princeton ein, wo er sich mathematisch
gesehen “wie ein Kind im Süßwarenladen” fühlte.261 Besonders prägend
war seine Begegnung mir Marston Morse,262 der eine Theorie entwickelt
hatte, die Topologie mit Analysis auf eine neuartige Weise verband;
wir werden sie gleich erklären. Nach seinem Princeton-Aufenthalt war
Bott von 1951 bis 1959 Professor an der University of Michigan, wo er
in Zusammenarbeit mit Hans Samelson263 die Theorie von Morse auf
Gruppen anwendete, die gleichzeitig Mannigfaltigkeiiten sind (Liegruppen),264 wie zum Beispiel On . Ab 1959 war er Professor in Harvard.
In der Morsetheorie geht es darum, die Topologie einer Mannigfaltigkeit M mit Hilfe einer geeigneten Funktion h : M → R zu verstehen.
Wir stellen uns dazu den Graphen von h als ein Gebirge auf M vor
260R. Bott, The stable homotopy of the classical groups, Ann. Math. 70 (1959),
313–337. http://www.maths.ed.ac.uk/ aar/papers/bott4.pdf
Siehe auch: J. Milnor: Morse Theory, Princeton 1963,
http://web.xidian.edu.cn/pinganzhang/files/20111207 212010.pdf
261Interview with Raoul Bott (2000),
www.ams.org/publications/journals/all/fea-bott.pdf
262Harold Calvin Marston Morse, 1892 (Waterville, Maine, USA) - 1977 (Princeton, New Jersey, USA)
263Hans Samelson, 1916 (Straßburg / Strasbourg) - 2005 (Palo Alto, USA)
264Sophus Lie, 1842 (Nordfjordeid, Norwegen) - 1899 (Kristiania, heute Oslo)
STERNSTUNDEN DER MATHEMATIK
149
und beobachten den Fluss des Regenwassers, das von den Bergen herunter in die Täler fließt. Das Regenwasser läuft dem Gradienten der
Funktion h entgegen; der Gradient ∇h ist das Vektorfeld, das überall
in die Richtung größten Anstieges der Funktion zeigt.265 Berge und
Täler sind die lokalen Maxima und Minima der Funktion h; dort ist
der Gradient Null. Es gibt aber noch weitere Punkte, wo der Gradient
Null ist (“kritische Punkte”), nämlich die Sättel oder Pässe in unserem
Graphen-Gebirge.
Nicht alle Regentropfen laufen in die Täler; ein paar Tropfen bleiben
auch an den Sätteln hängen, wenn sie zufällig genau dorthin laufen.
Aber je mehr (linear unabhängige) Richtungen von einem (hochdimensionalen) Sattel ausgehen, in denen die Funktion h kleiner wird, desto unbedeutender ist dieser Effekt; das meiste Regenwasser wird am
Sattel vorbei ins Tal fließen, zur Menge Mo der Minima. Die Anzahl
dieser “nach unten führenden” Richtungen heißt der Index des kritischen Punktes.266 Wenn eine Teilmenge von M sich so deformieren
lässt, dass keiner ihrer Punkte mehr unter dem Regenwasserfluss an
einem der Sättel hängen bleibt, wird sie in die Menge Mo der Minima
gespült und ist damit homotop zu einer Teilmenge von Mo .
In Botts Anwendung ist M überraschenderweise nicht etwas On selbst,
sondern der Raum ΛOn aller Wege267 λ : [0, 1] → On von I nach
−I, alsoR λ(0) = I und λ(1) = −I, und die Funtion Rh ist die Länge
1
1
h(λ) = 0 |λ′ | oder besser noch die Energie h(λ) = 0 |λ′ |2 (im Wesentlichen das Quadrat der Länge). Dazu müssen wir die Geometrie
von On kennen. Nun liegt ja die Mannigfaltigkeit On im Raum aller
linearen Abbildungen von Rn auf sich (n × n-Matrizen), Rn×n . Dies ist
2
der euklidischer Raum Rn , und On liegt darin so wie eine Fläche im
265Der
Regenwasserfluss ist also der Fluss der Vektorfeldes −∇h, das noch mit
einem ortsabhängigen positiven Faktor multipliziert werden darf.
266Genau definiert ist der Index eines kritischen Punktes p die Dimension
des größten Unterraums, auf dem die Matrix der zweiten partiellen Ableitungen
∂i ∂j h(p), die Hessematrix, negativ definit ist.
267Statt aller Wege kann man auch nur geodätische Polygone mit höchstens N
Ecken betrachten; dann hat der Raum M endliche Dimensionszahl.
150
J.-H. ESCHENBURG
Anschauungsraum R3 . Jeder Weg in On wird als Weg in Rn×n aufgefasst und in diesem euklidischen Raum werden seine Länge und Energie gemessen. Der Gradient von h an einer Stelle λ ∈ ΛOn ist das
Krümmungsvektorfeld längs λ, das ist in jedem Punkt des Weges die
Richtung, in die er sich am stärksten krümmt. Die kritischen Punkte
von h sind daher die Wege, die überhaupt nicht gekrümmt sind, die
Geodäten, und die Minima von h sind die kürzesten oder minimalen
Geodäten. Bott zeigte, dass die nichtminimalen Geodäten in On einen
hohen Index haben, also im Sinne des Regenwasserflusses unbedeutend
sind. Dieses Phänomen kann man schon an Sphären beobachten: Eine Geodäte zwischen Nord- und Südpol auf S2 , die nicht minimal ist,
muss die Sphäre auf einem Großkreis mindestens anderthalbmal umrunden und kann deshalb leicht verkürzt werden, sobald er den Südpol
überschritten hat.
N
S
Wenn wir die 2-Sphäre S2 durch eine hochdimensionale Sphäre Sn ersetzen, liegt jeder Großkreisbogen in vielen 2-Sphären, und sobald er
den Südpol passiert hat, kann er in jeder dieser 2-Sphären verkürzt werden, also in vielen unabhängigen Richtungen; der Index einer solchen
Geodäten ist somit hoch. Ein ähnliches Argument gilt für die Gruppe
On für genügend großes n.
Was aber hat ΛOn , der Raum der Wege von I nach −I in On , mit den
Löchern von On zu tun? Ein Loch in On wird ja durch eine Abbildung
f : Sk → On repräsentiert. Diese kann aber als eine Abbildung fˆ :
Sk−1 → ΛOn aufgefasst werden. Wir zeichnen dazu in Sk einen Punkt N
aus und nennen ihn “Nordpol”, ihr Antipode S = −N heißt ”Südpol”.
Auf halbem Weg dazwischen liegt die “Äquatorsphäre” Sk−1 = Sk ∩
Rk . Die beiden Pole sind durch die Meridiane verbunden; durch jeden
Punkt v ∈ Sk−1 geht genau ein Meridian mv .
STERNSTUNDEN DER MATHEMATIK
151
N
S
k
mv
k
R
S
k−1
v
S
Unsere Abbildung f bildet jeden Meridian mv auf einen Weg f (mv )
von f (N ) nach f (S) ab. Wir deformieren f zunächst so, dass f (N ) und
f (S) auf einer sehr speziellen Position in On sitzen, nämlich f (N ) = I
und f (S) = −I. Dann ist f (mv ) ein Weg von I nach −I, und wir haben
aus f eine Abbildung fˆ : Sk−1 → ΛOn gemacht, die v ∈ Sk−1 auf den
Weg f (mv ) abbildet.
Diese Wege f (mv ) ∈ ΛOn werden nun mit dem Regenwasserfluss
der Energiefunktion auf ΛOn zu kürzesten Geodäten γv von I nach −I
deformiert.
k
N
S
k−1
S
On
I
γv
f
mv
I
On
v
P1
"
Verkurzung
S
−I
−I
Die Abbildung fˆ : Sk−1 → ΛOn kann die vergleichsweise dünnen
Ströme, die an nicht-minimalen Geodäten (den Sattelpunkten) enden,
durch eine kleine Deformation vermeiden, weil der Index dieser Geodäten
hoch ist, insbesondere größer als die Dimension k − 1. Nach dieser Deformation wird fˆ(Sk−1 ) durch den Regenwasserfluss ganz in die Menge
der Minima, d.h. der kürzesten Geodäten hineingespült. Damit haben
wir f so deformiert, dass f (mv ) eine kürzeste Geodäte γv : [0, 1] → On
von γv (0) = I nach γv (1) = −I geworden ist.
Wir sehen in der obigen Figur, dass On in gewisser Weise der Sphäre
S ähnlich sieht und dass die Abbildung f so deformiert wurde, dass
sie diese Ähnlichkeit hervorhebt: Die Punkte I und −I spielen in On
die Rolle von Nord- und Südpol N und S,268 und wie diese sind sie sind
k
268
Das lässt sich genauer sagen. Die Pole in der Sphäre sind dadurch gekennzeichnet, dass ihre Punktspiegelungen übereinstimmen: Die Isometrie der Sphäre
Sk , die N als Fixpunkt hat und jede Geodäte γ in Sk mit γ(0) = N reflektiert,
sγ(t) = γ(−t), ist die 180-Grad-Drehung um die Achse durch N und S; sie lässt
S ebenfalls fix und reflektiert die Geodäten durch S. (Geodäten in Sk sind die
Großkreise, die Schnitte von Sk mit einer Ebene durch den Ursprung) Die gleiche
152
J.-H. ESCHENBURG
durch eine große Schar von kürzesten Geodäten verbunden, die die Rolle der Meridiane einnehmen, u.a. die γv . Es gibt sogar eine Art Äquator
P1 ⊂ On , das ist der Ort, auf dem die Mittelpunkte der Meridiane liegen. So erhalten wir aus f die Mittelpunkts-Abbildung f1 : Sk−1 → P1 ,
die jedes v ∈ Sk−1 auf den Mittelpunkt γv ( 21 ) des Meridians γv abbildet. Umgekehrt lässt sich f aus f1 zurückgewinnen, denn eine kürzeste
Geodäte γ von I nach −I wird durch ihren Mittelpunkt γ( 21 ) eindeutig
bestimmt: Gäbe es neben γ noch eine weitere Geodäte γ̃ von I nach
−I mit gleicher Länge und denselben Mittelpunkt γ( 12 ) = γ̃( 12 ), dann
wäre keine der beiden Geodäten minimal, weil man sie abkürzen könnte
(Abschneiden der Ecke bei γ( 21 )).
~
γ
I
γ
γ (2_1 )
−I
Die Abbildung [f ] 7→ [f1 ] : πk (On ) → πk−1 (P1 ) definiert daher einen
Isomorphismus
(92)
πk (On ) ∼
= πk−1 (P1 ).
Aber wie sieht P1 genau aus? Dazu müssen wir die Geodäten in On
von I nach −I kennen, aus deren Mittelpunkten P1 besteht. Geodäten
γ : R → On mit γ(0) = I sind Einparameter-Untergruppen: Sie erfüllen
das Gruppengesetz γ(s + t) = γ(s)γ(t) für alle s, t ∈ R.269 Wegen
γ(1) = −I erfüllt J = γ( 21 ) also die Beziehung
Eigenschaft gilt für das Punktepaar I und −I in On ; die Punktspiegelung ist hier die
Inversion g 7→ g −1 auf On . Riemannsche Mannigfaltigkeiten, die in jedem Punkt
eine Punktspiegelung besitzen, heißen symmetrische Räume. Viele symmetrische
Räumen (aber nicht alle) besitzen Pole, Punktepaare mit der gleichen Punktspiegelung. Die im weiteren Verlauf auftretenden Räume sind symmetrisch und besitzen
Pole; diese Eigenschaft ist wesentlich für die Konstruktion.
269Geometrisch ist es leicht einzusehen, dass Einparametergruppen in O
n
Geodäten sind. Wäre es anders, gäbe es in jedem Punkt einer Einparametergruppe
γ eine Richtung, in die sich die Kurve biegt, das Krümmungsvektorfeld. Deformieren
wir die Kurve γ in diese Richtung, so erhalten wir eine kürzere Nachbarkurve. Die
Multiplikation mit γ(t) von links ist eine Isometrie von On , die die Kurve γ erhält
(nämlich in sich selbst verschiebt) und deshalb auch das Krümmungsvektorfeld von
γ erhält. Die Nachbarkurve ist deshalb vom Typ γx, siehe Figur. Aber die Multiplikation mit x von rechts ist auch eine Isometrie, deshalb haben die Kurven γx
und γ in Wahrheit die gleiche Länge, ein Widerspruch!
I
γ
x
γx
STERNSTUNDEN DER MATHEMATIK
153
J 2 = γ( 21 )γ( 21 ) = γ( 12 + 21 ) = γ(1) = −I.
Eine solche lineare Abbildung J mit J 2 = −I heißt eine komplexe Struktur. Der Raum P1 besteht also aus komplexen Strukturen J ∈ On .270
Er ist ein besonders schöner Unterraum von On , ähnlich einer Ebene
(im Gegensatz zu einer krummen Fläche) im Anschauungsraum, siehe die nachfolgende Figur. Die Gleichung J 2 = −I kann man nämlich
umschreiben zu −J −1 = J. Die Abbildung τ : g 7→ −g −1 ist eine
längentreue Abbildung von On , eine Isometrie271 von On , und damit
ist P1 ein reflektiver Unterraum von On , eine Zusammenhangskomponente der Fixpunktmenge einer Isometrie auf On . Da es in jeder Riemannschen Mannigfaltigkeit genau eine kürzeste Geodäte zwischen nahe benachbarten Punkten gibt, muss die Kürzeste γ in On zwischen
zwei benachbarten Punkten J, J ′ ∈ P1 ganz in P1 verlaufen, sonst gäbe
es ja zwei Kürzeste zwischen J und J ′ , nämlich γ und τ (γ). Unterräume
mit der Eigenschaft, dass jede ihrer Geodäten auch Geodäte im umgebenden Raum ist, nennen wir totalgeodätisch; Ebenen im Raum haben
diese Eigenschaft.
γ
P1
J
τ(γ)
J’
τ
Diese Konstruktion können wir wiederholen, wobei als nächstes P1
die Rolle von On übernimmt. Gegeben ist dann eine stetige Abbildung
f1 : Sk−1 → P1 . Wir können wieder f1 (N1 ) und f1 (S1 ) in eine spezielle Position in P1 verschieben, wobei wir mit N1 und S1 Nord- und
Südpol der Sphäre Sk−1 bezeichnen. Dazu wählen wir ein beliebiges
Element J1 ∈ P1 und deformieren f1 zunächst so, dass f1 (N ) = J1
und f1 (S) = −J1 gilt.272 Damit setzt sich f1 jetzt aus Wegen f1 (mv )
von J1 nach −J1 zusammen, wobei mv der Meridian von N1 nach S1
durch den Äquatorpunkt v ∈ Sk−2 ist. Wieder verkürzen wir diese
270Genauer: Die Menge aller komplexen Strukturen in O
n besteht aus zwei gleichartigen Zusammenhangskomponenten (anders als der Äquator der Sphäre). Das
Bild f1 (Sk−1 liegt in einer Komponente; diese nennen wir P1 .
271Für eine Matrix g ∈ O (eine orthogonale Matrix) ist die Umkehrmatrix
n
gleich der Transponierten, g −1 = g T , und die Transponierte hat ja die gleichen
Einträge wie die Ausgangsmatrix, nur in geänderter Reihenfolge, also ist g 7→ g T
eine Isometrie von Rn×n , die die Teilmenge On erhält, also auch eine Isometrie von
On .
272 Zweifellos ist −J wieder eine komplexe Struktur, denn (−J )2 = (J )2 = −I.
1
1
1
Aber dass −J1 und J1 in durch einen stetigen Weg in P1 verbindbar sein sollen,
schränkt die Wahl der Zusammenhangskomponente P1 von komplexen Strukturen
ein.
154
J.-H. ESCHENBURG
Wege simultan,273 bis sie zu kürzesten Geodäten γv von J1 nach −J1
geworden sind. Deren Mittelpunkte liegen in einer Untermannigfaltigkeit P2 ⊂ P1 , und ähnlich wie vorher können wir f1 durch die Mittelpunktsabbildung f2 : Sk−2 → P2 , f2 (v) = γv ( 12 ) beschreiben. Da eine
Kürzeste bei gegebenen Endpunkten durch ihren Mittelpunkt eindeutig
bestimmt ist, erhalten wir analog
(93)
πk−1 (P1 ) ∼
= πk−2 (P2 ).
Was ist der Unterraum P2 ? Dazu müssen wir die kürzesten Geodäten
in P1 kennen. Diese sind zum Glück auch Geodäten in On , weil P1 ja
totalgeodätisch in On ist. Die Geodäten in On kennen wir aber schon:
Eine Geodäte γ mit γ(0) = J1 ist vom Typ γ(t) = φ(t)J1 , wobei φ
eine Einparameter-Untergruppe ist. Weil zusätzlich γ(1) = −J gelten
soll, ist φ(1) = −I, und φ( 12 ) =: J2 ist daher eine weitere komplexe
Struktur. Der Mittelpunkt von γ ist demnach γ( 21 ) = J2 J1 . Dieser soll
aber wieder in P1 liegen, d.h. eine komplexe Struktur sein: (J2 J1 )2 = −I
oder
J2 J1 = −(J2 J1 )−1 = −J1−1 J2−1 = −J1 J2 .
Also ist J2 J1 genau dann selbst wieder eine komplexe Struktur, wenn J1
und J2 anti-kommutieren, J1 J2 = −J2 J1 , und P2 ist damit eine Zusammenhangskomponente der Menge aller komplexen Strukturen J2 ∈ P ,
die mit J1 antikommutieren. Ein solches Paar (J1 , J2 ) nennt man eine
quaternionale Struktur, denn es macht den zugrundeliegenden Raum
Rn zum quaternionalen Raum
Hm = {(x1 , . . . , xm ) : x1 , . . . , xm ∈ H},
mit m = n/4, wobei die Abbildungen J1 , J2 und J1 J2 wie die Multiplikationen der Einheitsquaternionen i, j, k mit jeder Komponente
des Vektors x = (x1 , . . . , xm ) wirken. Man kann den Raum Rn auf
viele verschiedene Weise mit Hm identifizieren, selbst wenn man J1
vorschreibt. Die Gleichung J2 J1 = −J1 J2 lässt sich zu der Fixpunktgleichung −J1 J2 J1 = J2 für J2 umschreiben, also ist P2 ⊂ P1 wieder
reflektiv, nämlich Fixpunktkomponente der Isometrie τ1 (J) = −J1 JJ2
auf P1 .
Diesen Prozess können wir fortsetzen und immer neue Unterräume Ps
von komplexen Strukturen Js definieren, die reflektive Unterräume ihrer Vorgänger Ps−1 sind und deren Elemente mit vorher bereits gewählten Elementen Jr von Pr (r < s) antikommutieren, Jr Js = −Js Jr , und
(94)
πk (On ) ∼
= πk−1 (P1 ) ∼
= πk−2 (P2 ) ∼
= ... ∼
= πk−s (Ps ) .
273Hierzu
benutzen wir den “Regenwasserfluss” für die Energiefunktion auf dem
Raum ΛP1 aller Wege von J1 nach −J1 in P1 .
STERNSTUNDEN DER MATHEMATIK
155
Jeder dieser Räume Ps besitzt noch eine andere geometrische Beschreibung, und die Periodizität kommt dadurch zustande, dass eins der Ps
wieder isometrisch zum Ausgangsraum On ist, nur mit kleinerer Dimension: P̂8 = On/16 .274
Um dies zu beweisen, müssen wir die 8 Räume P1 , . . . , P8 verstehen.
Viel einfacher ist der Fall des komplexen Analogons von On , der Gruppe
Un der komplex linearen längentreuen Abbildungen auf Cn . Hier ist
die Periode nämlich 2 statt 8, wir benötigen also nur zwei Schritte:
P1 ist die komplexe Grassmann-Mannigfaltigkeit der n2 -dimensionalen
Unterräume von Cn und P2 ist isometrisch zu Un/2 .
Das sehen wir folgendermaßen. Für jede komplexe Struktur J ∈ Un
ist S := iJ eine Involution, d.h. ihr Quadrat ist die identische Abbildung, denn S 2 = (iJ)2 = i2 J 2 = (−1)(−I) = I. Jede unitäre Involution
S auf V = Cn zerlegt V in den Fixraum V+ = {v : Sv = v} und den
dazu senkrechten Antifixraum V− = {v : Sv = −v}.275 Beides sind
komplexe Unterräume, d.h. iV± = V± . Umgekehrt erhalten wir S und
J = −iS aus V+ zurück, denn V− = (V+ )⊥ = {v ∈ V : v ⊥ V+ }.
Dabei kann V+ ein beliebiger komplexer Unterraum von V = Cn sein.
Komplexe Strukturen in Un entsprechen also komplexen Unterräumen
von Cn , und die Unterräume einer festen Dimension p bilden die Zusammenhangskomponenten, die komplexe Grassmann-Mannigfaltigkeit
Gp (Cn ). Eine davon ist P1 .276
Ebenso einfach beschreiben wir P2 : Die zweite komplexe Struktur J2
antikommutiert mit der Involution S = iJ1 . Damit vertauscht J2 den
Fix- und den Antifixraum V± von S, denn
v ∈ V+ ⇒ Sv = v ⇒ J2 v = J2 Sv = −SJ2 v ⇒ J2 v ∈ V− ,
274P̂
8 = On/16 hat (ebenso wie On ) zwei Zusammenhangskomponenten (die orientierungstreuen und die orientierungsumkehrenden Transformationen), was wir
durch die Bezeichnung P̂8 ausdrücken; für den nächsten Schritt muss eine davon
ausgewählt werden; das ist P8 . Wenn wir mit P̂k statt Pk arbeiten, können wir die
Gleichung (94) auf Seite 154 auch noch für p = k anwenden: πk (On ) = π0 (P̂k ). Wir
brauchen also nur noch die Zusammenhangskomponenten von P̂k zu zählen. Zum
Beispiel hat π1 (On ) zwei Elemente, weil P1 zwei Zusammenhangskomponenten hat.
Die Verkleinerung der Dimension von n auf n/16 ändert übrigens die Homotopiegruppen πk (On ) nicht, wenn n >> k: Die Gruppe On−1 liegt als Untergruppe
in On , und jede Abbildung f : Sk → On kann leicht nach On−1 deformiert werden.
275Für jeden Vektor v ∈ V gilt offensichtlich v = v + v mit v := 1 (v ± Sv),
+
−
±
2
und v± ∈ V± .
276Eine analoge Beschreibung für die komplexen Strukturen in O gibt es nicht,
n
weil wir in Rn nicht mit i multiplizieren können.
156
J.-H. ESCHENBURG
und ebenso v ∈ V− ⇒ J2 v ∈ V+ . Insbesondere haben V+ und V− die
gleiche Dimension p = n2 , was die Wahl der Zusammenhangskomponente P1 nachträglich einschränkt. Die Abbildung A = J2 |V+ bildet V+
längentreu und komplex linear nach V− ab; ebenso wird V− nach V+
durch B = J2 |V− abgebildet, und (J2 )2 = −I ⇐⇒ B = −A−1 . Also
wird J2 eindeutig durch eine unitäre Abbildung A : V+ → V− beschrieben und umgekehrt. Wenn wir V+ und V− jeweils mit Cn/2 identifizieren, wird A ein (beliebiges) Element von Un/2 , und damit haben wir
die 2-Periodizität bewiesen:
πk (Un ) ∼
= πk−1 (Gn/2 (Cn )) ∼
= πk−2 (Un/2 ).
Bei den Gruppen On und Spn ist die Periode 8; die Argumente sind
ähnlich, aber deutlich komplizierter.277
Neben der allgemeinen Idee der Homotopie sind es vor allem zwei Ideen, die in den Beweis des Satzes eingegangen sind: der Regenwasserfluss
277Im
Fall von On erhalten wir erst mit der dritten komplexen Struktur J3 eine
Involution S = J1 J2 J3 , deren Fix- und Antifixräume eine Zerlegung V = V+ ⊕ V−
von V = Rn bildet. Da J1 und J2 mit S kommutieren, sind die Teilräume V± invariant unter J1 , J2 und damit quaternionale Unterräume von V = Hn/4 , und P3 ist eine
quaternionale Grassmann-Mannigfaltigkeit. J3 J4 schließlich antikommutiert mit S
und vertauscht daher die beiden Räume V+ und V− , die somit gleiche Dimension
haben müssen. Da J3 J4 mit J1 und J2 kommutiert, ist es eine H-lineare orthogonale Abbildung zwischen V+ und V− . Die H-linearen orthogonalen Abbildungen auf
V± ∼
= Rn/2 = Hn/8 bilden die symplektische Gruppe Spn/8 , und wir erhalten
πk (On ) ∼
= πk−4 (Spn/8 ).
Wenn man andererseits mit G = Spm auf V = Hm startet, liegen alle komplexen Strukturen Jr in Spm , sind also H-linear. Wie in Un ordnet man der ersten
komplexen Struktur J1 den Fixraum W der Involution S1 = iJ1 zu (wobei i die
Skalarmultiplikation mit i ∈ H bezeichnet); weil iJ1 mit i kommutiert und mit j
antikommutiert, ist W ein komplexer Unterraum (iW = W ) mit jW = W ⊥ , und
P1 ist die Menge dieser “totalkomplexen” Unterräume. Die zweite Struktur J2 ersetzen wir durch die Involution S2 = jJ2 , die mit S1 kommutiert, also W erhält,
und mit i antikommutiert; der Fixraum X von S2 auf W ist daher “totalreell”:
senkrecht zu iX mit W = X + iX. Im dritten Schritt erhalten wir die Involution
S3 = kJ3 , die mit S1 , S2 kommutiert und deren Fix- und Antifixräume X± den
(reellen) Raum X zerlegen: X = X+ + X− . Da X+ ein beliebiger Unterraum von
X ist und J3 = −kS3 bestimmt, ist P3 eine reelle Grassmann-Mannigfaltigkeit.
Der vierte Schritt schließlich ist ähnlich wie der zweite im Un -Fall: Die komplexe
Struktur J3 J4 vertauscht mit S1 und S2 und antikommutiert mit S3 ; deshalb bildet
sie X+ und X− aufeinander ab, woraus Dimensionsgleichheit folgt. Wenn wir X+
und X− jeweils mit Rm/2 identifizieren, ergibt sich P̂4 ∼
= Om/2 . Damit folgt die
8-Periodizität für On und Spn :
πk (On ) ∼
= πk−8 (On/16 ).
= πk−4 (Spn/8 ) ∼
STERNSTUNDEN DER MATHEMATIK
157
in der Morse-Theorie, der fast jeden Punkt in die Menge der Minima
schwemmt, angewandt auf den Wegeraum mit der Energiefunktion, sowie die sehr spezielle Geometrie der Gruppe der längentreuen linearen
Abbildungen On , die Pole, Meridiane und einen Äquator besitzt. Ein
Element von πk (On ) wird durch eine stetige Abbildung f : Sk → On
oder fˆ : Sk−1 → ΛOn repräsentiert. Mit dem Regenwasserfluss für die
Energiefunktion auf ΛOn werden daraus kürzeste Geodäten von I nach
−I. Die Mittelpunkte dieser Geodäten liegen auf dem “Äquator” P1 ,
der totalgeodätisch in On liegt. Damit ist f zu einer Abbildung deformiert worden, die Meridiane auf Meridiane und den Äquator von
Sk in den Äquator von On abbildet; sie wird durch die MittelpunktsAbbildung f1 : Sk−1 → P1 bestimmt. Dieses Verfahren lässt sich wiederholen mit P1 anstelle von On und Sk−1 anstelle von Sk . Durch Iteration erhalten wir eine Kette On ⊃ P1 ⊃ P2 ⊃ . . . von Räumen,
die jeweils ein “Äquator” ihres Vorgängers sind. Insbesondere folgt
πk (On ) = πk−j (Pj ). Durch Identifizierung von P8 als On/16 folgt der
Periodizitätssatz.
Im Jahr 2000 erhielt Bott (gemeinsam mit Jean-Pierre Serre)278 den
renomierten Wolf-Preis für Mathematik.279 Er wurde für den Periodizitätssatz und weitere Entdeckungen im Bereich der Topologie ausgezeichnet. In der Tat steht der Periodizitätssatz am Beginn der großartigen Entwicklung der Topologie in der zweiten Hälfte des 20. Jahrhunderts, zu denen Bott, Atiyah, Singer, Hirzebruch, Borel280 und viele
andere beigetragen haben.
Übungen
15.1. Mittelpunkte kürzester Geodäten. Gegeben sei eine Riemannsche Mannigfaltigkeit P und zwei Punkte p, q ∈ P . Zeigen Sie,
dass jede kürzeste Geodäte γ von p nach q, genauer γ : [0, 1] → P mit
γ(0) = p und γ(1) = q durch ihren Mittelpunkt m = γ( 12 ) eindeutig
bestimmt ist, d.h. es gibt keine zweite Geodäte γ̃ von p nach q mit
Mittelpunkt m. (Vgl. Figur auf Seite 152.)
278Jean-Pierre
Serre, geb. 1926 (Bages, Frankreich), lebt in Paris.
279http://www.wolffund.org.il/index.php?dir=site&page=winners&cs=167
Aus der Begründung: “His first major contribution was the application of Morse
theory to the topology of Lie groups and homogeneous spaces, culminating in the
famous ‘periodicity theorems’ for the stable homotopy of the classical groups.”
280Sir Michael Francis Atiyah, geb. 1929 (London), Isadore Manuel Singer, geb.
1924 (Detroit), Friedrich Ernst Peter Hirzebruch, 1927 (Hamm) - 2012 (Bonn),
Armand Borel, 1923 (La Chaux-de-Fonds, Schweiz) - 2003 (Princeton)
158
J.-H. ESCHENBURG
15.2. Rechnen mit antikommutierenden komplexen Strukturen. Gegeben seien antikommutierende komplexe Strukturen J1 , J2 , J3 , . . .
auf einem Raum V , also Jr2 = −I und Jr Js = −Js Jr für r 6= s. Zeigen
Sie, dass S = J1 J2 J3 eine Involution ist (S 2 = I, die mit J1 , J2 , J3
antikommutiert und mit J2 J3 kommutiert.
15.3. 1. Homotopie der Kreislinie. Zeigen Sie π1 (S1 ) = Z.
Anleitung: Wir können jeden Punkt v der Kreislinie S1 ⊂ R2 = C
durch seinen Winkel t ∈ [0, 2π] zur positiven x-Achse beschreiben:
it
t
v = ( cos
sint ) = e . Dieser ist eindeutig bestimmt bis auf Vielfache von
2π. Eine stetige Abbildung f : S1 → S1 können wir mit Hilfe des Winkels als Abbildung f : [0, 2π] → S1 auffassen, und f (t) = eig(t) mit einer
stetigen Funktion g : [0, 2π] → R. Wir dürfen (ggf. nach Deformation)
f (0) = e1 und g(0) = 0 annehmen. Da f (2π) = f (0), ist g(2π) = n · 2π
für eine Zahl n = n(f ) ∈ Z, genannt Umlaufszahl. Wenn wir f stetig
deformieren, ändert sich auch die Umlaufszahl nur stetig, also gar nicht,
weil sie nicht zur nächsten ganzen Zahl springen kann. Die Umlaufszahl ist also nur von der Homotopieklasse [f ] abhängig. Umgekehrt,
wenn f, f˜ : S1 → S1 die gleiche Umlaufszahl n haben, dann haben die
Graphen der Abbildungen g, g̃ : [0, 2π] → R die gleichen Randpunkte
(0, 0) und (2π, n·2π) (Zeichnung!) und können bei festen Randpunkten
ineinander deformiert werden, also sind f und f˜ homotop. Bei Verkettung f1 f2 von zwei stetigen Abbildungen f1 , f2 : S1 → S1 addieren sich
die Umlaufszahlen. Die Abbildung φ : [f ] 7→ n(f ) : π1 → Z ist also ein
Gruppen-Isomorphismus.
15.4. 1. Homotopie von SO3 . Zeigen Sie π1 (SO3 ) ∼
= {±1}.
Anleitung: SO3 ist die Gruppe der orientierungserhaltenden orthogonalen Abbildungen (Drehungen) des euklidischen Raums R3 . Jede solche
Drehung können wir durch Konjugation mit Einheitsquaternionen realisieren, durch die Abbildung Ad(a)x = axā mit a ∈ S3 ⊂ H und x ∈ H.
Offensichtlich erhält Ad(a) die Länge, |axā| = |x|, und Ad(a)1 = 1, also erhält Ad(a) den Imaginärteil H′ = 1⊥ ∼
= R3 . Für a = cos t + v sin t
′
für v ∈ H mit |v| = 1 ist Ad(a) die Drehung mit Drehachse Rv und
Drehwinkel 2t in H′ . Damit ist Ad : S3 → SO3 surjektiv und außerdem
ein Gruppen-Homomorphismus, und Ad(a) = Ad(b) ⇐⇒ b = ±a. Jeder Meridian von 1 nach −1 wird auf eine geschlossene Kurve in SO3
abgebildet (eine 360-Grad-Drehung um eine feste Achse), und diese
Kurve ist nicht auf einen Punkt zusammenziehbar, weil ihr Urbild, der
Meridian in S3 , keine geschlossene Kurve ist. Aber wenn ich diese Kurve zweimal durchlaufe (720-Grad-Drehung), dann durchlaufe ich auf
STERNSTUNDEN DER MATHEMATIK
159
S3 einen vollen Großkreis, den ich zusammenziehen kann. Deshalb hat
π1 (SO3 ) genau zwei Elemente.
15.5. 1. Homotopie von Spn . Zeigen Sie, dass Spn für alle n ≥ 1
einfach zusammenhängend ist, d.h. π1 (Spn ) = 0.
Anleitung; Sp1 = S3 ⊂ H, und Sn ist einfach zusammenhängend für
alle n ≥ 2: Wenn eine Abbildung f : S1 → Sn mindestens einen Punkt
von p ∈ Sn nicht trifft (diese Voraussetzung kann durch lokale Deformationen immer erfüllt werden) dann ist ganz Sn \ {p} auf den Punkt
−p zusammenziehbar (längs der Großkreise von p nach −p), damit
auch f . Nun beachte man, dass die Abbildung π : Spn → S4n−1 ⊂ Hn ,
A 7→ Ae1 die Nebenklassen von Spn−1 als Urbilder der Punkte (Fasern)
hat (vgl. Fußnote 269) und schließe aus dem einfachen Zusammenhang
von Spn (Induktionsvoraussetzung) und S4n+1 auf den einfachen Zusammenhang von Spn+1 , ähnlich wie in Fußnote 269.
16. Klingenberg: Krümmung und Gestalt (1961)
Die Riemannsche Geometrie in der Gestalt, die Riemann, Christoffel,
Levi-Civita und andere ihr gegeben haben, ist zunächst eine Theorie,
die in lokalen Koordinaten im Rn beschrieben wird. Sie beruht auf
den drei Größen, die in Einsteins Allgemeiner Relativitätstheorie (siehe
Abschnitt 7) dem Gravitationspotential, der Gravitationskraft und der
Massendichte bei Newton entsprechen:
P
• die Metrik ds2 = g = ij gij (x)dxi dxj ,
P
• die Levi-Civita-Ableitung ∇i ej = l Γlij el mit Γlij wie in (85),
• der Krümmungstensor Rij = [∇i , ∇j ], gegeben durch (86).
Dabei sind x = (x1 , . . . , xn ) die Koordinaten, e1 , . . . , en die Koordinatenvektoren: e1 = (1, 0, . . . , 0), e2 = (0, 1, 0, . . . , 0) usw. und ∇i die
Levi-Civita-Ableitung nach xi (d.h. in Richtung ei ).
1968 erschien ein Buch mit dem Titel “Riemannsche Geometrie im
Großen”.281 “Im Großen” bedeutete, dass ein einzelnes Koordinatensystem nicht mehr zur Beschreibung ausreichte; dafür mussten ganz
neue Methoden entwickelt werden. Das Buch hatte seinen Ursprung
1961 in einer Gastvorlesung von Wilhelm Klingenberg282 an der Universität Bonn über den gerade von ihm und Marcel Berger (geb. 1927,
281D.
Gromoll, W. Klingenberg, W. Meyer: Riemannsche Geometrie im Großen,
Springer Lecture Notes in Math. 55, 1968
282Wilhelm Paul Albert Klingenberg, 1924 (Rostock) - 2010 (Bonn)
160
J.-H. ESCHENBURG
Paris) bewiesenen “Sphärensatz”. Zwei Bonner Studenten, Detlef Gromoll und Wolfgang Meyer283 arbeiteten den Inhalt der Vorträge aus
und schufen damit ein Lehrbuch für eine ganze Generation. Der Satz
lautete:
Liegt die Schnittkrümmung einer einfach zusammenhängenden und vollständigen Riemannschen Mannigfaltigkeit M der Dimension n überall zwischen 14 und 1 (Grenzen ausgeschlossen), dann ist sie vom topologischen Typ
einer n-dimensionalen Sphäre.
Die Schnittkrümmung K einer Riemannschen Mannigfaltigkeit wird
durch bestimmte Koeffizienten des Kümmungstensors gegeben, nämlich
durch Rijji (x) für jedes Koordinatensystem mit gij (x) = δij . Schon
Riemann hat die geometrische Bedeutung dieser Koeffizienten erkannt
als Gaußkrümmung der Fläche, die aus den Geodäten durch x in der
xi xj -Ebene zusammengesetzt ist. Die Voraussetzung “einfach zusammenhängend” bedeutet, dass jede Schlinge in M sich auf einen Punkt
zusammenziehen lässt; auf einem Zylinder ist das zum Beispiel nicht
möglich, wenn sich die Schlinge einmal um den Zylinder wickelt. Eine Riemannsche Mannigfaltigkeit heißt “vollständig”, wenn sie nicht
irgendwo “zu Ende” ist, wenn sich also jede “Gerade” (Geodäte) von
jedem ihrer Punkte aus beliebig weit fortsetzen lässt. Die Krümmungsschranken 14 und 1 sind optimal: Sobald die Schnittkrümmung beide
Schranken irgendwo annehmen darf, gibt es Gegenbeispiele zur Behauptung des Sphärensatzes, nämlich die projektiven Ebenen und projektiven Räume über den normierten Algebren C, H, O (vgl. Abschnitt
10). Der Satz behauptet nämlich, dass die vorgelegte Mannigfaltigkeit
M bei diesen Voraussetzungen unter den unzählbar vielen Möglichkeiten
nur die denkbar einfachste wählen kann: Sie lässt sich umkehrbar eindeutig und in beiden Richtungen stetig (“homöomorph”) durch die ndimensionale Sphäre Sn parametrisieren, die Menge der Vektoren von
Länge Eins im (n + 1)-dimensionalen euklidischen Raum Rn+1 .
Eine Schwäche allerdings hat diese Behauptung: Die Parametrisierung ist nur stetig in beiden Richtungen; sie kann zwar nicht zerreißen,
wohl aber knicken. Es gibt in der Tat ab Dimension 7 Mannigfaltigkeiten, die homöomorph, aber nicht diffeomorph (s.u.) zur Sphäre sind,
283Detlef
Gromoll, 1938 (Berlin) - 2008 (Stony Brook, N.Y., USA),
Wolfgang T. Meyer, geb. 1936 (Münster)
STERNSTUNDEN DER MATHEMATIK
161
sogenannte exotische Sphären; die ersten wurden von John Milnor entdeckt.284 Erst 2009 haben Brendle und Schoen285 diesen Mangel behoben und gezeigt, dass die Parametrisierung durch die Sphäre tatsächlich
in beiden Richtungen glatt ist, ein “Diffeomorphismus”. Möglich wurde
dieser Beweis durch neuartige Methoden, die Perelman286 beim Beweis
der Poincaré-Vermutung287 entwickelt hatte. Ein ehemaliger Augsburger Mathematikstudent, Christoph Böhm, hat einen wesentlichen Beitrag zu diesem “glatten Sphärensatz” geleistet.288 Die folgende Beweisskizze des ursprünglichen (nur stetigen) Sphärensatzes beruht auf Ideen
von Michail Gromov und Hermann Karcher.289
Die Schnittkrümmung hat verschiedene geometrische Bedeutungen.
Eine der wichtigsten ist: Sie misst die lokale Konkavität der Kugeln,
d.h. die Konvexität ihres Komplements.290 Die Kugel vom Radius r
um einen Punkt p ∈ M ist definiert als die Menge der Punkte, die
mit p durch eine Geodäte von einer Länge ≤ r verbunden sind. Eine
konvexe Teilmenge C ⊂ M hat nach außen gebogenen Rand; alle ihre Punkte sind durch geodätische Strecken verbunden, die nicht nach
außen gelangen. Im euklidischen Raum zum Beispiel sind alle Kugeln
konvex, ihre Komplemente konkav. Aber je größer die Kugeln werden,
desto ebener wird ihr Rand; mit wachsendem Abstand geht die Konkavität der Komplemente also zurück. Betrachtet man statt des euklidischen Raums eine Sphäre (beliebiger Dimension) mit Radius r, so
284Milnor, John W.: On manifolds homeomorphic to the 7-sphere. Annals of
Mathematics 64 (1956), 399405
285Brendle, Simon; Schoen, Richard: Manifolds with 1/4-pinched curvature are
space forms, Journal of the American Mathematical Society 22 (2009) 287307
286Grigori Jakowlewitsch Perelman, geb. 1966 in Leningrad (St. Petersburg)
287Die einzige geschlossene einfach zusammenhängende 3-dimensionale Mannigfaltigkeit ist die 3-dimensionale Sphäre S3 , vermutet von Henri Poincaré 1904, bewiesen von Grigori Perelman 2003.
288
Böhm, Christoph; Wilking, Burkhard: Manifolds with positive curvature operators are space forms. Ann. of Math. 167 (2008), 1079-1097.
289
Michail Gromov, Paris (geb. 1943), Hermann Karcher, Bonn (geb. 1938).
Details des Beweises bei J.-H. Eschenburg: Local convexity and nonnegative curvature - Gromov’s proof of the sphere theorem. Invent. math. 84, 507 - 522 (1986).
Der ursprüngliche Beweis findet sich in D. Gromoll, W. Klingenberg, W. Meyer:
Riemannsche Geometrie im Großen. Springer Lecture Notes in Math. 55, (1975).
290
Je kleiner die Krümmung, desto mehr divergieren die von einem Punkt ausgehenden Geodäten und desto konvexer sind die Abstandskugeln.
K<0
K=0
K>0
162
J.-H. ESCHENBURG
ist dieses Phänomen noch ausgeprägter: Die Kugeln von sphärischem
Radius ≤ π2 r sind noch konvex. Die Halbsphäre, die Abstandskugel
mit (sphärischem) Radius π2 r, ist auch bereits konkav, d.h. ihr Komplement, die andere Halbsphäre, ist konvex. Die Abstandskugeln mit
Radius > π2 r dagegen sind streng konkav; nur noch ihr Komplement
ist konvex.
< π/2
= π/2
> π/2
Vergleichen wir Kugeln mit gleichem Radius in zwei Sphären mit unterschiedlicher Krümmung, dann sehen wir:
Je größer die Schnittkrümmung, desto mehr krümmen
sich die Kugeln nach außen, desto konkaver sind sie.
Diesen Umstand machen wir uns beim Beweis der Sphärensatzes zunutze. Da eine Sphäre vom Radius R die Krümmung 1/R2 hat, dürfen
wir nach Voraussetzung annehmen, dass die Schnittkrümmung von M
zwischen der einer kleinen Sphäre vom Radius 1+ǫ und der einer großen
Sphäre vom Radius 2 − ǫ liegt. Wir betrachten nun überall die “lokalen” Kugeln: Von einem beliebigen Punkt p ∈ M ausgehend wählen wir
Riemannsche Normalkoordinaten (siehe Seite 105), die die von 0 ausgehenden radialen Strahlen in Rn längentreu auf von p ausgehenden
Geodäten abbilden, wobei die Winkel der Anfangsvektoren erhalten
bleiben. Es ist allerdings nicht sicher, ob diese Koordinaten überall 1:1
sind; es könnte ja sein, dass zwei von p ausgehende geodätische Strahlen
sich wieder treffen, wie es auf der Kugel- und der Zylinderfläche (siehe Figur) ja wirklich geschieht; der Treffpunkt wäre dann durch zwei
verschiedene Koordinatensätze beschrieben. Dieses Phänomen müssen
wir zunächst in Kauf nehmen.291
p
p
In diesem Sinn betrachten wir eine Kugel K vom Radius π sowohl in
M als auch in den beiden Vergleichssphären S1+ǫ und S2−ǫ .
291Zu
sehen, dass es im vorliegenden Fall nicht eintrifft, ist einer der wichtigsten
Beiträge von Wilhelm Klingenberg.
STERNSTUNDEN DER MATHEMATIK
163
p
π
π
K
K
π
K
C
S1+ε
M
S2−ε
In der kleinen Sphäre (mit größerer Krümmung) ist das eine Kugel, die
fast bis zum Antipodenpunkt reicht, aber noch nicht ganz. Die Kugel K
hat einen zwar stark nach außen gekrümmten, aber immer noch glatten
Rand. Auf der schwächer gekrümmten Mannigfaltigkeit M bleibt das
so: Der Rand der Kugel K ist sogar schwächer nach außen gekrümmt
und immer noch glatt.
In der großen (weniger gekrümmten) Sphäre S2−ǫ dagegen reicht die
Kugel vom Radius π nur gerade über den Äquator bis in die Südhalbkugel hinein und ist daher bereits konkav. Das Gleiche muss erst recht
auf der stärker gekrümmten Mannigfaltigkeit M gelten. Der Rand Σπ
der Kugel vom Radius π ist also lokal streng konkav; er krümmt sich
nach außen. Allerdings darf Σπ Selbstschnitte haben, denn zwei von
p ausgehende Strahlen von Länge π, die nicht mehr nahe benachbart
sind, dürfen sich sehr wohl wieder treffen, wie auf einem Zylinder, siehe
Figur oben.
Die nach außen gekrümmte Hyperfläche Σπ können wir nun nach
außen hin immer kleiner machen, indem wir zu Hyperflächen in konstantem Abstand übergehen, und schließlich auf einen Punkt zusammenziehen.
Das Komplement C ist also lokal diffeomorph zu einer Halbsphäre, und
das gleiche gilt für die Kugel K. Damit ist M lokal diffeomorph zu einer Mannigfaltigkeit M̃ , die sich aus zwei Halbsphären zusammensetzen
lässt (“Nordhalbkugel” und “Südhalbkugel”) und daher homöomorph
zur Sphäre Sn ist.292 Weil M einfach zusammenhängt, ist der lokale
Diffeomorphismus ein globaler: Wenn ein Punkt p ∈ M zwei Urbilder
292Die
beiden Halbsphären müssen noch am Rand “zusammengeklebt” werden.
Dies geschieht durch einen Diffeomorphismus zwischen den beiden Rändern, die
beide diffeomorph zu Sn−1 sind. Bei der Standardsphäre ist dieser Diffeomorphismus eine orthogonale lineare Abbildung. Es gibt aber auch Diffeomorphismen auf
164
J.-H. ESCHENBURG
p̃1 , p̃2 in M̃ hätte, könnten wir sie mit einem Weg verbinden. Dieser
würde auf eine Schleife mit Anfangs- und Endpunkt p in M abgebildet, und eine solche Schleife ließe sich nicht zusammenziehen, im Widerspruch zur Voraussetzung “einfach zusammenhängend”. Damit ist
M diffeomorph zu M̃ und damit homöomorph zu Sn .
Wir haben allerdings einige Schwierigkeiten unterschlagen. Zwar ist
Σπ eine glatte Hyperfläche, aber Selbstschnitte sind zugelassen. Wir
müssen zeigen, dass eine immersierte293 streng lokal konvexe Hyperfläche in M mit K ≥ 0 eine immersierte Halbsphäre berandet. Bei
Dimension n = 2 stimmt das nicht, denn schon in der Ebene R2 gibt
lokal konvexe geschlossene Kurven mit Selbstschnitten; diese beranden
keine immersierte Kreisscheibe oder Halbsphäre.
5
4
5 4
3
1
2
1
3
2
Aber für Dimension n ≥ 3 kann man solche Komplikationen ausschließen. Im euklidischen Rn zum Beispiel kann eine lokal streng konvexe geschlossene Hyperfläche keine Selbstschnitte mehr haben und
muss eine eingebettete Sphäre sein, denn die in der Figur angedeutete Gaußsche Normalenabbildung, die jedem Punkt der Hyperfläche
den Normaleneinheitsvektor, den Einheitsvektor senkrecht zur Tangentialhyperfläche zuordnet, ist ein lokaler Diffeomorphismus auf die
Sphäre Sn−1 und muss damit injektiv (1:1) sein, wie gerade gezeigt,
weil nämlich Sn−1 einfach zusammenhängend ist für n ≥ 3. Dieses Argument von Hadamard294 kann man in unserer Situation lokal benutzen
- lokal ist ja jede Mannigfaltigkeit fast isometrisch zum Rn - und damit
den Satz zeigen.
Übungen
16.1. Die Krümmungsschranken für die Projektiven Räume.
Der eindimensionale Projektive Raum, die Projektive Gerade P1 über
den komplexen Zahlen C ist die Menge der eindimensionalen Unterräume
[x, y] = C(x, y) in C2 . Mit der Abbildung [x, y] 7→ x/y können wir P1
mit Ĉ = C ∪ {∞} und dies wiederum (mit Hilfe der stereographischen
Sn−1 , die sich nicht auf eine orthogonale Abbildung deformieren lassen; die damit zusammengesetzten Mannigfaltigkeiten sind nur noch homöomorph, aber nicht
mehr diffeomorph zu Sn , es sind “exotische Sphären”.
293“immersiert” bedeutet, dass Selbstschnitte zugelassen sind.
294Jacques Salomon Hadamard 1865 (Versailles) - 1963 (Paris)
STERNSTUNDEN DER MATHEMATIK
165
Projektion) mit S2 identifizieren. Wenn wir (x, y) in der Einheitssphäre
S3 ⊂ C2 wählen, dann hat jeder Großkreis von und nach (x, y) die
Länge 2π. Aber der Großkreis geht auch durch den Punkt (−x, −y),
der unter der Projektion (x, y) 7→ x/y auf denselben Punkt wie (x, y)
abgebildet wird. Das Bild des Großkreises ist also schon nach der Länge
π wieder am Ausgangspunkt. Deshalb ist die S2 geometrisch gesehen
nur halb so groß wie die S3 , ihre Krümmung ist also viermal so groß. In
höheren Dimensionen gibt es durch jeden Punkt zwei extreme Sorten
von Ebenen: Die Tangentialebenen von komplex-projektiven Geraden
und die Tangentialebenen von reell-projektiven Ebenen; die Krümmung
der einen ist um den Faktor 4 größer als die der anderen. Ähnlich ist
es für die anderen normierten Algebren H und sogar O.
17. Shechtman: Quasikristalle (8.4.1982)
Der Nobelpreis für Chemie des Jahres 2011 war eine Überraschung.
Er ging an einen israelischen Physiker, Daniel Shechtman (geb. 1941
in Tel Aviv), für eine fast 30 Jahre zurückliegende Leistung: die Entdeckung der Quasikristalle. Es war eine späte Ehrung für einen Wissenschaftler, der die Kristallographie revolutionierte und es schwer gehabt
hatte, die Anerkennung der Fachwelt dafür zu finden. Es gebe keine
Quasikristalle, sondern nur Quasi-Wissenschaftler, soll der zweifache
Nobelpreisträger Linus Pauling295 über ihn gesagt haben.
Die Entdeckung, die am 8. Dezember 2011 mit dem Nobelpreis ausgezeichnet wurde, nahm ihren Anfang am 8. April 1982 am National
Bureau of Standards in Washington, wo Shechtman, Professor am Technion in Haifa, eine Forschungszeit verbrachte. Er untersuchte gerade
Beugungsbilder von Kristallstrukturen, die bei schnell abgekühlten Legierungen von Aluminium mit Mangan, Eisen und Chrom entstanden
waren, und sah plötzlich Bilder wie dieses:296
295Linus
Carl Pauling, 1901 (Portland, Oregon) - 1994 (Big Sur, Kalifornien),
Nobelpreis für Chemie 1954, Friedensnobelpreis 1962
296www.youtube.com/watch?v=EZRTzOMHQ4s&feature=player embedded
166
J.-H. ESCHENBURG
Ein solches Beugungsbild war nach der Überzeugung aller Kristallographen gar nicht möglich, denn die 5 Achsen deuteten eine Drehsymmetrie von Ordnung 5 an; der Kristall musste also bei einer Drehung um
360/5 = 72 Grad wieder in sich selbst übergehen. Derartige Drehungen
können aber in einem Kristall gar nicht vorkommen! Warum nicht?
Ein Kristall besteht aus Atomen, die in drei Raumrichtungen periodisch angeordnet sind. Viel vertrauter sind uns die zweidimensionalen
Analoga; das sind periodische Muster, wie sie uns an vielen Stellen im
Alltag begegnen.
Diese Muster wiederholen sich in zwei Richtungen der Ebene, sie lassen
sich also in sich selbst verschieben, wenn man sie sich über die ganze
Ebene ausgedehnt vorstellt; sie haben eine Verschiebungs-Symmetrie.
Zusätzlich gibt es auch Drehsymmetrie, es gibt also Drehungen, die das
Muster in sich selbst überführen, wobei die Drehordnungen im linken
STERNSTUNDEN DER MATHEMATIK
167
Muster 2, 3 und 6, im rechten 4 sind. Die Ordung 5 kommt nicht vor,
ebenso wenig wie die Ordnungen 7 und mehr.
Wir können leicht einsehen, warum das so ist. Stellen wir uns in dem
Muster ein Drehzentrum einer beliebigen Ordnung n vor; wir können
das Muster also um 360/n Grad um diesen Punkt drehen, ohne dass
danach eine Veränderung sichtbar ist. Wenn das Muster periodisch ist,
wiederholt sich alles periodisch; insbesondere gibt es weitere Drehzentren der Ordnung n. Unter denen suchen wir uns zwei Drehzentren
A und B aus, die so nahe wie möglich beieinander liegen; keine zwei
Drehzentren der Ordnung n haben also kleineren Abstand zueinander
als A und B. Zwei weitere Drehzentren A′ und B ′ gewinnen wir, wenn
wir A um den Punkt B nach rechts und B um den Punkte A nach
links drehen, jeweils um den Winkel 360/n Grad. Da das ganze Muster
unter diesen Drehungen erhalten bleibt, gehen dabei Drehzentren in
Drehzentren über. Wenn man das für n = 2 oder n = 3 macht, sind A′
und B ′ weiter voneinander entfernt als A und B.
B’
A’
360/3
360/2
B’
B
A
A’
A
B
Ordnung 3
Ordnung 2
Für n = 4, 5, 6, 7 sehen die Bilder dagegen so aus:
B’
B’=A’
A’
B’
A’
A’
B’
360/7
A
B
Ordnung 7
B
A
Ordnung 6
360/4
360/5
360/6
A
B
Ordnung 5
A
B
Ordnung 4
Bei n = 4 sind die Abstände von B ′ , A′ und von A, B gleich, und bei
n = 6 fallen B ′ und A′ genau aufeinander. Diese Fälle sind noch möglich
und realisierbar, wie wir ja in den Figuren auf Seite 166 gesehen haben.
Aber bei n = 5 sind B ′ und A′ näher beisammen als A und B, was
wir gerade ausgeschlossen hatten, und bei n ≥ 7 sind bereits B ′ und
B näher beisammen als A und B, weil der Winkel bei A kleiner ist als
der Winkel im gleichseitigen Dreieck (60 Grad). Alle diese Fälle sind
also ausgeschlossen.
Das gleiche Argument trifft auch noch in drei Dimensionen zu, bei
Kristallen. Dort sind die Drehzentren durch Drehachsen zu ersetzen.
168
J.-H. ESCHENBURG
Die Verschiebungen, die das Kristallgitter erhalten, werden jede Drehachse auf eine andere, dazu parallele Drehachse der gleichen Ordnung
verschieben. Wieder wählen wir uns zwei parallele Drehachsen der gleichen Drehordnung n in minimalem Abstand. Die obigen Bilder behalten ihre Gültigkeit, wenn wir sie etwas anders interpretieren: Die
Zeichenebene steht senkrecht auf den Drehachsen, und jedes Drehzentrum steht jetzt für eine Drehachse, die senkrecht auf den Betrachter
zukommt. Der Schluss ist der gleiche: Es kann keine Drehachsen von
Ordnung 5 oder 7 und größer in einem periodischen Kristallgitter geben.
Was also hatte Shechtman gesehen? Wenn Physiker eine Entdeckung
machen, bitten sie manchmal Mathematiker um Hilfe (oder sie sind
selbst gute Mathematiker). Dann fangen die Mathematiker an zu denken, und nach 10 Jahren kommen sie zurück und sagen, dass sie jetzt
einen Teil dieser Beobachtungen verstehen. Sehr gut, sagen die Physiker, aber leider interessieren wir uns jetzt für ganz andere Fragen. Wir
hätten aber auch da ein kleines Problem, bei dem wir eure Hilfe gebrauchen könnten ... Aber in diesem Fall war es anders. Die Mathematiker
waren tatsächlich einmal früher dran als die Physiker und hatten bereits
Modelle für diese Situation studiert. Das war purer Zufall, denn der Ursprung dieser Modelle war nicht Kristallographie, sondern eine Frage
der Logik und Informatik, das “Entscheidungsproblem”: Welche Fragen
lassen sich durch Algorithmen entscheiden? Man kann entscheiden, ob
eine ganze Zahl prim ist oder nicht, oder ob eine ganze Zahl Teiler einer
anderen ist oder nicht, aber es kann zum Beispiel keinen Algorithmus
geben, der entscheidet, ob eine ganzzahlige Polynomgleichung in mehreren Variablen eine ganzzahlige Lösung besitzt oder nicht; das Problem
ist “unentscheidbar”.297 Alan Turing298 zeigte 1936, dass das Halteproblem für Computerprogramme (Turing-Maschinen) unentscheidbar ist:
Kein Algorithmus kann für jedes beliebige Computerprogramm entscheiden, ob es irgendwann anhält oder nicht.
Manche Fragen dieser Art lassen sich auf Kachelungsprobleme zurückführen. Der chinesisch-amerikanische Informatiker und Mathematiker
Hao Wang299 vermutete 1961, dass von einem gegebenen endlichen
Satz von Kacheltypen mit Kantenfärbungen immer entschieden werden
297Zehntes
Hilbertsches Problem (David Hilbert, geb. 1862 in Königsberg, gest.
1943 in Göttingen), negativ beantwortet von Yuri Vladimirovich Matiyasevich, geb.
1947 in Leningrad (St. Petersburg)
298Alan Mathison Turing, 1912 (London) - 1954 (Wilmslow, England)
299Hao Wang, 1921 (Jinan, China) - 1995 (New York)
STERNSTUNDEN DER MATHEMATIK
169
kann, ob (unter Berücksichtigung der Kantenfärbungen) eine Pflasterung der ganzen Ebene mit Kacheln dieser Typen existiert oder nicht.
Wenn nämlich nicht, argumentierte Hao Wang, dann müsste man einen
“aperiodischen” Satz von Kacheltypen finden können, mit denen man
die ganze Ebene zwar pflastern kann, aber nur aperiodisch, also ohne Wiederholungen; das galt als extrem unwahrscheinlich, denn ohne
Wiederholung musste sich das Muster “unendlich viel Neues” einfallen
lassen, um die unendlich ausgedehnte Ebene zu pflastern. Doch 1964
konnte sein Doktorand Robert Berger (geb. 1936) zeigen, dass das Kachelungsproblem äquivalent zum Halteproblem und damit unentscheidbar ist. Tatsächlich fand Berger auch einen “aperiodischen” Satz von
Kacheltypen, zunächst mit mehr als 20 000 Typen, die später auf 13
reduziert werden konnten. Roger Penrose fand 1974 schließlich einen
aperiodischen Satz von zunächst drei, später zwei Typen,300 beruhend
auf der Geometrie des regelmäßigen Fünfecks. Die einfachsten Kacheltypen sind die beiden gleichschenkligen Dreiecke, die im regelmäßigen
Fünfeck vorkommen. Man gewinnt sie gleich mit der richtigen Unterteilung, wenn man im Fünfeck zwei sich kreuzende Diagonalen einzeichnet
und durch ihren Schnittpunkt die Parallele zur eingeschlossenen Seite
zieht (mittlere Figur).
An eine Dreiecksseite kann immer nur eine gleichartige Dreiecksseite angelegt werden, deshalb kommen die Penrose-Dreiecke immer im
Doppelpack, als Rauten vor. Die Unterteilung besteht aus den gleichen
zwei Sorten von Dreiecken, nur im verkleinerten Maßstab Eins zu Goldener Schnitt. Die kleineren Dreiecke können ebenso unterteilt werden,
und wenn die Unterteilungen an den Kanten zusammenpassen sollen,
gibt es dafür genau eine Möglichkeit. Diese können wir ebenso auf die
noch kleineren Dreiecke anwenden und noch feiner unterteilen. Da das
300Roger
Penrose (geb. 1931 in Colchester, Essex, England, 1973-1998 Professor
in Oxford): Pentaplexity: A class of non-periodic tilings of the plane, The Mathematical Intelligencer 2 (1979), 32-37
170
J.-H. ESCHENBURG
schmale Dreieck eigentlich ein Teil des breiten ist, brauchen wir nur
das letztere anzusehen.
Durch Wiederholung dieses Vorgangs (“Inflation”) entstehen feinere
und feinere Unterteilungen, und wenn wir das Dreieck so weit vergrößern, dass die kleinsten Teile wieder die Größe der ursprünglichen
Dreiecke haben, werden wir größere und immer größere Teile der Ebene
überdecken. Umgekehrt muss jedes Penrosemuster so zusammengesetzt
sein. Auf den ersten Blick sieht es so aus, als seien diese Muster dadurch vollkommen festgelegt, aber das ist nicht so. Es gibt unendlich
(sogar überabzählbar) viele verschiedene von ihnen. Sie gleichen darin
den Wegen in einem Baum, dessen Äste von einer Wurzel ausgehend
sich endlos wieder und wieder verzweigen. Von jedem Punkt aus gibt
es nur einen einzigen Weg nach unten zur Wurzel, und alle Wege der
gleichen Länge sehen absolut gleich aus (dies entspricht der Kongruenz
der Unterteilungen), nach oben hin aber gibt es unendlich viele verschiedene Wege, denn in jedem weiteren Verzweigungspunkt hat man
eine neue Wahlmöglichkeit. So gibt es auch immer neue Möglichkeiten,
ein gegebenes endliches Stück eines Penrose-Musters fortzusetzen: Die
Verzweigungen entsprechen den beiden möglichen Rollen eines schmalen Dreiecks als Teil entweder einer breiten oder einer schmalen Raute.
Da diese Dreiecke in beliebigen Größenordnungen auftreten, legt kein
endlicher Teil des Musters fest, wie das ganze Muster aussehen wird;
insofern unterscheiden sich die Quasikristalle von ihren periodischen
Brüdern durch eine stärkere Unbestimmtheit.
STERNSTUNDEN DER MATHEMATIK
171
Die Abbildung301 zeigt eins der beiden Penrose-Muster mit einer
globalen fünfzähligen Drehsymmetrie, die durch Inflation wechselseitig auseinander hervorgehen. Das Drehzentrum liegt in der Figur ganz
unten in der Mitte und sieht wie eine Rose mit fünf Blütenblättern
aus. Man sieht deutlich ein Inflationsmuster (bei dreifacher Inflation),
dessen Eckpunkte die aus fünf Rauten zusammengesetzten Sterne sind;
sie bilden das zweite Penrose-Muster mit fünfzähliger Drehsymmetrie.
Die Eckpunkte eines noch größeren Inflationsmuster vom zweiten Typ
(nach zwei weiteren Inflationsschritten) werden von den Sternen gebildet, die ganz von einem Kranz breiter Rauten umgeben sind, einem
301http://www.geom.uiuc.edu/apps/quasitiler/
172
J.-H. ESCHENBURG
Kranz, der entweder wie ein Fünfeck oder wie ein fünfzackiger Stern
geformt ist.
Übrigens gibt es eine Verbindung der Penrose-Muster zur islamischen
Kunst im Iran, wo verwandte Muster seit über 300 Jahren verwendet
wurden.302
Penrose-Muster sind ebene nichtperiodische Muster mit einer strengen Ordnung und lokaler Fünfeck-Symmetrie, die wir im Bild sehen
konnten. Würde man sie senkrecht durchleuchten, so würde ihr Beugungsbild eine genaue Fünfecksymmetrie aufweisen. Die von Shechtman gefundenen Quasikristalle sind aber dreidimensional, und die Beugungsbilder weisen in manchen Ebenen Fünfeck-Symmetrie, in anderen
aber eine Dreieck-Symmetrie auf. Die lokale Symmetriegruppe ist also komplizierter als die Fünfeck-Gruppe, es ist die Ikosaedergruppe A5 .
Ein Modell für solche Quasikristalle war bereits von dem Tübinger Physiker Peter Kramer (geb. 1933) und seinem Mitarbeiter Reinhardt Neri
entwickelt worden.303 Es beruhte auf einer anderen Beschreibung des
Penrosemusters, nämlich als eine zweidimensionale Projektion eines regelmäßigen Musters in 5 Dimensionen; diese Beschreibung stammt von
dem holländischen Mathematiker Nicolaas de Bruijn.304
E
Σ
302Peter
J. Lu and Paul J. Steinhardt: Decagonal and Quasi-Crystalline Tilings
in Medieval Islamic Architecture, Science 315 (2007), 1106 - 1110,
Saskia Franziska Mayer: Penrose-Muster und ihr Zusammenhang zu islamischen
Mustern, Augsburg 2013, www.math.uni-augsburg.de/∼eschenbu
303P. Kramer, R. Neri: On periodic and non-periodic space fillings obtained by
projection, Acta Cryst A 40 (1984) 580-587
304Nicolaas Govert de Bruijn, 1918 (Den Haag) - 2012 (Nuenen, Nordbrabant):
Algebraic theory of Penrose’s non-periodic tilings of the plane I,II, Proc. Koninklijke
Nederlandse Akademie van Wetenschapen, 84 (1981) 39-66
STERNSTUNDEN DER MATHEMATIK
173
Die Figur zeigt das Projektionsverfahren allgemein. Dabei ist E ein
sehr speziell gewählter305 k-dimensionaler affiner Unterraum (Gerade,
Ebene, ...) des Rn , der durch keinen der Gitterpunkte Zn ⊂ Rn geht.
Man projiziert nun die Gitterpunkte “nahe” E orthogonal auf E, genauer die Vektoren mit ganzzahligen Komponenten, die im “Streifen”
Σ = E + I n liegen, wobei I = (0, 1) = {t ∈ R : 0 < t < 1} das
offene Einheitsintervall und I n der Einheitswürfel ist. Die projizierten
Punkte bilden die Eckpunkte des Musters, sozusagen die Atome des
Quasikristalls. Im Fall des Penrosemusters ist n = 5 und k = 2, in
dem von Kramer und Neri untersuchten Fall dagegen ist n = 6 und
k = 3. Die zugehörige dreidimensionale Pflasterung ist aus zwei Typen
von “Pflastersteinen”, zwei dreidimensionalen Rhomben, zusammengesetzt. In einer Augsburger Doktorarbeit306 konnte die Unterteilung
der zwei Pflastersteine durch “Inflation” konstruiert werden. Alternativ gäbe es noch ein weiteres Modell mit n = 10, das bisher noch kaum
untersucht worden ist.
Die Figur auf Seite 171 hat einen sehr wörtlichen Bezug zu unserem Titel “Sternstunden”. Sie führt uns auch an den Anfang dieser
Vorlesungsreihe zurück, wo es um die Entdeckung der Irrationalität
und den Goldenen Schnitt ging; beides spielt bei den Quasikristallen
eine entscheidende Rolle. Die Irrationalität hat mit dem Unendlichen
zu tun: Eine irrationale Zahl kann durch Brüche mit noch so großen
Zählern und Nennern niemals genau ausgedrückt, nur angenähert werden. Aber diese Art “Unendlichkeit” ist kaum sichtbar; die Abweichung
von einem Bruch ist ja beliebig klein. Im Penrosemuster dagegen ist die
Unendlichkeit sozusagen nach außen entfaltet und führt zu dem unendlichen Reichtum an Formen, der für Quasikristalle charakteristisch ist.
So können die Quasikristalle als ein später Triumph des PythagorasSchülers Hippasos gelten, dem wir vermutlich die Entdeckung der Irrationalität verdanken.
Übungen
17.1. Drehzentren. Finden Sie alle Drehzentren in den Mustern auf
Seite 166 und die zugehörigen Drehordnungen. Finden Sie zu zwei Drehzentren A, B gleicher Ordnung mit minimalem Abstand auch die Drehzentren A′ , B ′ der Figuren auf Seite 167.
305E
ist parallel zu einem irreduziblen Darstellungsmodul der lokalen Symmetriegruppe, die Fünfeckgruppe im Penrosefall, die Ikosaedergruppe im Fall von Kramer
und Neri.
306Ruth Dietl: Dreidimensionale Penrose-Muster und Selbstähnlichkeit, Augsburg 2011, www.math.uni-augsburg.de/∼eschenbu
174
J.-H. ESCHENBURG
17.2. Symmetrische Penrosemuster. Setzen Sie die beiden symmetrischen Sterne zu Penrosemustern fort, indem Sie die Unterteilung des
einen Musters jeweils als Plan für das andere Muster verwenden. Im
nächsten Schritt müssen schmale Rauten angefügt werden; sie entsprechen den gefärbten Teilen in der Unterteilung. Nach der Färbung des
linken Muster muss der rechte Stern durch fünf schmale Rauten zu einem Zehneck erweitert werden, nach der Färbung des rechten Muster
dagegen müssen zehn schmale Rauten an die Seiten der Zacken des
linken Sterns angelegt werden. Achten Sie darauf, dass die Färbungen
an den Kanten zusammenpassen; es gibt immer eine richtige und eine
falsche Möglichkeit.
17.3. Das Penrose-Zehneck. Zeigen Sie, dass die Unterteilung des
Penrose-Zehnecks (siehe nachfolgende Figur) wieder ein kleineres PenroseZehneck enthält, das um 180 Grad gedreht ist. Machen Sie sich klar,
dass man damit ein beliebig großes Penrosemuster konstruieren kann,
indem man das unterteilte Innere der jeweiligen Figur als Plan zum
Weiterbau verwendet.
Literatur
[1] Alten, H.-W., et al.: 4000 Jahre Algebra. Geschichte, Kulturen, Menschen,
Springer 2003
[2] Attali, Jacques: Blaise Pascal, Biographie eines Genies, Klett-Cotta 2007
[3] Baez, John: The Octonions, Bulletin Amer. Math. Soc. 39, S. 145 - 205 (2001),
http://math.ucr.edu/home/baez/octonions
[4] Bell, E.T.: Men of Mathematics, Fireside 1937/1965
STERNSTUNDEN DER MATHEMATIK
175
[5] Klein, Felix: Vorlesungen über die Entwicklung der Mathematik im 19. Jahrhundert, Springer 1926
[6] Klein, Felix: Vorlesungen über das Ikosaeder, Hg.: Peter Slodowy, Birkhäuser
1993
[7] Linden, Sebastian: Die Algebra des Omar Chayyam, Edition Avicenna 2012
[8] Mania, Hubert: Gauß. Eine Biographie, Rowohlt 2008
[9] Neffe, Jürgen: Einstein. Eine Biographie, Rowohlt 2006
[10] Penrose, Roger: The Road to Reality, New York 2005
[11] Pesic, Peter: Abels Beweis, Springer 2005
[12] Riemann, Bernhard: Über die Hypothesen, welche der Geometrie zu Grunde
liegen. Historisch und mathematisch kommentiert von Jürgen Jost, Springer
2013
[13] Scriba, C.J., Schreiber, P.: 5000 Jahre Geometrie. Geschichte, Kulturen, Menschen, Springer 2001
[14] Wußing, H.: 6000 Jahre Mathematik. Eine kulturgeschichtliche Zeitreise,
Springer 2008, 2013
Index
Cayley-Zahlen, 95
Cayleytripel, 99
Chance, 62, 64
Christoffel, 106
Cohen, 145
A5 , 88, 110, 119, 172
Abbildung, 85
Abel, 80, 81, 109
Ackermann, 141
Addition, 55
Ähnlichkeit, 7, 19, 23
Al-Chwarizmi, 41
Alberti, 33
Algebra, 41, 96
Algorithmus, 5, 41
anti-assoziativ, 97
Antifixraum, 155
Äquator, 152
Archimedes, 9, 17, 43
Argand, 54, 69, 78
Aristoteles, 124
Artmann, 11
Äther, 124
Atiyah, 157
Atombombe, 126
Ausnahmegruppen, 98, 119
Automorphismus, 99
d’Alembert, 69, 74
de Bruijn, 172
de Méré, 60, 67
de Moivre, 61
del Ferro, 46
del Fiore, 46
della Nave, 48
Desargues, 35, 36, 59
Descartes, 60
Diffeomorphismus, 161
Differentialgleichung, 118
Diophantos, 42
Diskriminante, 110
Divergenz, 128, 130
Divisionsalgebra, 93, 98
Doppelspalt, 53
Drehzentrum, 167
Dürer, 33, 39
Berger, 159
Bernoulli, 69
Best Case, 66
Betrag, 56, 93, 95
Binom, 65
Binomialkoeffizienten, 62, 64, 65, 68
Binomische Formel, 65, 68, 69
Bogenmaß, 71
Böhm, 161
Bolyai, 101
Bombelli, 51, 53, 58, 77
Borel, 157
Bott, 147
Brioschi, 109
Brouwer, 136
Brunelleschi, 31, 33
Bush, 67
e, 71, 72
Ebene, 54
Eigenzeit, 129
einfach zusammenhängend, 159, 160,
164
einfache Gruppe, 97
Einparametergruppe, 152
Einstein, 59, 123
Einsteintensor, 130, 131
elementarsymmetrisch, 83, 115
Energie, 149
Entscheidungsproblem, 168
Eschenburg, 73
Eudoxos, 9
Euklid, 5, 59, 100, 134
euklidischer Algorithmus, 5
Euler, 69, 70, 72, 74, 145
Exhaustion, 23
Exotische Sphäre, 161, 164
Exponentialfunktion, 69, 70
Cantor, 137
Cardano, 46, 48, 52, 63
Cartan, 98
Casus Irreducibilis, 51
Cauchy, 82
Cavalieri, 20
Fakultät, 64
176
STERNSTUNDEN DER MATHEMATIK
Fermat, 61, 63
Fernebene, 35
Ferngerade, 35
Fernpunkt, 35, 38
Ferrari, 47, 48
Fibonacci, 45
Fixpunktmenge, 153
Fixraum, 155
Folge, 136
Formalismus, 136
Foto, 39
Fourier, 81
Français, 54
Freudenthal, 98
Fünfeck, 7
Galilei, 122–124
Galileitransformation, 123
Galois, 79
Galoisgruppe, 85
Gaußkrümmung, 106
Gauß, 69, 72, 80, 81, 101, 102, 106,
108, 145, 164
Gemeinsames Maß, 5
Geodäte, 129, 150
Geodäten, 104, 105
Geometrie, 30, 35, 37, 38, 41, 56, 59,
93, 97, 99, 101, 103, 107, 121
Gleichheit, 2
Gödel, 140
Gödelzahl, 142
Goldener Schnitt, 6, 8, 115, 169, 173
Gradient, 128, 149
Grassmann, 38, 155
Gromoll, 160
Gromov, 161
Größen, 2, 53, 56
Großkreis, 151
Gruppe, 85, 97, 135, 147, 148
günstig, 62
Hadamard, 164
Hahn, 140
Halbebene, 114
Hamilton, 93
Hao Wang, 168
Hausdorff, 145
Helmholtz, 104
Hermite, 109
Hieron, 17
Hieronymus, 39
Hilbert, 136, 137, 168
Hippasos, 6, 173
Hirzebruch, 157
homogen, 121
homotop, 146
Homotopiegruppe, 147
Horizont, 31
Hurewicz, 146
Hurwitz, 93
Hyperboloid, 134
hypergeometrische Reihe, 119
i, 52, 72
Ikosaeder, 11, 110, 119, 172
Ikosaederfunktion, 111
imaginär, 52, 53, 71
Imaginärteil, 55
Index, 149
Inertialsystem, 123, 126
inkommensurabel, 8
Interferenz, 53
Intuitionisten, 136
Involution, 155
irrational, 8
Isometrie, 153
Jacobi, 81
Kalender, 42
Kant, 100, 123
Karcher, 161
Kegel, 22, 23
Kegelschnitt, 44
Kepler, 31, 60, 122, 127
Kervaire, 93
Kettenbruch, 5, 12, 15
Khayyam, 42, 48, 49, 63
Killing, 98
Kirchgraber, 45
Klein, 109
Klingenberg, 159, 162
Koeffizienten, 82
komplex, 155
Komplexe Struktur, 153
Komplexe Zahlen C, 53, 55
Konjugation, 56, 71, 87, 98
Konjugation in Gruppen, 86
177
178
J.-H. ESCHENBURG
konkav, 161
Konstruktion, 134
Kontinuumshypothese, 137, 145
Konvergenz, 136
konvex, 161
Kopernikus, 31, 122
Körper, 55
Krümmungsvektor, 150, 152
Kramer, 172
Kreis, 19, 21, 40, 71, 72
Kristall, 166
kritischer Punkt, 149
Kronecker, 109
Krümmungstensor, 105, 107
Kubikzahl, 49, 52
Kubische Gleichung, 43, 45
Kugel, 21, 77, 112, 161
Kugelkappe, 26
Kürzeste, 150
Länge, 149
Lagrange, 15, 73, 74, 80, 91
Λ, 149
Lange, 123
Legendre, 80, 101
Levi-Civita, 106, 130, 159
lichtartig, 129
Lie, 98
linear, 147
Lipschitz, 106
Lobachevski, 101
Loch, 146
Lorentz, 125
Lorentzmetrik, 129
Lorentztransformation, 125
Lotto, 64
Mach, 131
Mächigkeit, 138
Mächtigkeit, 145
Mannigfaltigkeit, 102, 145
Masaccio, 33
Masse, 127
Massetensor, 130
Matyasevich, 168
Menge, 2, 137
Menger, 140
Mersenne, 59
Messen, 3, 103
Metamathematik, 142
Metrik, 103, 104, 125, 129, 130
Meyer, 160
Michelson-Experiment, 124
Milnor, 93, 148, 161
Minkowski, 125, 129
Möbius, 16, 76
Moltke, 67
Monodromiegruppe, 119
Monom, 90
Morse, 148
Multiplikation, 72
Muster, 166
Nebenklasse, 86
Newton, 83, 90, 110, 118, 122, 127
nichteuklidisch, 101
Normalkoordinaten, 105
Normalteiler, 86, 97
normiert, 82
Normierte Algebra, 96
Nullstelle, 74
Oktaeder, 110
Oktaven, 94
Oktaven O, 93
Olinde Rodrigues, 93
Ordnung, 88
orthogonal, 96
Orthogonale Gruppe, 147
orthonormal, 97
Parallelverschiebung, 106
Pascal, 40, 59, 60
Pascalsches Dreieck, 63
Pauling, 165
Penrose, 169
Perelman, 161
Permutationen, 85, 109
Pfaff, 74
π, 19, 21, 72
Platon, 11
Poincaré, 125, 145, 161
Poisson, 81, 129
Pol, 152
Poncelet, 36
Potential, 128
Potenz, 52
Potenzmenge, 138
STERNSTUNDEN DER MATHEMATIK
Prisma, 22
Projektionszentrum, 33
Projektive Geometrie, 35
Projektiver Raum, 38, 117, 164
Punktspiegelung, 151
Pyramide, 22
Pythagoras, 6, 21, 95, 104
Quadrat, 21
Quadratische Gleichung, 15
Quadratwurzel, 11, 51, 53
Quadratzahl, 11, 16, 73
Quadrik, 117, 134
Quantenmechanik, 53, 59
Quartische Gleichung, 47, 82
Quasikristall, 165, 170
quaternional, 156
quaternionale Struktur, 154
Quaternionen H, 93
Quelle, 128
Quintische Gleichung, 50, 80
rationale Funktion, 115
Rationale Zahlen, 8
Raum, 35, 37, 38, 93, 100, 102, 122
Raumzeit, 126
Realteil, 55
Rechenmaschine, 60
rechnen, 85
Reelle Gerade, 96
Reelle Zahlen, 145
Reelle Zahlen R, 9
reflektiv, 153
Regenwasserfluss, 149
Relativitätstheorie, 126
Renaissance, 45
Resolvente, 83, 91
Richard, 139
Riemann, 77, 100, 115, 145, 159
Ruffini, 109
Russell, 138
Samelson, 148
Scheibe, 146
Schickard, 60
Schnittkrümmung, 160
Schwarz, 115, 118
Schwarzschild, 132
Schwimmen, 19
Sekante, 23
Selbstähnlichkeit, 11, 17
Serre, 157
Shechtman, 165
Singer, 157
Slodowy, 119
Sphäre, 146
Sphäre, 160
Sphärensatz, 160
Spinoza, 135
Spirale, 25
Sporadische Gruppen, 97
Stereographische Projektion, 112,
165
Substitution, 45, 143
Symmetrie, 89, 111, 113, 166, 172
symmetrisch, 90
Symmetrische Funktion, 83
Symmetrischer Raum, 147, 152
Tartaglia, 46, 63
Taylor, 105
Tertium non datur, 136
Tetraeder, 22
Theaitetos, 11
Theodoros, 11
Tits, 98
Topologie, 74, 103, 145
totalgeodätisch, 153
totalkomplex, 156
totalreell, 156
Transformation, 16
Tschirnhaus, 116, 120
Turing, 168
Umkehrung, 85
Umlaufszahl, 76, 158
Unbekannte, 42
unendlich, 20, 24, 173
unentscheidbar, 144, 145
Untergruppe, 85, 86
unvollständig, 141, 144
Vakuum, 60
Vektor, 56, 95
Vergleichen, 2
Verhältnis, 3, 19
Vieta, 82, 83, 87, 110, 115
Vinberg, 98
179
180
J.-H. ESCHENBURG
vollständig, 134, 141, 160
Volumen, 21, 22
Wahrscheinlichkeit, 59, 69
Wechselwegnahme, 5, 9
Weg, 146, 149
Weltlinie, 132
Wessel, 54
Weyl, 102, 105, 106, 136, 148
Whitehead, 139
Whitney, 146
Widerspruchsfreiheit, 136, 141, 144
Winkel, 71
Worst Case, 66
Zählen, 3
Zahl, 2, 6, 8, 53, 71
Zahlbereichserweiterung, 9, 53, 92
Zahlenebene, 54
Zahlenkugel, 77, 112
Zeit, 124, 125
zeitartig, 129
Zentralperspektive, 30, 38
Zentralprojektion, 33
Zinseszins, 69
Zusammenhangskomponente, 146
Zweig, Stefan, 1
STERNSTUNDEN DER MATHEMATIK
181
Inhaltsverzeichnis
Einführung
1. Pythagoras: Verhältnis und Unendlichkeit (−500)
1.1. Kommensurabel = Rationales Verhältnis
1.2. Anwendung auf das “Zählbare”
1.3. Gemeinsame Verfeinerung
1.4. Zahlen ohne eindeutige Primfaktorzerlegung
1.5. Fibonacci-Zahlen:
2. Theodoros: Quadratwurzeln und Selbstähnlichkeit (−430)
2.1. Konstruierbarkeit von Quadratwurzeln
2.2. Beweis der Selbstähnlichkeit
2.3. Einfache Perioden
2.4. “Goldenes Rechteck”
2.5. Variablenverschiebung
2.6. Anzahl der Tripel (a, b2 , c)
3. Archimedes: Die Rechnung mit dem Unendlichen (−220)
3.1. Kugelkappen
3.2. Integralrechnung
3.3. Kreisberechnung 1
3.4. Kreisberechnung 2
3.5. Kreisberechnung 3
4. Brunelleschi: Wo schneiden sich Parallelen? (1420)
4.1. Perspektive
4.2. Photographische Abbildung
4.3. Dürers Heiliger Hieronymus
4.4. Satz von Pascal
5. Cardano: Kubische und quartische Gleichung (1545)
5.1. Omar Khayyams Teilung des Viertelkreises
5.2. Omar Khayyams Dreieck
5.3. Cardanos Formel
5.4. Lösung der quintischen Gleichung?
6. Bombelli: Die Zahl, die es nicht gibt (1572)
6.1. Kubische Gleichung, Casus Irreducibilis
6.2. Beispiel von Bombelli:
6.3. Multiplikation mit i und Jim Knopfs Vergesslichkeit
7. Pascal: Gott würfelt nicht, aber der Mensch (1654)
7.1. de Mérés Beobachtung (1)
7.2. de Mérés Beobachtung (2)
7.3. Gerechte Aufteilung nach Pascal
7.4. Binomialkoeffizienten
7.5. Binomische Formel (1)
1
2
9
9
9
10
10
11
16
17
17
17
17
17
17
25
26
26
27
28
30
38
39
39
40
41
48
49
49
50
51
58
58
58
59
67
68
68
68
68
182
J.-H. ESCHENBURG
7.6. Binomische Formel (2)
68
8. Gauß: Der Fundamentalsatz der Algebra (1799)
68
8.1. Negative Potenzen
77
8.2. Rationale Potenzen von e
77
8.3. Bombellis Kubikwurzel
77
8.4. Kubische Gleichung
77
8.5. Beweis von Argand
78
9. Galois: (Un-)Lösbarkeit von Gleichungen (29.5.1832)
78
9.1. Permutationen und Würfeldrehungen
89
9.2. Lösung der quartischen Gleichung:
89
9.3. Newtons Algorithmus
90
9.4. Lagrange-Resolventen
91
9.5. Lösung der kubischen Gleichung nach Lagrange
91
10. Graves: Die Grenze des Zahlenreichs (26.12.1843)
92
10.1. Konjugation in normierten Algebren
98
10.2. Divisionsalgebren
98
10.3. Automorphismen der Oktaven
99
10.4. Automorphismen der Quaternionen
99
11. Riemann: Die Geometrie des Raumes (10.6.1854)
99
11.1. Diagonalisierung quadratischer Formen
108
11.2. Riemanns Definition des Krümmungstensors
108
12. F. Klein: Ikosaeder und quintische Gleichung (1884)
109
12.1. Tschirnhaus-Transformation
120
12.2. Ikosaederdrehungen
120
12.3. Geraden auf dem einschaligen Hyperboloid
120
12.4. Hyperboloid und Produktfläche
121
13. Einstein: Philosophisches Rätsel gelöst (25.11.1915)
121
13.1. Die Massendichte ist nicht Lorentz-invariant
132
13.2. Das Zwillingsparadoxon
133
13.3. Die De-Sitter-Raumzeit
133
14. Gödel: Ist die Mathematik axiomatisierbar? (1931)
134
15. Bott: Der Periodizitätssatz (1959)
145
15.1. Mittelpunkte kürzester Geodäten
157
15.2. Rechnen mit antikommutierenden komplexen Strukturen 158
15.3. 1. Homotopie der Kreislinie
158
15.4. 1. Homotopie von SO3
158
15.5. 1. Homotopie von Spn
159
16. Klingenberg: Krümmung und Gestalt (1961)
159
16.1. Die Krümmungsschranken für die Projektiven Räume
164
17. Shechtman: Quasikristalle (8.4.1982)
165
17.1. Drehzentren
173
17.2. Symmetrische Penrosemuster
174
STERNSTUNDEN DER MATHEMATIK
17.3. Das Penrose-Zehneck
Literatur
Index
183
174
174
176
Institut für Mathematik, Universität Augsburg, 86135, Augsburg
E-mail address: [email protected]